31.07.2014 Views

Mecânica - IFSC - USP

Mecânica - IFSC - USP

Mecânica - IFSC - USP

SHOW MORE
SHOW LESS

You also want an ePaper? Increase the reach of your titles

YUMPU automatically turns print PDFs into web optimized ePapers that Google loves.

Mecânica<br />

Esmerindo Bernardes 1<br />

L.I.A. – Laboratório de Instrumentação Algébrica<br />

Departamento de Física e Ciência dos Materiais<br />

Instituto de Física de São Carlos<br />

Universidade de São Paulo<br />

www.lia.if.sc.usp.br<br />

18 de Junho de 2010<br />

1 email: sousa@if.sc.usp.br


Conteúdo<br />

1 Introdução 1<br />

2 Cinemática 3<br />

2.1 O espaço Euclidiano . . . . . . . . . . . . . . . . . . . . . . . . . . . . . . . . . . . . . . . 3<br />

2.1.1 Exercícios . . . . . . . . . . . . . . . . . . . . . . . . . . . . . . . . . . . . . . . . . 4<br />

2.2 Vetor posição . . . . . . . . . . . . . . . . . . . . . . . . . . . . . . . . . . . . . . . . . . . 5<br />

2.2.1 Exercícios . . . . . . . . . . . . . . . . . . . . . . . . . . . . . . . . . . . . . . . . . 6<br />

2.3 Produto escalar . . . . . . . . . . . . . . . . . . . . . . . . . . . . . . . . . . . . . . . . . . 6<br />

2.3.1 Exercícios . . . . . . . . . . . . . . . . . . . . . . . . . . . . . . . . . . . . . . . . . 8<br />

2.4 Produto vetorial . . . . . . . . . . . . . . . . . . . . . . . . . . . . . . . . . . . . . . . . . 8<br />

2.4.1 Exercícios . . . . . . . . . . . . . . . . . . . . . . . . . . . . . . . . . . . . . . . . . 11<br />

2.5 Trajetórias . . . . . . . . . . . . . . . . . . . . . . . . . . . . . . . . . . . . . . . . . . . . 12<br />

2.5.1 Exercícios . . . . . . . . . . . . . . . . . . . . . . . . . . . . . . . . . . . . . . . . . 13<br />

2.6 Velocidade e aceleração . . . . . . . . . . . . . . . . . . . . . . . . . . . . . . . . . . . . . 14<br />

2.6.1 Exercícios . . . . . . . . . . . . . . . . . . . . . . . . . . . . . . . . . . . . . . . . . 20<br />

2.7 Espaço percorrido . . . . . . . . . . . . . . . . . . . . . . . . . . . . . . . . . . . . . . . . 20<br />

2.7.1 Exercícios . . . . . . . . . . . . . . . . . . . . . . . . . . . . . . . . . . . . . . . . . 23<br />

3 Translações 25<br />

3.1 Os princípios . . . . . . . . . . . . . . . . . . . . . . . . . . . . . . . . . . . . . . . . . . . 25<br />

3.1.1 Exercícios . . . . . . . . . . . . . . . . . . . . . . . . . . . . . . . . . . . . . . . . . 30<br />

3.2 As forças da natureza . . . . . . . . . . . . . . . . . . . . . . . . . . . . . . . . . . . . . . 30<br />

3.2.1 Exercícios . . . . . . . . . . . . . . . . . . . . . . . . . . . . . . . . . . . . . . . . . 33<br />

3.3 Aplicações da segunda lei de Newton . . . . . . . . . . . . . . . . . . . . . . . . . . . . . . 34<br />

3.3.1 Forças constantes . . . . . . . . . . . . . . . . . . . . . . . . . . . . . . . . . . . . . 34<br />

3.3.2 Forças dependentes da velocidade . . . . . . . . . . . . . . . . . . . . . . . . . . . . 36<br />

3.3.3 Forças dependentes da posição . . . . . . . . . . . . . . . . . . . . . . . . . . . . . 40<br />

3.3.4 Exercícios . . . . . . . . . . . . . . . . . . . . . . . . . . . . . . . . . . . . . . . . . 43<br />

4 Leis de conservação I 45<br />

4.1 Trabalho . . . . . . . . . . . . . . . . . . . . . . . . . . . . . . . . . . . . . . . . . . . . . . 45<br />

4.1.1 Exercícios . . . . . . . . . . . . . . . . . . . . . . . . . . . . . . . . . . . . . . . . . 47<br />

4.2 Energia cinética . . . . . . . . . . . . . . . . . . . . . . . . . . . . . . . . . . . . . . . . . . 47<br />

4.2.1 Exercícios . . . . . . . . . . . . . . . . . . . . . . . . . . . . . . . . . . . . . . . . . 48<br />

4.3 Energia potencial . . . . . . . . . . . . . . . . . . . . . . . . . . . . . . . . . . . . . . . . . 48<br />

4.3.1 Exercícios . . . . . . . . . . . . . . . . . . . . . . . . . . . . . . . . . . . . . . . . . 52<br />

4.4 Energia mecânica . . . . . . . . . . . . . . . . . . . . . . . . . . . . . . . . . . . . . . . . . 52<br />

4.4.1 Exercícios . . . . . . . . . . . . . . . . . . . . . . . . . . . . . . . . . . . . . . . . . 55<br />

iii


CONTEÚDO<br />

CONTEÚDO<br />

4.5 Períodos . . . . . . . . . . . . . . . . . . . . . . . . . . . . . . . . . . . . . . . . . . . . . . 55<br />

4.5.1 Exercícios . . . . . . . . . . . . . . . . . . . . . . . . . . . . . . . . . . . . . . . . . 58<br />

4.6 Comentários gerais . . . . . . . . . . . . . . . . . . . . . . . . . . . . . . . . . . . . . . . . 59<br />

5 Rotações 61<br />

5.1 Cinemática . . . . . . . . . . . . . . . . . . . . . . . . . . . . . . . . . . . . . . . . . . . . 61<br />

5.2 Energia rotacional . . . . . . . . . . . . . . . . . . . . . . . . . . . . . . . . . . . . . . . . 63<br />

5.3 Dinâmica rotacional e leis de conservação II . . . . . . . . . . . . . . . . . . . . . . . . . . 66<br />

5.4 As equações de Euler . . . . . . . . . . . . . . . . . . . . . . . . . . . . . . . . . . . . . . . 72<br />

5.5 Exercícios . . . . . . . . . . . . . . . . . . . . . . . . . . . . . . . . . . . . . . . . . . . . . 80<br />

6 Gravitação 83<br />

6.1 Um sistema de dois corpos . . . . . . . . . . . . . . . . . . . . . . . . . . . . . . . . . . . . 84<br />

6.2 A energia potencial de Kepler . . . . . . . . . . . . . . . . . . . . . . . . . . . . . . . . . . 86<br />

6.3 Trajetórias . . . . . . . . . . . . . . . . . . . . . . . . . . . . . . . . . . . . . . . . . . . . 88<br />

6.4 O campo gravitacional . . . . . . . . . . . . . . . . . . . . . . . . . . . . . . . . . . . . . . 91<br />

6.5 Correções . . . . . . . . . . . . . . . . . . . . . . . . . . . . . . . . . . . . . . . . . . . . . 91<br />

6.6 Exercícios . . . . . . . . . . . . . . . . . . . . . . . . . . . . . . . . . . . . . . . . . . . . . 91<br />

7 Relatividade restrista 93<br />

7.1 Transformações de Galileu . . . . . . . . . . . . . . . . . . . . . . . . . . . . . . . . . . . . 93<br />

7.2 Transformações de Lorentz . . . . . . . . . . . . . . . . . . . . . . . . . . . . . . . . . . . 94<br />

7.3 Mecânica relativística . . . . . . . . . . . . . . . . . . . . . . . . . . . . . . . . . . . . . . 95<br />

7.4 Exercícios . . . . . . . . . . . . . . . . . . . . . . . . . . . . . . . . . . . . . . . . . . . . . 97<br />

A Análise dimensional 99<br />

B Séries de Taylor 101<br />

B.1 Exercícios . . . . . . . . . . . . . . . . . . . . . . . . . . . . . . . . . . . . . . . . . . . . . 101<br />

C Coordenadas polares 103<br />

iv


Capítulo 1<br />

Introdução<br />

Estas notas de aulas não pretendem substituir<br />

qualquer livro texto sobre mecânica Newtoniana.<br />

Elas existem apenas como uma forma de organização<br />

de vários tópicos importantes que são comumente<br />

abordados em um primeiro curso sobre as<br />

leis de Newton para o movimento e suas aplicações.<br />

Como tal, elas pretendem ser concisas e enfatizar<br />

apenas conceitos fundamentais, indispensáveis<br />

à aquisição e manipulação de muitos outros conceitos<br />

físicos utilizados em Ciências.<br />

Cabe a você estudante ser extremamente crítico<br />

e procurar pela coerência de cada informação contida<br />

nestas notas. Além disto, estas notas de aulas<br />

devem ser usadas como uma extensa lista de<br />

exercícios. O caminho para chegar nos resultados<br />

exibidos é sempre comentado, cabendo a você a<br />

tarefa de executar os passos intermediários. Assim,<br />

você saberá exatamente de onde veio qualquer<br />

resultado, fórmula ou teorema, bem como suas<br />

condições de validade. Em geral, cada resultado é<br />

válido em um determinado contexto, o qual é revelado<br />

durante a execussão dos passos intermediários.<br />

Outro benefício decorrente da execussão dos passos<br />

intermediários é o aumento da nossa capacidade intelectual<br />

para lidar com conhecimentos novos.<br />

Adotaremos uma abordagem construtivista onde<br />

as leis da mecânica serão derivadas de observações<br />

meticulosas da natureza através de experimentos<br />

simples, porém extremamente ilustrativos. Numa<br />

etapa seguinte, iremos analisar os resultados das<br />

observações realizadas e propor modelos que os descrevam<br />

de forma acurada. Para que estes modelos<br />

possam ser descritos de forma elegante, prática e<br />

com capacidade de fazer previsões, construiremos<br />

diversas “ferramentas matemáticas”, as quais farão<br />

parte do conjunto de ferramentas de trabalho de<br />

qualquer profissional em Ciências Exatas. Todas<br />

estas ferramentas serão estudadas em detalhes nos<br />

cursos de Geometria, Cálculo Diferencial e Integral,<br />

Equações Diferenciais, Eletromagnetismo e Termodinâmica.<br />

Desta forma, estas notas de aula pretendem<br />

criar um laboratório para promover a integração<br />

de todos os demais cursos do Ciclo Básico<br />

em Ciências. Estes cursos estão todos entrelaçados,<br />

interligados por conduítes que permitem entrada e<br />

saída de conhecimentos.<br />

Em geral, desejamos usar conhecimentos adquiridos<br />

para tornar nossa sociedade mais agradável,<br />

justa e auto-suficiente. Hoje, vivemos em um momento<br />

da história do conhecimento humano onde<br />

a taxa de transferência de conhecimentos básicos<br />

para o setor produtivo está crescendo rapidamente.<br />

Apesar deste crescimento, ainda encontramos uma<br />

distância muito longa entre a aquisição de conhecimentos<br />

e o uso destes. Treinamento é uma<br />

forma bastante eficaz de encurtar esta distância.<br />

Vários exercícios são propostos no final de cada<br />

capítulo para ressaltar aspectos importantes dos<br />

resultados obtidos, além de evidenciar como estes<br />

resultados devem ser utilizados. Na medida do<br />

possível, procuraremos realizar exercícios de empreendimento<br />

e transferência de conhecimento construindo<br />

máquinas simples e/ou fazendo simulações<br />

computacionais. Procuraremos construir máquinas<br />

simples cujos princípios de funcionamento dependam<br />

exclusivamente das leis de Newton para o<br />

movimento, como foguetes de água e giroscópios.<br />

Também simularemos via computação algébrica<br />

(ou simbólica) o movimento de objetos sujeitos<br />

às forças elástica, gravitacional e eletromagnética,<br />

sempre agregando alguns efeitos realistas como dissipações<br />

e ressonâncias.<br />

1


Capítulo 1. Introdução<br />

Várias rotinas em computação algébrica serão<br />

criadas, passo a passo, para implementarem os conhecimentos<br />

adquiridos através dos textos de referência<br />

e notas de aula bem como aqueles adquiridos<br />

em sala de aula. Computação algébrica é sem<br />

dúvida uma ferramenta indispensável ao ensino e<br />

pesquisa. Além disto, assim como os atuais estudantes,<br />

este curso pretende ser pós-computadores.<br />

Os computadores serão usados para executar tarefas<br />

que nos tomariam muito tempo, nos liberando<br />

para construir modelos e analisar seus resultados,<br />

exercitando assim ainda mais a nossa capacidade<br />

intelectual. Mais especificamente, usaremos<br />

o computador para resolver as equações diferenciais<br />

decorrentes das leis de Newton, desenhar<br />

as trajetórias decorrentes e realizar as animações<br />

correspondentes. Derivadas, integrais e soluções<br />

numéricas de equações diferenciais serão efetuadas<br />

computacionalmente por um processo similar<br />

ao uso de calculadoras de bolso na realização de<br />

operações aritméticas.<br />

Não basta sabermos resolver determinados<br />

exercícios, em geral com o intuito de sermos aprovados<br />

em um determinado curso. Devemos aprender a<br />

aplicar nossos conhecimentos adquiridos num contexto<br />

mais amplo. Também não podemos esperar<br />

por um momento mágico para começarmos a desenvolver<br />

nossa capacidade empreendedora. O melhor<br />

momento para isto é agora. Ao estudar as leis de<br />

Newton para o movimento e suas aplicações, desenvolva<br />

concomitantemente a sua capacidade empreendedora.<br />

Imagine o computador a sua fábrica e<br />

uma determinada linguagem de programação como<br />

matéria prima. Você é capaz de produzir com seus<br />

conhecimentos adquiridos? Simular o movimento<br />

de uma simples bolinha de gude sob ação da gravidade<br />

(com alguma dissipação) é um excelente produto.<br />

O objetivo deste curso é oferecer oportunidades,<br />

através de exemplos e exercícios, para que<br />

você aprenda a aprender. Assim, você estará caminhando<br />

na direção de se tornar uma pessoa e<br />

um profissional crítico, analítico, com capacidade<br />

de absorver, gerar e transmitir conhecimentos, conforme<br />

requer o projeto pedagógico desta universidade.<br />

Nestas notas, abordaremos os tópicos seguintes:<br />

1. Cinemática. Iniciaremos estudando as propriedades<br />

básicas do espaço Euclidiano, palco<br />

de quase todos os movimentos que iremos estudar.<br />

Precisaremos saber como representar<br />

pontos (objetos) e curvas (trajetórias) neste<br />

espaço. Para tal, teremos que construir sistemas<br />

de coordenadas, vetores e algumas ferramentas<br />

específicas, como os produtos escalar<br />

e vetorial, para lidar com vetores. Mais<br />

detalhes serão vistos no curso de Geometria<br />

Analítica. Em seguida, construiremos mais<br />

ferramentas específicas (derivadas e integrais)<br />

para lidarmos com velocidades, acelerações,<br />

forças e trajetórias. Mais detalhes sobre derivadas<br />

e integrais serão vistos nos diversos cursos<br />

de Cálculo. Finalmente, faremos uso de<br />

computao (algébrica ou simbólica) para simular<br />

objetos reais em movimento. Estes tópicos<br />

estão discutidos no Cap. 2 (Cinemática) das<br />

notas de aulas.<br />

2. Dinâmica. Estabeleceremos aqui as leis que<br />

governam um movimento geral (translação +<br />

rotação). Iniciaremos pelas leis que governam<br />

as translações. Utilizaremos de alguns<br />

experimentos para estabelecermos tais leis.<br />

Em seguida, construiremos uma estrutura matemática<br />

para expressarmos e manipularmos<br />

tais leis. Esta estrutura matemática, como<br />

veremos, será auto-consistente, elegante e capaz<br />

de fazer previsões. Em seguida, estudaremos<br />

as leis que governam as rotações. Veja os<br />

Caps. 3 (Translações), 4 (Rotações) e 5 (Leis<br />

de Conservação) das notas de aulas.<br />

3. Gravitação. Veremos como Newton usou<br />

o conceito de força, recém inventado por<br />

ele mesmo, para explicar como objetos são<br />

atraídos pela Terra, bem como nossos planetas<br />

orbitão o Sol. Também deduziremos as leis<br />

de Kepler da força da gravidade proposta por<br />

Newton. Finalizaremos nossas discussões estudando<br />

os fundamentos da Relatividade Restrita.<br />

Veja o Cap. 6 (Gravitação).<br />

Além das notas de aulas, as seguintes referências<br />

podem ser úteis:<br />

1. Herch Moysés Nussenzveig – Curso de Física<br />

Básica: Mecânica.<br />

2. Alaor Silvério Chaves – Curso Básico para Estudantes<br />

de Ciências Físicas e Engenharias.<br />

Vol. 1: Mecânica.<br />

2


Capítulo 2<br />

Cinemática<br />

Estudaremos aqui os conceitos de posição, velocidade<br />

e aceleração, sem nos preocupar com as<br />

suas causas, ao invés, procuraremos construir ferramentas<br />

matemáticas adequadas a uma descrição<br />

elegante e prática destas grandezas físicas.<br />

Em geral, um objeto qualquer move-se em um<br />

determinado espaço. Portanto, precisaremos construir<br />

uma forma efetiva de representar posição, velocidade<br />

e aceleração deste objeto em cada instante<br />

de tempo neste espaço. Para efetuarmos esta<br />

construção, denominada de sistemas de coordenadas,<br />

iremos precisar de ferramentas matemáticas.<br />

Precisaremos de pontos para localizar nossos objetos<br />

físicos e de curvas para representar suas trajetórias.<br />

Precisaremos também de vetores (por<br />

exemplo, os vetores posição, velocidade, aceleração<br />

e forças) bem com de matrizes (por exemplo, momento<br />

de inércia) para representar diversas quantidades<br />

físicas. Também iremos construir ferramentas<br />

específicas, como derivadas e integrais, além de<br />

produtos escalares e vetorias, para modificarmos<br />

quantidades físicas.<br />

2.1 O espaço Euclidiano<br />

O que é o espaço? Pode parecer inacreditável,<br />

mas ainda não dispomos de uma resposta concreta<br />

a esta pergunta e, talvez, nunca venhamos tê-la.<br />

No entanto, veremos que, mesmo desprovidos de<br />

uma definição, seremos capazes de usar o espaço de<br />

forma operacional. Encontraremos outras duas situações<br />

análogas envolvendo os conceitos de tempo<br />

e massa. Esta capacidade de operar com conceitos<br />

desprovidos de uma definição única é, sem dúvidas,<br />

uma das grandes conquistas humanas.<br />

Por enquanto vamos admitir a “existência” de<br />

um espaço caracterizado pelas seguintes qualidades:<br />

(i) a soma dos espaços das partes é igual ao<br />

espaço do todo contendo estas partes; (ii) o espaço<br />

é desprovido de matéria e, portanto, desprovido de<br />

qualquer propriedade que possa influenciar no movimento<br />

dos corpos; (iii) o espaço é homogêneo, isto<br />

é, qualquer posição ao longo de uma determinada<br />

direção é sempre igual às demais; (iv) o espaço é<br />

isotrópico, isto é, estando em uma posição fixa, todas<br />

as direções são idênticas; (v) o espaço apresenta<br />

três dimensões independentes (por exemplo,<br />

largura, profundidade e altura) e é infinito. Como<br />

veremos a seguir, estas propriedades tornam operacional<br />

o conceito de espaço.<br />

Tendo estabelecido estas propriedades, podemos<br />

afirmar que objetos podem ser localizados neste<br />

espaço através de coordenadas, medidas em relação<br />

a alguma posição fixa, previamente escolhida, a<br />

qual chamaremos de referencial. Note, portanto,<br />

que coordenadas são medidas relativas de distância.<br />

Também é importante mencionar que espaço e<br />

tempo são conceitos distintos: relógios sincronizados<br />

podem ser colocados simultaneamente em<br />

qualquer posição neste espaço que estamos considerando.<br />

Desta forma, trabalharemos com a noção<br />

de tempo absoluto, ou seja, relógios sincronizados<br />

sempre marcarão os mesmos intervalos de tempo<br />

em qualquer posição do espaço. Discutiremos as<br />

propriedades operacionais da noção de tempo na<br />

segunda parte contendo as leis de Newton.<br />

É importante ter em mente que o conceito de<br />

espaço que estamos usando aqui, comumente denominado<br />

de espaço Euclidiano, uma homenagem<br />

ao matemático grego Euclides que viveu no Séc. III<br />

a.c., considerado o fundador da geometria plana, foi<br />

estabelecido somente a partir do Séc. XIV. As principais<br />

propriedades dos objetos geométricos da geo-<br />

3


2.1. O espaço Euclidiano Capítulo 2. Cinemática<br />

metria Euclidiana serão estudadas detalhadamente<br />

no curso de Geometria Analítica. No entanto, usaremos<br />

aqui as noções de ponto como sendo um objeto<br />

adimensional, de curva como sendo um objeto<br />

unidimensional (tendo apenas comprimento, como<br />

retas, parábolas, circunferências, elipses, espirais,<br />

etc.) e de superfície como sendo um objeto bidimensional<br />

(tendo apenas área, como planos, cascas<br />

esféricas e cilíndricas, etc.). Também utilizaremos<br />

a noção de vetor como sendo uma flecha<br />

(segmento de reta) tendo comprimento, direção e<br />

sentido. Também assumiremos que o teorema de<br />

Pitágoras 1 para triângulos retângulos seja conhecido.<br />

Muito bem, tendo estabelecido as principais propriedades<br />

do espaço Euclidiano, o qual está prontinho<br />

para receber objetos, devemos fixar uma<br />

notação para as coordenadas que irão localizar um<br />

determinado objeto. A Figura 2.1 ilustra uma<br />

situação típica: um ponto m, com coordenadas<br />

m(x, y, z), representado em um sistema de coordenadas<br />

com eixos X, Y e Z mutuamente perpendiculares<br />

(ortogonais), conhecido como sistema de<br />

coordenadas cartesiano, introduzido por René Descartes<br />

no Século XVII. Os números reais (x, y, z)<br />

são determinados graficamente da seguinte forma:<br />

primeiro traçamos uma reta paralela ao eixo Z passando<br />

por m até interceptarmos o plano XY . Passando<br />

por este ponto de intersecção, traçamos retas<br />

paralelas aos eixos Y e X, as quais interceptam os<br />

eixos X e Y em x e y, respectivamente. A coordenada<br />

z é obtida traçando uma reta paralela ao<br />

plano XY até interceptarmos o eixo Z em z.<br />

É possível calcular a distância entre dois pontos,<br />

digamos A(x a , y a , z a ) e B(x b , y b , z b ), neste espaço<br />

Euclidiano usando somente suas coordenadas, sem<br />

a necessidade de uma régua? Sim, é perfeitamente<br />

possível. Uma possível maneira é definir a distância<br />

entre dois pontos como sendo o comprimento do<br />

segmento de reta que os une. Por exemplo, o<br />

segmento de reta que une a origem O(0, 0, 0) e<br />

o ponto m(x, y, z) na Figura 2.1 tem um comprimento<br />

igual a √ x 2 + y 2 + z 2 , como podemos obter<br />

aplicando o teorema de Pitágoras duas vezes (faça<br />

o Exercício 2). Assim, a distância entre a origem<br />

O e o ponto m é √ x 2 + y 2 + z 2 . Este exemplo nos<br />

ensina que em geral a distância d(A, B) entre dois<br />

1 A soma dos quadrados dos catetos é igual ao quadrado<br />

da hipotenusa.<br />

X<br />

x<br />

î<br />

z<br />

ˆk<br />

O<br />

Z<br />

Figura 2.1: Um objeto m localizado no espaço Euclidiano<br />

XY Z pelo vetor posição ⃗r com coordenadas<br />

(x, y, z). Os versores î, ˆȷ e ˆk são ortogonais e<br />

estão normalizados a unidade.<br />

pontos quaisquer A(x a , y a , z a ) e B(x b , y b , z b ) pode<br />

ser calculada através de suas coordenadas por<br />

d(A, B) =<br />

⃗r<br />

ĵ<br />

= √ (x b − x a ) 2 + (y b − y a ) 2 + (z b − z a ) 2 . (2.1)<br />

Como um vetor é formado por dois pontos no<br />

espaço (origem e ponta), então podemos usar a expressão<br />

da Eq. (2.1) para calcularmos o comprimento<br />

de vetores (faça os Exercícios 1–3).<br />

2.1.1 Exercícios<br />

Exercício 1<br />

Use o teorema de Pitágoras para determinar o comprimento<br />

do vetor ⃗ρ no plano XY na Figura 2.1 em<br />

termos das coordenadas x e y.<br />

Exercício 2<br />

Use o resultado do exercício anterior e novamente<br />

o teorema de Pitágoras para determinar o comprimento<br />

do vetor ⃗r na Figura 2.1.<br />

Exercício 3<br />

Mostre que o comprimento do vetor ⃗r calculado no<br />

exercício anterior também pode ser obtido pela expressão<br />

dada na Eq. (2.1) com o ponto A sendo a<br />

⃗ρ<br />

m<br />

y<br />

Y<br />

4


Capítulo 2. Cinemática<br />

2.2. Vetor posição<br />

origem O e o ponto B sendo o ponto m (veja a<br />

Figura 2.1).<br />

2.2 Vetor posição<br />

Na Figura 2.1 estamos usando o vetor ⃗r para indicar<br />

a posição do objeto m. Um vetor com a origem<br />

fixa na origem de um sistema de coordenadas<br />

e com a ponta na posição de um objeto (em movimento<br />

ou não) é denominado de vetor posição. Em<br />

princípio não precisamos usar um vetor para localizar<br />

um ponto no espaço. No entanto, a noção de<br />

velocidade requer a presença de um vetor posição,<br />

isto é, requer a especificação de uma direção e de<br />

um sentido, além do seu valor. Quando há movimento,<br />

é importante especificar também a direção<br />

e o sentido deste movimento. Em outras palavras,<br />

precisamos saber para onde estamos indo, literalmente.<br />

Como veremos, vetores constituem uma<br />

linguagem matemática extremamente concisa, elegante<br />

e prática para descrevermos posições, velocidades,<br />

acelerações e outras quantidades físicas,<br />

como forças, que necessitam de direção e sentido,<br />

além de intensidade, para serem especificadas completamente.<br />

Em geral, quando a origem de um vetor estiver<br />

na origem de um sistema de coordenadas, escreveremos<br />

um vetor como uma matriz linha, ⃗r = (x, y, z),<br />

formada pelas componentes x, y e z da ponta do<br />

vetor. Note que as coordenadas (x, y, z) do vetor<br />

⃗r são as mesmas coordenadas do ponto m. Isto<br />

ocorre toda vez que colocamos a origem de qualquer<br />

vetor na origem do sistema de coordenadas. As coordenadas<br />

(x, y, z) do vetor ⃗r representam as suas<br />

projeções (ortogonais) sobre os três eixos ortogonais<br />

X, Y e Z, respectivamente (veja a Figura 2.1).<br />

Também devemos lembrar que as propriedades de<br />

homogeneidade (o epaço é o mesmo ao longo de<br />

uma dada direção) e isotropia (o espaço é o mesmo<br />

em todas as direções) nos permitem fixar a origem<br />

do sistema de coordenadas em qualquer posição do<br />

espaço, ou, equivalentemente, nos podemos sempre<br />

transladar nossos vetores sem girá-los. Observe que<br />

esta translação deve ser feita de tal forma a manter<br />

o vetor na nova posição, sempre paralelo ao vetor<br />

original. Esta translação sem rotação é conhecida<br />

por transporte paralelo.<br />

Essencialmente, o que estamos fazendo é usando<br />

vetores para representar posições. vejamos então<br />

algumas propriedades importantes sobre vetores.<br />

Primeiro, vamos denotar por ˆr um vetor de comprimento<br />

unitário, o qual chamaremos de versor.<br />

Sendo r o comprimento do vetor ⃗r, então ˆr = ⃗r/r.<br />

Observe na Figura 2.1 que o teorema de Pitágoras<br />

para triângulos retângulos nos permite escrever ⃗ρ =<br />

xî+yˆȷ, bem como ⃗r = ⃗ρ+zˆk, ou seja ⃗r = xî+yˆȷ+zˆk.<br />

Este resultado está nos informando que podemos<br />

fazer combinações lineares de vetores, isto é, multiplicação<br />

de vetores por números 2 e soma entre vetores.<br />

Seja α e β dois números reais e ⃗r 1 = (x 1 , y 1 , z 1 )<br />

e ⃗r 2 = (x 2 , y 2 , z 2 ) dois vetores (posição). Então,<br />

somando componentes multiplicadas por números,<br />

podemos formar um terceiro vetor,<br />

⃗r 3 = α⃗r 1 + β⃗r 2<br />

= (αx 1 + βx 2 , αy 1 + βy 2 , αz 1 + βz 2 ). (2.2)<br />

Note que combinação linear é uma operação entre<br />

dois vetores (operação binária), fornecendo um terceiro<br />

vetor. Esta propriedade, compartilhada por<br />

todos os vetores, é fundamental para atribuirmos<br />

ao conjunto dos vetores a condição de “espaço vetorial”,<br />

um dos tópicos centrais do curso de Álgebra<br />

Linear.<br />

Outra característica de um vetor, muito importante<br />

para a Física, é o seu comportamento em<br />

relação a rotações em torno de um eixo fixo e à<br />

inversões espaciais. Quando um vetor é rodado em<br />

torno de um eixo fixo, o comprimento do vetor não<br />

é alterado. Apenas a sua direção (e sentido) é alterada.<br />

Inversão espacial significa trocar simultaneamente<br />

o sinal de todas as coordendas (“virar ao<br />

avesso”). Portanto, uma inversão espacial mantém<br />

a direção e muda apenas o sentido do vetor. Um<br />

candidato a vetor (que tem comprimento, direção e<br />

sentido) que permanece invariante a uma inversão<br />

espacial é denominado de pseudo-vetor (exemplos<br />

de pseudo-vetores serão apresentados na Sec. 2.4,<br />

após estudarmos o produto vetorial). Por falar em<br />

vetor e pseudo-vetor, o vetor resultante ⃗r 3 em (2.2)<br />

tem o mesmo comportamento que os vetores ⃗r 1 e<br />

⃗r 2 mediante rotações e inversões espaciais?<br />

2 Números também são conhecidos por escalares, ou seja,<br />

quantidades que precisam apenas de sua magnitude para<br />

serem especificados completamentes.<br />

5


2.3. Produto escalar Capítulo 2. Cinemática<br />

2.2.1 Exercícios<br />

Exercício 4<br />

Desenhe os vetores posição R ⃗ 1 = (1, 2, 1) e R ⃗ 2 =<br />

(1, 1, 2) em um mesmo sistema cartesiano de coordenadas<br />

(ortonormal). Determine as coordenadas<br />

da soma R ⃗ 1 + R ⃗ 2 e da diferença R ⃗ 1 −R ⃗ 2 e representeos<br />

no mesmo sistema de coordenadas anterior. Use<br />

o teorema de Pitágoras para determinar o comprimento<br />

de cada vetor. Repita este procedimento<br />

para outras combinações lineares.<br />

2.3 Produto escalar<br />

Há outras operações binárias que podemos realizar<br />

com vetores, além da combinação linear (2.2)? É<br />

possível inventar uma operação entre dois vetores<br />

que nos dê informações sobre seus comprimentos<br />

e o ângulo entre eles? Sim, é possível. Vejamos<br />

como.<br />

Também podemos realizar uma operação binária<br />

envolvendo dois vetores cujo resultado é um número<br />

real. Esta operação binária entre os vetores ⃗r 1 e ⃗r 2 ,<br />

denotada por ⃗r 1 · ⃗r 2 , será denominada de produto<br />

escalar. O produto escalar é definido requerendo<br />

que ele satisfaça quatro propriedades: que ele seja<br />

simétrico e linear,<br />

⃗r 1 · ⃗r 2 = ⃗r 2 · ⃗r 1 ∈ R, (2.3)<br />

⃗r 3 · (α⃗r 1 + β⃗r 2 ) = α ⃗r 3 · ⃗r 1 + β ⃗r 3 · ⃗r 2 , (2.4)<br />

respectivamente, onde α e β são são números reais;<br />

que ele seja positivo definido,<br />

{<br />

= 0 se ⃗r = ⃗0<br />

⃗r · ⃗r<br />

; (2.5)<br />

> 0 se ⃗r ≠ ⃗0<br />

e que o comprimento r de um vetor ⃗r qualquer<br />

possa ser calculado por<br />

r = ||⃗r|| = √ ⃗r · ⃗r. (2.6)<br />

A propriedade (2.3) nos diz que o produto escalar<br />

é uma operação binária simétrica (ou comutativa).<br />

A propriedade (2.4) significa que o produto escalar<br />

é linear, pois obedece a propriedade distributiva.<br />

Note que os número α e β no lado direito de (2.4)<br />

podem ser retirados de dentro dos produtos escalares.<br />

A propriedade (2.5) nos garante que o produto<br />

escalar é bem comportado (não-degenerado), pois<br />

ela evita que o produto escalar de um vetor com ele<br />

mesmo seja nulo sem que o vetor seja nulo. A propriedade<br />

(2.6) nos diz que o comprimento, também<br />

conhecido por módulo ou norma, pode ser calculado<br />

pelo produto escalar. Podemos notar então<br />

que a propriedade (2.5) está em sintonia com a<br />

definição (2.6) de comprimento como sendo uma<br />

quantidade real positiva ou nula. Nula quando, e<br />

somente quando, o vetor for nulo.<br />

Uma questão importante é: como realizar esta<br />

operação binária, denominada de produto escalar,<br />

em termos de coordenadas? ou seja, como tornar o<br />

produto escalar operacional? Graças à propriedade<br />

distributiva (2.4), basta conhecermos todos os produtos<br />

escalares possíveis entres os versores î, ˆȷ e ˆk,<br />

colocados ao longo dos três eixos independentes do<br />

espaço Euclidiano (veja a Figura 2.1). Desta forma,<br />

teremos, em princípio, nove produtos escalares a serem<br />

determinados, pois cada vetor pode ser escrito<br />

como uma combinação linear dos versores î, ˆȷ e ˆk<br />

do sistema de coordenadas escolhido. No entanto,<br />

a propriedade de simetria, presente na definição do<br />

produto escalar em (2.3), reduz para seis o número<br />

de produtos escalares que devemos conhecer. Uma<br />

forma eficiente de guardarmos estes produtos escalares<br />

é utilizando um arranjo matricial, isto é, numa<br />

matriz 3 × 3,<br />

⎛<br />

î · î î ·ˆȷ î · ˆk ⎞<br />

g = ⎝ˆȷ · î ˆȷ · ˆȷ ˆȷ · ˆk ⎠ . (2.7)<br />

ˆk · î ˆk ·ˆȷ ˆk · ˆk<br />

Utilizando a matriz (2.7), contendo todos os produtos<br />

escalares entre os versores de base, denominada<br />

de métrica, podemos mostrar que o produto<br />

escalar entre dois vetores quaisquer é calculado<br />

em termos de suas componentes da seguinte<br />

forma (faça o Exercício 5),<br />

⃗r 1 · ⃗r 2 = ⃗r 1 g ⃗r T 2 , (2.8)<br />

na qual ⃗r 2<br />

T é uma matriz coluna, obtida da matriz<br />

linha representando o vetor ⃗r 2 pela transposição de<br />

suas linhas por colunas. As operações matriciais<br />

em (2.8) são totalmente equivalentes ao uso da propriedade<br />

de linearidade (2.4) com os vetores escritos<br />

explicitamente em termos de seus versores (tenho<br />

certeza que você irá verificar isto). Esta parece<br />

uma estória sem fim. Afinal de contas, como poderemos<br />

calcular os produtos escalares que aparecem<br />

na métrica (2.7)? Não podemos! Estes produtos<br />

escalares devem ser fornecidos. Decepcionado?<br />

6


Capítulo 2. Cinemática<br />

2.3. Produto escalar<br />

Para entendermos melhor esta situação, devemos<br />

nos perguntar: qual é o significado geométrico do<br />

produto escalar?<br />

ĵ<br />

(a)<br />

î + ĵ<br />

î<br />

⃗C<br />

Ĉ<br />

θ<br />

ˆB<br />

(b)<br />

Figura 2.2: O produto escalar está associado com<br />

a projeção geométrica de um vetor sobre o outro.<br />

Veja os teoremas 1 e 2.<br />

Para entendermos melhor o significado<br />

geométrico do produto escalar, devemos calcular<br />

primeiro o produto escalar entre dois vetores<br />

perpendiculares (ortogonais). Vamos usar os versores<br />

î e ˆȷ como dois representantes típicos de vetores<br />

ortogonais. Como indicado na Figura 2.2 (a), o<br />

comprimento da soma vetorial î + ˆȷ é a hipotenusa<br />

do triângulo retângulo formado pelos versores<br />

î e ˆȷ. Como os versores possuem comprimento<br />

unitário, por definição, então a hipotenusa pode<br />

ser calculada usando o teorema de Pitágoras,<br />

||î +ˆȷ|| 2 = ||î|| 2 + ||ˆȷ|| 2 . (2.9)<br />

Sim, você tem razão. Naturalmente, o valor da<br />

hipotenusa também pode ser calculado usando somente<br />

a ferramenta (2.6) para calcular o comprimento<br />

de um vetor,<br />

||î+ˆȷ|| 2 = (î+ˆȷ)·(î+ˆȷ) = ||î|| 2 +||ˆȷ|| 2 +2î·ˆȷ, (2.10)<br />

onde usamos também a propriedade distributiva do<br />

produto escalar. Comparando estes dois resultados,<br />

(2.9) e (2.10), podemos concluir que î · ˆȷ = 0.<br />

Isto significa que o produto escalar entre dois vetores<br />

ortogonais é nulo. Caso os vetores não sejam<br />

unitários, seguindo o mesmo raciocínio anterior,<br />

este resultado continua valendo. Portanto ele<br />

é um teorema:<br />

Teorema 1<br />

Sejam ⃗ A e ⃗ B dois vetores perpendiculares. Então,<br />

⃗A · ⃗B = 0 ⇔ ⃗ A ⊥ ⃗ B. (2.11)<br />

⃗A<br />

⃗B<br />

Portanto, já sabemos o significado e o valor de<br />

cada elemento na diagonal da métrica (2.7) associada<br />

ao sistema de coordenadas cartesiano da Figura<br />

2.1, pois nossos versores î, ˆȷ e ˆk possuem comprimentos<br />

iguais a um (são unitários). E quando<br />

os vetores não são perpendiculares? qual é o significado<br />

do produto escalar? Considere dois vetores<br />

arbitrários A ⃗ e B, ⃗ formando um ângulo θ entre eles.<br />

Dois vetores sempre estão em um plano, como mostrado<br />

na Figura 2.2 (b). Escolha neste plano um vetor<br />

C ⃗ perpendicular a B. ⃗ Naturalmente, os versores<br />

ˆB e Ĉ formam uma base ortonormal para o vetor A, ⃗<br />

isto é, podemos escrever A ⃗ = A cos θ ˆB + A sin θ Ĉ.<br />

Efetue agora o produto escalar A ⃗ · ˆB e use o Teorema<br />

1. Resulta que A cos θ = A ⃗ · ˆB. Isto é exatamente<br />

a projeção do vetor A ⃗ sobre o vetor B ⃗ (ou<br />

na direção do vetor B). ⃗ Note na Figura 2.1 que<br />

⃗r · î = x, ⃗r · ˆȷ = y e ⃗r · ˆk = z. Naturalmente, os papeis<br />

de ⃗ A e ⃗ B podem ser perfeitamente invertidos.<br />

Assim, temos um segundo teorema:<br />

Teorema 2<br />

Sejam ⃗ A e ⃗ B dois vetores formando um ângulo θ entre<br />

eles. Então,<br />

⃗A · ⃗B = AB cos θ. (2.12)<br />

Note que este teorema nos permite calcular o produto<br />

escalar entre dois vetores sem a necessidade<br />

de escrevê-los em um determinado sistema de coordenadas,<br />

basta conhecermos seus comprimentos e<br />

o ângulo entre eles. Em geral, escreveremos nossos<br />

vetores em algum sistema de coordenadas (ortonormal,<br />

de preferência) e usaremos (2.12) para calcular<br />

o ângulo entre eles.<br />

Resumindo: além do comprimento, o produto escalar<br />

nos dá também uma informação sobre a orientação<br />

relativa entre vetores. Também é importante<br />

notar que a expressão (2.12) nos fornece uma<br />

forma operacional para calcularmos o produto escalar<br />

entre vetores quando seus comprimentos e o<br />

ângulo entre eles são conhecidos previamente.<br />

Voltemos ao nosso problema original: o sistema<br />

cartesiano da Figura 2.1. Nele, escolhemos os três<br />

versores î, ˆȷ e ˆk mutuamente ortogonais (perpendiculares),<br />

isto é, os ângulos entre estes versores é<br />

de 90 graus. Portanto, usando o Teorema 2, nossa<br />

métrica (2.7) pode ser escrita explicitamente,<br />

⎛<br />

î · î î ·ˆȷ î · ˆk ⎞ ⎛<br />

g = ⎝ˆȷ · î ˆȷ ·ˆȷ ˆȷ · ˆk ⎠ = ⎝ 1 0 0 ⎞<br />

0 1 0⎠ , (2.13)<br />

ˆk · î ˆk · ˆȷ ˆk · ˆk 0 0 1<br />

7


2.4. Produto vetorial Capítulo 2. Cinemática<br />

ou seja, ela é a matriz identidade. Isto simplifica<br />

muito nossa vida e explica a importância prática de<br />

usarmos sistemas ortonormais (versores ortogonais<br />

e unitários) de coordenadas. Assim, podemos escrever<br />

o produto escalar entre dois vetores arbitrários<br />

⃗A = (A x , A y , A z ) e ⃗ B = (B x , B y , B z ), usando a<br />

prescrição (2.8), em um sistema de coordenadas ortonormal<br />

cartesiano como (verifique)<br />

⃗A · ⃗B = ⃗ Ag ⃗ B T = A x B x + A y B y + A z B z . (2.14)<br />

Use (2.14) para calcular o comprimento do vetor<br />

posição ⃗r da Figura 2.1 e veja que é o mesmo valor<br />

obtido diretamente das projeções indicadas na<br />

mesma figura.<br />

Também é importante ter em mente que a expressão<br />

(2.14) é válida somente em um sistema ortonormal<br />

de coordenadas. Caso a base não seja<br />

ortonormal, devemos usar a métrica (2.7) em todas<br />

as operações envolvendo o produto escalar, como<br />

em (2.8). Também devemos ter em mente que a<br />

métrica vem junto com a base, ela é um conjunto de<br />

“especificações técnicas” sobre a base, uma espécie<br />

de “manual de instruções”. Se alguém lhe vender<br />

uma base sem a métrica, entre em contato com o<br />

Procon mais próximo.<br />

2.3.1 Exercícios<br />

Exercício 5<br />

Usando a propriedade distributiva (2.4) escreva<br />

explicitamente o produto escalar entre os vetores<br />

⃗r 1 = (x 1 , y 1 , z 1 ) e ⃗r 2 = (x 2 , y 2 , z 2 ). Verifique que<br />

este resultado é idêntico às operações matriciais do<br />

lado direito da Eq. (2.8).<br />

Exercício 6<br />

Efetue o produto escalar entre os vetores posição<br />

⃗R 1 = (1, 2, 1) e ⃗ R 2 = (1, 1, 2). Calcule também o<br />

comprimento de cada um deles bem como o ângulo<br />

entre eles. Repita este procedimento para os vetores<br />

resultantes da soma e da diferença entre ⃗ R 1 e<br />

⃗R 2 . Considere um sistema ortonormal de coordenadas.<br />

Exercício 7<br />

Escreva uma rotina em computação algébrica para<br />

calcular o comprimento de um vetor a partir de suas<br />

componentes em uma base ortonormal. Use uma<br />

estrutura do tipo lista (list) para estocar as coordenadas<br />

de um vetor. Escreva também uma rotina<br />

para calcular o ângulo em radianos e em graus entre<br />

dois vetores a partir de suas coordenadas. Verifique<br />

o funcionamento de suas rotinas comparando<br />

os resultados delas com os resultados do exercício<br />

anterior.<br />

Exercício 8<br />

Refaça as duas rotinas do exercício anterior assumindo<br />

que a base seja arbitrária, definida pela<br />

métrica (2.7). Use uma estrutura do tipo matriz<br />

(Matrix) para estocar a métrica.<br />

Exercício 9<br />

Suponha que uma determinada base tenha a seguinte<br />

métrica:<br />

⎛<br />

1 0.5<br />

⎞<br />

0<br />

g = ⎝0.5 1 0⎠ . (2.15)<br />

0 0 1<br />

Determine os ângulos entre os versores desta base<br />

e desenhe-a. Suponha que ⃗ R 1 = (1, 2, 1) e ⃗ R 2 =<br />

(1, 1, 2) sejam dois vetores escritos nesta base. Determine<br />

seus comprimentos e o produto escalar entre<br />

eles. Desenhe-os nesta base.<br />

2.4 Produto vetorial<br />

Há uma outra operação binária com vetores que<br />

é muito importante. Trata-se do produto vetorial.<br />

Neste tipo de operação, defini-se um novo produto<br />

entre os vetores ⃗ A e ⃗ B, denotado por ⃗ A × ⃗ B (ou<br />

⃗A ∧ ⃗ B), denominado de produto vetorial de ⃗ A por<br />

⃗B, do qual resulta um novo vetor. Lembre-se que o<br />

produto escalar produz um número (escalar) real.<br />

O produto vetorial é definido pelas seguintes propriedades:<br />

⃗A × ⃗ B = − ⃗ B × ⃗ A, (2.16)<br />

⃗C × (α ⃗ A + β ⃗ B) = α( ⃗ C × ⃗ A) + β( ⃗ C × ⃗ B), (2.17)<br />

⃗A × ⃗ B = ⃗ C ⇔ ⃗ C · ⃗A = ⃗ C · ⃗B = 0, (2.18)<br />

( ⃗ A, ⃗ B, ⃗ A × ⃗ B) : destrogiro. (2.19)<br />

A propriedade (2.16) significa que o produto vetorial,<br />

ao contrário do produto escalar, é antisimétrico<br />

(troca de sinal). A propriedade (2.17)<br />

significa que o produto vetorial, assim como o produto<br />

escalar, é linear. Note que os escalares α e<br />

β no lado direito de (2.17) podem ser retirados de<br />

8


Capítulo 2. Cinemática<br />

2.4. Produto vetorial<br />

dentro do produto vetorial. A propriedade (2.18)<br />

significa que o vetor resultante ⃗ A × ⃗ B é, simultaneamente,<br />

perpendicular aos vetores ⃗ A por ⃗ B. A<br />

propriedade (2.19) significa que o sentido do vetor<br />

resultante ⃗ C = ⃗ A× ⃗ B é indicado pelo nosso polegar<br />

direito quando movimentamos os demais dedos da<br />

mão direita no sentido de ⃗ A para ⃗ B (regra da mão<br />

direita; veja a Figura 2.3).<br />

⃗C<br />

θ<br />

⃗A<br />

Figura 2.3: O produto vetorial está associado com<br />

a área do paralelogramo formado pelos vetores ⃗ A e<br />

⃗B. O sentido do vetor resultante ⃗ C = ⃗ A× ⃗ B é dado<br />

pela regra da mão direita.<br />

Como podemos calcular as componentes do vetor<br />

resultante C ⃗ = A× ⃗ B ⃗ a partir das componentes dos<br />

vetores A ⃗ = (A x , A y , A z ) e B ⃗ = (B x , B y , B z )? Isto<br />

pode ser feito em duas etapas. Primeiro observe<br />

que a propriedade (2.18) nos fornece as seguintes<br />

relações (verifique):<br />

C x A x + C y A y + C z A z = 0, (2.20)<br />

C x B x + C y B y + C z B z = 0. (2.21)<br />

Note que estamos pressupondo que estes vetores estejam<br />

decompostos numa base ortonormal. Caso<br />

a base não seja ortonormal, devemos efetuar os<br />

produtos escalares usando a métrica apropriada.<br />

Das relações (2.20) e (2.21) podemos escrever, por<br />

exemplo, as componentes C x e C y em função de C z<br />

(verifique),<br />

⃗B<br />

C x = A yB z − A z B y<br />

A x B y − A y B x<br />

C z , (2.22)<br />

C y = A zB x − A x B z<br />

A x B y − A y B x<br />

C z . (2.23)<br />

Naturalmente, podemos re-escrevê-las na forma<br />

(verifique)<br />

C x<br />

A y B z − A z B y<br />

=<br />

C y<br />

A z B x − A x B z<br />

C z<br />

=<br />

= β. (2.24)<br />

A x B y − A y B x<br />

Estas razões devem ser válidas para quaisquer vetores<br />

A ⃗ e B. ⃗ Desta forma, temos as componentes<br />

do vetor C, ⃗ resultante do produto vetorial A ⃗ × B, ⃗<br />

em termos das componentes dos vetores A ⃗ e B ⃗ e da<br />

constante arbitrária β. Mas como esta constante β<br />

é arbitrária, então podemos escolher um valor para<br />

ela: β = 1. Não se assuste, como veremos adiante,<br />

há várias razões práticas para tal escolha. Com<br />

β = 1 em (2.24), temos:<br />

Teorema 3<br />

As componentes do vetor ⃗ C = ⃗ A × ⃗ B são<br />

C x = A y B z − A z B y ,<br />

C y = A z B x − A x B z ,<br />

C z = A x B y − A y B x .<br />

(2.25)<br />

Como veremos através de vários exemplos, escolher<br />

β como sendo um número positivo está condizente<br />

com a propriedade (2.19) (regra da mão<br />

direita). Caso tivéssemos escolhido um número<br />

negativo para β, teríamos que usar uma regra<br />

da mão esquerda. Note que a disposição dos<br />

índices x, y e z nas expressões dadas no Teorema<br />

3 segue uma ordem cíclica, com valores positivos<br />

no sentido horário, {(x, y, z), (z, x, y), (y, z, x)},<br />

e com valores negativos no sentido anti-horário,<br />

{(x, z, y), (y, x, z), (z, y, x)}. Veja a Figura (2.4)<br />

com (i, j, k) trocados por (x, y, z).<br />

Tendo as componentes (2.25), podemos calcular<br />

o módulo do vetor C ⃗ usando o produto escalar<br />

(2.6). Após um pouco de paciência (Faça o<br />

Exercício 10; use computação algébrica para checar<br />

os resultados), encontraremos<br />

C 2 = A 2 B 2 − ( ⃗ A · ⃗B) 2 , (2.26)<br />

a qual pode ser perfeitamente re-escrita, usando a<br />

propriedade (2.12), em termos do ângulo θ entre ⃗ A<br />

e ⃗ B,<br />

C 2 = A 2 B 2 − A 2 B 2 cos 2 θ = (AB) 2 sin 2 θ. (2.27)<br />

Portanto, temos:<br />

9


2.4. Produto vetorial Capítulo 2. Cinemática<br />

Teorema 4<br />

O módulo do vetor ⃗ C = ⃗ A × ⃗ B pode ser convenientemente<br />

calculado por<br />

î<br />

C = AB |sin θ|. (2.28)<br />

ˆk<br />

+<br />

ĵ<br />

+<br />

Assim, este Teorema 4 nos permite calcular o comprimento<br />

do vetor resultante de um produto vetorial<br />

a partir dos comprimentos dos vetores iniciais e<br />

do ângulo entre eles, sem a necessidade de escrevêlos<br />

em um sistema de coordenadas. Esta situação<br />

é análoga àquela relacionada com o Teorema 2.<br />

Vimos anteriormente que um produto escalar<br />

está associado com a projeção de um vetor sobre<br />

o outro. E o produto vetorial? ele tem alguma<br />

interpretação geométrica relevante? Sim, ele tem.<br />

Note que a expressão (2.28) é numericamente igual<br />

à área do paralelogramo formado pelos vetores ⃗ A<br />

e ⃗ B (veja a Figura 2.3 para se convencer disto e<br />

faça o Exercício 13). Esta propriedade será usada<br />

para derivarmos uma das leis de Kepler para o movimento<br />

planetário.<br />

Vejamos outras conseqüências do Teorema 4. A<br />

propriedade (2.28) significa que o produto vetorial<br />

entre dois vetores paralelos (ou anti-paralelos) resulta<br />

em um vetor nulo, pois neste caso o ângulo<br />

entre eles é 0 graus (ou 180 graus). Esta propriedade,<br />

juntamente com a regra da mão direita nos<br />

permite calcular todos os produtos vetoriais entre<br />

os versores î, ˆȷ e ˆk de uma base ortonormal:<br />

î = ˆȷ × ˆk, ˆȷ = ˆk × î, ˆk = î × ˆȷ. (2.29)<br />

Notas: (1) observando os produtos vetoriais (2.29),<br />

percebemos que a regra da mão direita é equivalente<br />

a efetuarmos permutações circulares nos versores<br />

î, ˆȷ e ˆk, tomando o sentido horário como positivo,<br />

como indicado na Figura (2.4); (2) podemos<br />

ver em (2.29) que a escolha β = 1 em (2.24) garante<br />

que o versor resultante î = ˆȷ × ˆk seja unitário; (3)<br />

usando (2.29) e a propriedade de linearidade (2.17),<br />

podemos calcular o produto vetorial entre dois vetores<br />

arbitrários escritos explicitamente em termos<br />

de uma base ortonormal (faça o Exercício 14).<br />

Figura 2.4: O sentido do produto vetorial é dado<br />

pela regra da mão direita. Esta regra é equivalente<br />

à execução de permutações circulares dos versores<br />

î, ˆȷ e ˆk: î ׈ȷ = ˆk, ˆȷ × ˆk = î e ˆk × î = ˆȷ.<br />

Há muitas outras propriedades importantes envolvendo<br />

simultaneamente produtos escalares e<br />

produtos vetoriais (numa base ortonormal) as quais<br />

serão estudadas no curso de Geometria Analítica.<br />

Entretanto, iremos precisar em breve de uma<br />

operação envolvendo três vetores. Nesta nova<br />

operação iremos usar um produto vetorial e um<br />

produto escalar. Por isto ela será denominada de<br />

produto misto. A sua definição é: dado três vetores<br />

⃗A, B ⃗ e C ⃗ quaisquer, o produto misto é um número<br />

definido por C ⃗ · ( A ⃗ × B) ⃗ (veja a Figura 2.5). Note<br />

que temos de executar primeiro o produto vetorial<br />

⃗A × B, ⃗ o qual resultará em um vetor, para depois<br />

calcularmos o produto escalar com C. ⃗ Que acontece<br />

se as posições dos três vetores no produto misto<br />

⃗C·( A× ⃗ B) ⃗ forem modificadas simultaneamente, por<br />

exemplo para B ⃗ · ( C ⃗ × A)? ⃗ Nada! O produto misto<br />

é invariante por permutações circulares das letras<br />

A, B e C,<br />

⃗A · ( ⃗ B × ⃗ C) = ⃗ C · ( ⃗ A × ⃗ B) = ⃗ B · ( ⃗ C × ⃗ A). (2.30)<br />

Portanto, do ponto de vista operacional, o produto<br />

misto está muito bem compreendido. E o significado<br />

geométrico do produto misto? Muito bem,<br />

você está aprendendo rápido as regras do nosso<br />

jogo. O produto misto é numericamente igual ao<br />

volume do paralelepípedo formado pelos três vetores<br />

A, ⃗ B ⃗ e C ⃗ conforme ilustrado pela Figura 2.5<br />

(estude a Figura 2.5 e faça o Exercício 16).<br />

Algumas observações importantes sobre produtos<br />

escalares e vetoriais a serem lembradas sempre:<br />

(1) o produto escalar é um número real e, geometricamente,<br />

está associado à projeção de um vetor sobre<br />

o outro; (2) o produto vetorial fornece um novo<br />

10


Capítulo 2. Cinemática<br />

vetor (na verdade, um pseudo-vetor) cuja norma<br />

(módulo) é numericamente igual à área do paralelogramo<br />

subentendido pelos dois vetores que foram<br />

usados no produto vetorial.<br />

Mencionamos na Seção 2.2 que um pseudo-vetor<br />

não inverte o seu sentido perante uma inversão espacial.<br />

Desta forma, pseudo-vetores podem ser produzidos<br />

através de produtos vetoriais, pois invertendo<br />

os sentidos de A ⃗ e B, ⃗ simultaneamente, o<br />

produto vetorial A ⃗ × B ⃗ não altera o seu sentido,<br />

isto é, C ⃗ = A ⃗ × B ⃗ é um pseudo-vetor. Em Física,<br />

temos vários pseudo-vetores importantes. Iremos<br />

trabalhar com dois deles em Mecânica: momentum<br />

angular L ⃗ = ⃗r × ⃗p e torque ⃗τ = ⃗r × F ⃗ , onde ⃗r é o<br />

vetor posição, ⃗p é o momentum linear (⃗p = m⃗v) e<br />

⃗F é a força resultante atuando no centro de massa<br />

de um objeto de massa m. Nas nossas aplicações<br />

iremos usar a força de Lorentz, F ⃗ L = (q/c)⃗v × B, ⃗<br />

produzida por um campo magnético B ⃗ sobre uma<br />

carga q em movimento com uma velocidade ⃗v, como<br />

exemplo de outro pseudo-vetor importante.<br />

2.4.1 Exercícios<br />

Exercício 10<br />

Use as componentes dadas em (2.25) para verificar<br />

(ou provar, demonstrar) que a expressão em (2.26)<br />

está correta.<br />

Exercício 11<br />

Efetue o produto vetorial entre os vetores posição<br />

⃗R 1 = (1, 2, 1) e R ⃗ 2 = (1, 1, 2). Calcule também<br />

o comprimento do vetor resultante deste produto<br />

vetorial usando (2.6) e (2.28). Calcule explicitamente<br />

o produto escalar deste vetor resultante com<br />

os vetores posição R ⃗ 1 e R ⃗ 2 . Considere um sistema<br />

ortonormal de coordenadas.<br />

Exercício 12<br />

Escreva uma rotina em computação algébrica para<br />

calcular as componentes do vetor resultante de um<br />

produto vetorial em termos das componentes dos<br />

vetores originais. Considere uma base ortonormal.<br />

Verifique o funcionamento de suas rotinas<br />

comparando-as com os resultados do exercício anterior.<br />

Exercício 13<br />

Calcule a área do paralelogramo formado pelos vetores<br />

⃗ A e ⃗ B mostrados na Figura 2.3 em termos dos<br />

comprimentos A e B e do ângulo θ entre ⃗ A e ⃗ B.<br />

2.4. Produto vetorial<br />

Exercício 14<br />

Efetue o produto vetorial entre ⃗ A = A x î+A yˆȷ+A zˆk<br />

e ⃗ B = B x î+B yˆȷ+B zˆk explicitamente, usando apenas<br />

a propriedade distributiva (2.17) e os produtos<br />

vetoriais (2.29). Mostre que este procedimento nos<br />

possibilita re-obter as expressões (2.25).<br />

Exercício 15<br />

Verifique que as componentes (2.25) também podem<br />

ser calculadas através do determinante<br />

∣<br />

⃗A × B ⃗ î ˆȷ ˆk ∣∣∣∣ =<br />

A<br />

∣ x A y A z . (2.31)<br />

B x B y B z<br />

Refaça o Exercício 11 usando este método para calcular<br />

o produto vetorial.<br />

Exercício 16<br />

Calcule o volume do paralelepípedo formado pelos<br />

vetores A, B e C da Figura 2.5 e mostre que este<br />

volume é numericamente igual ao produto misto<br />

⃗C · ( A ⃗ × B). ⃗ Portanto, o produto misto está relacionado<br />

com volume. Calcule o produto misto<br />

entre os vetores posição ⃗ A = (1, 2, 1), ⃗ B = (1, 1, 2)<br />

e ⃗ C = (1, 1, −1) mostrados na Figura 2.5.<br />

β<br />

⃗A× ⃗ B<br />

⃗C<br />

α<br />

⃗B<br />

Figura 2.5: O produto misto ⃗ C·( ⃗ A× ⃗ B) é numericamente<br />

igual ao volume do paralelepípedo tracejado<br />

formado pelos vetores A, B e C.<br />

⃗A<br />

11


2.5. Trajetórias Capítulo 2. Cinemática<br />

2.5 Trajetórias<br />

Tendo estabelecido propriedades importantes sobre<br />

o espaço Euclidiano, temos que precisar a noção<br />

de trajetória de um objeto em movimento neste<br />

espaço. Qual é a noção cotidiana de trajetória<br />

que temos? Eu a vejo como um seqüência de<br />

fotografias de um objeto em movimento, tiradas<br />

em intervalos de tempo muito pequenos, com as<br />

posições do objeto ligadas por retas. Se os intervalos<br />

de tempo são muito pequenos, a trajetória terá<br />

a aparência de uma curva suave em três dimensões.<br />

Isto é tudo que precisamos para estabelecer uma estrutura<br />

matemática geral para qualquer trajetória.<br />

Portanto, nosso problema agora é como representar<br />

uma curva no espaço Euclidiano de forma eficiente,<br />

i.e., tendo um sistema de coordenadas Cartesiano<br />

(de preferência ortonormal), temos de encontrar<br />

uma forma adequada (para a Mecânica) para<br />

representarmos analiticamente uma curva em termos<br />

de coordenadas.<br />

Vamos iniciar pelo começo: com uma reta. E<br />

bem no começo: com uma reta no plano XY . Estamos<br />

bem familiarizados com uma reta no plano?<br />

Sim, uma reta no plano XY é descrita pela equação<br />

y = a + bx, (2.32)<br />

introduzir um parâmetro real, o qual denotaremos<br />

por t, da seguinte forma. Primeiro, faça x = t.<br />

Então, de (2.32) devemos ter y = a + bt. Desta<br />

forma, para cada valor do parâmetro t no intervalo<br />

real [t 0 , t 1 ], teremos um ponto (x(t), y(t)) entre os<br />

pontos (x 0 , y 0 ) e (x 1 , y 1 ) do plano XY . Assim, a<br />

reta (2.32) pode ser re-escrita como<br />

x(t) = t, y(t) = a + bt, t ∈ [t 0 , t 1 ]. (2.34)<br />

Esta é a forma paramétrica de uma reta no plano<br />

XY . Por exemplo, a reta y = 2x entre os pontos<br />

(0, 0) e (2, 4) pode ser representada na forma paramétrica<br />

por x = t e y = 2t, na qual t ∈ [0, 2].<br />

Sim, de fato esta representação paramétrica parece<br />

ser desnecessária, pelo menos para uma reta.<br />

Entretanto, esta forma paramétrica é natural em<br />

Mecânica, pois em geral a posição de um objeto<br />

em movimento será uma função do tempo. Veja<br />

uma outra situação, ainda no plano, na qual a representação<br />

paramétrica é inerentemente de grande<br />

valia.<br />

y<br />

y 0<br />

R θ<br />

na qual b é conhecido como o coeficiente angular<br />

da reta. Ele é calculado conhecendo-se dois pontos<br />

quaisquer (x 0 , y 0 ) e (x 1 , y 1 ) da reta,<br />

b = y 1 − y 0<br />

x 1 − x 0<br />

. (2.33)<br />

Naturalmente, temos nenhuma dificuldade em representar<br />

graficamente uma reta no plano desde<br />

que as constantes a e b da equação (2.32) sejam<br />

dadas. Para cada valor de x, há um único valor de<br />

y. Desta forma, vários pontos (x, y) pertencentes<br />

à reta (2.32) podem ser encontrados rapidamente.<br />

Por ser uma relação direta entre as coordenadas x<br />

e y, a Eq. (2.32) é conhecida por forma direta da<br />

equação da reta em duas dimensões (mais detalhes<br />

serão vistos no curso de Geometria Analítica).<br />

Apesar de não haver dificuldades em calcular os<br />

pontos da reta (2.32), podemos interpretar este<br />

processo computacional de forma ligeiramente diferente,<br />

a qual será muito útil quando tivermos de<br />

lidar com curvas tridimensionais. Esta nova maneira<br />

de ver uma reta consiste simplesmente em<br />

Figura 2.6: Uma circunferência de raio R centrada<br />

em (x 0 , y 0 ).<br />

Vamos considerar agora uma curva fechada no<br />

plano XY . Algum palpite para uma curva fechada?<br />

Isto mesmo, uma circunferência como aquela mostrada<br />

na Figura 2.6. Conta-se que Giotto, considerado<br />

o mais importante pintor da pré-renascença,<br />

ao ser indagado sobre o que é simetria, desenhou<br />

uma circunferência perfeita (a mão-livre, naturalmente).<br />

A equação de uma circunferência de raio<br />

R, centrada no ponto (x 0 , y 0 ), é bem conhecida,<br />

x 0<br />

(x − x 0 ) 2 + (y − y 0 ) 2 = R 2 , (2.35)<br />

onde (x, y) são as coordenadas de um ponto sobre<br />

a circunferência. Como proceder agora para repre-<br />

x<br />

12


Capítulo 2. Cinemática<br />

2.5. Trajetórias<br />

sentar esta circunferência graficamente? Uma dificuldade<br />

em fazer este desenho surge por não dispormos<br />

no momento de uma prescrição única de<br />

como escrever os valores de x e y que irão satisfazer<br />

(2.35). Tentar isolar y em função de x,<br />

t 1<br />

t 0<br />

y = y 0 ± √ R 2 − (x − x 0 ) 2 , (2.36)<br />

⃗r 1<br />

⃗r 0<br />

não ajuda muito, pois não é qualquer valor de x que<br />

torna a raiz quadrada real em (2.36). A solução é<br />

representar a circunferência (2.35) numa forma paramétrica.<br />

Isto é feito como indicado na Figura 2.6<br />

com θ = ωt e t ∈ [0, 2π/ω],<br />

x − x 0 = R cos(ωt), y − y 0 = R sin(ωt). (2.37)<br />

Assim, para cada valor do parâmetro t no intervalo<br />

[0, 2π/ω] teremos uma circunferência completa.<br />

Esta é a forma mais eficiente de desenhar<br />

uma circunferência. Faça um exemplo.<br />

Sumariando: uma trajetória é representada matematicamente<br />

por uma curva γ no espaço Euclidiano<br />

(mais tópicos de Geometria Analítica). Em<br />

geral, a representação paramétrica<br />

Figura 2.7: Uma trajetória entre os instantes t 0 e<br />

t 1 representada como uma curva suave no espaço<br />

Euclidiano.<br />

Uma questão importante: como determinar<br />

a equação de uma reta tangente em um dado ponto<br />

da curva γ? Outra questão importante: como<br />

determinar o comprimento da trajetória entre os<br />

instantes t 0 e t 1 na Figura 2.7?<br />

Veja como estes problemas são resolvidos na<br />

seção seguinte. As soluções destes dois problemas<br />

nos fornece uma forma eficiente para calcularmos a<br />

velocidade (bem como sua aceleração) instantânea<br />

e o espaço percorrido por um objeto conhecendose<br />

a forma paramétrica de sua trajetória. As<br />

soluções destes problemas marcou o início (Newton,<br />

novamente) do Cálculo Diferencial e Integral<br />

no Século XVII.<br />

γ : t → ( x(t), y(t), z(t) ) , t ∈ [t 0 , t 1 ], (2.38)<br />

é a forma mais eficiente de lidarmos com tais curvas<br />

(veja a Figura 2.7). Além disto, a representação paramétrica<br />

é natural em Mecânica com o parâmetro<br />

t desempenhando o papel do tempo e com as coordenadas<br />

( x(t), y(t), z(t) ) de um ponto na curva<br />

identificadas com as coordenadas do vetor posição<br />

⃗r(t) = ( x(t), y(t), z(t) ) do objeto no instante t.<br />

Note que cada coordenada é também uma função<br />

do tempo quando a posição é escrita em termos de<br />

coordenadas. Assim, o tempo (mecânico) desempenha<br />

naturalmente o papel de um parâmetro na<br />

forma paramétrica de uma curva.<br />

2.5.1 Exercícios<br />

Exercício 17<br />

Use computação algébrica para desenhar as seguintes<br />

trajetórias: (1) circular (partícula carregada<br />

em um campo magnético uniforme perpendicular<br />

ao vetor velocidade), x = cos(2πt), y = sin(2πt),<br />

z = 0; (2) elíptica (um planeta e um satélite),<br />

x = 2 cos(2πt), y = 3 sin(2πt), z = 0; (3) helicoidal<br />

(partícula carregada em um campo magnético uniforme),<br />

x = cos(2πt), y = sin(2πt), z = t; (4) parabólico<br />

(massa na presença da gravidade), x = t,<br />

y = t, z = 20t − 5t 2 . Em cada caso, indique no<br />

mesmo gráfico o vetor posição em três instantes<br />

distintos: no início, no final e em algum instante<br />

intermediário.<br />

13


2.6. Velocidade e aceleração Capítulo 2. Cinemática<br />

2.6 Velocidade e aceleração<br />

O que são velocidade e aceleração? É a velocidade<br />

a taxa de variação do espaço percorrido no devido<br />

intervalo de tempo? ou é ela a taxa de variação<br />

temporal do vetor posição? No primeiro caso ela<br />

é um escalar enquanto que no segundo caso ela é<br />

um vetor. Em ambos os casos a velocidade tem<br />

dimensão de comprimento por tempo. Veremos<br />

que estas duas definições estão corretas e intimamente<br />

relacionadas entre si. No entanto, por ser<br />

mais completa, vamos iniciar nossa discussão com<br />

a definição vetorial para a velocidade ⃗v de um objeto<br />

entre os instantes t 0 e t 1 , como mostrado na<br />

Figura 2.7,<br />

⃗v = ⃗r 1 − ⃗r 0<br />

t 1 − t<br />

( 0<br />

x1 − x 0<br />

= , y 1 − y 0<br />

, z )<br />

1 − z 0<br />

.<br />

t 1 − t 0 t 1 − t 0 t 1 − t 0<br />

(2.39)<br />

Agora observe a Figura 2.7 e compare o comprimento<br />

do vetor diferença ∆⃗r = ⃗r 1 − ⃗r 0 com o comprimento<br />

real da trajetória percorrida entre os instantes<br />

t 0 e t 1 , o qual podemos chamar de ∆S. Estes<br />

comprimento são iguais em geral? Qual é a única<br />

situação em que eles são exatamente os mesmos?<br />

Sim, numa trajetória retilínea. Estas observações<br />

nos revela que ∆S > ||∆⃗r||, tornando assim a definição<br />

escalar incompatível com a definição vetorial<br />

para a velocidade.<br />

Como podemos proceder para compatibilizar as<br />

duas definições de velocidade? A resposta está nas<br />

mesmas observações anteriores. A única maneira de<br />

compatibilização é fazermos com que a definição vetorial<br />

forneça como seu módulo a definição escalar.<br />

Isto ocorrerá somente quando fixarmos o instante<br />

t 0 e permitirmos que o instante t 1 se aproxime indefinidamente<br />

de t 0 . Quando ∆⃗r = ⃗r 1 − ⃗r 0 for muito<br />

pequeno, devido ao vetor posição ⃗r 1 estar muito<br />

próximo do vetor posição inicial ⃗r 0 , certamente o<br />

comprimento de ∆⃗r será confundido com o comprimento<br />

da trajetória. Portanto, a velocidade no<br />

instante t 0 deve ser definida através de um processo<br />

limite (um tópico do curso de Cálculo),<br />

⃗r 1 − ⃗r 0 ∆⃗r<br />

⃗v(t 0 ) = lim = lim ∣<br />

t 1 →t 0 t 1 − t 0 ∆t→0 ∆t<br />

⃗r(t + ∆t) − ⃗r(t)<br />

= lim<br />

∆t→0 ∆t<br />

∣<br />

t=t0<br />

∣ ,<br />

t=t0<br />

(2.40)<br />

na qual estamos escrevendo também t 0 = t e<br />

t 1 = t 0 + ∆t. Qual é o significado geométrico da<br />

expressão (2.40)? Como podemos torná-la operacional?,<br />

isto é, dada a dependência temporal das coordenadas<br />

do vetor posição descrevendo a trajetória<br />

de um determinado movimento, como podemos determinar<br />

as respectivas componentes do vetor velocidade<br />

em qualquer instante de tempo? Ainda relacionada<br />

com a questão operacional: o limite (2.40)<br />

realmente existe? Esta pergunta é pertinente pois,<br />

se ∆t tende a zero a trajetória também tende a zero<br />

(repouso). Assim, quem vai a zero primeiro, o numerador<br />

∆⃗r ou o denominador ∆t? Numerador e<br />

denominador ambos indo a zero resultará em uma<br />

razão mensurável?<br />

Não devemos esquecer que há três limites a serem<br />

calculados em (2.40), pois a velocidade é um vetor.<br />

Entretanto, basta usarmos uma componente, digamos<br />

a componente X, para exemplificarmos como<br />

aqueles limites devem ser calculados. Também não<br />

precisamos substituir t = t 0 imediatamente após o<br />

limite ter sido calculado. Assim, temos que entender<br />

o limite<br />

d<br />

x(t + ∆t) − x(t)<br />

x(t) = ẋ(t) = lim<br />

dt ∆t→0 ∆t<br />

(2.41)<br />

do ponto de vista geométrico e torná-lo operacional.<br />

Note que estamos usando uma notação especial<br />

para representar este limite, denominada de<br />

derivada de x(t) em relação a t. Importante: o<br />

símbolo d/dt deve ser entendido como um símbolo<br />

único, contendo o argumento da função sendo derivada<br />

no denominador. Caso tivéssemos que derivar<br />

a função f(x), escreveríamos d/dx. Vamos tornar<br />

a derivada operacional através de alguns exemplos.<br />

Suponha que a equação horária no eixo X seja<br />

uma constante, x(t) = a, isto é, repouso. Então,<br />

levando a função constante x(t) = a em (2.41), teremos<br />

d<br />

x(t + ∆t) − x(t)<br />

x(t) = lim<br />

dt ∆t→0 ∆t<br />

= lim<br />

∆t→0<br />

a − a<br />

∆t<br />

= lim<br />

∆t→0<br />

(2.42)<br />

0<br />

∆t = 0,<br />

pois o numerador já era nulo antes de executarmos<br />

o limite. Acabamos de aprender que a derivada<br />

de uma função constante é nula. Do ponto<br />

de vista geométrico, devemos perceber que uma<br />

função constante é uma reta paralela ao eixo X.<br />

14


Capítulo 2. Cinemática<br />

2.6. Velocidade e aceleração<br />

Assim, qualquer função constante tem uma inclinação<br />

(ângulo formado com o eixo X) nula, cuja<br />

tangente (coeficiente angular) também é nula. Portanto,<br />

o coeficiente angular de uma reta paralela ao<br />

eixo X, x(t) = a, é numericamente igual à sua derivada<br />

em qualquer ponto, ou seja, nulo. Vejamos<br />

se esta relação entre derivada e tangente é mantida<br />

em um outro exemplo.<br />

Considere agora uma equação horária linear no<br />

tempo, x(t) = a + bt, correspondendo a um movimento<br />

uniforme. Então, levando a função x(t) =<br />

a + bt em (2.41), teremos<br />

d<br />

x(t + ∆t) − x(t)<br />

x(t) = lim<br />

dt ∆t→0 ∆t<br />

{a + b(t + ∆t)} − {a + bt} (2.43)<br />

∆t<br />

b = b,<br />

∆t→0<br />

= lim<br />

∆t→0<br />

= lim<br />

pois o numerador já era igual a b antes de executarmos<br />

o limite. Portanto, aprendemos que a<br />

derivada de uma função linear é igual ao seu coeficiente<br />

angular, confirmando assim nossa conjectura<br />

que a derivada calculada em um ponto t é numericamente<br />

igual ao coeficiente angular da reta<br />

tangente à função x(t) (no mesmo ponto t). Note<br />

que a reta tangente de uma reta coincide com a<br />

própria reta. Para confirmar esta conjectura sobre<br />

a interpretação geométrica da derivada, vejamos o<br />

próximo exemplo.<br />

Considere agora uma função quadrática para a<br />

equação horária, x(t) = a+bt+ct 2 , correspondendo<br />

a um movimento com aceleração constante. Então,<br />

levando esta função em (2.41), teremos<br />

d<br />

x(t + ∆t) − x(t)<br />

x(t) = lim<br />

dt ∆t→0<br />

(<br />

∆t<br />

)<br />

= lim b + 2ct + c∆t (2.44)<br />

∆t→0<br />

= b + 2ct + lim c∆t = b + 2ct,<br />

∆t→0<br />

pois o limite do termo c∆t é obtido substituindo<br />

∆t = 0. Uma regra para calcular limites: simplifique<br />

antes suas expressões. Até aqui aprendemos<br />

que a derivada de um polinômio t n parece obedecer<br />

à regra nt n−1 . Também nos parece que a derivada<br />

do produto de uma função f(t) por uma constante<br />

c obedece à regra c df(t)/dt, isto é, a constante<br />

pode sair para fora da derivada. Ao compararmos<br />

o resultado em (2.43) com o resultado<br />

(2.44) podemos conjecturar que a derivada obedece<br />

a propriedade de linearidade, d(f(t) + cg(t))/dt =<br />

df(t)/dt+c dg(t)/dt. De fato, estas conjecturas podem<br />

ser provadas (no curso de Cálculo). Em geral,<br />

as propriedades seguintes nos permitem calcular a<br />

derivada de qualquer função suave.<br />

Derivada de uma constante:<br />

Linearidade:<br />

d<br />

a = 0, (2.45)<br />

dt<br />

d ( ) d<br />

f(t) + bg(t) =<br />

dt<br />

dt f(t) + b d g(t), (2.46)<br />

dt<br />

Regra do produto:<br />

d ( ) d<br />

f(t)g(t) = g(t)<br />

dt<br />

dt f(t)<br />

Regra da função composta:<br />

d<br />

dt f( g(t) ) =<br />

+ f(t) d g(t), (2.47)<br />

dt<br />

[ ] d d<br />

dg f(g) g(t). (2.48)<br />

dt<br />

Em particular, usaremos com freqüência derivadas<br />

de funções elementares:<br />

d<br />

dt tn = nt n−1 ,<br />

d<br />

dt et = e t ,<br />

d<br />

d<br />

sin t = cos t,<br />

dt<br />

d<br />

ln t = 1/t,<br />

(2.49)<br />

dt<br />

dt cos t = − sin t.<br />

Estas propriedades e resultados serão demonstrados<br />

no curso de Cálculo. Até lá, use-as e memorizeas.<br />

Por exemplo, suponha y = sin(2t 2 ). Esta é uma<br />

função composta na forma y = f(g(t)), na qual<br />

f(g) = sin(g) e g(t) = 2t 2 . Assim, devemos usar a<br />

regra da função composta, (2.48), para efetuar sua<br />

derivada,<br />

ẏ = dg<br />

dt<br />

df<br />

dg = (4t) cos(g) = 4t cos(2t2 ). (2.50)<br />

Vejamos este outro exemplo: y = cos(2t)e t2 .<br />

Desta vez temos um produto de duas funções (um<br />

15


2.6. Velocidade e aceleração Capítulo 2. Cinemática<br />

cosseno vezes uma exponencial), na qual cada parcela<br />

é uma função composta. Assim, devemos usar<br />

primeiro a regra do produto, (2.47), e depois a regra<br />

da função composta, (2.48),<br />

{ }<br />

{ }<br />

d d<br />

ẏ =<br />

dt cos(2t) e t2 + cos(2t)<br />

dt et2<br />

= −2 sin(2t) e t2 + cos(2t)2t e t2<br />

= 2 [ t cos(2t) − sin(2t) ] e t2 .<br />

(2.51)<br />

Invente outros exemplos. Use computação<br />

algébrica para checar seus resultados. Pratique a<br />

vontade. Faça a primeira parte do Exercício 18.<br />

E sobre a interpretação geométrica de derivada?<br />

Para fazermos esta interpretação corretamente, devemos<br />

resolver um outro problema (geométrico).<br />

Como determinar a equação da reta tangente em<br />

um dado ponto t 0 de uma dada curva x(t)? Veja<br />

a Figura 2.8. A única informação que temos é<br />

que esta reta tangente deve passar pelo ponto<br />

(t 0 , x(t 0 )), e somente por este ponto numa vizinhança<br />

muito pequena em torno de t 0 . No entanto,<br />

sabemos que precisaremos conhecer também o coeficiente<br />

angular desta reta tangente e que para isto<br />

precisaremos de um segundo ponto. Isto mesmo,<br />

o problema é que não temos este segundo ponto.<br />

Que fazer? A única atitude sensata é usar um outro<br />

ponto, digamos t 1 , da curva x(t), como indicado<br />

na Figura 2.8.<br />

x 1<br />

x<br />

x(t)<br />

De fato, esta reta secante não é a mesma reta tangente<br />

que estamos procurando, mas se mantivermos<br />

t 0 fixo e aproximarmos t 1 de t 0 , obteremos o coeficiente<br />

angular da reta tangente que procuramos,<br />

tan θ 0 = lim<br />

t 1→t 0<br />

x 1 − x 0<br />

t 1 − t 0<br />

. (2.53)<br />

Assim, como este processo limite é o mesmo processo<br />

limite usado para definir a velocidade instantânea<br />

(2.40), podemos concluir que a derivada<br />

nos dá informação sobre retas tangentes de curvas:<br />

a derivada de uma função qualquer f(t) é numericamente<br />

igual ao coeficiente angular da reta tangente<br />

passando por (t, f(t)). De fato, isto é uma generalização<br />

do problema matemático de encontrar a<br />

reta tangente a uma curva plana (veja a Figura 2.8).<br />

Uma aplicação imediata desta interpretação<br />

geométrica: ela é muito útil para determinarmos os<br />

pontos extremos (máximos e mínimos) de funções,<br />

pois, nestes pontos de máximos e mínimos, a reta<br />

tangente é sempre horizontal (logo o seu coeficiente<br />

angular é nulo). Portanto, a derivada deve ser nula<br />

nestes pontos extremos (faça o Exercício 19).<br />

Esta interpretação geométrica de derivada é<br />

muito importante também para a Mecânica, pois<br />

ela afirma que qualquer vetor velocidade<br />

θ 0<br />

θ 1<br />

x 0<br />

t 0 t 1<br />

t<br />

⃗v(t) = d dt ⃗r(t)<br />

= d dt x(t) î + d dt y(t)ˆȷ + d z(t) ˆk<br />

dt<br />

(2.54)<br />

Figura 2.8: A secante de uma trajetória entre os<br />

instantes t 0 e t 1 e a sua reta tangente em t 0 .<br />

O coeficiente angular da reta secante passando<br />

pelos pontos x 0 = x(t 0 ) e x 1 = x(t 1 ) é<br />

tan θ 1 = x 1 − x 0<br />

t 1 − t 0<br />

. (2.52)<br />

será sempre tangente à trajetória definida pelo vetor<br />

posição ⃗r(t). Então devemos repetir os passos<br />

anteriores e mostrar que o vetor velocidade (2.54)<br />

está sobre a reta tangente à trajetória definida pelo<br />

vetor posição ⃗r(t).<br />

16


Capítulo 2. Cinemática<br />

2.6. Velocidade e aceleração<br />

1<br />

0.8<br />

0.6<br />

0.4<br />

0.2<br />

z<br />

0 0<br />

1<br />

0.8<br />

0.6<br />

0.4<br />

0.2<br />

0<br />

z<br />

0<br />

0.2<br />

x<br />

0.2<br />

y<br />

0.4<br />

0.4<br />

0.6<br />

0.6<br />

0.8<br />

0.8<br />

1 0<br />

0.2<br />

0.8<br />

0.4<br />

0.6<br />

0.6<br />

y<br />

0.8<br />

0.2<br />

0.4<br />

x<br />

Figura 2.9: As secantes da trajetória ⃗r(t) =<br />

(t 2 , t 3 , t) coincidirão com o vetor velocidade em<br />

t = 0.25 s.<br />

A Figura 2.9 mostra a trajetória definida pelo<br />

vetor posição ⃗r(t) = (t 2 , t 3 , t) entre os instantes<br />

t = 0 s e t = 1 s. Esta figura também mostra<br />

o vetor velocidade em t 0 = 0.25 s (calcule este vetor).<br />

Esta mesma figura também mostra duas retas<br />

secantes passando por t 0 = 0.25 s (mantido fixo),<br />

t 1 = 0.5 s e t 1 = 0.75 s. Como no caso bidimensional,<br />

à medida que t 1 se aproxima de t 0 , a secante<br />

se aproxima do vetor velocidade. Note que a trajetória<br />

da Figura 2.9 não é uma reta e nem é plana.<br />

Como podemos mostrar matematicamente o que<br />

estamos vendo na Figura 2.9? Estamos vendo na<br />

Figura 2.9 que o vetor velocidade se aproxima da<br />

reta tangente em t 0 à medida que t 1 se aproxima<br />

de t 0 . Comecemos com a equação da reta em três<br />

dimensões na forma paramétrica (mais Geometria<br />

Analítica):<br />

¯x(t) = b 1 t + c 1 ,<br />

ȳ(t) = b 2 t + c 2 ,<br />

¯z(t) = b 3 t + c 3 ,<br />

1<br />

0<br />

(2.55)<br />

na qual ¯x, ȳ e ¯z representam as coordenadas de uma<br />

reta arbitrária. Os três números b i são os coeficientes<br />

angulares desta reta. Note que as funções do<br />

parâmetro t em (2.55) são lineares. Cada ponto na<br />

trajetória tem coordenadas ⃗r(t) = (x(t), y(t), z(t)).<br />

O nosso problema agora é: dado um ponto na trajetória,<br />

digamos em t = t 0 , qual é a reta tangente<br />

(em três dimensões) passando por este ponto?<br />

Qualquer reta, mesmo em três dimensões, é determinada<br />

por dois pontos. Um ponto foi dado em t 0 ,<br />

(x 0 , y 0 , z 0 ). Como no caso bidimensional, vamos<br />

escolher (note bem, estamos fazendo uma escolha)<br />

um segundo ponto também sobre a trajetória em<br />

t = t 1 , com t 1 > t 0 , (x 1 , y 1 , z 1 ). Levando estes dois<br />

pontos nas equações (2.55), podemos determinar os<br />

coeficientes angulares (faça os detalhes):<br />

b 1 = x 1 − x 0<br />

t 1 − t 0<br />

,<br />

b 2 = y 1 − y 0<br />

t 1 − t 0<br />

,<br />

b 3 = z 1 − z 0<br />

t 1 − t 0<br />

.<br />

(2.56)<br />

Estes são os coeficientes angulares de uma reta secante<br />

passando pelos pontos da trajetória determinados<br />

por t = t 0 e t = t 1 . Matendo t 0 fixo e permitindo<br />

que t 1 se aproxime indefinidamente de t 0 ,<br />

encontraremos a reta tangente que estamos procurando.<br />

Note que este processo envolve três limites<br />

idênticos ao limite (2.41) que usamos para definir<br />

o operador derivada. Portanto, os coeficientes angulares<br />

da reta tangente que estamos procurando<br />

podem ser calculados por derivadas,<br />

b 1 = d dt x(t) ∣<br />

∣∣∣t=t0<br />

,<br />

b 2 = d dt y(t) ∣<br />

∣∣∣t=t0<br />

,<br />

b 3 = d dt z(t) ∣<br />

∣∣∣t=t0<br />

.<br />

(2.57)<br />

Os coeficientes angulares (2.57) são da reta tangente<br />

à trajetória ⃗r(t) = (x(t), y(t), z(t)) no ponto<br />

t = t 0 . Em qualquer instante de tempo, o vetor<br />

velocidade desta trajetória arbitrária é ⃗v(t) =<br />

(ẋ(t), ẏ(t), ż(t)), isto é, a derivada do vetor posição.<br />

Podemos observar agora que os coeficientes angulares<br />

(2.57) são exatamente iguais às componentes<br />

do vetor velocidade avaliado em t = t 0 , mostrando<br />

assim que o vetor velocidade está de fato sobre a<br />

reta tangente em t = t 0 , mesmo em três dimensões.<br />

17


2.6. Velocidade e aceleração Capítulo 2. Cinemática<br />

De forma análoga à definição de velocidade, a<br />

aceleração (instantânea) é a taxa de variação temporal<br />

do vetor velocidade,<br />

⃗a(t) = d dt<br />

d2<br />

⃗v(t) = ⃗r(t), (2.58)<br />

dt2 ou, equivalentemente, a derivada segunda do vetor<br />

posição. Note como a derivada segunda é indicada<br />

em (2.58). A taxa de variação temporal do<br />

vetor aceleração não tem um nome específico, pois<br />

é apenas a aceleração que aparece explicitamente<br />

na segunda lei de Newton, a qual estabelece uma<br />

relação entre cinemática e dinâmica (forças), formando<br />

assim a mecânica. Faça a segunda parte do<br />

Exercício 18.<br />

Questões são sempre bem vindas. Qual é a interpretação<br />

geométrica da aceleração? Vamos tentar<br />

entender esta pergunta estudando alguns exemplos.<br />

Primeiro, consideremos uma trajetória retilínea:<br />

⃗r = (t, 1+t, 0). Os vetores velocidade e aceleração<br />

são ⃗v = (1, 1, 0) e ⃗a = (0, 0, 0), respectivamente.<br />

Obviamente, o produto vetorial ⃗v×⃗a é nulo.<br />

Vejamos se este mesmo produto vetorial é nulo para<br />

uma trajetória que não é retilínea. Consideremos<br />

a parábola ⃗r = (t, 1 + t 2 , 0) com ⃗v = (1, 2t, 0) e<br />

⃗a = (0, 2, 0). Desta vez, o produto vetorial ⃗v × ⃗a<br />

não é nulo, ⃗v × ⃗a = (0, 0, 2) (faça a terceira parte<br />

do Exercício 18). A quantidade escalar<br />

κ =<br />

||⃗v × ⃗a||<br />

v 3 , v = ||⃗v||, (2.59)<br />

denominada de curvatura, com dimensão de inverso<br />

de comprimento, é nula para uma reta (qualquer)<br />

e vale κ = 2/ √ 1 + 4t 2 para a parábola ⃗r =<br />

(t, 1+t 2 , 0). Note que uma parábola se aproxima de<br />

retas (assímptotas) para valores muito grandes de t<br />

(t ≫ 1) e que a curvatura κ = 2/ √ 1 + 4t 2 é nula no<br />

limite t → ∞. Note também que κ(0) = 2. Faça o<br />

Exercício 21 para perceber o significado geométrico<br />

da curvatura: a parábola ⃗r = (t, 1 + t 2 , 0) se aproxima<br />

muito de uma circunferência de raio 1/κ(0),<br />

centrada em (0, 3/2), na região em torno de t = 0.<br />

Assim, a curvatura em t = 0 está nos informando<br />

o quanto a parábola se desvia de uma reta.<br />

A curvatura tem uma interpretação bastante<br />

simples e direta em uma trajetória circular. Em<br />

geral,<br />

⃗r = (R cos ωt, R sin ωt, 0) (2.60)<br />

representa uma circunferência de raio R, centrada<br />

na origem. Conseqüentemente (faça o<br />

Exercício 22),<br />

⃗v = (−ωR sin ωt, ωR cos ωt, 0), (2.61)<br />

⃗a = (−ω 2 R cos ωt, −ω 2 R sin ωt, 0). (2.62)<br />

Assim, recuperamos três informações importantes<br />

sobre um movimento circular qualquer: (i) a aceleração<br />

em um movimento circular é voltada para o<br />

centro (centrípeta), pois ⃗a = −ω 2 ⃗r; (ii) o vetor velocidade<br />

é sempre perpendicular ao vetor posição<br />

⃗r · ⃗v = 0; e (iii) os módulos dos vetores posição<br />

(R), velocidade (Rω) e aceleração (v 2 /R) permanecem<br />

constantes durante o movimento (apenas<br />

suas direções e sentidos é que variam no tempo).<br />

A informação (ii) é equivalente à interpretação<br />

geométrica do vetor velocidade como sendo tangente<br />

à trajetória. A curvatura (2.59) neste caso<br />

é (faça o Exercício 22)<br />

κ = 1 R . (2.63)<br />

Note que a curvatura é constante e seu inverso é<br />

igual ao raio da trajetória circular. Desta forma,<br />

podemos afirmar que a curvatura é um escalar que<br />

mede o quanto uma curva desvia-se de uma reta, a<br />

qual possui uma curvatura nula (raio infinito). Há<br />

uma outra maneira de ver a relação entre curvatura<br />

e aceleração ainda mais frutífera (veja as discussões<br />

subsequentes).<br />

Seja v o módulo do vetor velocidade e ˆv o seu<br />

versor, ⃗v = vˆv. Note que o versor ˆv é sempre tangente<br />

à trajetória ⃗r, pois ⃗v = ˙⃗r (derivada do vetor<br />

posição). Por isso, ˆv também é conhecido por versor<br />

tangente da curva ⃗r. Então, o vetor aceleração<br />

correspondente, calculado pela derivada do vetor<br />

velocidade, é<br />

⃗a = ˙⃗v = d dt (vˆv) = ˙vˆv + v ˙ˆv. (2.64)<br />

Embora não esteja indicado, mas todas as quantidades<br />

aqui dependem explicitamente do tempo.<br />

Podemos ver em (2.64) que a aceleração possui<br />

uma componente tangencial (na direção do versor<br />

tangente) igual a ˙v. Em que direção está a derivada<br />

˙ˆv do versor tangente? Podemos encontrar<br />

esta direção facilmente se usarmos o fato de que ˆv<br />

é um versor, ˆv · ˆv = 1, pois derivando os dois lados<br />

desta igualdade, temos<br />

d<br />

dt (ˆv · ˆv) = 2ˆv · ˙ˆv = 0. (2.65)<br />

18


Capítulo 2. Cinemática<br />

2.6. Velocidade e aceleração<br />

Este resultado nos mostra que ˙ˆv é perpendicular a<br />

ˆv. Note que este resultado vale para qualquer versor<br />

(dependente do tempo), ou para qualquer vetor<br />

cujo módulo não varia com o tempo. É comum denominarmos<br />

de normal a direção perpendicular ao<br />

versor tangente. O versor ˆn na direção normal é conhecido<br />

por versor normal. Como o versor normal<br />

ˆn e a derivada ˙ˆv são paralelos, podemos esolher<br />

˙ˆv = vκˆn, (2.66)<br />

onde κ é um escalar, em geral dependente do<br />

tempo, denominado de curvatura, o mesmo escalar<br />

dado em (2.59). Portanto, a aceleração (2.64)<br />

pode ser re-escrita como<br />

⃗a = ˙⃗v = ˙v ˆv + v 2 κ ˆn. (2.67)<br />

Fizemos acima a escolha (2.66). Vejamos como esta<br />

escolha é compatibilizada com a expressão (2.59)<br />

para a curvatura. Multiplique vetorialmente os dois<br />

lados da aceleração (2.67) por ⃗v. Como o produto<br />

vetorial entre vetores paralelos é nulo, então<br />

⃗v × ⃗a = ˙v ⃗v × ˆv + v 2 κ ⃗v × ˆn = v 3 κ ˆb, (2.68)<br />

onde fizemos ˆb = ˆv × ˆn, (versor binormal). Assim,<br />

calculando o módulo dos dois lados, podemos isolar<br />

a curvatura para re-obter a expressão (2.59).<br />

Como vimos anteriormente, a curvatura de uma<br />

circunferência é o inverso de seu raio (κ = 1/R).<br />

Lembrando que o módulo do vetor velocidade não<br />

muda em um movimento circular, então ˙v = 0.<br />

Desta forma, para um movimento circular, a aceleração<br />

(2.67) reduz-se à aceleração centrípeta ⃗a =<br />

−(v 2 /R)ˆr, com ˆn = −ˆr.<br />

Curvas também podem ser classificadas como<br />

pertencentes a um plano, como uma circunferência<br />

ou uma parábola, para citar dois exemplos conhecidos,<br />

ou não pertencentes a um plano, como uma<br />

hélice, outro exemplo bem conhecido. Considere<br />

novamente uma trajetória circular. Nela, o vetor<br />

velocidade (tangencial) é sempre perpendicular ao<br />

vetor aceleração (centrípeta), ambos contidos no<br />

plano da trajetória circular. Desta forma, o produto<br />

vetorial entre os versores tangencial (velocidade)<br />

e normal (aceleração centrípeta) do movimento<br />

circular é sempre perpendicular ao plano da<br />

trajetória, portanto nunca muda de direção. Sendo<br />

um pouco mais preciso, é a taxa de variação do<br />

versor binormal ˆb = ˆv × ˆn que mede o quanto uma<br />

determinada curva afasta-se de um plano. Elaborando<br />

um pouco mais este exemplo, podemos construir<br />

uma ferramenta para medir o quanto uma<br />

curva se afasta de um plano, ou seja, para medir<br />

o grau de torcimento de uma curva espacial.<br />

Antes de mais nada, é importante notar que os<br />

três versores ˆv, ˆn e ˆb, com ˆb = ˆv × ˆn, são ortonormais<br />

e obedecem a regra da mão-direita, assim<br />

como os versores î, ĵ e ˆk de um sistema ortonormal<br />

de coordenadas. Os três versores {ˆv, ˆn, ˆb} são<br />

conhecidos por tríade de Frenet. Note que estes versores<br />

são funções (vetoriais) do tempo, ou seja, ao<br />

contrário dos versores {î, ĵ, ˆk} que estão fixos, a<br />

tríade de Frenet é móvel. Note também que os<br />

versores da tríade de Frenet são linearmente independentes.<br />

Sendo linearmente independentes, podemos<br />

escrever a primeira derivada de qualquer um<br />

deles como uma combinação linear deles mesmos,<br />

como fizemos em (2.66). Como eles são versores,<br />

então podemos usar o que aprendemos em (2.65),<br />

isto é, ˙ˆn · ˆn = 0 (perpendiculares). Assim, ˙ˆn deve<br />

ser uma combinação linear da forma ˙ˆn = αˆv + βˆb,<br />

com α e β sendo dois números reais. Multiplicando<br />

escalarmente esta combinação linear por ˆv, obteremos<br />

α = ˙ˆn · ˆv = −ˆn · ˙ˆv = −vκ, onde usamos<br />

também a derivada (2.66). De forma análoga, multiplicando<br />

˙ˆn = αˆv + βˆb escalarmente por ˆb, obteremos<br />

β = ˙ˆn · ˆb = −ˆn · ˙ˆb. Infelizmente ainda não<br />

decompusemos a derivada ˙ˆb em termos da tríade de<br />

Frenet.<br />

Procedendo de forma similar ao parágrafo anterior,<br />

sabemos que ˙ˆb · ˆb = 0. Assim, devemos ter<br />

˙ˆb = ᾱˆv+ ¯βˆn. Multiplicando escalarmente esta combinação<br />

linear por ˆv obteremos ᾱ = ˙ˆb · ˆv = −ˆb · ˙ˆv =<br />

−vκˆb · ˆn = 0, onde usamos também a derivada<br />

(2.66). Portanto, ˙ˆb = ¯βˆn. Levando esta informação<br />

em β = −ˆn· ˙ˆb, obtida no final do parágrafo anterior,<br />

teremos que β = − ¯β. Desta forma, temos que fazer<br />

uma escolha para ¯β. A mais utilizada é ¯β = −vτ,<br />

˙ˆb = −vτ ˆn, (2.69)<br />

onde o número τ, geralmente uma função do tempo,<br />

é conhecido por torção da curva ⃗r. Tendo escolhido<br />

o valor de ¯β, a primeira derivada ˙ˆn torna-se em<br />

˙ˆn = −vκ ˆv + vτ ˆb. (2.70)<br />

As equações (diferenciais) (2.66), (2.69) e (2.70) são<br />

19


2.7. Espaço percorrido Capítulo 2. Cinemática<br />

conhecidas por equações de Frenet. Elas determinam<br />

univocamente uma curva a partir do conhecimento<br />

de apenas duas funções escalares: a curvatura<br />

κ = κ(t) e da torção τ = τ(t).<br />

Podemos deduzir uma expressão bastante conveniente<br />

para calcular a torção de forma análoga<br />

à expressão (2.59), usada para calcular a curvatura.<br />

Para isto, precisamos da derivada do vetor<br />

aceleração,<br />

˙⃗a = (¨v − κ 2 v 3 )ˆv + [ κv ˙v + d dt (κv2 ) ]ˆn<br />

+ κτv 3ˆb. (2.71)<br />

Multiplicando escalarmente esta expressão por ⃗v ×<br />

⃗a = v 3 κ ˆb, obtida em (2.68), teremos<br />

τ =<br />

⃗v × ⃗a · ˙⃗a<br />

v 6 κ 2 , (2.72)<br />

ou, numa forma mais simétrica,<br />

⃗v × ⃗a · ˙⃗a<br />

τ =<br />

||⃗v × ⃗a|| 2 = ˙⃗r × ¨⃗r · ...<br />

⃗r<br />

|| ˙⃗r<br />

. (2.73)<br />

× ¨⃗r||<br />

2<br />

Faça o Exercício 23 para ver que a torção de curvas<br />

planas (retas, parábolas, circunferências, etc.)<br />

é nula, como deveríamos esperar. No entanto, note<br />

que a torção de uma curva como uma hélice é diferente<br />

de zero.<br />

Estes conceitos bastante intuitivos de curvatura e<br />

torção são fundamentais na concepção moderna de<br />

espaço e tempo. Como você está percebendo, existe<br />

uma relação muito íntima entre Física e Geometria.<br />

Esta relação é confirmada em muitas outras áreas<br />

da Física e atinge seu clímax na Teoria da Relatividade<br />

Geral elaborada por Albert Einstein.<br />

2.6.1 Exercícios<br />

Exercício 18<br />

(1) Calcule os vetores velocidades, e seus respectivos<br />

módulos, usando todos os vetores posição do<br />

Exercício 17. (2) Calcule os vetores acelerações, e<br />

seus respectivos módulos, usando todos os vetores<br />

posição do Exercício 17, ou, equivalentemente, os<br />

vetores velocidades do item anterior. (3) Calcule<br />

também os produtos vetoriais entre os vetores velocidade<br />

e aceleração. As trajetórias planas estão<br />

relacionadas de que forma com estes produtos vetoriais?<br />

Reveja a discussão apresentada no parágrafo<br />

anterior.<br />

Exercício 19<br />

Determine os pontos extremos da função f(t) =<br />

1 − t + 2t 2 + t 3 . Desenhe esta função e localize<br />

estes pontos extremos e classifique-os (máximos ou<br />

mínimos). Observe o valor da segunda derivada de<br />

f(t) nestes pontos.<br />

Exercício 20<br />

Determine, usando derivadas e condições físicas,<br />

o tempo gasto para um objeto atingir a altura<br />

máxima na trajetória parabólica do Exercício 17.<br />

Exercício 21<br />

Desenhe a curva ⃗r = (t, 1 + t 2 , 0) (parábola)<br />

no intervalo t ∈ [−0.5, 0.5] e guarde este desenho<br />

na variável g1. Desenhe a circunferência de<br />

raio 1/κ(0), com κ(t) = 2/ √ 1 + 4t 2 , centrada em<br />

(0, 3/2), e guarde este desenho numa variável g2<br />

(coloque este desenho no plano z = 0.1). Mostre<br />

estes dois desenhos simultaneamente e tire suas<br />

conclusões a respeito da curvatura em t = 0.<br />

Exercício 22<br />

Deduza as expressões (2.61)–(2.62) diretamente de<br />

(2.60). Determine o ângulo entre os vetores ⃗r e ⃗v<br />

dados em (2.60) e (2.61), respectivamente. Calcule<br />

a curvatura (2.59) para estes dois vetores. Com<br />

base no exercício anterior e neste, interprete a curvatura<br />

geometricamente.<br />

Exercício 23<br />

Use a expressão (2.73) para calcular a torção da<br />

circunferência ⃗r(t) = (R cos(ωt), R sin(ωt), 0) e da<br />

hélice circular ⃗r(t) = (R cos(ωt), R sin(ωt), αt).<br />

2.7 Espaço percorrido<br />

Pronto para o nosso último problema? Estabelecer<br />

uma ferramenta eficiente para determinar o<br />

espaço percorrido. Observe a trajetória da Figura<br />

2.7. A não ser no caso de uma trajetória retilínea,<br />

em geral o comprimento de uma trajetória<br />

(espaço percorrido) não é obtido calculando-se o<br />

módulo da diferença entre vetores posição. No<br />

entanto, podemos subdividir a trajetória em muitos<br />

pedaços pequenos de forma que cada pedaço<br />

possa ser considerado um trajetória retilínea muito<br />

pequena. Sendo retilíneos, podemos calcular seus<br />

comprimentos através do módulo do vetor diferença<br />

associado a cada um deles. O comprimento total<br />

20


Capítulo 2. Cinemática<br />

2.7. Espaço percorrido<br />

da trajetória será a soma desses inúmeros comprimentos<br />

retilíneos pequenos (veja a Figura 2.10).<br />

Pode não parecer, talvez por envolver uma soma<br />

com muitos termos, mas este mecanismo é extremamente<br />

eficiente. Vamos torná-lo operacional.<br />

Para isto precisaremos de um intervalo pequeno da<br />

trajetória, digamos, entre os instantes t e t + ∆t,<br />

com ∆t muito pequeno, ∆⃗r(t) = ⃗r(t + ∆t) − ⃗r(t).<br />

Para ∆t infinitesimalmente pequeno (isto significa<br />

que a trajetória foi dividida em infinitas partes) é<br />

razoável imaginarmos que este trecho da trajetória<br />

será praticamente retilíneo (veja a Figura 2.10).<br />

Assim, seu comprimento será praticamente igual<br />

ao módulo de ∆⃗r,<br />

||∆⃗r(t)|| = √ ∆⃗r · ∆⃗r = √ ⃗v · ⃗v ∆t<br />

= ||⃗v|| ∆t,<br />

(2.74)<br />

na qual aproveitamos para introduzir o vetor velocidade<br />

no instante t, ⃗v = ∆⃗r/∆t. É na verdade<br />

uma velocidade média, mas será a velocidade instantânea<br />

no limite ∆t → 0.<br />

Agora estamos numa posição muito confortável,<br />

pois o comprimento total da trajetória entre os instantes<br />

t 0 e t 1 pode ser obtido somando-se todos os<br />

intervalos (2.74). Esta soma pode ser efetuada no<br />

limite ∆t → 0, ou seja, sub-dividindo a trajetória<br />

em infinitos intervalos. A nossa capacidade em executar<br />

estas somas infinitas é simplesmente incrível.<br />

Como estamos lidando com uma soma muito especial,<br />

precisaremos de uma notação especial. Esta<br />

soma será denominada de integral e denotada por<br />

S 01 =<br />

∫ t1<br />

t 0<br />

v(t) dt = S(t)<br />

∣<br />

1<br />

0<br />

= S(1) − S(0), (2.75)<br />

onde fizemos ||⃗v|| = v(t). Note que o limite ∆t → 0<br />

foi indicado simplesmente por dt. O símbolo dt (diferencial)<br />

em (2.75) indica que a integral (soma)<br />

está sendo feita na variável t (variável de integração).<br />

A integral (2.75) é denominada de definida,<br />

t 0 indicando o limite inferior e t 1 indicando o<br />

limite superior. Sem os limites de integração, uma<br />

integral é denominada de indefinida e, em geral,<br />

fornece uma nova função da variável de integração.<br />

Vejamos um exemplo. Suponha que o movimento<br />

seja em uma dimensão, ⃗r = x(t) î com x(t) = t<br />

(movimento uniforme). Portanto, usando (2.54),<br />

o vetor velocidade é ⃗v = ẋ î com ẋ = 1. Assim,<br />

v(t) = ||⃗v|| = 1 e o espaço percorrido (em metros)<br />

entre os instantes t 0 = 0 e t 1 = 1 (em segundos) é<br />

∫ 1 ∫ 1<br />

1<br />

S 01 = v(t) dt = dt = t<br />

∣ (2.76)<br />

0<br />

= 1 − 0 = 1.<br />

Note que este resultado coincide com o módulo da<br />

diferença entre os vetores posição nos instantes inicial<br />

e final, S 01 = x(1) − x(0) = 1 − 0 = 1, pois a<br />

trajetória aqui é retilínea.<br />

Suponha agora que a trajetória seja uma<br />

parábola no plano XY , ⃗r = t î + (1 + t 2 ) ĵ. Assim,<br />

o vetor velocidade é ⃗v = î + 2t ĵ, cujo módulo é<br />

v(t) = √ 1 + 4t 2 . Quem é o espaço percorrido entre<br />

os instantes t 0 = 0 s e t 1 = 1 s? Devemos efetuar a<br />

integral (2.75) (via computação algébrica),<br />

S 01 =<br />

∫ 1<br />

0<br />

v(t) dt =<br />

= 1.479 m.<br />

∫ 1<br />

0<br />

0<br />

0<br />

√<br />

1 + 4t2 dt<br />

(2.77)<br />

Note agora que este espaço percorrido é diferente<br />

de ||⃗r(1) − ⃗r(0)|| = 1.414 m, pois a trajetória não é<br />

linear como no exemplo anterior. Espero que este<br />

exemplo tenha mostrado a importância desta ferramenta<br />

nova, denominada de integral (mais detalhes<br />

no curso de Cálculo).<br />

f(t i )<br />

f(t)<br />

•<br />

•<br />

•<br />

•<br />

•<br />

•<br />

t 0 t i t i + ∆t<br />

t 1<br />

•<br />

Figura 2.10: A área total abaixo das retas secantes<br />

se aproxima cada vez mais da área abaixo da curva<br />

f(t), entre t 0 e t 1 , à medida que o número N de<br />

sub-intervalos aumenta (N → ∞).<br />

Opa! Espere um pouco. Como as integrais em<br />

(2.76) e (2.77) foram efetuadas? Será que não podemos<br />

apreender como executá-las, pelo menos em<br />

t<br />

21


2.7. Espaço percorrido Capítulo 2. Cinemática<br />

algumas situações simples? Muito bem. Precisamos<br />

Portanto, iremos calcular integrais respondendo<br />

Então,<br />

onde calculamos o espaço percorrido na trajetória<br />

d<br />

dt F (t) = f(t). (2.81) ⃗r = t î + (1 + t 2 )ˆȷ [confira novamente a integral<br />

em (2.77)], não podemos adivinhar que a integral<br />

saber como calcular uma integral. Também à seguinte pergunta: “quem é a função F (t) cuja<br />

precisamos saber qual é a interpretação geométrica<br />

de uma integral.<br />

Vamos continuar na mesma linha de raciocínio<br />

usada anteriormente onde fizemos v(t) = ||⃗v||. Vejamos<br />

primeiro a interpretação geométrica. A Figura<br />

2.10 mostra um trecho de uma função arbitrária<br />

derivada é igual ao integrando f(t) em (2.80)?”.<br />

Isto significa que primeiro devemos saber derivar<br />

para depois efetuarmos uma integral. Note também<br />

que o resultado de uma integral indefinida como<br />

em (2.80) pode muito bem conter uma constante<br />

aditiva, pois a derivada de qualquer constante é<br />

f(t) a ser integrada entre t 0 e t 1 , dando zero. A integral definida é efetuada da seguinte<br />

destaque para o i-ésimo sub-intervalo compreendido<br />

forma:<br />

entre t i e t i + ∆t. Como podemos ver, este<br />

∫<br />

sub-intervalo é um retângulo de altura f(t i ) e base<br />

t1<br />

t 1<br />

f(t) dt = F (t)<br />

∆t, cuja área é f(t i ) ∆t. Portanto, a área A abaixo<br />

∣ = F (t 1 ) − F (t 0 ).<br />

t 0<br />

t 0<br />

da curva f(t), entre t 0 e t 1 , é aproximadamente a<br />

(2.82)<br />

soma das N áreas f(t i ) ∆t de cada retângulo, Vejamos alguns exemplos. Primeiro um exemplo<br />

onde o resultado é bem conhecido: a área de um<br />

N∑<br />

A ≈ f(t i ) ∆t. (2.78)<br />

triângulo reto, de base 1 e altura 1, é 1/2. Esta é<br />

a área delimitada pela curva f(t) = t entre t = 0 e<br />

i=1<br />

t = 1. Usando (2.79) e o Teorema (5),<br />

No limite ∆t → 0 ou, equivalentemente, no limite<br />

de haver infinitos sub-intervalos (N → ∞), a área<br />

∫ 1<br />

( )∣ t<br />

2 ∣∣∣<br />

1<br />

A abaixo da curva f(t) será calculada exatamente A = t dt =<br />

0 2 + C 0<br />

por uma soma infinita, denominada de integral (definida)<br />

da função f(t) entre t 0 e t 1 ,<br />

1 0<br />

( ) ( )<br />

=<br />

2 + C −<br />

2 + C = 1 (2.83)<br />

2 .<br />

N∑<br />

∫ t1<br />

A = lim f(t i ) ∆t = f(t) dt. (2.79) A área debaixo de uma parábola, f(t) = t 2 ,<br />

N→∞<br />

i=1<br />

t 0<br />

é outro exemplo no qual o resultado é desconhecido<br />

(pelo menos para os normais). Assim, usando<br />

Desta forma, uma integral está relacionada com a<br />

área abaixo de uma curva.<br />

(2.79) e o Teorema (5), a área abaixo da parábola<br />

Newton (1665–1666) e Leibniz (1684-1686) são<br />

f(t) = t 2 no intervalo t = 0 e t = 2 pode ser calculada<br />

facilmente,<br />

considerados os fundadores do cálculo diferencial<br />

(derivadas) e do cálculo integral (integrais). Eles<br />

∫ 1<br />

( )∣<br />

já conheciam a inter-relação entre derivadas (tangentes)<br />

e integrais (áreas) devido aos trabalhos de<br />

0 3 + C 0<br />

t<br />

A = t 2 3 ∣∣∣<br />

2<br />

dt =<br />

( ) ( )<br />

Barrow (1663-1669) relacionando tangentes de curvas<br />

com áreas delimitadas por estas mesmas cur-<br />

= 8 0<br />

3 + C −<br />

3 + C = 8 3 .<br />

vas. Esta relação é conhecida hoje como o teorema<br />

(2.84)<br />

fundamental do cálculo. Hoje, este teorema é enunciado<br />

da seguinte forma:<br />

Naturalmente, resolvemos estas duas integrais<br />

respondendo uma pergunta: “Quem é a função F (t)<br />

cuja derivada é igual ao integrando f(t) em (2.80)?”<br />

Teorema 5 (Teorema Fundamental)<br />

simplesmente porque já sabíamos da resposta, consultando<br />

as derivadas elementares (2.49). No en-<br />

Seja f(t) uma função contínua e F (t) a sua integral<br />

(indefinida),<br />

∫<br />

tanto, nem sempre, mesmo em situações relativamente<br />

f(t) dt = F (t). (2.80)<br />

simples, poderemos ter este mesmo êxito.<br />

Por exemplo, no exemplo do parágrafo anterior<br />

22


Capítulo 2. Cinemática<br />

2.7. Espaço percorrido<br />

indefinida correspondente é<br />

∫ √1<br />

+ 4t2 dt = 1 2 t√ 1 + 4t 2<br />

+ 1 4 sinh−1 (2t). (2.85)<br />

Por isto é que existe um conjunto muito grande<br />

de regras para nos ajudar a calcular integrais. Estas<br />

regras serão exploradas durante os vários cursos<br />

de Cálculo. Entretanto, ao contrário de derivadas,<br />

deve ser mencionado que nem sempre temos<br />

êxito em resolver uma integral. Neste caso, devemos<br />

usar diretamente a definição (2.83) (executando<br />

uma soma) em um ambiente de computação<br />

(numérica ou algébrica) para obtermos valores de<br />

integrais definidas.<br />

2.7.1 Exercícios<br />

Exercício 24<br />

Calcule as integrais indefinidas de 2t + sin(t), t 2 +<br />

e −t , te −t2 .<br />

Exercício 25<br />

Calcule o espaço percorrido na trajetória circular<br />

⃗r(t) = (R cos(2πt), R sin(2πt), 0) entre os instantes<br />

t = 0 e t = t. Compare este resultado com a<br />

expressão que é conhecida para o comprimento de<br />

uma circunferência de raio R.<br />

Exercício 26<br />

Calcule o espaço percorrido na trajetória parabólica<br />

⃗r(t) = (t, t, 20t − 5t 2 ) (mks) entre os instantes t =<br />

0 s e t = 4 s. Qual é a distância no plano XY (solo)<br />

entre o ponto de lançamento (t = 0 s) e o ponto de<br />

chegada (t = 4 s)?<br />

Exercício 27<br />

Faça um programa para calcular numericamente<br />

uma derivada e uma integral (definida). Consulte<br />

seu professor para obter as devidas ferramentas<br />

computacionais.<br />

23


2.7. Espaço percorrido Capítulo 2. Cinemática<br />

24


Capítulo 3<br />

Translações<br />

3.1 Os princípios<br />

Que é uma lei (ou princípio)? No que diz respeito<br />

às leis da natureza, o dicionário nos diz que uma<br />

lei (da natureza) é “aquilo que se impõe ao homem<br />

por sua razão, consciência ou por determinadas<br />

condições ou circunstâncias”. Em outras palavras,<br />

uma lei física é uma afirmação fundamental,<br />

passível de verificação, formando a base de uma teoria.<br />

Portanto, uma teoria é um conjunto de leis.<br />

É importante fazermos um paralelo entre Física e<br />

Matemática. Em Matemática, o postulado é o equivalente<br />

de uma lei em Física, porém com uma diferença<br />

muito importante: um postulado nem sempre<br />

está sujeito à verificação. Por estarem sempre sujeitas<br />

a verificações, as leis físicas estão sempre em<br />

estado de alerta.<br />

A história tem nos mostrado que uma lei física<br />

não é imutável. Basta uma única experiência<br />

bem sucedida que mostre claramente a violação de<br />

uma determinada lei para que ela seja abandonada<br />

ou corrigida. Por exemplo, até a década de 60,<br />

acreditava-se que um processo físico e a sua imagem<br />

(como num espelho) fossem idênticos, isto é,<br />

produzissem os mesmos resultados. Esta lei ficou<br />

conhecida como a lei da conservação da paridade<br />

(sem distinção entre direito e esquerdo; similaridade).<br />

Todavia, dois jovens físicos, C. N. Yang e<br />

T. D. Lee, fizeram em 1956 uma previsão (ou observação)<br />

onde esta lei da conservação da paridade<br />

pudesse ser violada em processos de desintegração<br />

nuclear (decaimento beta). Esta previsão foi confirmada<br />

em 1957 em um belíssimo experimento conduzido<br />

por Madame Wu (C. S. Wu). Neste caso, a<br />

conservação da paridade foi descartada como uma<br />

lei.<br />

Um outro exemplo: a teoria Newtoniana da gravitação<br />

é capaz de explicar razoavelmente bem,<br />

usando a lei da gravitação Newtoniana, o comportamento<br />

mecânico do nosso sistema solar, ou seja,<br />

reproduz as leis de Kepler. Todavia, existem alguns<br />

efeitos minuciosos que a teoria Newtoniana<br />

da gravitação não consegue explicar (reproduzir).<br />

Dois deles, o desvio da luz ao passar próxima de<br />

um corpo estelar como o nosso sol e a precessão<br />

do periélio do planeta Mercúrio (voltaremos a estes<br />

dois casos mais tarde), quando calculados com<br />

a teoria Newtoniana não concordam plenamente<br />

com as observações. A reconciliação entre teoria<br />

e experimento só é possível no contexto da relatividade<br />

geral de Einstein (1915). Neste caso, a teoria<br />

Newtoniana foi corrigida pela relatividade geral.<br />

Mesmo as leis de Newton para o movimento translacional<br />

que estamos prestes a discutir valém somente<br />

para velocidades baixas em comparação com<br />

a velocidade da luz. Para velocidades comparáveis<br />

à da luz, a mecânica Newtoniana é corrigida pela<br />

mecânica relativística (Einstein, 1905), como veremos<br />

mais tarde.<br />

Que acontece se alguém viola uma lei? Se você<br />

viola uma lei na nossa sociedade atual, você sofre<br />

as devidas punições (pelo menos deveria). No<br />

entanto, Yang e Lee dividiram um prêmio de um<br />

milhão de dólares (prêmio Nobel) ao violarem uma<br />

lei em Física. Moral: caso precise quebrar alguma<br />

lei, quebre uma lei física! E aproveite o prêmio<br />

para praticar um pouco de filantropia. Por outro<br />

lado, aprofundamos nossos conhecimentos sobre<br />

nossa natureza. Por exemplo, além de corrigir<br />

a mecânica Newtoniana, a relatividade especial de<br />

Einstein nos revelou a fusão entre espaço e tempo<br />

(o tempo não é absoluto como na mecânica Newto-<br />

25


3.1. Os princípios Capítulo 3. Translações<br />

niana) bem como entre energia e massa (E = mc 2 ).<br />

A relatividade geral nos revelou que a matéria (e/ou<br />

energia) diz ao espaço-tempo como se deformar<br />

(curvar e torcer) e, por sua vez, o espaço deformado<br />

dita à matéria como ela deve se mover. Em<br />

outras palavras, evoluímos, ficamos mais próximos<br />

da humanidade.<br />

Estudamos até o momento o movimento de um<br />

ponto material. Sabemos construir sua trajetória<br />

usando curvas espaciais em um espaço Euclidiano<br />

tridimensional. Também sabemos determinar velocidade<br />

e aceleração, bem como o espaço percorrido,<br />

em qualquer instante de tempo. Portanto, é hora de<br />

passarmos ao estudo das leis físicas que determinam<br />

e controlam o movimento de corpos macroscópicos<br />

(que não sejam muito mais massivos que o Sol)<br />

com velocidades muito inferiores à velocidade da<br />

luz no vácuo (aproximadamente 300 000 km/s). Estas<br />

leis são conhecidas por “leis de Newton” do<br />

movimento. As duas restrições mencionadas (corpos<br />

macroscópicos de baixa massa e com baixas<br />

velocidades) são importantes, pois as leis Newtonianas<br />

deixam de ser válidas em três ocasiões: (1) no<br />

mundo microscópico de dimensões atômicas (neste<br />

caso as leis Newtonianas são corrigidas pelas leis da<br />

física quântica), (2) quando as velocidades envolvidas<br />

são comparáveis à velocidade da luz no vácuo<br />

(neste caso as leis Newtonianas são corrigidas pelas<br />

leis da relatividade especial de Einstein) e (3)<br />

para corpos extremamente massivos , como estrelas<br />

de nêutrons (neste caso as leis Newtonianas são<br />

corrigidas pela relatividade geral de Einstein).<br />

A seguir, apresentaremos os dois princípios que<br />

governam o movimento. Iremos reservar a palavra<br />

lei (sinônimo de princípio) para as três leis Newtonianas,<br />

as quais são equivalentes aos dois princípios<br />

apresentados abaixo. Deve ser enfatizado que estas<br />

leis têm sido submetidas a verificações refinadas por<br />

séculos sem qualquer contradição, exceto no mundo<br />

microscópico, ou no regime de altas velocidades, ou<br />

no caso de corpos extremamente massivos.<br />

Princípio 1<br />

Existem certos referenciais, denominados de inerciais<br />

(no sentido de incapacitados), com as seguintes propriedades.<br />

1. Cada corpo isolado move-se em uma linha reta<br />

neste referencial inercial.<br />

2. A noção de tempo pode ser quantificada: uma<br />

unidade de tempo pode ser definida através do<br />

movimento uniforme de um corpo isolado neste<br />

referencial inercial.<br />

3. Cada corpo isolado move-se com uma velocidade<br />

constante neste referencial inercial.<br />

Note que estas três propriedades definem um referencial<br />

inercial. Note também que um corpo isolado<br />

não tem aceleração em um referencial inercial.<br />

Assim, podemos definir, pelo menos operacionalmente,<br />

tempo como sendo um parâmetro t proporcional<br />

ao comprimento s da trajetória de um corpo<br />

isolado (velocidade constante) em um referencial<br />

inercial:<br />

∆s(t) =<br />

∫ ∆t<br />

0<br />

v dt = v ∆t ⇒ ∆t = ∆s<br />

v . (3.1)<br />

Naturalmente, qualquer outro parâmetro t ′ definido<br />

como tempo, usando um outro corpo isolado,<br />

estará relacionado de forma linear com o<br />

primeiro parâmetro t, pois os comprimentos de<br />

duas trajetórias quaisquer são sempre proporcionais.<br />

Note também que podemos medir apenas<br />

intervalos de tempo, proporcionais ao comprimento<br />

da trajetória, e nunca os valores absolutos<br />

do “tempo”.<br />

Como vimos anteriormente, o comprimento de<br />

uma trajetória é calculado através de uma integral.<br />

Assim, usar o comprimento de uma trajetória para<br />

medir intervalos de tempo não é prático. Desde há<br />

muito tempo, temos usado o movimento de rotação<br />

da Terra em torno de seu próprio eixo, praticamente<br />

constante, como um instrumento (relógio)<br />

para medir intervalos de tempo. Qualquer processo<br />

repetitivo, com uma freqüência praticamente<br />

constante, pode ser usado como um relógio. Os<br />

relógios mais precisos que dispomos no momento<br />

são os chamados “relógios atômicos”, baseados na<br />

impressionante regularidade de certos processos nucleares<br />

(portanto, pertencentes ao domínio da física<br />

quântica). Em um relógio destes, o erro é de apenas<br />

um segundo em um milhão de anos. Um fato importante:<br />

embora não sabemos exatamente o que é<br />

o tempo, sabemos operar com ele, isto é, sabemos<br />

medir intervalos de tempo.<br />

Princípio 2<br />

Considere dois corpos isolados dos demais, mas nãoisolados<br />

entre si, sendo observados em um referencial<br />

26


Capítulo 3. Translações<br />

3.1. Os princípios<br />

inercial. Isto significa que estes dois corpos poderão<br />

interagir entre si. Em geral, não podemos afirmar que<br />

o movimento deles será uniforme, pois, podendo interagir,<br />

eles poderão alterar suas velocidades, as quais<br />

denotaremos por ⃗v i (t), i = 1, 2. Então as duas propriedades<br />

seguintes se verificam.<br />

1. Existem experimentos (colisões) envolvendo estes<br />

dois corpos cujos resultados podem ser expressados<br />

como<br />

⃗v 1 (t) + µ 12 ⃗v 2 (t) = ⃗ P 12 . (3.2)<br />

Caso estes experimentos sejam realizados usando<br />

o corpo 1 e um terceiro corpo 3, ou usando o<br />

corpo 2 e um terceiro corpo 3, sob condições<br />

idênticas, resultados similares são obtidos,<br />

⃗v 1 (t) + µ 12 ⃗v 2 (t) = ⃗ P 12 ,<br />

⃗v 2 (t) + µ 23 ⃗v 3 (t) = P ⃗ 23 , (3.3)<br />

⃗v 3 (t) + µ 31 ⃗v 1 (t) = P ⃗ 31<br />

nos quais, os escalares µ ij e os vetores P ⃗ ij são<br />

constantes, isto é, são independentes do tempo.<br />

Mais importante, as constantes escalares µ ij são<br />

positivas e não dependem do experimento e nem<br />

do referencial inercial. Elas dependem apenas<br />

dos dois corpos envolvidos. A situação é bem<br />

diferente para as constantes vetoriais P ⃗ ij : elas<br />

dependem do experimento, do referencial inercial<br />

e dos dois corpos envolvidos.<br />

2. As constantes (escalares) µ ij > 0, obtidas em experimentos<br />

diferentes, estão relacionadas da seguinte<br />

forma:<br />

µ 12 µ 23 µ 31 = 1. (3.4)<br />

Algumas observações são necessárias. Observe<br />

que devido ao fato das constantes µ ij > 0 obedecerem<br />

à condição (3.4), elas podem ser escritas numa<br />

forma racional µ ij = m j /m i , com m i > 0 (faça o<br />

Exercício 28). Em termos destas novas constantes,<br />

os resultados (3.3) podem ser re-escritos como<br />

m i ⃗v i (t) + m j ⃗v j (t) = ⃗p ij , ⃗p ij = m i<br />

⃗ Pij . (3.5)<br />

A quantidade vetorial<br />

m i ⃗v i (t) ≡ ⃗p i (t) (3.6)<br />

é denominada de momentum linear (ou quantidade<br />

de movimento) do i-ésimo corpo. Note que (3.6) é<br />

uma definição. A constante vetorial ⃗p ij = ⃗p i (t) +<br />

⃗p j (t) é o momentum linear total dos dois corpos<br />

envolvidos em cada experimento e é uma constante<br />

quando estes dois corpos estão isolados. A constante<br />

m i > 0 é denominada de carga inercial (ou<br />

massa inercial, ou simplesmente massa, por razões<br />

históricas). Note que apenas a razão µ ij é determinada<br />

pelo experimento. Portanto, podemos determinar<br />

apenas razões entre massas, µ ij = m j /m i .<br />

Isto significa que temos de eleger um corpo padrão,<br />

cuja massa deve ser assumida como uma unidade<br />

de massa, digamos m i = 1 kg. Então a massa<br />

m j = µ ij é determinada univocamente do experimento.<br />

Novamente, mesmo não sabendo o que<br />

é massa, somos capazes de operar com ela, escolhendo<br />

um padrão e realizando comparações.<br />

Como nestes experimentos o momentum linear<br />

total ⃗p ij = ⃗p i (t) + ⃗p j (t) é constante, então, derivando<br />

os dois lados desta expressão, sabendo que<br />

as massas m i são constantes, temos<br />

m i ⃗a i (t) + m j ⃗a j (t) = 0. (3.7)<br />

Apesar dos dois corpos estarem isolados, eles interagem<br />

entre si no momento da colisão. Esta interação<br />

produz mudanças nas velocidades (ou na<br />

quantidade de movimento) acarretando na relação<br />

(3.7). Naturalmente, as acelarações em (3.7) são diferentes<br />

de zero apenas durante o (curto) intervalo<br />

de tempo da colisão. Como temos apenas dois corpos,<br />

isto significa que cada parcela em (3.7) deve<br />

ser idêntica à interação sofrida por um dos corpos<br />

no momento da colisão. Esta interação, capaz<br />

de mudar o vetor velocidade, denominaremos<br />

de força. Assim, podemos concluir que uma força<br />

⃗f i (t) agindo num objeto de massa m i (constante)<br />

altera seu movimento de acordo com<br />

⃗f i (t) = m i ⃗a i (t) = d dt ⃗p i. (3.8)<br />

No nosso experimento, esta é a forma com que a<br />

força devida ao j-ésimo objeto atua sobre o i-ésimo<br />

objeto durante a colisão. Note que, ao contrário<br />

da definição (3.6) do momentum linear, a Eq. (3.8)<br />

não é uma definição de força. A Eq. (3.8) é o resultado<br />

de uma observação experimental. Então, o<br />

resultado experimental (3.7) pode ser re-escrito na<br />

forma<br />

⃗F R = f ⃗ ij (t) + f ⃗ ji (t) = 0. (3.9)<br />

27


3.1. Os princípios Capítulo 3. Translações<br />

Note que f ⃗ ij (t) = m i ⃗a i (t) é a força que o corpo j faz<br />

no corpo i e f ⃗ ji (t) = m j ⃗a j (t) é a força que o corpo<br />

i faz no corpo j, portanto elas atuam em corpos diferentes.<br />

Ainda mais, a soma vetorial nula em (3.9)<br />

implica em | f ⃗ ij | = | f ⃗ ji | e f ⃗ ij // − f ⃗ ji (vetores antiparalelos).<br />

Conseqüentemente, (3.9) expressa a terceira<br />

lei de Newton (ação e reação). Aprendemos<br />

também com estes experimentos que é necessário<br />

um agente físico (força) para alterar o estado de<br />

movimento de um corpo material e que esta força<br />

é a taxa de variação do momentum linear (segunda<br />

lei de Newton).<br />

Vimos também, através do Princípio 2, que tanto<br />

o momentum linear total em (3.5) quanto a força<br />

total (ou resultante) em (3.9) são obtidos somando<br />

todas as contribuições individuais, soma esta que<br />

é uma combinação linear de vetores. Em Física,<br />

quantidades que são determinadas através de combinações<br />

lineares de outras são ditas satisfazerem o<br />

princípio de superposição.<br />

E as famosas três leis de Newton para o movimento?<br />

Onde estão? Elas estão contidas nestes<br />

dois princípios. A primeira lei de Newton, ou<br />

lei da inércia está contida no Princípio 1. Uma<br />

partícula isolada em um referencial inercial ou<br />

está em repouso ou está em movimento uniforme.<br />

Note que não estamos usando o conceito de forças<br />

no Princípio 1. Não devemos esquecer que o<br />

Princípio 1 também nos dá uma definição operacional<br />

de tempo.<br />

A segunda lei de Newton está contida no<br />

Princípio 2,<br />

⃗F R = d ⃗p, ⃗p = m⃗v, (3.10)<br />

dt<br />

onde ˙⃗p = m⃗a para massas constantes. Em geral,<br />

a força resultante F ⃗ R é conhecida independentemente<br />

do momentum linear, ou seja, a segunda<br />

lei de Newton não é uma mera definição de força.<br />

Ela representa um fato experimental: uma força<br />

muda o estado de movimento de uma massa m de<br />

acordo com a expressão em (3.10). A terceira lei de<br />

Newton também está contida no Princípio 2 [veja<br />

a Eq. (3.9)].<br />

Note o papel do momentum linear na definição<br />

de força (3.10): se a força é nula, então o momentum<br />

linear é constante. Uma quantidade constante<br />

no tempo é denominada de quantidade conservada.<br />

Então temos nosso primeiro teorema, denominado<br />

de teorema de conservação do momentum linear:<br />

Teorema 6<br />

Se a força resultante em um sistema físico é nula em<br />

um referencial inercial, então o momentum linear total<br />

deste sistema é conservado (não varia no tempo).<br />

A utilidade de uma quantidade conservada reside<br />

no fato dela possuir os mesmos valores em<br />

tempos diferentes. Por exemplo, a quantidade de<br />

movimento total ⃗p ij (t) em (3.5) é conservada no<br />

nosso experimento. Então, se denotarmos por t 1<br />

um tempo antes da colisão e por t 2 um tempo depois<br />

da colisão, teremos ⃗p ij (t 1 ) = ⃗p ij (t 2 ), a qual<br />

pode ser usada para determinar alguma quantidade<br />

desconhecida contida nela (faça o Exercício 29).<br />

É importante frisar que todas as medidas (experimentos)<br />

estão sendo realizadas em um referencial<br />

inercial. Caso contrário, a segunda lei de Newton<br />

(3.10) não é válida, ou seja, em um referencial nãoinercial<br />

a segunda lei de Newton precisa ser corrigida<br />

(veremos isto ao estudarmos as leis que governam<br />

o movimento rotacional). Naturalmente,<br />

o momentum linear tem um papel importante na<br />

mecânica Newtoniana. Esta importância é devida<br />

ao Teorema 6 sobre a conservação do momentum linear.<br />

Portanto, podemos dizer que a sua definição<br />

em (3.6) está justificada pelo seu conteúdo físico.<br />

Note que o Teorema 6 é uma conseqüência direta<br />

da relação (3.10) entre força e momentum linear.<br />

Além disto, a relação (3.10) permite que a segunda<br />

lei de Newton possa ser usada para descrever sistemas<br />

com massas variáveis, como em um foguete,<br />

por exemplo.<br />

Qual é a importância da massa m que aparece<br />

na quantidade conservada em (3.5)? Lá, o experimento<br />

nos revelou que o momentum linear total<br />

do sistema (isolado) formado apenas pelos corpos<br />

i e j é constante. Se fizermos m j = 0, então resulta<br />

que ⃗v i é também uma constante, ou seja, o<br />

corpo i está em movimento uniforme. No entanto,<br />

se este mesmo corpo i também tivesse uma massa<br />

nula, m i = 0, então a sua quantidade de movimento<br />

seria nula. Isto nos sugere que é necessário<br />

que um corpo tenha massa não-nula para poder<br />

ter uma quantidade de movimento não-nula. Desta<br />

forma, podemos afirmar que a condição de estar em<br />

movimento uniforme em um referencial inercial depende<br />

exclusivamente de ter uma massa não-nula.<br />

Vale mencionar que até a época de Newton (até o<br />

Séc. 16, portanto um pouco mais de 2000 anos depois<br />

de Aristóteles) acreditava-se que era necessário<br />

28


Capítulo 3. Translações<br />

3.1. Os princípios<br />

uma força para manter um corpo em movimento,<br />

mesmo em movimento uniforme. Assim, a massa<br />

m em (3.10) passou a ser conhecida também por<br />

massa inercial. Em (3.10) podemos ver que aumentando<br />

a massa, devemos aumentar também a<br />

intensidade da força para manter a intensidade da<br />

aceleração constante. Isto significa que temos de esforçar<br />

mais para mudar o estado de movimento de<br />

um corpo com uma maior massa. Esta resistência<br />

em mudar o estado de movimento é quantificada<br />

pela massa inercial. Em outras palavras, a massa<br />

inercial mede a resistência de um corpo em mudar<br />

seu estado de movimento. A massa inercial é uma<br />

propriedade intrínsica de qualquer objeto.<br />

Qual é a importância do vetor força, definido em<br />

(3.8)? Qual é o seu conteúdo físico? Observamos<br />

que na imensa maioria dos casos é a força que é conhecida<br />

em primeira mão, independentemente da<br />

aceleração. Veremos vários exemplos logo em seguida.<br />

Em geral, para massas constantes, esta força<br />

dependerá da posição, da velocidade e do próprio<br />

tempo,<br />

⃗F ( ⃗r, d⃗r<br />

dt ; t) = m d2 ⃗r<br />

dt 2 . (3.11)<br />

Assim, esta equação (segunda lei de Newton) envolve<br />

o vetor posição ⃗r e suas duas primeiras derivadas<br />

d⃗r/dt e d 2 ⃗r/dt 2 . É por isto que as quantidades<br />

cinemáticas (posição, velocidade e aceleração)<br />

terminam na aceleração.<br />

E o que nós queremos de um sistema físico do<br />

ponto de vista da mecânica? Queremos caracterizar<br />

o seu comportamento mecânico. Que significa<br />

isto? Isto significa que dado uma força resultante,<br />

a posição inicial ⃗r 0 = ⃗r(t 0 ) e a velocidade inicial<br />

⃗v 0 = ⃗v(t 0 ), queremos determinar a posição ⃗r(t) em<br />

qualquer instante de tempo posterior ao instante<br />

inicial t 0 . Isto significa que (3.11) é uma equação<br />

que determina ⃗r(t). Como aparece nela também as<br />

derivadas de ⃗r(t), então esta equação é uma equação<br />

diferencial para ⃗r(t). É uma equação diferencial de<br />

segunda ordem, pois a derivada de ordem maior<br />

é a derivada segunda no tempo. A pergunta que<br />

temos de responder é: dado o lado esquerdo de<br />

(3.11), isto é, a força externa resultante, e esta informação<br />

deve ser obtida de experimentos, quem<br />

é ⃗r(t) satisfazendo as condições iniciais impostas<br />

sobre a posição e a velocidade? Note que (3.11) é<br />

uma equação diferencial vetorial, ou seja, há de fato<br />

três equações diferenciais a serem resolvidas, uma<br />

para cada componente nos eixos X, Y e Z (faça o<br />

Exercício 31).<br />

Bem, temos uma equação diferencial de segunda<br />

ordem a ser resolvida para encontramos as componentes<br />

do vetor posição em função do tempo.<br />

Esta equação é o conteúdo da segunda lei de Newton.<br />

Conhecendo as forças que agem em um determinado<br />

sistema, bem como técnicas matemáticas<br />

necessárias para resolver uma equação diferencial<br />

de segunda ordem, determinamos o vetor posição.<br />

Então, devemos perguntar: a equação diferencial<br />

(3.11) admite uma solução? se admitir, esta solução<br />

é única? Sim, sob certas condições, a equação diferencial<br />

(3.11) admite uma solução única. É necessário<br />

apenas fornecermos duas informações [note<br />

que a ordem da equação diferencial (3.11) também é<br />

dois]. Estas duas informações podem ser, por exemplo,<br />

os valores da posição ⃗r 0 = ⃗r(t 0 ) e da velocidade<br />

⃗v 0 = ⃗v(t 0 ) em algum instante inicial t 0 . Dado estas<br />

informações, denominadas de condições iniciais,<br />

existirá uma solução única, ⃗r(t), t >= t 0 , para a<br />

equação diferencial (3.10) (faça o Exercício 30).<br />

Então estamos numa posição bastante confortável.<br />

Dado as condições iniciais, ⃗r 0 = ⃗r(t 0 )<br />

e ⃗v 0 = ⃗v(t 0 ), determinamos ⃗r(t) e, conseqüentemente,<br />

⃗v(t) [a aceleração ⃗a(t) já está determinada<br />

em (3.11)]. Isto significa que podemos prever todo o<br />

comportamento do nosso sistema em qualquer instante<br />

de tempo t >= t 0 . Desta forma, podemos<br />

afirmar que a física Newtoniana é completamente<br />

determinística. Mas existe alguma área da Física<br />

que não é determinística? Sim, há pelo menos duas:<br />

a mecânica estatística, a qual é muito importante<br />

em termodinâmica, e a física quântica, a qual descreve<br />

os fenômenos do mundo microscópico.<br />

Embora a mecânica Newtoniana (também denominada<br />

de mecânica clássica) seja determinística,<br />

ela pode apresentar comportamentos caóticos.<br />

Para entendermos este comportamento caótico devemos<br />

perguntar o quão estável são as soluções de<br />

(3.11). Para examinarmos esta questão, suponha<br />

que temos a disposição duas soluções para (3.11):<br />

uma solução ⃗r 1 (t) determinada pelas condições iniciais<br />

⃗r 1 (t 0 ) e ⃗v 1 (t 0 ), e uma segunda solução ⃗r 2 (t) determinada<br />

pelas condições iniciais ⃗r 2 (t 0 ) = ⃗r 1 (t 0 )+<br />

δ⃗r(t 0 ) e ⃗v 1 (t 0 ). O termo δ⃗r(t 0 ) na condição inicial<br />

⃗r 2 (t 0 ) significa que ⃗r 2 (t 0 ) está muito próxima de<br />

⃗r 1 (t 0 ). Assim, é razoável supor que as duas soluções<br />

⃗r 1 (t) e ⃗r 2 (t) estarão muito próximas, ⃗r 2 (t) = ⃗r 1 (t)+<br />

δ⃗r(t), pelo menos nos instantes próximos a t 0 . Mas<br />

29


3.2. As forças da natureza Capítulo 3. Translações<br />

a medida que o tempo passa, como será o comportamento<br />

de δ⃗r(t)? Aumentará indefinidamente?<br />

Permanecerá pequeno? ou diminuirá até anularse?<br />

Em outras palavras, estas duas trajetórias<br />

com condições iniciais muito próximas permanecerão<br />

sempre muito próximas? ou a separação entre<br />

elas poderá oscilar? ou quem sabe aumentar indefinidamente?<br />

Uma solução de uma equação diferencial<br />

é denominada de solução estável se a separação<br />

δ⃗r(t) aproximar-se de zero ou permanecer constante<br />

e de solução instável quando esta separação aumentar<br />

indefinidamente com o tempo t. Vamos pensar<br />

em termos práticos: em geral é impossível fornecer<br />

as condições iniciais com uma precisão absoluta,<br />

pois nossos instrumentos de medidas sempre possuem<br />

uma precisão limitada. Portanto, o estudo<br />

da estabilidade das soluções de equações diferenciais<br />

é indispensável e é hoje uma área de pesquisa<br />

em desenvolvimento.<br />

Sistemas caóticos, pois é, vamos a eles. Suponha<br />

por um momento que exista um sistema tal que<br />

dado as condições iniciais a (uma notação simplificada<br />

para os valores da posição e da velocidade<br />

inicial) ele atinja um estado A (ou seja, depois de<br />

certo tempo ele terá uma posição e uma velocidade<br />

final). Suponha também que dado outra condição<br />

inicial b, ele atinja o estado B. Então se houver uma<br />

região onde as condições iniciais a e b estejam misturadas,<br />

o sistema pode exibir instabilidades (no<br />

sentido definido acima). De fato há sistemas onde<br />

as condições estão inextrincavelmente misturadas:<br />

em cada sub-região, não importa o quão pequena<br />

ela seja, sempre haverá condições iniciais dadas pela<br />

superposição de a e b. Estes sistemas são denominados<br />

de caóticos. Isto significa que precisaremos conhecer<br />

as condições iniciais em um sistema caótico<br />

com uma precisão infinita para que o estado final<br />

possa ser predito com exatidão, o que é impossível.<br />

3.1.1 Exercícios<br />

Exercício 28<br />

Escreva as constantes µ ij > 0 numa forma racional<br />

µ ij = m j /m i , com m i > 0 e mostre que elas satisfazem<br />

a relação (3.4). Estas constantes positivas<br />

m i > 0 são denominadas de “massas”.<br />

Exercício 29<br />

Sabendo que as quantidades P ⃗ ij em (3.3) são conservadas,<br />

determine as constantes µ ij em termos<br />

das velocidades antes e depois das colisões, as quais<br />

são medidas (conhecidas). Sugestão: se uma quantidade<br />

(vetorial ou escalar) A(t) é conservada, então<br />

A(t 1 ) = A(t 2 ).<br />

Exercício 30<br />

Resolva as equações diferenciais contidas em ⃗ F =<br />

m¨⃗r, com ⃗ F independente do tempo, da posição e da<br />

velocidade. Assuma que o corpo esteja em repouso<br />

na origem no instante inicial. Sugestão: re-escreva<br />

esta equação diferencial em coordenadas e resolva<br />

cada uma delas sepradamente por tentativas.<br />

Exercício 31<br />

Faça um paralelo entre uma equação diferencial e<br />

uma equação polinomial. O que você quer determinar<br />

em ambos os casos?<br />

Exercício 32<br />

Quais são as inconsistências presentes na forma tradicional<br />

de apresentação das leis de Newton? Como<br />

estas inconsistências são resolvidas pelos Princípios<br />

1 e 2?<br />

3.2 As forças da natureza<br />

Exemplos de forças? sim podemos examinar alguns<br />

exemplos. Durante esta apresentação, consulte<br />

também o Apêndice A, no qual há discussões<br />

importantes acerca das dimensões das quantidades<br />

presentes nas forças apresentadas nos exemplos<br />

subseqüentes.<br />

Em muitas situações práticas, principalmente<br />

nas engenharias Mecânica e Civil, as forças encontradas<br />

são constantes, F ⃗ = F ⃗ 0 , isto é, independentes<br />

da posição, velocidade e do tempo. Geralmente<br />

estas forças constantes são forças de contato entre<br />

corpos, geralmente de origem eletromagnética.<br />

Como a força resultante é constante (independente<br />

do tempo, da posição e da velocidade), o vetor aceleração<br />

em (3.11) fica determinado e é também uma<br />

constante, ⃗a = F ⃗ 0 /m. Desta forma, a equação diferencial<br />

resultante da segunda lei de Newton [veja<br />

a Eq. (3.11)] que temos de resolver é<br />

d 2<br />

dt 2 ⃗r(t) = ⃗ F 0<br />

m . (3.12)<br />

Naturalmente, com uma integração, podemos determinar<br />

o vetor velocidade. Finalmente, integrando<br />

o vetor velocidade, podemos obter o vetor<br />

30


Capítulo 3. Translações<br />

3.2. As forças da natureza<br />

posição. Faça tudo isto usando componentes (reveja<br />

o Exercício 30 e faça o Exerçício 33).<br />

A força elástica é também uma força de contato,<br />

de origem elétrica, mas não é constante. Ela mede<br />

como um corpo reage a deformações (compressões<br />

ou distensões). Não havendo deformações permanentes,<br />

o corpo reage no sentido de restaurar a deformação<br />

sofrida. Vamos quantificar isto. Seja x a<br />

deformação sofrida ao longo do eixo X. Por simplicidade<br />

vamos considerar que a deformação seja<br />

apenas ao longo do eixo X. Então, experimentos<br />

nos revela que a força restauradora produzida pelo<br />

corpo em resposta a esta deformação (pequena) é<br />

⃗F (x) = −k x î, (3.13)<br />

na qual k é uma constante característica do material<br />

(independente de sua massa) e é determinada<br />

experimentalmente. Esta força é conhecida como<br />

lei de Hooke. Assim, usando a segunda lei de Newton<br />

(3.11) e a lei de Hooke (3.13), devemos encontrar<br />

uma função x(t) cuja derivada segunda seja<br />

proporcional a ela mesma,<br />

ou<br />

d 2<br />

− k x = m d2<br />

x, (3.14)<br />

dt2 dt 2 x(t) = − k x(t). (3.15)<br />

m<br />

A solução desta equação diferencial é discutida no<br />

Exercício 34.<br />

Vejamos mais alguns exemplos importantes.<br />

Sim, podemos ver agora alguns exemplos de forças<br />

que não são forças de contato. Por exemplo, as<br />

forças elétrica e magnética. Coulomb observou que<br />

a força elétrica entre duas cargas elétricas Q e q é<br />

Qq<br />

⃗F e = C e<br />

r 2 ˆr = C Qq<br />

e ⃗r, (3.16)<br />

r3 na qual ˆr é um versor ao longo da reta que une<br />

as duas cargas (veja a Figura 3.1). Lembre-se que<br />

cargas elétricas podem ser positivas ou negativas.<br />

Q<br />

ˆr<br />

Figura 3.1: Duas cargas elétricas exercem uma<br />

força entre elas, como foi observado primeiramente<br />

por Coulomb.<br />

r<br />

⃗ Fe<br />

q<br />

Para simplificar, podemos colocar uma das duas<br />

cargas na origem, digamos Q. Então a distância r<br />

entre elas é r = √ x 2 + y 2 + z 2 , sendo ⃗r = (x, y, z)<br />

o vetor posição da outra carga q. Se a carga Q<br />

permanecer fixa na origem e se a carga q for suficientemente<br />

pequena (infinitesimalmente pequena)<br />

para não perturbar Q, então podemos usar a segunda<br />

lei para determinar o vetor posição de q em<br />

função do tempo. Nestas condições, esta carga q, a<br />

única carga em movimento, é denominada de carga<br />

de prova (ou teste). Neste caso, temos uma carga<br />

de prova q movimentando-se numa região onde ela<br />

sente uma força elétrica criada pela carga Q (denominada<br />

de fonte).<br />

Para não termos de falar sobre a fonte ao determinarmos<br />

o movimento da carga de prova, podemos<br />

introduzir um novo vetor,<br />

Q<br />

⃗E = C e ˆr, (3.17)<br />

r2 denominado de campo elétrico, tal que ⃗ F e = q ⃗ E.<br />

De forma similar, podemos ter a presença de um<br />

vetor campo magnético ⃗ B, o qual também produz<br />

uma força em um corpo com carga elétrica q<br />

movimentando-se com velocidade ⃗v,<br />

⃗F m = q ⃗v × ⃗ B, (3.18)<br />

na qual o campo magnético pode depender da<br />

posição e do tempo. Esta força é conhecida como<br />

força de Lorentz. Portanto, em geral, o movimento<br />

de uma carga q será determinado pela segunda lei<br />

de Newton (3.11),<br />

⃗F R = q ⃗ E ( ⃗r, t ) + q ˙⃗r × ⃗ B ( ⃗r, t ) = m¨⃗r. (3.19)<br />

Faça o Exercício35.<br />

Estas duas forças, elétrica e magnética, estão presentes<br />

em quase tudo ao nosso redor, como em tubos<br />

de televisores, equipamentos médicos, aceleradores<br />

de partículas, câmaras para resfriamento de<br />

átomos e moléculas, etc. Também vale ressaltar<br />

que estas duas forças atuam a distância, isto é, não<br />

precisam de contato. Outra observação importante<br />

(ou intrigante?): até o presente, ainda não observamos<br />

a presença de cargas magnéticas na nossa<br />

natureza.<br />

Outra força que estamos talvez ainda mais habituados<br />

é a força gravitacional. Ela é muito parecida<br />

com a força elétrica entre duas cargas elétricas.<br />

31


3.2. As forças da natureza Capítulo 3. Translações<br />

São tão parecidas que podemos usar a mesma Figura<br />

3.1, trocando as cargas elétricas por cargas<br />

gravitacionais, as quais serão denotadas por M g e<br />

m g . Newton observou que a força entre as cargas<br />

gravitacionais M g e m g é dada por<br />

⃗F g = −C g<br />

M g m g<br />

r 2 ˆr = m g<br />

⃗ Eg . (3.20)<br />

Em (3.20), da mesma forma que a carga Q é a<br />

fonte do campo elétrico E ⃗ em (3.17), M g é a fonte<br />

do campo gravitacional Eg ⃗ . Estas cargas gravitacionais<br />

não devem ser confundidas com as cargas<br />

inerciais introduzidas no Princípio 2. De fato,<br />

as constantes m em (3.5) e em (3.10) são cargas<br />

inerciais, pois elas nos permitem relacionar forças<br />

que não são de natureza eletromagnética ou gravitacional<br />

com aceleração. Para sermos completos,<br />

também devemos tomar cuidados para não confundirmos<br />

cargas elétricas com cargas inerciais e gravitacionais.<br />

Outra observação importante: a expressão (3.20)<br />

é válida somente para uma fonte com o formato<br />

esférico de raio R, com a distância r > R medida<br />

desde o centro da fonte até a carga gravitacional<br />

de prova m g . Por exemplo, podemos considerar<br />

a Terra como esférica com uma boa aproximação.<br />

Neste caso, fazendo r = R, com R sendo o raio<br />

médio da Terra, o campo gravitacional E ⃗ g (R) terá o<br />

seu módulo constante na superfície da Terra (onde<br />

nos estamos neste momento). Vamos agora levar<br />

esta situação para a segunda lei de Newton (3.11),<br />

m g<br />

⃗ Eg (R) = m I ⃗a, (3.21)<br />

na qual fizemos m = m I para ressaltar o caráter<br />

inercial da massa m, como definido no Princípio 2.<br />

Agora vem a parte mais interessante. Tanto o valor<br />

do campo gravitacional na superfície da Terra,<br />

⃗E g (R), quanto a aceleração de uma carga gravitacional<br />

m g com uma massa inercial m I , colocada<br />

próxima à superfície da Terra, digamos em cima<br />

do nosso edifício central conhecido como E1, podem<br />

ser medidas independentemente. Surpreendentemente,<br />

resulta que ⃗a = E ⃗ g (R). Isto significa<br />

duas coisas: (1) que a carga gravitacional<br />

é igual, dentro da precisão experimental,<br />

à carga inercial, m g = m I = m e (2) todos<br />

os corpos caem com a mesma aceleração<br />

na superfície da Terra. Esta igualdade entre<br />

massa gravitacional e massa inercial foi conjecturada<br />

por Galileu, verificada por Newton e muitos<br />

outros desde então. A precisão deste experimento<br />

chega hoje a uma parte em 10 16 .<br />

Esta igualdade entre massa inercial (capaz de<br />

produzir uma quantidade de movimento) e massa<br />

gravitacional (capaz de produzir e sentir uma força<br />

gravitacional) implica que todos os corpos são<br />

atraídos pela Terra com a mesma aceleração, independentemente<br />

de suas massas (gravitacionais ou<br />

inerciais). Assim, no vácuo, uma bolinha de tênis<br />

e o porta-aviões Minas Gerais gastam o mesmo<br />

tempo para percorrerem a mesma distância em<br />

direção ao centro da Terra (queda livre). Este fato<br />

também está entre os mais marcantes acerca da<br />

nossa natureza. Vale mencionar também que não<br />

há uma igualdade entre massa inercial (ou gravitacional)<br />

e carga elétrica. Também não há cargas<br />

magnéticas na nossa natureza, ou pelo menos nunca<br />

foram vistas.<br />

Note também que a igualdade m g = m I = m<br />

implica que o campo gravitacional Eg ⃗ tenha dimensões<br />

de aceleração. Na superfície da Terra, o<br />

seu módulo é E g (R) = g = 9.81 m/s 2 . Mais uma<br />

observação igualmente intrigante: no caso da força<br />

gravitacional, por mais estranho que possa parecer,<br />

é a aceleração E g que é conhecida em primeira mão.<br />

Faça os Exercícios 36 e 37.<br />

Falando em forças, quais são as forças conhecidas<br />

na nossa natureza? A força gravitacional<br />

com certeza é a mais popular. Na física Newtoniana,<br />

ela é responsável pela atração entre duas<br />

massas. Ela atua a distância e de forma instantânea.<br />

No entanto, ela, segundo a Relatividade<br />

Geral, não é exatamente uma força: ela é o<br />

efeito de estarmos em movimento em um espaçotempo<br />

curvo. Falaremos mais sobre isto mais adiante.<br />

Temos também a força eletromagnética.<br />

Ela também atua a distância mas, diferentemente<br />

da força gravitacional Newtoniana, ela não age<br />

instantaneamente, pois o efeito de uma carga sobre<br />

uma outra, colocadas no vácuo, propaga-se<br />

na velocidade da luz. A força eletromagnética<br />

é responsável tanto por fenômenos macroscópicos<br />

quanto microscópicos. Ela é a única força que sobrevive<br />

nestes dois mundos.<br />

Estas duas primeiras forças são de longo alcance,<br />

ou seja, a dependência com o inverso do<br />

quadrado da distância possibilita que elas sejam<br />

32


Capítulo 3. Translações<br />

3.2. As forças da natureza<br />

sentidas mesmo quando os corpos estão separados<br />

por uma distância considerável. Vejamos algumas<br />

distâncias interessantes (em ordens de grandeza).<br />

A distância média entre a Terra e o Sol é de aproximadamente<br />

10 11 m, uma distância denominada<br />

de “unidade astronômica”. O diâmetro do Sol é<br />

1.3 × 10 9 m. A distância Terra-Lua é 3.8 × 10 8 m.<br />

O diâmetro da Lua é 1/4 do diâmetro da Terra<br />

que é 1.28 × 10 7 m. Portanto, a razão entre os<br />

diâmetros do Sol e da Terra é cerca de 111. A razão<br />

entre a distância Terra-Sol e o diâmetro do Sol é<br />

da ordem de 100. Façamos estas mesmas análises<br />

para o átomo de hidrogênio. A distância elétronpróton<br />

no átomo de hidrogênio é aproximadamente<br />

10 −15 m. O diâmetro do próton é da ordem de<br />

10 −10 m. Portanto, temos agora 10 5 para a razão<br />

entre a distância elétron-próton pelo diâmetro do<br />

próton. Note que temos muito mais (proporcionalmente)<br />

espaço vazio no interior do átomo de hidrogênio<br />

do que no espaço entre o Sol e a Terra.<br />

Além das forças gravitacional e eletromagnética,<br />

temos mais duas outras e somente duas: a força<br />

forte e a força fraca. A força forte mantém os<br />

prótons e nêutrons unidos no interior do núcleo<br />

atômico. Ela atua a distância e é de curto alcance,<br />

restringindo-se ao interior do núcleo (cerca<br />

de 10 −15 m). A força fraca atua entre partículas<br />

elementares confinadas em regiões muito pequenas,<br />

como aquelas que existem no interior de prótons e<br />

nêutrons (numa região menor que 10 −15 m). Ela<br />

também é de curto alcance. Ela é responsável por<br />

alguns processos radiotivos como o decaimento beta<br />

(um núcleo do átomo de lítio).<br />

É muito interessante fazermos uma comparação<br />

entre as intensidades destas quatro forças fundamentais.<br />

Qual delas você acha que tem a maior intensidade<br />

relativa? Vamos estabelecer intensidade<br />

igual a 1 para a força nuclear forte. Então a força<br />

nuclear fraca terá uma intensidade relativa de 10 −5 .<br />

A força eletromagnética terá uma intensidade relativa<br />

de 10 −2 . A força gravitacional, surpreendentemente,<br />

terá uma intensidade relativa de 10 −39 .<br />

Faça o Exerçicio 38.<br />

3.2.1 Exercícios<br />

Exercício 33<br />

Verifique que<br />

⃗r(t) = ⃗c + ⃗v t + 1 2 ⃗a t2 , (3.22)<br />

com ⃗a sendo o vetor aceleração, ⃗v sendo o vetor<br />

velocidade e ⃗r 0 sendo a posição inicial em t = 0,<br />

todos constantes, é uma solução da equação diferencial<br />

(3.12). Reveja o Exercício 30.<br />

Exercício 34<br />

Verifique que<br />

x(t) = x 0 cos ( ωt + ϕ ) , ω 2 = k m , (3.23)<br />

com ϕ sendo uma constante, é uma solução da<br />

equação diferencial (3.15). Qual é a dimensão da<br />

constante ω (consulte o Apêndice A)? A constante<br />

ϕ é denominada de constante de fase, enquanto que<br />

θ(t) = ωt + ϕ é a fase. A constante ω é denominada<br />

de freqüência angular ou velocidade angular da<br />

fase θ(t), pois ω = ˙θ. Invente condições iniciais e<br />

desenhe a equação horária x(t).<br />

Exercício 35<br />

Considere ⃗ E = 0 e ⃗ B = B 0 ˆk, com B0 constante.<br />

Determine de (3.19) as equações diferenciais (decorrentes<br />

da segunda lei de Newton) que controlam o<br />

movimento de um corpo com uma carga elétrica q e<br />

uma massa m na presença de um campo magnético<br />

constante. Veremos mais tarde que a trajetória correspondente<br />

é uma circunferência e o movimento<br />

será (circular) uniforme.<br />

Exercício 36<br />

Refaça o Exercício 33 considerando que F ⃗ 0 =<br />

mE ⃗ g (R) = −mg ˆk, com E ⃗ g sendo o campo gravitacional<br />

definido em (3.20) e R o raio médio da<br />

Terra. Compare seus resultados com o movimento<br />

uniformemente variado que você já conhece.<br />

Exercício 37<br />

Use R = 6.37 × 10 6 m e M = 5.98 × 10 24 kg como<br />

o raio médio e a massa da Terra para calcular o<br />

valor do campo gravitacional (definido em (3.20))<br />

na superfície da Terra. Faça a mesma coisa para o<br />

Sol (R = 6.96 × 10 8 m e M = 1.99 × 10 30 kg), para<br />

a Lua (R = 1.74 × 10 6 m e M = 7.35 × 10 22 kg) e<br />

para uma estrela de nêutrons (R = 1.50 × 10 4 m e<br />

M = 2.80 × 10 30 kg). Compare os valores.<br />

Exercício 38<br />

Calcule as forças elétrica (3.16) e gravitacional<br />

(3.20) entre um próton (1.67 × 10 −27 kg) e um<br />

elétron (9.11 × 10 −31 kg) no átomo de hidrogênio<br />

33


3.3. Aplicações da segunda lei de Newton Capítulo 3. Translações<br />

(r = 5 × 10 −11 m). Use C e = 8.99 × 10 9 N·s 2 /C 2 e<br />

C g = 6.67 × 10 −11 N·m 2 /kg 2 . Compare os valores.<br />

3.3 Aplicações da segunda lei<br />

de Newton<br />

Antes de mais nada, é importante ficar claro que<br />

para haver interação é necessário haver dois ou mais<br />

corpos e que esta interação é representada matematicamente<br />

pelo vetor “força”. A relação entre<br />

força e movimento é caracterizada pela segunda lei<br />

de Newton. A seguir vamos considerar algumas<br />

aplicações importantes da segunda lei de Newton<br />

(ou equação de Newton) (3.11). Nestas aplicações<br />

queremos enfatizar a importância de uma equação<br />

diferencial para compreendermos a dinâmica proveniente<br />

da interação entre dois ou mais corpos.<br />

Não apresentaremos técnicas sofisticadas para a resolução<br />

de equações diferenciais por falta de espaço.<br />

Entretanto, procuraremos desenvolver nossa habilidade<br />

em adivinhar a solução de uma determinada<br />

equação diferencial, uma das melhores técnicas.<br />

Além disto, procuraremos usar rotinas em computação<br />

algébrica para resolver nossas equações diferenciais.<br />

Encare isto, por enquanto, como a utilização<br />

de uma calculadora para efetuar operações<br />

aritméticas e outros cálculos envolvendo funções<br />

transcendentais como funções trigonométricas, exponenciais<br />

e logarítmicas.<br />

Procuraremos desenvolver nossas aplicações em<br />

condições ideais (no vácuo) e também, sempre que<br />

possível, na presença de forças dissipativas como<br />

atrito e resistência de um fluido (gases e líquidos).<br />

Ao introduzirmos estas forças dissipativas estaremos<br />

trabalhando com um dos procedimentos mais<br />

frutíferos em ciência: a idéia da elaboração de modelos.<br />

Certamente uma descrição detalhada destas<br />

forças dissipativas seria muito difícil ou mesmo<br />

impossível. Imagine como deve ser o mecanismo<br />

de ação de uma força de atrito, a qual resulta de<br />

interações eletromagnéticas entre duas superfícies<br />

em contato. Felizmente fomos capazes, enquanto<br />

seres inteligentes, de superar tais dificuldades aparentes.<br />

Criamos a idéia do modelo. Um modelo<br />

não procura reproduzir todos os detalhes de<br />

um determinado fenômeno, ao contrário, ele procura<br />

incorporar apenas as características mais relevantes.<br />

Em compensação, haverá constantes arbitrárias<br />

nestes modelos que deverão ser determinadas<br />

através de experimentos cuidadosamente preparados.<br />

Por exemplo, na lei de Hooke para a deformação<br />

elástica, há a constante elástica (ou constante<br />

de mola) que precisa ser conhecida para cada<br />

material antes que ela (a lei de Hooke) possa ser<br />

usada.<br />

Também é fundamental que você se envolva profundamente,<br />

a fim de procurar entender todos os aspectos<br />

discutidos nestas aplicações, fazendo e refazendo<br />

os trabalhos manuais e computacionais, sempre<br />

com um olhar inquisidor. Melhor que apresentar<br />

um conjunto de exercícios para treiná-lo a fazer<br />

uma boa prova, espero que possamos apresentar um<br />

conjunto de aplicações que ilustrem bem o comportamento<br />

crítico e analítico de um bom profissional.<br />

Estas aplicações serão também fundamentais<br />

para estabelecermos as leis de conservação, nosso<br />

próximo assunto.<br />

3.3.1 Forças constantes<br />

Como já mencionamos, forças constantes, isto é,<br />

independentes da posição e do tempo, são importantes.<br />

As forças em uma determinada estrutura<br />

mecânica são constantes. A força gravitacional na<br />

superfície da Terra é praticamente constante. Vamos<br />

considerar um corpo de massa m constante sujeito<br />

a uma força ⃗ F 0 também constante na ausência<br />

de efeitos dissipativos. De acordo com a segunda<br />

lei (3.11), sendo a força uma constante, então a<br />

aceleração também será uma constante. Portanto,<br />

a equação diferencial que temos de resolver para<br />

encontrarmos a equação horária ⃗r(t) deste corpo é<br />

d 2<br />

dt 2 ⃗r(t) = ⃗a 0, ⃗a 0 = ⃗ F 0<br />

m . (3.24)<br />

Note que estamos usando uma notação vetorial.<br />

Como há três direções independentes, portanto temos<br />

de resolver em geral três equações diferenciais,<br />

d 2<br />

dt 2 x(t) = a x0,<br />

d 2<br />

dt 2 y(t) = a y0,<br />

d 2<br />

dt 2 z(t) = a z0,<br />

a x0 = F x0<br />

m ,<br />

a y0 = F y0<br />

m , (3.25)<br />

a z0 = F z0<br />

m .<br />

Como o lado direito destas equações diferenciais são<br />

constantes, nós temos condições de compreender a<br />

34


Capítulo 3. Translações<br />

3.3. Aplicações da segunda lei de Newton<br />

técnica de solução deste tipo de equação diferencial.<br />

Que tipo de equação diferencial temos aqui?<br />

Primeiro um pouco de nomenclatura: x(t), y(t)<br />

e z(t) são funções de t, ou seja, elas são as variáveis<br />

dependentes. O parâmetro t, nosso tempo, é a<br />

variável independente. Então o tipo de equação diferencial<br />

que estamos tratando aqui pode ser escrita<br />

na forma onde o lado direito contenha apenas constantes,<br />

como em (3.25), e/ou a variável independente<br />

explicitamente. Este tipo de equação diferencial<br />

deve ser resolvido por integrações (também<br />

denominadas de quadraturas neste caso específico).<br />

Observe com atenção o modelo a seguir. Para facilitar<br />

nossa visão, vamos re-escrever as derivadas de<br />

segunda ordem em x, y e z aparecendo em (3.25)<br />

como derivadas de primeira ordem nas respectivas<br />

componentes do vetor velocidade. Então o plano é<br />

determinar primeiro as componentes do vetor velocidade:<br />

d<br />

dt v x(t) = a x0 ⇔ dv x = a x0 dt. (3.26)<br />

As quantidades infinitesimais dv x e dt são denominadas<br />

de diferenciais. Em (3.26), dizemos que os<br />

diferenciais das variáveis dependente (v x ) e independente<br />

(t) estão separados, completamente separados.<br />

Neste caso, podemos integrar independentemente<br />

os dois lados da última expressão em (3.26),<br />

∫ ∫<br />

dv x = a x0 dt<br />

⇒<br />

v x + C 1 = a x0 (t + C 2 )<br />

⇔<br />

v x (t) = a x0 t + C. (3.27)<br />

Note que usamos integrais indefinidas. Note<br />

também a presença das duas constantes de integração<br />

C 1 e C 2 . Naturalmente, como a x0 também é<br />

uma constante, fizemos C = a x0 C 2 −C 1 . Em suma,<br />

determinamos a função v x (t). Resta descobrirmos<br />

quem é a constante arbitrária C. Certamente, sem<br />

uma informação adicional, não temos como descobrir<br />

quem é esta constante. Devemos conhecer o<br />

valor de v x (t) em algum instante particular, por<br />

exemplo no início do movimento. Suponha que o<br />

movimento comece em t = t 0 e que neste instante<br />

inicial sabemos que v x (t 0 ) = v 0x seja conhecida.<br />

Isto é uma condição inicial. Então, substituindo<br />

esta condição inicial na última expressão em (3.27),<br />

teremos<br />

v x (t) = v 0x + a x0 (t − t 0 ). (3.28)<br />

Como a velocidade é a derivada da posição, podemos<br />

repetir o procedimento anterior e determinar<br />

x(t) integrando (3.28),<br />

v x (t) = dx<br />

dt = a x0 t + v 0x − a x0 t 0 ⇒<br />

dx = a x0 tdt + (v 0x − a x0 t 0 )dt. (3.29)<br />

Integrando os dois lados desta última expressão,<br />

temos<br />

∫<br />

dx = a x0<br />

∫<br />

∫<br />

tdt + (v 0x − a x0 t 0 )<br />

dt<br />

⇒<br />

x(t) = 1 2 a x0 t 2 + (v 0x − a x0 t 0 ) t + C. (3.30)<br />

Note que já fizemos o truque de juntar todas as<br />

constantes de integração em uma só. Novamente,<br />

temos que usar uma condição inicial para determinar<br />

a constante arbitrária C. Vamos supor que a<br />

posição no eixo X seja conhecida no instante inicial<br />

t 0 , x 0 = x(t 0 ). Então, após um pouco de álgebra<br />

(faça todos os detalhes), temos (faça o Exercício 39)<br />

x(t) = x 0 + v 0x (t − t 0 ) + 1 2 a x0(t − t 0 ) 2 . (3.31)<br />

Naturalmente, há resultados similares nos demais<br />

eixos (refaça-os):<br />

y(t) = y 0 + v 0y (t − t 0 ) + 1 2 a y0(t − t 0 ) 2 , (3.32)<br />

z(t) = z 0 + v 0z (t − t 0 ) + 1 2 a z0(t − t 0 ) 2 . (3.33)<br />

Podemos até re-escrever estas funções do tempo de<br />

volta numa notação vetorial,<br />

⃗r(t) = ⃗r 0 + ⃗v 0 (t − t 0 ) + 1 2 ⃗a 0 (t − t 0 ) 2 (3.34)<br />

com ⃗r 0 , ⃗v 0 e ⃗a 0 constantes. Conclusão: quando a<br />

força é constante, o movimento é parabólico, isto<br />

é, a sua equação horária é uma função quadrática<br />

(parábola) do tempo.<br />

Um exemplo importante é o movimento de um<br />

objeto de massa m próximo à superfície da Terra,<br />

onde a força gravitacional é praticamente constante,<br />

⃗ F 0 = −mg ˆk, g = 9.81 m/s 2 . Vamos ainda<br />

supor a ausência de qualquer efeito dissipativo.<br />

Note que escolhemos um referencial fixo na Terra<br />

com o eixo Z passando pelos centros de massa da<br />

35


3.3. Aplicações da segunda lei de Newton Capítulo 3. Translações<br />

Terra e do corpo. Neste caso, o nosso vetor aceleração<br />

em (3.24) é ⃗a 0 = −g ˆk. Podemos realizar<br />

dois experimentos interessantes: (1) queda livre<br />

e (2) lançamento oblíquo. Na queda livre, soltamos<br />

o objeto de uma determinada altura h = z 0<br />

(x 0 = y 0 = 0) com velocidade inicial nula. Levando<br />

estas informações na solução vetorial (3.34),<br />

teremos<br />

⃗r(t) =<br />

(h − 1 )<br />

2 g t2 ˆk ⇒<br />

z(t) = h − 1 2 g t2 , x(t) = y(t) = 0. (3.35)<br />

Note que o movimento é vertical. Note também que<br />

o corpo chega ao solo (z = 0) após um certo intervalo<br />

de tempo com uma velocidade diferente de zero<br />

(faça o Exercício 40). Também podemos ver que<br />

o módulo do vetor velocidade é sempre crescente<br />

no tempo (faça o Exercício 40). No lançamento<br />

oblíquo, o corpo é lançado com uma velocidade inicial<br />

diferente de zero. Para simplificar um pouco,<br />

vamos supor t 0 = 0 no instante do lançamento e<br />

que x 0 = y 0 = z 0 = 0 (origem). Então, de (3.34)<br />

temos<br />

⃗r(t) = ⃗v 0 t − 1 2 g t2 ˆk, (3.36)<br />

ou, em termos de componentes,<br />

x(t) = v 0x t,<br />

y(t) = v 0y t,<br />

z(t) = v 0z t − 1 2 g t2 .<br />

(3.37)<br />

Isto significa que o movimento é acelerado apenas<br />

ao longo do eixo Z. Vale observar que a altura<br />

máxima atingida (acima do plano XY ) pode ser determinada<br />

calculando o ponto de máximo da função<br />

z(t),<br />

v z (t) = d dt z(t) = v 0z − g t ∴ v z (t M ) = 0<br />

⇒<br />

t M = v 0z<br />

g , z M = 1 v0z<br />

2<br />

2 g . (3.38)<br />

Note que do ponto de vista de Cálculo, o máximo da<br />

função z(t) é determinado pela condição ż(t M ) = 0,<br />

com o ponto especificando a derivada no tempo, ou<br />

seja, a reta tangente a z(t) em t M deve ser horizontal.<br />

Acontece que do ponto de vista da Mecânica,<br />

ż é justamente a componente ao longo do eixo Z<br />

do vetor velocidade, a qual deve ser nula no ponto<br />

mais alto. Isto é mais uma demostração de que a<br />

base matemática da mecânica está muito bem estabelecida<br />

(faça o Exercício 41).<br />

3.3.2 Forças dependentes da velocidade<br />

Que tal um toque de realismo? Você já deve ter sentido<br />

o quão difícil é caminhar dentro de uma piscina<br />

cheia de água, embora a água esteja tranqüilamente<br />

em repouso. Também podemos notar neste experimento<br />

que é mais difícil correr que caminhar na<br />

água. Na prática, isto significa duas coisas: (1) a<br />

água nos aplica uma força contrária ao nosso movimento,<br />

portanto uma força dissipativa (cujo significado<br />

preciso será entendido mais tarde, após uma<br />

discussão sobre o conceito de energia), e (2) esta<br />

força dissipativa deve ser proporcional à nossa velocidade.<br />

Agora vem a essência do modelo: não importa<br />

o quão complicada seja nossa interação com<br />

a água, o efeito pode ser modelado por uma força<br />

dissipativa proporcional à velocidade,<br />

⃗F d = −b⃗v = −b d ⃗r(t), (3.39)<br />

dt<br />

onde o parâmetro b é uma característica do meio<br />

e deve ser determinado experimentalmente (como<br />

a constante de mola da lei de Hooke). Note que<br />

estamos supondo uma dependência linear na velocidade.<br />

Reconsiderando os exemplos discutidos anteriomente<br />

na Seção 3.3.1, temos agora duas forças<br />

agindo no nosso corpo de testes, a força constante<br />

⃗F 0 e a força dissipativa F ⃗ d definida em (3.39),<br />

⃗F 0 + ⃗ F d = m⃗a. (3.40)<br />

Note que a aceleração não é mais uma constante.<br />

Isto significa que teremos de resolver agora<br />

equações diferenciais ligeiramente mais complicadas.<br />

Este modelo descreve muito bem corpos pequenos<br />

como partículas de poeira no ar ou de corpos<br />

não muito extensos a baixas velocidades como uma<br />

bolinha de gude caindo em um fluido qualquer (gás<br />

ou líquido). Façamos alguns exemplos.<br />

Partícula em suspensão<br />

Primeiro vamos considerar uma situação mais simples:<br />

queda livre ao longo do eixo Z. Na queda<br />

36


Capítulo 3. Translações<br />

3.3. Aplicações da segunda lei de Newton<br />

livre, podemos escolher F ⃗ 0 = −mg ˆk. Portanto, o Veja que interessante: τ é uma quantidade positiva.<br />

v z (t) = −gτ ( 1 − e −t/τ ) , τ = m microscópicos a interação entre um corpo em movimento<br />

e o fluido no qual ele está imerso.<br />

b . (3.47)<br />

movimento é unidimensional, ao longo do eixo Z,<br />

e a equação diferencial (3.40) pode ser re-escrita<br />

como (faça o Exercício 42)<br />

Então, após um intervalo de tempo maior que<br />

τ a velocidade em (3.47) é praticamente igual −gτ,<br />

ou seja, independente do tempo, pois a exponential<br />

− mg − bż = m¨z ⇒<br />

decresce rapidamente a zero. O módulo deste valor<br />

constante para a velocidade é conhecido como<br />

¨z + 1 τ ż + g = 0, τ = m b . (3.41)<br />

“velocidade terminal”,<br />

V T = gτ = gm b . Neste caso, não podemos mais usar a técnica de<br />

(3.48)<br />

separação de variáveis, pois aparece uma derivada Faça um gráfico da velocidade (3.47). Note também<br />

primeira e uma derivada segunda. No entanto, que se a velocidade terminal (3.48) for substituída<br />

podemos tentar determinar primeiro a velocidade<br />

v z (t) = ż ao longo do eixo Z,<br />

na segunda lei (3.41) (faça ż = V T ), a aceleração<br />

torna-se nula. Isto significa que a velocidade terminal<br />

é atingida quando a força dissipativa, a<br />

˙v z + 1 τ v z + g = 0, v z = ż. (3.42)<br />

qual está aumentando sua intensidade proporcionalmente<br />

à velocidade, iguala à força gravitacional.<br />

Atingida a velocidade terminal, não tem mais<br />

Esta equação pode ser resolvida através de uma<br />

mudança da variável dependente v z para u =<br />

aceleração. Experimente em casa usando uma pequena<br />

bolinha em queda livre em um líquido qual-<br />

τ −1 v z + g. Assim,<br />

quer. Talvez seja necessário usar óleo para observar<br />

u = 1 τ v z + g, ˙v z = τ ˙u. (3.43)<br />

o efeito da velocidade terminal. E a equação<br />

horária deste movimento? Podemos usar a velocidade<br />

Levando estes resultados de volta em (3.42), teremos<br />

(3.47) para determinarmos z(t),<br />

˙u + 1 v<br />

u = 0. (3.44) z (t) = dz<br />

dt = −gτ( 1 − e −t/τ )<br />

⇒<br />

τ<br />

dz = −gτ ( 1 − e −t/τ ) dt. (3.49)<br />

Agora esta equação diferencial pode ser resolvida<br />

pela técnica de separação de variáveis,<br />

Integrando independentemente os dois lados desta<br />

última expressão, obteremos (faça o Exercício 43)<br />

du<br />

dt + 1 τ u = 0 ⇔ du u = − 1 τ dt<br />

z(t) = −gτ ( C + t + τe −t/τ ) . (3.50)<br />

⇒ ln u = − t τ + C′ . (3.45) Vamos supor que o corpo seja solto de uma altura<br />

h em t = 0, z(0) = h. Então, usando esta condição<br />

Invertendo o logaritmo na última expressão em<br />

(3.45), determinamos a função intermediária u(t)<br />

inicial em (3.50) determinamos C = −τ − h/(gτ)<br />

(faça o Exercício 43),<br />

e depois, usando a última expressão em (3.43), determinamos<br />

v z (t),<br />

z(t) = h − gτ t + gτ 2( 1 − e −t/τ ) . (3.51)<br />

Note que para um tempo t > τ, ou seja, após o<br />

ln u = − t τ + C′ ⇒ u(t) = Ce −t/τ<br />

corpo ter alcançado sua velocidade terminal, esta<br />

equação horária é praticamente linear no tempo,<br />

⇒ v z (t) = Cτe −t/τ − gτ. (3.46) como deve ser, pois não há mais aceleração. Estas<br />

observações que fizemos são verificadas na prática<br />

A constante de integração C é determinada, como e confirmam a validade do modelo que foi feito.<br />

sempre, pelas condições iniciais. Na queda livre, De novo, devemos frisar a importância do modelo<br />

podemos supor v z (0) = 0. Esta condição inicial para ultrapassarmos a enorme barreira apresentada<br />

pela tentativa de entendermos em nos fornece C = g (faça o Exercício 42). Assim,<br />

detalhes<br />

37


3.3. Aplicações da segunda lei de Newton Capítulo 3. Translações<br />

Para-quedas<br />

Vale a pena fazermos duas observações: (1) em<br />

√ g e 2t/τ − 1<br />

v z (t) = ż(t) = −<br />

η e 2t/τ + 1 . (3.57) Da definição de forçaSegunda Lei de Newton, F ⃗ B =<br />

m¨⃗r, teremos as seguintes equações diferenciais (faça<br />

ambos os casos de queda livre na presença da atmosfera,<br />

o empuxo não foi levado em consideração.<br />

Nosso primeiro modelo descreve bem corpos pequenos<br />

com velocidades baixas. E corpos extensos<br />

Como será visto em nossos estudos com fluidos, o<br />

como um pára-quedas em queda livre? Neste caso,<br />

empuxo faz com que o “peso aparente” torne-se um<br />

também um movimento unidimensional, o modelo<br />

pouco menor. (2) Até aqui, supusemos uma densidade<br />

constante ρ = ρ<br />

que funciona melhor tem uma força dissipativa com<br />

uma dependência quadrática com a velocidade de<br />

0 . No entanto, a densidade<br />

atmosférica varia com a altura z. Em um modelo<br />

queda,<br />

⃗F δ = 1 2 ρδA v2 ˆk,<br />

simplificado podemos escrever<br />

z (3.52)<br />

ρ(z) = ρ<br />

na qual ρ é a densidade do ar (ou de um fluido qualquer),<br />

0 e −z/H , (3.58)<br />

δ é uma grandeza adimensional, denominada<br />

com ρ<br />

de coeficiente de arrasto, relacionada com a forma<br />

0 = 1.25 kg/m 3 no nível do mar e H =<br />

8 × 10<br />

geométrica e A é a área da seção reta do corpo em<br />

m. Levando esta dependência da densidade<br />

com a altura z(t), a equação diferencial (3.53)<br />

queda livre. O coeficiente de arrasto para corpos<br />

torna-se em<br />

arrendondados é 0.5, aproximadamente. Note em<br />

(3.52) que a força é dissipativa, isto é, contrária<br />

ao sentido do vetor velocidade. Neste caso, devemos<br />

resolver uma equação diferencial um pouquinho<br />

¨z − η ż 2 e −z/H + g = 0, η = ρ 0δA<br />

2m . (3.59)<br />

mais complicada. Fazendo F ⃗ 0 = −mgˆk, temos<br />

de resolver<br />

Neste caso, não temos uma solução analítica conhecida<br />

como (3.56). Nossa única opção é fazer uso de<br />

⃗F 0 + F ⃗ δ = m⃗a ⇒<br />

técnicas numéricas para resolver a equação diferencial<br />

(3.59). Note também que a velocidade terminal<br />

¨z − η ż 2 + g = 0, η = ρδA<br />

2m . (3.53) (¨z = 0) não é mais uma constante,<br />

√ √ (<br />

g g<br />

Neste caso, também não podemos usar diretamente V T =<br />

o método da separação de variáveis porque temos<br />

η ez/2H ≃ 1 + z )<br />

, (3.60)<br />

η 2H<br />

derivadas de ordens diferentes. Também não ajuda<br />

muito tentar resolver primeiro a equação diferencial<br />

na qual estamos supondo z ≪ H.<br />

para a velocidade,<br />

˙v z = −g + η vz, 2 v z = ż, (3.54)<br />

Partícula carregada em um campo eletromagnético<br />

pois não podemos imaginar uma mudança experta<br />

na variável dependente que nos possibilite usar o Vejamos mais um exemplo importante no qual a<br />

método da separação de variáveis. No entanto, podemos<br />

usar a equação diferencial (3.54) para deter-<br />

um corpo de massa m e carga Q no vácuo. Supo-<br />

força também depende da velocidade. Considere<br />

minarmos a velocidade terminal,<br />

nha também que no instante inicial t 0 = 0 ele esteja<br />

√ na posição inicial ⃗r 0 = (x 0 , y 0 , z 0 ) e com velocidade<br />

g<br />

V T =<br />

η . (3.55) inicial ⃗v 0 = (v 0x , v 0y , v 0z ), em relação a um sistema<br />

de coordenadas ortonormal (fixo). Suponha<br />

também que um determinado campo magnético,<br />

A solução da equação diferencial em (3.53) é<br />

z(t) = 1 [<br />

hη + ln(2) + t ]<br />

⃗B = (0, 0, B 0 ), com B 0 constante, esteja presente.<br />

η<br />

τ − ln(1 + Então, este corpo sentirá o efeito da força de Lorentz,<br />

e2t/τ ) , (3.56)<br />

com τ = 1/ √ gη e η = ρδA/2m. A derivada primeira<br />

desta função nos dá a velocidade instantânea,<br />

⃗F B = Q⃗v × B ⃗ = Q ( v y B 0 , −v x B 0 , 0 ) . (3.61)<br />

38


Capítulo 3. Translações<br />

3.3. Aplicações da segunda lei de Newton<br />

o Exercício 43),<br />

mẍ = +QB 0 ẏ ⇒ ẍ = ωẏ, ω = QB 0<br />

m (3.62)<br />

mÿ = −QB 0 ẋ ⇒ ÿ = −ωẋ, (3.63)<br />

m¨z = 0 ⇒ ¨z = 0. (3.64)<br />

Note que temos uma novidade: as equações diferenciais<br />

(3.62) e (3.63) estão acopladas, isto é, as<br />

derivadas das variáveis dependentes x(t) e y(t) aparecem<br />

misturadas nestas duas equações. No entanto,<br />

esta mistura é linear nestas derivadas. Esta<br />

linearidade faz com que possamos resolver este sistemas<br />

de duas equações diferenciais de uma forma<br />

muito simples. Derive a equação diferencial (3.62)<br />

em relação ao tempo e use a equação diferencial<br />

(3.63) para eliminar ÿ. Depois introduza a velocidade<br />

v x = ẋ,<br />

¨v x = −ω 2 v x , v x = ẋ. (3.65)<br />

Note que esta equação diferencial (EDO) para v x (t)<br />

tem a mesma forma da EDO do sistema massamola<br />

(oscilador harmônico) apresentada em (3.15)<br />

com ω 2 = k/m (reveja o Exercício 34). Conseqüentemente,<br />

a EDO (3.65) para v x pode ser resolvida<br />

facilmente respondendo a pergunta “Quem é a<br />

função cuja derivada sengunda é proporcional a ela<br />

mesma?” Conhecemos duas candidatas: funções<br />

trigonométricas (senos e cossenos) e a função exponencial.<br />

Vamos usar um seno na forma<br />

v x (t) = −ωA sin(ωt + ϕ), (3.66)<br />

com A (amplitude) e ϕ (fase) constantes, as<br />

quais devem ser determinadas com o auxílio das<br />

condições iniciais. Note que esta solução também<br />

pode ser escrita na forma (faça o Exercício 49)<br />

v x (t) =<br />

− ωA ( cos(ϕ) sin(ωt) + sin(ϕ) cos(ωt) )<br />

= C 1 sin(ωt) + C 2 cos(ωt). (3.67)<br />

É importante observar também que: (1) a solução<br />

de uma equação diferencial de segunda ordem sempre<br />

terá duas constantes arbitrárias e (2) se duas<br />

funções independentes satisfazem a mesma equação<br />

diferencial, então a solução mais geral será uma<br />

combinação linear delas. A solução (3.67) é uma<br />

combinação linear de duas soluções independentes.<br />

E a função exponencial? Ela não é uma terceira<br />

solução? Não, ela é de fato equivalente à solução<br />

(3.67) (faça o Exercício 49). Tendo determinado<br />

v x (t), podemos usar (3.62) para determinar v y (t),<br />

v y (t) = 1 ω ˙v x = −ωA cos(ωt + ϕ). (3.68)<br />

O próximo passo é integrar as velocidades (3.66)<br />

e (3.68) para obtermos as respectivas equações<br />

horárias:<br />

x(t) = A cos(ωt + ϕ) + C 1 , (3.69)<br />

y(t) = −A sin(ωt + ϕ) + C 2 , (3.70)<br />

z(t) = C 3 + C 4 t, (3.71)<br />

onde aproveitamos também para resolver a equação<br />

diferencial (3.64). Todas as seis constantes aparecendo<br />

em (3.69)–(3.71) devem ser determinadas a<br />

partir de seis condições iniciais,<br />

⃗r(0) = ( x 0 , y 0 , z 0<br />

)<br />

, ⃗v(0) =<br />

(<br />

vx0 , v y0 , v z0<br />

)<br />

. (3.72)<br />

Usando estas informações, teremos (faça o<br />

Exercício 50)<br />

√<br />

tan ϕ = v x0 vx0 2 , A =<br />

+ v2 y0<br />

v y0 ω 2 (3.73)<br />

e<br />

C 1 = x 0 − A cos ϕ, C 2 = y 0 + A sin ϕ,<br />

C 3 = z 0 , C 4 = v z0 . (3.74)<br />

Qual é a trajetória determinada pelas equações<br />

horárias (3.69)–(3.71)? Podemos notar que no<br />

plano XY , a trajetória é uma circunferência de raio<br />

A, centrada em (C 1 , C 2 ), pois<br />

(x − C 1 ) 2 + (y − C 2 ) 2 = A 2 . (3.75)<br />

Portanto, temos duas possibilidades: (1) se a velocidade<br />

inicial no eixo Z for nula, v z0 = 0, então<br />

a trajetória será uma circunferência no plano XY<br />

(faça o Exercício 51); (2) se a velocidade inicial no<br />

eixo Z não for nula, então a trajetória será uma<br />

hélice de base circular e passo uniforme ao longo<br />

do eixo Z. Note o seguinte: no caso da trajetória<br />

ser circular, teremos<br />

A = v ω<br />

⇒ Q m = v<br />

AB 0<br />

, (3.76)<br />

39


3.3. Aplicações da segunda lei de Newton Capítulo 3. Translações<br />

√<br />

com v = vx0 2 + v2 y0 . Também fizemos uso da definição<br />

da freqüência ω dada em (3.62). Note que<br />

a última expressão em (3.76) nos permite calcular<br />

a razão entre a carga e a massa de um elétron, por<br />

exemplo, em função do raio A da trajetória, o qual<br />

é determinado pela velocidade inicial e pelo valor<br />

do campo magnético.<br />

3.3.3 Forças dependentes da posição<br />

k<br />

k<br />

0 x<br />

Figura 3.2: Uma massa m é presa a uma mola de<br />

constante k e deformada por x.<br />

Forças dependentes da posição constituem outra<br />

classe importante. Iremos considerar aqui dois sistemas<br />

importantes sob a ação de forças dependentes<br />

da posição: uma massa presa a uma mola e um<br />

pêndulo simples.<br />

Oscilador Harmônico<br />

Considere uma mola como sendo um corpo fixo capaz<br />

de reagir a pequenas deformações oferecendo<br />

uma força restauradora linearmente proporcional à<br />

deformação ∆⃗r (lei de Hooke),<br />

⃗F = −k x ∆x î − k y ∆yˆȷ − k z ∆z ˆk, (3.77)<br />

na qual (k x , k y , k z ) são constantes características do<br />

corpo, denominadas de constantes de mola. Considere<br />

agora um corpo de massa m preso a esta mola.<br />

Despreze a ação de quaisquer outras forças. Situação<br />

ideal. Suponha também que este corpo produza<br />

uma deformação na mola em algum instante<br />

inicial e que permaneça preso à mola nos instantes<br />

posteriores. Como será o movimento do corpo de<br />

massa m? Para facilitar um pouco, vamos supor<br />

que a deformação na mola ocorra apenas ao longo<br />

do eixo X, k x = k, k y = k z = 0, e que a origem<br />

F<br />

m<br />

do sistema de coordenadas coincida com a extremidade<br />

da mola não-deformada, assim ∆x = x (veja a<br />

Figura 3.2). Portanto, da segunda lei (já re-escrita<br />

em componentes) temos<br />

da qual resulta a EDO<br />

− k x = mẍ, (3.78)<br />

ẍ + ω 2 0x = 0, ω 0 =<br />

√<br />

k<br />

m . (3.79)<br />

Esta EDO aparece em muitas situações e, por isso,<br />

é uma das equações diferenciais fundamentais em<br />

ciências. Ela é denominada de EDO do oscilador<br />

harmônico.<br />

Como a equação diferencial (3.79) tem a mesma<br />

estrutura da equação diferencial (3.65), então<br />

x(t) = A cos(ω 0 t + ϕ), ω 0 =<br />

√<br />

k<br />

m , (3.80)<br />

com A e ϕ sendo duas constantes arbitrárias,<br />

as quais serão determinadas de acordo com as<br />

condições iniciais dadas (posição e velocidade iniciais).<br />

Note que a massa m oscila harmonicamente (a<br />

função cosseno é uma função harmônica de período<br />

2π) entre −A e A, indefinidamente (veja a Figura<br />

3.3(a)), com o período 2π/ω 0 . Por isto, este<br />

sistema massa-mola é também conhecido por oscilador<br />

harmônico. A constante ω 0 é a freqüência<br />

(angular) natural 1 deste oscilador harmônico. A<br />

constante A é denominada de amplitude e a constante<br />

ϕ é denominada de constante de fase. A fase é<br />

ω 0 t+ϕ = θ(t). Dado as condições iniciais x(0) = x 0<br />

e ẋ(0) = ẋ 0 , a amplitude e a fase ficam determinadas<br />

(faça o Exercício 51),<br />

√<br />

ẋ0<br />

tan ϕ = − , A =<br />

ω 0 x 0<br />

x 2 0 + (<br />

ẋ0<br />

ω 0<br />

) 2<br />

. (3.81)<br />

A Figura 3.3 mostra em 3.3(a) a equação horária<br />

(3.80) com A = 1, ϕ = 0 e ω 0 = 2π (unidades<br />

arbitrárias) juntamente com a velocidade correspondente,<br />

ẋ(t). Você deve interpretar esta Figura<br />

3.3(a) em termos da força elástica, acompanhando<br />

o gráfico da equação horária e da velocidade<br />

e imaginando como é a ação da força elástica<br />

1 Há dois tipos de freqüência em um movimento periódico:<br />

uma freqüência que é o inverso do período, f = 1/P, medida<br />

em Hertz (Hz), e a freqüência angular, ω = 2πf, medida em<br />

rad/s. O movimento periódico de um determinado sistema<br />

é harmônico quando a sua freqüência angular for igual a<br />

freqüência angular natural deste sistema.<br />

40


Capítulo 3. Translações<br />

3.3. Aplicações da segunda lei de Newton<br />

em cada instante. A Figura 3.3(b) exibe a curva<br />

(<br />

x(t), ẋ(t), 0<br />

)<br />

, parametrizada pelo tempo. Este<br />

plano x × ẋ é denominado de espaço de fase. No<br />

caso do oscilador harmônico, podemos mostrar explicitamente<br />

que esta curva é uma elipse, com o<br />

semi-eixo maior a = 1 e semi-eixo menor b = ω 0<br />

(faça o Exercício 52). Esta informação geométrica,<br />

que a curva no espaço de fase é uma elipse, pode ser<br />

usada para caracterizar um movimento harmônico.<br />

Uma curva fechada no espaço de fase x × ẋ caracteriza<br />

um movimento periódico.<br />

x<br />

1<br />

0.8<br />

0.6<br />

0.4<br />

0.2<br />

0<br />

-0.2<br />

-0.4<br />

-0.6<br />

-4<br />

-0.8<br />

x(t)<br />

-1<br />

ẋ(t)<br />

-6<br />

0 0.2 0.4 0.6 0.8 1<br />

t<br />

8<br />

(a)<br />

6<br />

4<br />

2<br />

0<br />

-2<br />

ẋ<br />

coordenadas) e com uma massa m presa na outra<br />

extremidade, conforme indicado na Figura 3.4.<br />

Neste sistema temos apenas as forças da gravidade<br />

⃗F = mg î agindo na massa m e a tensão ⃗ T na haste.<br />

Deslocando a massa de sua posição de equilíbrio (na<br />

vertical) por um ângulo θ, imediatamente a força<br />

gravitacional faz com que o sistema retorne à sua<br />

posição de equilíbrio. Nosso objetivo é descrever o<br />

movimento desta massa m, ou seja, encontrar como<br />

o ângulo θ (ou fase) varia com o tempo. Note que,<br />

embora o movimento ocorra no plano (dois graus<br />

de liberdade), temos um movimento com apenas<br />

um grau de liberdade, pois a condição de inflexibilidade<br />

da haste impõe que x 2 (t) + y 2 (t) = l 2 , com<br />

x(t) e y(t) dependentes do tempo e l independente<br />

do tempo. Naturalmente, a equação horária para<br />

o ângulo θ = θ(t) deve ser encontrada resolvendo<br />

a equação diferencial proveniente da segunda lei de<br />

Newton,<br />

⃗F R = ⃗ F + ⃗ T = m⃗a. (3.82)<br />

O<br />

θ<br />

l<br />

y<br />

ĵ<br />

6<br />

4<br />

2<br />

x<br />

⃗T<br />

ẋ<br />

0<br />

-2<br />

-4<br />

-6<br />

-8<br />

-1 -0.8 -0.6 -0.4 -0.2 0 0.2 0.4 0.6 0.8 1<br />

x<br />

(b)<br />

Figura 3.3: Equação horária x(t), velocidade ẋ(t)<br />

e espaço de fase para o oscilador harmônico com<br />

A = 1, ϕ = 0 e ω 0 = 2π (unidades arbitrárias).<br />

Pêndulo simples<br />

Vamos considerar agora um pêndulo ideal. Um<br />

pêndulo ideal é formado por uma haste rígida, inflexível,<br />

de massa nula e de comprimento l, com<br />

uma das extremidades fixa em algum lugar (o qual<br />

usaremos como origem para o nosso sistema de<br />

î<br />

⃗F<br />

Figura 3.4: Pêndulo simples (ideal) de comprimento<br />

l e massa m sob a ação da gravidade F ⃗ =<br />

mg î.<br />

Usando o sistema de coordenadas mostrado na<br />

Figura 3.4, teremos<br />

F Rx = mg − T cos θ<br />

= mẍ = −ml¨θ sin θ − ml ˙θ 2 cos θ, (3.83)<br />

F Ry = −T sin θ<br />

= mÿ = ml¨θ cos θ − ml ˙θ 2 sin θ. (3.84)<br />

Estas duas equações diferenciais são equivalentes a<br />

uma equação diferencial para θ(t),<br />

¨θ + ω 2 0 sin θ = 0, ω 0 =<br />

√ g<br />

l , (3.85)<br />

41


3.3. Aplicações da segunda lei de Newton Capítulo 3. Translações<br />

e uma equação para o módulo da tensão na haste,<br />

T = mg ( ˙θ2 ω0<br />

2 − ¨θ cot θ ) . (3.86)<br />

Uma observação importante: note que a constante<br />

ω 0 , a freqüência angular natural do pêndulo, em<br />

(3.85) não depende da massa do m do pêndulo.<br />

Assim, pêndulos construídos com uma bolinha de<br />

tênis e um elefante terão a mesma equação horária.<br />

A equação diferencial (3.85), aparentemente simples,<br />

oferece muitas dificuldades para ser resolvida<br />

analiticamente. Esta é uma situação onde é muito<br />

mais conveniente usarmos métodos numéricos diretamente.<br />

No entanto, podemos observar que, devido<br />

à série de Taylor (Exercício 108) para a função<br />

seno em torno de θ = 0, sin θ ≃ θ, a equação diferencial<br />

(3.85) torna-se na equação diferencial de<br />

um oscilador harmônico para pequenos valores de<br />

θ (sempre em radianos),<br />

¨θ + ω 2 0θ = 0, θ ≪ 1, ω 0 =<br />

√ g<br />

l . (3.87)<br />

Assim, dentro da aproximação de ângulos pequenos,<br />

podemos escrever<br />

θ(t) = A cos(ω 0 t + ϕ), ω 0 =<br />

√ g<br />

l . (3.88)<br />

Isto significa que o movimento do pêndulo será<br />

harmônico para baixas amplitudes (ângulos) e que<br />

a constante ω 0 será a sua freqüência natural, a<br />

qual é independente da massa m. Tendo um bom<br />

cronômetro e muita paciência podemos então calcular<br />

o valor do módulo da aceleração da gravidade<br />

no local do pêndulo. Portanto, esperamos<br />

ver uma elipse no espaço de fase (θ × ˙θ) deste<br />

pêndulo com ângulos (amplitudes) pequenos com<br />

a razão entre seus semi-eixos igual à freqüência natural<br />

ω 0 = √ g/l.<br />

A Figura 3.5 mostra três equações horárias (unidades<br />

MKS) para o mesmo pêndulo de freqüência<br />

angular natural ω 0 = 2π rad/s. Estas três funções<br />

do tempo foram obtidas resolvendo a equação diferencial<br />

(3.85) numericamente. Estas equações<br />

horárias diferem apenas pelas condições iniciais.<br />

Como podemos ver na Figura 3.5, as duas primeiras<br />

destas equações horárias representam um movimento<br />

oscilatório (amplitude limitada). A terceira<br />

equação representa um movimento de rotação<br />

(faça o Exercício 54). Estes três movimento são<br />

periódicos. Na Figura 3.5, cada curva foi devidamente<br />

normalizada,<br />

e<br />

5<br />

4<br />

3<br />

2<br />

1<br />

0<br />

-1<br />

Θ 1 = 20<br />

π θ 1(t), ω 0 = 2π,<br />

θ 1 (0) = π 20 , ˙θ1 (0) = 0,<br />

Θ 2 = 2 π θ 2(t), ω 0 = 2π,<br />

θ 2 (0) = π 2 , ˙θ2 (0) = 0,<br />

Θ 3 = 1 π θ 3(t), ω 0 = 2π,<br />

θ 3 (0) = 0, ˙θ2 (0) = 12.57 ≃ 4π.<br />

Θ 1<br />

Θ 2<br />

Θ 3<br />

(3.89)<br />

(3.90)<br />

(3.91)<br />

0 0.5 1.0 1.5 2.0 2.5 3.0 3.5 4.0<br />

t<br />

Figura 3.5: Três equações horárias para o mesmo<br />

pêndulo simples com ω 0 = 2π rad/s, diferindo apenas<br />

pela escolha das condições iniciais dadas em<br />

(3.89)–(3.91).<br />

Podemos ver na Figura 3.5 que apenas a primeira<br />

equação horária possui um período aproximadamente<br />

igual a 1 s. Portanto, a frequência<br />

angular correspondente é aproximadamente igual<br />

a 2π rad/s (faça o Exercício 54). Ela também<br />

possui uma amplitude inicial pequena π/20 rad.<br />

Estes dois fatos siginificam que apenas a primeira<br />

equação horária representa um movimento<br />

harmônico, ou seja, ela também deve satisfazer a<br />

equação diferencial (3.87) dentro de uma boa aproximação.<br />

Os respectivos espaços de fase destes três<br />

casos estão mostrados nas Figuras 3.6. Note que o<br />

espaço de fase da terceira equação horária é bem diferente<br />

dos demais. Este exemplo nos ensina que o<br />

espaço de fase de sistemas que executam um movimento<br />

oscilatório (confinado em uma certa região) é<br />

42


Capítulo 3. Translações<br />

3.3. Aplicações da segunda lei de Newton<br />

uma curva fechada. O espaço de fase para rotações<br />

é sempre uma curva aberta, como aquela mostrada<br />

na Figura 3.6(b).<br />

˙θ<br />

8<br />

6<br />

4<br />

2<br />

0<br />

-2<br />

-4<br />

-6<br />

Θ 1<br />

Θ 2<br />

-8<br />

-1 -0.8 -0.6 -0.4 -0.2 0 0.2 0.4 0.6 0.8 1<br />

θ<br />

˙θ<br />

4.5<br />

4.0<br />

3.5<br />

3.0<br />

2.5<br />

2.0<br />

1.5<br />

1.0<br />

0.5<br />

(a)<br />

0<br />

0 0.5 1.0 1.5 2.0 2.5 3.0 3.5 4.0 4.5 5.0<br />

θ<br />

(b)<br />

Figura 3.6: Três espaços de fases para o mesmo<br />

pêndulo simples com ω 0 = 2π rad/s, diferindo apenas<br />

pelas escolhas das condições iniciais dadas em<br />

(3.89)–(3.91). Unidades MKS.<br />

3.3.4 Exercícios<br />

Exercício 39<br />

Determine explicitamente as expressões (3.31)–<br />

(3.33). Determine também as três componentes<br />

do vetor velocidade a partir das expressões (3.31)–<br />

(3.33).<br />

Exercício 40<br />

Determine o tempo de queda e a velocidade final<br />

da queda livre descrita em (3.35).<br />

Exercício 41<br />

Determine as quantidades em (3.38).<br />

Θ 3<br />

Determine<br />

também o tempo de retorno ao plano XY . Determine<br />

também o alcance máximo deste lançamento.<br />

Exercício 42<br />

Determine as dimensões da constante τ em (3.41).<br />

Determine a constante de integração C em (3.47)<br />

usando a condição inicial v z (0) = 0.<br />

Exercício 43<br />

Determine a constante de integração C em (3.50)<br />

usando a condição inicial z(0) = h. Verifique explicitamente<br />

que a função z(t) em (3.51) satisfaz a<br />

equação diferencial em (3.41).<br />

Exercício 44<br />

Determine as dimensões da constante η e da constante<br />

τ em (3.56).<br />

Exercício 45<br />

Derive a solução z(t) em (3.56) para obter a velocidade<br />

(3.57).<br />

Exercício 46<br />

Verifique explicitamente que a função z(t) em<br />

(3.56) satisfaz a equação diferencial em (3.53).<br />

Exercício 47<br />

Determine a velocidade terminal efetuando diretamente<br />

o limite<br />

com v z (t) dada em (3.57).<br />

V T = lim<br />

t→∞<br />

v z (t), (3.92)<br />

Exercício 48<br />

Determine a velocidade terminal em (3.59).<br />

Exercício 49<br />

Mostre que<br />

v x (t) = B + e +iωt + B − e −iωt , i = √ −1, (3.93)<br />

também satisfaz a equação diferencial (3.65). Use o<br />

fato de que qualquer número complexo z = x + i y,<br />

de módulo r = √ zz ∗ = √ x 2 + y 2 , com z ∗ = x −<br />

i y (conjugado), pode ser representado por (faça o<br />

Exercício 109)<br />

z = re iθ = r ( cos θ + i sin θ), tan θ = y x , (3.94)<br />

para mostrar que as soluções (3.67) e (3.93) são<br />

idênticas. Determine as constantes C 1 e C 2 aparecendo<br />

em (3.67) em função das constantes B ±<br />

aparecendo em (3.93).<br />

43


3.3. Aplicações da segunda lei de Newton Capítulo 3. Translações<br />

Exercício 50<br />

Determine explicitamente as constantes de integração<br />

em (3.73)–(3.74).<br />

Exercício 51<br />

Obtenha explicitamente as constantes A e ϕ dadas<br />

em (3.81) usando as condições iniciais x(0) = x 0<br />

e ẋ(0) = ẋ 0 . Depois faça x 0 = 1, ẋ 0 = 0 (objeto<br />

solto do repouso) e ω = 2π. Use o sistema MKS de<br />

unidades.<br />

Exercício 52<br />

Considere a equação (forma direta) de uma elipse<br />

de semi-eixo maior a e semi-eixo menor b,<br />

X 2<br />

a 2 + Y 2<br />

= Cte. (3.95)<br />

b2 ( Mostre que ) a curva (forma paramétrica) γ(t) =<br />

x(t), ẋ(t), 0 , com x(t) dada em (3.80), satisfaz<br />

(3.95) para qualquer instante de tempo, ou seja,<br />

prove que γ é de fato uma elipse de semi-eixo maior<br />

a = 1 e semi-eixo menor b = ω 0 . Faça a identificação<br />

X = x(t) e Y = ẋ(t).<br />

Exercício 53<br />

Determine explicitamente as equações (3.83)–<br />

(3.86).<br />

Exercício 54<br />

A partir dos gráficos mostrados na Figura 3.5, calcule<br />

os respectivos períodos e freqüências angulares.<br />

Descreva precisamente o movimento do pêndulo em<br />

cada situação.<br />

44


Capítulo 4<br />

Leis de conservação I<br />

4.1 Trabalho<br />

Nós temos uma experiência cotidiana em lidar com<br />

forças. Temos uma idéia clara que é mais fácil fazer<br />

um pingüim de geladeira, feito de gesso e mal acabado,<br />

deslizar sobre uma mesa do que deslizar um<br />

monitor de computador. Também sabemos que é<br />

mais cansativo transportar o monitor escada acima<br />

we que este cansaço é (no mínimo) proporcional à<br />

distância percorrida. A questão que devemos responder<br />

é: como quantificar esta nossa noção cotidiana<br />

acerca do nosso esforço quando debatemos<br />

com uma determinada força? Em outras palavras,<br />

como podemos combinar força e deslocamento, dois<br />

vetores, para produzirmos um número, um escalar,<br />

que possa ser usado para quantificar nosso esforço?<br />

Apelando para a simplicidade, podemos tentar<br />

um produto escalar entre força F ⃗ = (F x , F y , F z )<br />

e deslocamento infinitesimal d⃗r = (dx, dy, dz) para<br />

quantificar nosso esforço, F ⃗ · d⃗r. Somando todas<br />

estas contribuições infinitesimais ao longo de uma<br />

determinada trajetória entre as posições A e B,<br />

∆W AB =<br />

∫ B<br />

A<br />

⃗F · d⃗r, (4.1)<br />

obtemos o número ∆W , denominado de trabalho.<br />

Dizemos que este número W AB representa o trabalho<br />

associado à força ⃗ F durante o movimento iniciado<br />

em A e terminado em B, ao longo de uma<br />

determinada trajetória descrita parametricamente<br />

pelo vetor posição ⃗r(t) = (x(t), y(t), z(t)). Esta trajetória<br />

foi produzida pela força resultante ⃗ F , través<br />

da segunda lei de Newton.<br />

Algumas observações importantes sobre como<br />

calcular o trabalho usando a integral (4.1): (i) os<br />

limites de integração A e B representam as posições<br />

inicial A = ⃗r(t 0 ) = (x 0 , y 0 , z 0 ) em t = t 0 e final<br />

B = ⃗r(t 1 ) = (x 1 , y 1 , z 1 ) em t = t 1 , sobre a<br />

trajetória ⃗r(t) = (x(t), y(t), z(t)); (ii) a força pode<br />

ser uma função da posição em geral, como a força<br />

elástica, por exemplo; (iii) não podemos esquecer<br />

que o produto escalar em (4.1) pode ser efetuado<br />

de duas formas, através de coordenadas,<br />

∆W AB =<br />

=<br />

∫ B<br />

A<br />

∫ B<br />

A<br />

⃗F · d⃗r<br />

F x dx +<br />

∫ B<br />

A<br />

∫ B<br />

F y dy + F z dz, (4.2)<br />

A<br />

bem como através de normas (comprimentos),<br />

∆W AB =<br />

∫ B<br />

A<br />

⃗F · d⃗r =<br />

∫ B<br />

A<br />

cos θ F dr, (4.3)<br />

com F = || F ⃗ ||, dr = ||d⃗r|| e θ sendo o ângulo entre o<br />

vetor força e o vetor deslocamento. Não podemos<br />

esquecer que tanto o módulo F da força quanto<br />

o ângulo θ podem depender da posição. Cada situação<br />

exigirá a escolha de uma destas duas maneiras<br />

de calcular o trabalho que tornará o cálculo<br />

o mais simples possível.<br />

Podemos notar que quanto maior a intensidade<br />

da força, maior o trabalho. Em uma dimensão, digamos<br />

ao longo do eixo X, qual é a interpretação<br />

geométrica do trabalho (4.1)? É a mesma interpretação<br />

geométrica de uma integral, ou seja, é a<br />

área abaixo da curva F x (x). Muito bem! Isto é<br />

importante, pois há muitas situações reais onde a<br />

força é unidimensional e conhecemos somente seu<br />

gráfico em função da posição. Mesmo não tendo<br />

uma expressão analítica para F (x), a qual é absolutamente<br />

necessária para efetuarmos a integral<br />

45


4.1. Trabalho Capítulo 4. Leis de conservação I<br />

(4.1), podemos calcular, aproximadamente, o valor<br />

da área abaixo da curva F x (x) usando técnicas<br />

geométricas, usando a definição de integral. No entanto,<br />

lidaremos aqui apenas com situações onde<br />

temos as devidas expressões analíticas.<br />

Vejamos alguns exemplos. Considere um sistema<br />

composto por um único corpo de massa m, o qual<br />

está em movimento uniforme com uma velocidade<br />

⃗v = v î. Suponha que uma força elástica externa<br />

⃗F = −kx î (lei de Hooke) comece a agir neste corpo<br />

no instante t = 0 (x = 0). Note que estamos usando<br />

um sistema de coordenadas ortonormal no qual a<br />

massa m está sendo localizada por ⃗r = x î. Conseqüentemente,<br />

o vetor deslocamento infinitesimal<br />

(diferencial do vetor posição) é d⃗r = dx î. Nossa<br />

tarefa é calcular o trabalho feito por esta força<br />

elástica (uma força externa) ao longo da trajetória<br />

retilínea iniciando em x = 0 [ponto A = (0, 0, 0)]<br />

e terminando em x = l [ponto B = (l, 0, 0)], sendo<br />

l a distância máxima percorrida pela massa m até<br />

parar (amplitude). Usando (4.2), o trabalho correspondente<br />

é ∆W = −kl 2 /2 (Faça a parte (i) do<br />

Exercício 55). Note que este trabalho é proporcional<br />

à constante de mola k e à amplitude l, mas é<br />

independente da massa m. Quanto maior a constante<br />

de mola (mola mais dura), maior o trabalho.<br />

Quanto maior a amplitude, maior o trabalho.<br />

No entanto, o trabalho é o mesmo para qualquer<br />

massa, fato este que está em desacordo com a nossa<br />

experiência diária.<br />

Considere novamente um sistema formado por<br />

uma única massa m mantida em repouso a uma altura<br />

h próxima à superfície da Terra. Portanto, de<br />

acordo com a gravitação Newtoniana, esta massa<br />

m está sujeita a uma força constante, F ⃗ = −mg ˆk.<br />

A força gravitacional aqui é externa ao sistema.<br />

Note que estamos usando um sistema de coordenadas<br />

ortonormal no qual a massa m está localizada<br />

por ⃗r = z ˆk. Conseqüentemente, o vetor deslocamento<br />

infinitesimal (diferencial do vetor posição)<br />

é d⃗r = dz ˆk. O trabalho efetuado por esta força<br />

externa, calculado pela definição (4.2), para levar a<br />

massa m de volta à superfície é ∆W = +mgh (Faça<br />

a parte (i) do Exercício 56). Desta vez, o trabalho<br />

depende não somente da distância percorrida, mas<br />

também da massa m.<br />

Note que, devido ao produto escalar presente na<br />

definição de trabalho (4.1), o qual é nulo para vetores<br />

perpendiculares, mantendo a força gravitacional<br />

na vertical, podemos imaginar uma trajetória<br />

horizontal na qual a força gravitacional não realiza<br />

trabalho. Embora isto também esteja em desacordo<br />

com nossa experiência cotidiana, temos aqui um resultado<br />

importante acerca da força gravitacional: o<br />

trabalho realizado por ela independe da trajetória<br />

escolhida. O trabalho realizado pela força gravitacional<br />

depende apenas da distância vertical correspondente<br />

ao deslocamento da massa m.<br />

Quais são as dimensões de trabalho? De acordo<br />

com a definição (4.1), [W ] =ML 2 T −2 . Estas unidades<br />

receberam o nome de Joule (J=N m). Note que<br />

o Joule tem as mesmas dimensões do produto massa<br />

por velocidade ao quadrado. Aviso importante: veremos<br />

na próxima seção que o Joule é a unidade de<br />

“energia” no sistema MKS. Entretanto, trabalho<br />

não é energia, ou seja, não é correto afirmar que<br />

um corpo tem trabalho. Trabalho é um processo<br />

de troca de energia (como veremos adiante). Assim,<br />

é correto dizer que um corpo realiza trabalho<br />

ao se movimentar na presença de uma força.<br />

Pense na seguinte situação: duas máquinas M 1 e<br />

M 2 são usadas para elevar (adiabaticamente) uma<br />

massa m a uma altura h acima da superfície da<br />

Terra. Como vimos anteriormente, ambas realizam<br />

a mesma quantidade de trabalho: mgh. No entanto,<br />

observa-se que a máquina M 2 leva menos<br />

tempo para executar esta tarefa. Isto significa que<br />

esta máquina M 2 consegue trocar mais “energia”<br />

por unidade de tempo através do mecanismo “trabalho”.<br />

Sendo trabalho um mecanismo de troca de<br />

energia, então é conveniente definirmos uma taxa<br />

de variação temporal do trabalho realizado. Em um<br />

certo intervalo de tempo dt, um corpo é deslocado<br />

por d⃗r = ⃗vdt sob a ação de uma força F ⃗ . O trabalho<br />

infinitesimal correspondente é dW = F ⃗ · ⃗vdt.<br />

Assim, podemos definir<br />

P = dW dt<br />

= ⃗ F · ⃗v. (4.4)<br />

Esta taxa temporal de energia é denominada de<br />

potência. A unidade de potência em MKS é o Watt<br />

(W=J/s). Você já reparou que a sua conta de luz<br />

registra o seu consumo em kWh (kilowatt-hora)?<br />

1 kWh=3, 6×10 6 J. Portanto, você paga pela energia<br />

fornecida e não pela potência consumida.<br />

46


Capítulo 4. Leis de conservação I<br />

4.2. Energia cinética<br />

4.1.1 Exercícios<br />

Exercício 55<br />

(i) Mostre que o trabalho (4.1) realizado pela força<br />

elástica F ⃗ = −kx î durante o deslocamento de uma<br />

massa m, localizada por ⃗r = x î, desde x 1 = 0 (velocidade<br />

⃗v 1 = v 0 î) até x 2 = l (amplitude; velocidade<br />

nula ⃗v 2 = 0) é ∆W = −kl 2 /2. Calcule também<br />

o trabalho realizado pela força elástica no sentido<br />

inverso (volta). Agora calcule o trabalho total realizado<br />

na trajetória fechada (ida e volta). (ii) Mostre<br />

também que o trabalho realizado na ida é igual à<br />

variação de energia cinética T 2 − T 1 [confira (4.5)<br />

e (4.6)]. Use o Teorema 7 para determinar a velocidade<br />

final (em x 1 = 0) na volta. Esta velocidade<br />

final é igual à inicial? Justifique.<br />

Exercício 56<br />

(i) Mostre que o trabalho (4.1) realizado pela força<br />

gravitacional (na superfície da Terra) F ⃗ = −mgˆȷ<br />

durante o deslocamento de um massa m, localizada<br />

por ⃗r = yˆȷ, desde y 1 = 0 (velocidade inicial<br />

⃗v 1 = v 0ˆȷ) até y 2 = h (velocidade nula ⃗v 2 = 0)<br />

é ∆W = −mgh. Calcule também o trabalho realizado<br />

pela força gravitacional no sentido inverso<br />

(volta). Agora calcule o trabalho total realizado na<br />

trajetória fechada (ida e volta). (ii) Mostre também<br />

que o trabalho realizado na ida é igual à variação<br />

de energia cinética T 2 − T 1 [confira (4.5) e (4.6)].<br />

4.2 Energia cinética<br />

De acordo com os dois exemplos apresentados na<br />

seção anterior, a integral (trabalho) definida em<br />

(4.1) atende muito bem aos quesitos iniciais, embora<br />

produza alguns fatos em desacordo com nossa<br />

experiência cotidiana. No entanto, veremos que a<br />

definição (4.1) é muito mais importante que uma<br />

simples quantificação do nosso esforço usual ou<br />

do custo de produzir um determinado movimento.<br />

Primeiramente, usando a segunda lei de Newton<br />

⃗F = m⃗a, podemos re-escrever a integral (4.1), a<br />

qual é uma integral na posição, numa integral na<br />

velocidade ⃗v = ˙⃗r (Faça o Exercício 57),<br />

∆W =<br />

∫ B<br />

A<br />

∫ vB<br />

⃗F · d⃗r = m ⃗v · d⃗v<br />

v A<br />

= 1 2 mv2 B − 1 2 mv2 A.<br />

(4.5)<br />

Isto significa que podemos calcular trabalho através<br />

do módulo do vetor velocidade (velocidade escalar),<br />

calculado apenas nos extremos da trajetória<br />

percorrida. Este é um resultado muito importante<br />

devido à sua praticidade de evitar a resolução de<br />

integrais. Tão importante que iremos denominar a<br />

quantidade<br />

T = 1 2 mv2 , v = ||⃗v||, (4.6)<br />

de energia cinética. Note que esta energia cinética<br />

é uma quantidade inerente ao corpo de massa m<br />

com um vetor velocidade ⃗v. Note também que as<br />

dimensões de energia cinética são as mesmas de trabalho,<br />

[T ] =ML 2 T −2 . Assim, podemos enunciar o<br />

resultado (4.5) na forma de um teorema:<br />

Teorema 7 (Energia cinética)<br />

Quando o trabalho realizado em um sistema produz<br />

modificações na velocidade, então<br />

∆W = ∆T. (4.7)<br />

Vejamos algumas aplicações deste teorema. Vamos<br />

considerar novamente o sistema formado por<br />

uma massa m na presença de uma força elástica<br />

⃗F = −kx î produzida por uma mola ideal. A massa<br />

m tem velocidade ⃗v A = v 0 î em x = 0 (ponto A<br />

da trajetória). Ao atingir a posição x = l (ponto<br />

B; distensão máxima; velocidade nula), a massa m<br />

deve retornar. Retorno significa inverter o sentido<br />

do vetor velocidade e isto significa que a velocidade<br />

precisa ser nula em x = l, ⃗v 1 = 0. O trabalho realizado<br />

pela força elástica nesta trajetória (ida de<br />

x = 0 para x = l) é ∆W = −kl 2 /2, como calculado<br />

anteriormente. Portanto, usando o Teorema 7, a<br />

velocidade inicial (em x = 0) deve ser v 0 = ω 0 A,<br />

ω0 2 = k/m (Faça a parte (ii) dos Exercícios 55–56).<br />

Este sistema massa-mola ainda pode nos ensinar<br />

muito mais. O trabalho realizado pela força<br />

elástica é negativo no trajeto de ida (de x = 0<br />

até x = l). Então estamos confirmando que uma<br />

força externa realiza trabalho sobre a massa m e<br />

produz uma alteração na energia cinética dela, isto<br />

é, altera a sua velocidade. Neste exemplo que estamos<br />

considerando, a energia cinética diminuiu,<br />

passou de mv0/2 2 para 0, com v 0 = ω 0 A. Portanto,<br />

a massa m perdeu energia cinética. Isto justifica<br />

interpretarmos o trabalho como sendo um processo<br />

de transferência de energia. Mas a massa m perde<br />

47


4.3. Energia potencial Capítulo 4. Leis de conservação I<br />

energia para quem? Além da massa, temos apenas<br />

a mola ideal de constante k neste sistema. Então<br />

é razoável imaginar a existência um mecanismo de<br />

troca de energia entre a massa m e a mola ideal k.<br />

Como esta energia é armazenada na mola? Embora<br />

a mola apresente movimentos internos (compresão<br />

e distensão), ela não se move como um todo. Então<br />

esta energia armazenada na mola não pode ser na<br />

forma cinética. Esta nova modalidade de energia, a<br />

qual depende da disposição espacial da mola, será<br />

discutida na próxima seção.<br />

Algumas observações importantes. Inicialmente<br />

dissemos que o trabalho serve muito bem para<br />

quantificar nossos esforços da experiência cotidiana.<br />

Na verdade, estamos vendo através destes<br />

exemplos simples que a definição (4.1) nos revela<br />

algumas surpresas que estão em desacordo ao nosso<br />

senso comum. Por exemplo, ao transportarmos<br />

um objeto perpendicularmente à força gravitacional<br />

nas proximidades da superfície da Terra, certamente<br />

ficamos exaustos. Então temos a sensação<br />

de termos realizado “trabalho”. No entanto o trabalho<br />

(4.1) é nulo nesta condição. Também sentimos<br />

que estamos realizando trabalho ao mantermos<br />

um objeto em equilíbrio a uma certa altura,<br />

como um halterofilista mantendo pesos em<br />

equilíbrio. No entanto, pelo Teorema 7, não havendo<br />

modificações na energia cinética, não há trabalho.<br />

Também é surpreendente que o trabalho realizado<br />

em um sistema possa ser calculado usando-se<br />

somente o módulo do vetor velocidade (velocidade<br />

escalar). Isto significa que o trabalho é nulo em<br />

um movimento circular uniforme, pois o módulo<br />

do vetor velocidade é constante. Evidentemente,<br />

todas as forças externas agindo numa massa m em<br />

movimento circular são perpendiculares ao deslocamento<br />

(tangente à trajetória circular). Desta<br />

forma, a definição (4.1) de trabalho em mecânica<br />

é bem diferente da nossa noção cotidiana de trabalho,<br />

embora ela nos tenha servido como inspiração.<br />

Algo que realmente não podemos fazer confusão:<br />

trabalho não é uma modalidade de energia. Trabalho<br />

é um mecanismo de troca de energia. Nesta troca<br />

de energia pelo mecanismo de trabalho (4.1), há<br />

variação de energia cinética (Teorema 7). Veremos<br />

mais adiante que calor também é um mecanismo<br />

de troca de energia envolvendo variação da energia<br />

interna (proporcional à temperatura, a qual está ligada<br />

à energia cinética média). Naturalmente, os<br />

mecanismos de troca de energia (trabalho, calor,<br />

etc.) possuem as mesmas unidades de energia e<br />

isto nos leva a interpretar (erroneamente) estes mecanismos<br />

como energia. Até mesmo na etimologia<br />

(grega e latina) da palavra energia, força em ação<br />

ou trabalho interno, já aparece a noção de trabalho.<br />

A única quantidade que temos até o momento<br />

que pode ser denominada de energia, é a energia<br />

cinética (4.6). Então o Joule (J=N m) é uma unidade<br />

de energia no sistema MKS. As dimensões de<br />

energia, usando (4.6), são [T ] =ML 2 T −2 . É importante<br />

mantermos em mente estas dimensões de<br />

energia. Curiosidade: a energia cinética da Terra é<br />

cerca de 3 × 10 33 J. A energia cinética de um atleta<br />

corredor é aproximadamente 4 × 10 3 J.<br />

4.2.1 Exercícios<br />

Exercício 57<br />

Faça explicitamente todas as passagens omitidas<br />

em (4.5). Use a segunda lei de Newton e uma integração<br />

por partes.<br />

4.3 Energia potencial<br />

Além da energia cinética, há um outro tipo de<br />

energia relacionada com a definição (4.1) de trabalho.<br />

Como sempre, vamos utilizar novamente a<br />

força elástica e a força gravitacional para aprendermos.<br />

Nosso sistema será formado por uma única<br />

massa m. Neste sistema atuará uma força externa<br />

(elástica ou gravitacional). Nós calculamos anteriormente<br />

(Exercício 55) o trabalho realizado pela<br />

força elástica sobre a massa m no trajeto de ida<br />

(de x 1 = 0 até x 2 = l). Vamos denotar este trabalho<br />

por ∆W 12 = −kl 2 /2. Note que a energia<br />

cinética da massa m diminuiu. O trabalho realizado<br />

no caminho inverso, de x 2 = A para x 1 = 0<br />

é ∆W 21 = +kl 2 /2. Note que neste caso, a energia<br />

cinética da massa m aumentou. Portanto, o<br />

trabalho total realizado pela força elástica é nulo,<br />

∆W = ∆W 12 + ∆W 21 = 0. Em outras palavras, o<br />

trabalho realizado pela força elástica na massa m<br />

ao longo de uma trajetória fechada (ida e volta) é<br />

nulo. Este mesmo resultado também é válido para<br />

a força gravitacional (verifique).<br />

Se há forças que produzem um trabalho nulo em<br />

trajetórias fechadas, isto significa que o trabalho realizado<br />

por estas forças dependem apenas dos pontos<br />

extremos das trajetórias. Note que isto está de<br />

48


Capítulo 4. Leis de conservação I<br />

4.3. Energia potencial<br />

pleno acordo com o Teorema 7 da energia cinética:<br />

o trabalho depende somente da velocidade escalar<br />

calculada nas posições inicial e final da trajetória.<br />

Denominaremos as forças com esta propriedade,<br />

ou seja, trabalho nulo numa trajetória fechada, de<br />

forças conservativas. As forças fundamentais da natureza<br />

são conservativas. Naturalmente, quando há<br />

qualquer tipo de atrito e/ou viscosidade, teremos<br />

forças não-conservativas ou dissipativas.<br />

Quando calculamos a variação de energia cinética<br />

no sistema massa-mola, vimos que a energia<br />

cinética diminuiu na ida e aumentou na volta. Concluímos<br />

que a mola deve ter absorvido esta energia<br />

perdida pela massa durante a ida e liberado a<br />

mesma quantidade durante a volta. Também concluímos<br />

que a mola não pôde ter armazenado esta<br />

energia na forma de energia cinética, pois houve<br />

apenas uma deformação da mesma, o que modificou<br />

a intensidade da força elástica. No momento<br />

em que a massa m está momentaneamente em repouso,<br />

a força elástica (restauradora) é máxima.<br />

Assim que a mola inicia sua volta ao seu comprimento<br />

natural, a massa m entra em movimento (no<br />

sentido oposto) e atinge sua velocidade máxima<br />

quando o alongamento da mola é nulo. Nesta situação,<br />

a mola cede para a massa m toda a energia<br />

que havia armazenado anteriormente. O trabalho<br />

é uma forma de quantificar esta troca de energias<br />

entre a massa m e a mola, numa interação mediada<br />

pela força elástica. Podemos concluir então que<br />

esta nova modalidade de energia está associada com<br />

a força elástica (produzida pela mola). Uma análise<br />

similar, nos leva a concluir também a existência de<br />

uma nova modalidade de energia associada com a<br />

força gravitacional. Em ambos os casos, esta nova<br />

modalidade de energia, através do trabalho, tem a<br />

capacidade (potencial) de se transformar em energia<br />

cinética. Por isso iremos denominá-la de energia<br />

potencial e denotá-la por V .<br />

Ainda considerando o mesmo sistema massamola<br />

que temos usado até aqui, o trabalho realizado<br />

pela força elástica na ida é negativo (∆W 12 =<br />

−kl 2 /2). Vamos interpretar este sinal negativo<br />

como uma representação da perda de energia<br />

cinética pela massa m. Conseqüentemente, a mola<br />

aumentou sua energia potencial, ∆V 12 = kl 2 /2 =<br />

−∆W 12 . Pelo teorema da energia cinética e trabalho,<br />

temos ∆W 12 = ∆T 12 . Portanto ∆V 12 =<br />

−∆T 12 , ou, equivalentemente, ∆(T + V ) 12 = 0.<br />

Na volta, podemos concluir que ∆(T + V ) 21 = 0.<br />

Assim, temos dois resultados importantes: (i) podemos<br />

aproveitar trabalho para calcular variações<br />

de energia potencial,<br />

V (⃗r) = −<br />

∫ ⃗r<br />

⃗r 0<br />

⃗ F · d⃗r, (4.8)<br />

a qual é uma função exclusivamente da posição final<br />

⃗r = ⃗r(t), pois a força é conservativa, ou seja, a<br />

integral (4.8) não depende da trajetória (caminho)<br />

escolhido entre o ponto inicial ⃗r 0 = ⃗r(t 0 ) (mantido<br />

fixo) e final ⃗r(t) (arbitrário).<br />

Note também que a função potencial em (4.8)<br />

está definida a menos de uma constante arbitrária<br />

(devido ao limite inferior ocorrer em uma posição<br />

⃗r(t 0 ) previamente escolhida). Deve ser mantido<br />

em mente que a função energia potencial está definida<br />

apenas para forças conservativas. Você ainda<br />

lembra do teorema fundamental do cálculo? Teorema<br />

5. Pois bem, usando este teorema nós<br />

podemos responder a seguinte questão fundamental:<br />

dado a energia potencial V = V (x, y, z) é<br />

possível obter o vetor força (resultante)? Em outras<br />

palavras, é possível inverter a relação (4.8)?<br />

Sim é possível, graças ao Teorema Fundamental do<br />

Cálculo, Teorema 5. Como os versores î, ˆȷ e ˆk são<br />

constantes, então (verifique)<br />

⃗F = − d<br />

dx V î − d<br />

dy V ˆȷ − d dz V ˆk = − ⃗ ∇V. (4.9)<br />

Lembre-se que a força contém toda a informação<br />

sobre o comportamento dinâmico de um sistema.<br />

Ela é uma quantidade vetorial e, portanto, contém<br />

três informações. Agora, usando (4.9), podemos<br />

calcular a força derivando a energia potencial, a<br />

qual é uma quantidade escalar (uma única informação).<br />

Ganhamos em dobro: primeiro economizamos<br />

dois terços na quantidade de informações<br />

a serem armazenadas para descrevermos o comportamento<br />

de qualquer sistema. Segundo, descobrimos<br />

uma ferramenta matemática espetacular ( ∇), ⃗<br />

pois ela tem a incrível capacidade de transformar<br />

uma função escalar em uma função vetorial. Esta<br />

ferramenta nova é denominada de operador gradiente<br />

e é denotado por ∇, ⃗ onde o síbolo ∇ é conhecido<br />

por nabla. O operador gradiente opera sempre<br />

em funções escalares, como a energia potencial<br />

V = V (⃗r) = V (x, y, z),<br />

⃗∇V = ∂<br />

∂x V î + ∂ ∂y V ˆȷ + ∂ ∂z V ˆk. (4.10)<br />

49


4.3. Energia potencial Capítulo 4. Leis de conservação I<br />

Entortamos um pouquinho o símbolo dos diferenciais.<br />

Isto é para nos lembrar que cada derivada deve<br />

ser efetuada em relação a uma variável, mantendo<br />

as demais variáveis independentes fixas (fazem o<br />

papel de parâmetros). Estes novos símbolos para<br />

as derivadas são denominadas de derivadas parciais<br />

(por razões óbvias).<br />

Vejamos algumas aplicações. Adivinhe quem<br />

será o primeiro sistema a ser usado? É impressionante<br />

a quantidade de conceitos fundamentais<br />

que podemos aprender com o oscilador harmônico.<br />

Considere um oscilador harmônico como descrito<br />

na Figura 3.2. Com a força elástica (lei de Hooke)<br />

escrita na forma F ⃗ = −kx î e energia potencial na<br />

forma V = V (x, y, z), então, de (4.9), temos<br />

⃗F = −k x î<br />

= − d<br />

dx V î − d<br />

dy V ˆȷ − d dz V ˆk,<br />

(4.11)<br />

a qual nos fornece três equações diferenciais de primeira<br />

ordem,<br />

∂V<br />

∂x = k x,<br />

∂V<br />

∂y = 0,<br />

∂V<br />

∂z<br />

= 0. (4.12)<br />

As duas últimas equações diferenciais em (4.12) nos<br />

diz que a função energia potencial depende apenas<br />

da coordenada x, isto é, V = V (x) é independente<br />

de y e z. Assim, levando esta informação na primeira<br />

equação diferencial em (4.12), determinamos<br />

(faça os detalhes)<br />

V (x) = 1 2 kx2 + C, (4.13)<br />

na qual C é uma constante arbitrária. Como observado<br />

anteriormente, a energia potencial (4.8) está<br />

definida a menos de uma constante arbitrária. O<br />

valor desta constante pode escolhido. Então vamos<br />

escolher C = 0 em x = 0. Assim, o potencial<br />

harmônico é parabólico na posição (veja também a<br />

Figura 4.2). Com ou sem a constante arbitrária,<br />

a variação de energia potencial durante o deslocamento<br />

da massa m desde x = 0 até x = l (trajeto<br />

de ida) é ∆V = V (l) − V (0) = kl 2 /2, o qual é exatamente<br />

o negativo do trabalho efetuado pela força<br />

elástica neste mesmo trajeto, ∆V = −∆W .<br />

Observe que a expressão no lado direito de (4.8)<br />

é de fato uma função da posição, independente da<br />

massa m e dependente da constante de mola k.<br />

Por isto, costuma-se dizer que a energia potencial<br />

está armazenada na mola (ideal, invisível). Mas é<br />

importante termos em mente que é necessário termos<br />

a massa m para observarmos qualquer movimento<br />

proveniente desta energia potencial, através<br />

da relação (4.11). Ou seja para haver um aproveitamento<br />

da energia potencial, é necessário haver uma<br />

interação (força). Note também que a aplicação do<br />

operador gradiente (4.10) em (4.13), como indicado<br />

em (4.9), realmente recupera a força elástica (faça<br />

o Exercício 58).<br />

Considere agora o sistema massa-Terra. Com a<br />

força gravitacional (outra lei de Newton) escrita na<br />

forma ⃗ F = −mg ˆk, a função energia potencial (4.8)<br />

para o sistema massa-Terra é (faça os detalhes)<br />

V (z) = mgz + C, (4.14)<br />

na qual C é uma constante arbitrária. Vamos<br />

escolher esta constante igual a zero na superfície<br />

(z = 0). Podemos ver em (4.14) que esta energia<br />

potencial depende da massa m e também da aceleração<br />

da gravidade g, a qual depende da massa da<br />

Terra. Portanto, não podemos afirmar que a energia<br />

potencial deste sistema massa-Terra está localizada<br />

em um dos dois corpos. A energia potencial<br />

está associada à força gravitacional. Note também<br />

que a aplicação do operador gradiente (4.10) em<br />

(4.14), como indicado em (4.9), realmente recupera<br />

a força gravitacional (faça o Exercício 59).<br />

Considere agora o pêndulo simples mostrado na<br />

Figura 3.4. Este é também um sistema com dois<br />

corpos: uma massa m e a Terra. No entanto, a<br />

massa m está obrigada a movimentar-se somente<br />

ao longo de uma trajetória circular de raio l, o comprimento<br />

do pêndulo. Portanto, este é um exemplo<br />

contendo um vínculo, x 2 +y 2 = l 2 (usando o mesmo<br />

sistema de coordenadas indicado na Figura 3.4).<br />

Vamos calcular a energia potencial deste sistema<br />

usando a definição (4.8). A primeira providência<br />

é escrevermos os vetores força (resultante, F ⃗ ) e o<br />

deslocamento (infinitesimal, d⃗r) em algum sistema<br />

de coordenadas. Usando o mesmo sistema de coordenadas<br />

indicado na Figura 3.4, a força resultante<br />

agindo na massa m é<br />

⃗F = (mg − T cos θ) î − T sin θˆȷ, (4.15)<br />

na qual T é o módulo da tensão no fio. A massa m<br />

está localizada na posição ⃗r = x î + yˆȷ. Escrevendo<br />

as componentes x e y em função da posição angular<br />

50


Capítulo 4. Leis de conservação I<br />

4.3. Energia potencial<br />

θ, o diferencial d⃗r pode ser calculado facilmente<br />

(faça o Exercício 60),<br />

d⃗r = −l sin θ dθ î + l cos θ dθˆȷ, (4.16)<br />

onde l é o comprimento do pêndulo, l 2 = x 2 +<br />

y 2 , mantido constante durante todo o movimento.<br />

Então, levando as expressões (4.15) e (4.16) na definição<br />

(4.8), temos (Exercício 61)<br />

com<br />

∫<br />

V = −<br />

⃗F · d⃗r = Iω 2 0<br />

= Iω 2 0(1 − cos θ),<br />

∫ θ<br />

0<br />

sin θ dθ<br />

(4.17)<br />

I = ml 2 , ω 0 =<br />

√ g<br />

l . (4.18)<br />

A constante I = ml 2 é denominada de momento<br />

de inércia do pêndulo. Ele mede a dificuldade de<br />

alterarmos o movimento de rotação da massa m<br />

em torno do eixo (fixo) onde está presa a outra<br />

extremidade do pêndulo. Note a semelhança do<br />

momento de inércia com a massa.<br />

Observe que o produto Iω0<br />

2 tem dimensões de<br />

energia. De fato, veremos, durante nossos estudos<br />

sobre o movimento de rotação em torno de um<br />

eixo fixo, que a energia cinética de rotação será<br />

Iω0/2.<br />

2 Note a semelhança desta energia cinética<br />

rotacional com a energia cinética (4.6) (translacional).<br />

As características gerais do potencial (4.17)<br />

estão mostradas na Figura 4.3. Ele é periódico de<br />

período 2π, tem um mínimo em θ = 0 e um máximo<br />

em θ = ±π/2. Note que este potencial depende<br />

também da massa m, de g e do comprimento l.<br />

Isto mostra que a energia potencial pertence ao sistema,<br />

ou seja, ela não está localizada somente em<br />

um dos dois corpos, a massa m ou a Terra.<br />

Você notou que o módulo T da força tensão na<br />

haste do pêndulo não aparece na expressão (4.17)<br />

da energia potencial? Então quando usarmos o operador<br />

gradiente (4.10) em (4.17), como indicado em<br />

(4.9), não iremos recuperar a força resultante (4.15)<br />

por completo. O que está acontecendo? O operador<br />

gradiente não sabe mecânica e nem enxerga um<br />

sistema físico como nós enxergamos. O operador<br />

gradiente simplesmente sabe recuperar a parte de<br />

uma força resultante que realiza trabalho conservativo.<br />

Como indicado na Figura 3.4, o movimento<br />

da massa m é circular (não é uniforme). Nesta trajetória<br />

circular, a tensão é sempre perpendicular ao<br />

vetor deslocamento, o qual é sempre tangente à trajetória.<br />

Portanto, o trabalho realizado pela tensão é<br />

nulo. Vale a mesma observação para a componente<br />

da força peso ao longo da haste (componente radial).<br />

Então, somente a componente da força peso<br />

que é tangente à trajetória (componente tangencial)<br />

é que realiza trabalho. Esta componente tangencial<br />

pode ser escrita na forma F ⃗ θ = −mg sin θ ˆθ,<br />

onde ˆθ é um versor na direção do ângulo θ. Vamos<br />

denotar por d⃗s o deslocamento infinitesimal ao<br />

longo trajetória circular. Assim, este deslocamento<br />

infinitesimal d⃗s também é um vetor na direção do<br />

versor ˆθ (tangente à trajetória), d⃗s = ldθ ˆθ (faça<br />

o Exercício 62). Desta forma, estamos aprendendo<br />

que o sistema cartesiano retangular nem sempre é o<br />

melhor sistema de coordenadas. Para o pêndulo da<br />

Figura 3.4, coordenadas polares é a melhor escolha<br />

(veja o Apêndice C). Este é o sistema de coordenadas<br />

que melhor se adapta à simetria circular da<br />

trajetória da massa m do pêndulo. Entendido isto,<br />

podemos usar o gradiente em coordenadas polares<br />

(C.5), com r = l e ϕ = π/2 constantes, ou seja,<br />

com dr = 0 e dϕ = 0, para re-obtermos a componente<br />

tangencial da força peso segundo a prescrição<br />

(4.9). Note então que a prescrição (4.9) recupera<br />

somente a parte da força resultante que realmente<br />

realiza trabalho. Nada é mais que perfeito.<br />

Como já observamos, o pêndulo simples é um excelente<br />

sistema para aprendermos mecânica. Vimos<br />

em (3.87) que a equação diferencial de um pêndulo<br />

simples se reduz à equação diferencial de um oscilador<br />

harmônico quando o deslocamento angular é<br />

pequeno, θ ≪ 1 rad. Então, para pequenas amplitudes,<br />

o potencial (4.17) deve ser idêntico ao potencial<br />

harmônico (4.13). Para θ ≪ 1 rad, a função<br />

cosseno pode ser aproximada para cos θ ≃ 1 − θ 2 /2<br />

(os dois primeiros termos da série de Taylor). Assim,<br />

o potencial (4.17) torna-se em<br />

V = Iω 2 0(1 − cos θ)<br />

≃ 1 2 Iω2 0θ 2 = 1 2 mω2 0 (lθ) 2 .<br />

(4.19)<br />

Note que lθ é o comprimento do arco compreendido<br />

pelo deslocamento angular θ e faz o papel<br />

da distensão x em (4.13). Ainda mais surpreendente:<br />

a última expressão em (4.19), quando comparada<br />

com o potencial harmônico (4.13), fornece<br />

uma “constante de mola” igual a mω0, 2 massa vezes<br />

a freqüência (angular) natural, a mesma definição<br />

para o sistema massa-mola.<br />

51


4.4. Energia mecânica Capítulo 4. Leis de conservação I<br />

Da geometria plana sabemos que o arco compreendido<br />

4.3.1 Exercícios<br />

⃗∇ = ˆθ d<br />

l dθ . (4.22) E = T + V. (4.26)<br />

pelo ângulo dθ numa circunferência de raio l<br />

Exercício 58<br />

Aplique o operador gradiente (4.10) em (4.13), é l dθ. Então, no sistema polar de coordenadas, o<br />

como indicado em (4.9), e mostre que esta operação vetor deslocamento infinitesimal ao longo de uma<br />

realmente recupera o vetor força elástica F ⃗ =<br />

circunferência de raio l é d⃗s = ldθ ˆθ, um vetor<br />

−kx î.<br />

na direção do versor ˆθ (tangente à trajetória) de<br />

módulo ldθ. Mostre que o vetor velocidade tangencial<br />

é<br />

Exercício 59<br />

Aplique o operador gradiente (4.10) em (4.14),<br />

⃗v = l ˙θ ˆθ. (4.23)<br />

como indicado em (4.9), e mostre que esta operação<br />

realmente recupera o vetor força gravitacional F ⃗ =<br />

−mgˆȷ.<br />

Seguindo este modelo, a componente tangencial da<br />

força peso (a única parte da força resultante que<br />

realmente realiza trabalho, pois a outra parte está<br />

na direção radial, perpendicular ao deslocamento,<br />

Exercício 60<br />

pode ser escrita como F ⃗ θ = −mg sin θ ˆθ. Então<br />

A massa m na Figura 3.4 está localizada na posição<br />

⃗r = x î + yˆȷ. Mostre que o diferencial deste vetor<br />

use a definição (4.8) para mostrar que a energia<br />

potencial calculada por<br />

posição é d⃗r = dx î + dyˆȷ. Agora vamos efetuar<br />

∫ θ<br />

uma mudança de coordenadas. Das coordenadas<br />

V = −<br />

cartesianas (x e y) para coordenadas polares (l e<br />

0<br />

⃗F θ · d⃗s (4.24)<br />

θ): x = l cos θ, y = l sin θ, com l constante. Mostre é igual à energia potencial encontrada em (4.17).<br />

que o diferencial do vetor posição torna-se Feito isto, use a prescrição (4.9), com o gradiente<br />

d⃗r = −l sin θ dθ î + l cos θ dθˆȷ<br />

na forma (C.5), para recuperar F ⃗ θ = −mg sin θ ˆθ.<br />

= −y dθ î + x dθˆȷ. (4.20)<br />

4.4 Energia mecânica<br />

Calcule o vetor velocidade d⃗r/dt, dividindo o diferencial<br />

da posição (4.20) pelo diferencial do tempo<br />

dt. Mostre que este vetor velocidade é perpendicular<br />

ao vetor posição ⃗r = x î + yˆȷ. Mostre que o<br />

módulo deste vetor velocidade é<br />

Considere um sistema contendo apenas forças conservativas.<br />

Então, usando a definição (4.8) de energia<br />

potencial, podemos substituir o trabalho pela<br />

energia potencial correspondente (∆W = −∆V )<br />

v = ∣ d⃗r<br />

no Teorema 7 para obtermos<br />

∣ = l<br />

dt<br />

(4.21)<br />

∆T = ∆W = −∆V<br />

Exercício 61<br />

⇒ ∆T + ∆V = ∆(T + V ) = 0. (4.25)<br />

Determine explicitamente a energia potencial Este resultado é simplesmente surpreendente pois,<br />

(4.17) para o pêndulo simples.<br />

em geral, tanto a energia cinética T quanto a energia<br />

potencial V variam ao longo de uma determinada<br />

Exercício 62<br />

Considere a trajetória circular do pêndulo mostrado<br />

na Figura 3.4. Esta trajetória pode ser descrita<br />

usando o sistema polar (ou esférico) de coordenadas<br />

apresentado no Apêndice C fazendo r = l e<br />

ϕ = π/2. Como r e ϕ são constantes, então seus diferenciais<br />

são nulos. Assim o operador gradiente em<br />

coordenadas polares (C.5), adaptado à trajetória<br />

circular de raio l no plano XY , pode ser re-escrito<br />

como<br />

trajetória, ou seja, são funções do tempo. A<br />

última expressão em (4.25) está nos dizendo que a<br />

soma T + V é uma constante. Além disso, como os<br />

extremos de qualquer trajetória podem ser escolhidos<br />

arbitrariamente, então a soma T + V deve ser<br />

independente do tempo, ou seja, é uma quantidade<br />

conservada. Encontrar uma quantidade conservada<br />

é como encontrar uma agulha em um palheiro. Esta<br />

quantidade conservada merece um nome próprio:<br />

energia mecânica,<br />

52


Capítulo 4. Leis de conservação I<br />

4.4. Energia mecânica<br />

Portanto, a energia mecânica de um sistema contendo<br />

forças conservativas é uma quantidade conservada.<br />

Isto justifica o adjetivo “conservativo”<br />

para as forças conservativas. Um sistema contendo<br />

somente forças conservativas será denominado<br />

também de sistema conservativo. Bem, encontramos<br />

então a outra lei de conservação (além<br />

da conservação do momentum linear):<br />

Teorema 8 (Energia mecânica)<br />

A energia mecânica em um sistema conservativo é<br />

uma quantidade conservada,<br />

∆E = 0, E = T + V. (4.27)<br />

Não podemos esquecer que a definição (4.8) de<br />

energia potencial somente está estabelecida para<br />

sistemas conservativos, ou seja, quando o trabalho<br />

numa trajetória fechado qualquer é nulo. Para sistemas<br />

não-conservativos, não é mais verdade que<br />

o trabalho dependerá apenas das posições inicial<br />

e final, impossibilitando assim a definição de uma<br />

função (potencial) da posição. Além disto, não podemos<br />

garantir que toda energia trocada com um<br />

sistema não-conservativo será armazenda somente<br />

nas formas de energia cinética e potencial, pois<br />

pode haver perdas por qualquer processo dissipativo,<br />

como atrito, por exemplo.<br />

Exemplos, comentários e aplicações. Vamos<br />

tomar novamente um oscilador harmônico constituído<br />

por uma massa m e uma força elástica de<br />

constante k como exemplo. Em um determinado<br />

instante de tempo t qualquer, a energia mecânica<br />

(4.26) deste sistema, juntando os resultados da<br />

Seção 4.2, definição (4.6), e da Seção 4.3, expressão<br />

(4.13), é<br />

E = T + V = 1 2 mẋ2 + 1 2 kx2 , (4.28)<br />

na qual fizemos a escolha V (0) = 0. Este sistema<br />

é um excelente caso de estudo porque nós temos<br />

a equação horária x(t) determinada como uma<br />

função explícita do tempo, a expressão (3.80),<br />

x(t) = A cos(ω 0 t + ϕ), ω 0 =<br />

√<br />

k<br />

m . (4.29)<br />

Podemos ver claramente que tanto a energia<br />

cinética T quanto a energia potencial V estão variando<br />

no tempo. No entanto, quando substituímos<br />

(4.29) em (4.28) (faça o Exercício 63), obtemos<br />

uma energia mecânica constante (independente do<br />

tempo),<br />

E = 1 2 kA2 = 1 2 mω2 0A 2 . (4.30)<br />

Considere agora uma massa m em queda livre de<br />

uma altura h acima da superfície da Terra. Despreze<br />

qualquer tipo de dissipação. Em um determinado<br />

instante de tempo t qualquer, a energia<br />

mecânica (4.26) deste sistema, juntando os resultados<br />

da Seção 4.2, definição (4.6), e da Seção 4.3,<br />

expressão (4.14), é<br />

E = T + V = 1 2 mż2 + mgz (4.31)<br />

na qual fizemos a escolha V (0) = 0. Este sistema<br />

também é um excelente caso de estudo, pois<br />

também temos a equação horária z(t) determinada<br />

como uma função explícita do tempo, expressão<br />

(3.35),<br />

z(t) = h − 1 2 g t2 . (4.32)<br />

Podemos ver claramente que tanto a energia<br />

cinética T quanto a energia potencial V estão variando<br />

no tempo. No entanto, quando substituímos<br />

(4.32) em (4.31) (faça o Exercício 64), obtemos<br />

uma energia mecânica constante (independente do<br />

tempo),<br />

E = mgh. (4.33)<br />

O pêndulo da Figura 3.4 é também um ótimo<br />

exemplo de um sistema conservativo. O módulo do<br />

vetor velocidade da massa m pode ser calculado seguindo<br />

o procedimento descrito no Exercício (60).<br />

A energia potencial deste pêndulo foi calculada anteriormente,<br />

expressão (4.17). Então, a energia<br />

mecânica deste pêndulo pode ser escrita na forma<br />

com<br />

E = T + V = 1 2 I ˙θ 2 + Iω 2 0(1 − cos θ), (4.34)<br />

I = ml 2 , ω 0 =<br />

√ g<br />

l . (4.35)<br />

Embora não tenhamos uma função simples para expressar<br />

a equação horária θ(t) explicitamente em<br />

função do tempo, como nos casos anteriores, para<br />

verificarmos que a energia mecânica (4.34) é de fato<br />

independente do tempo, isto pode ser verificado facilmente<br />

calculando a equação horária θ(t) numericamente<br />

(faça o Exercício 65).<br />

53


4.4. Energia mecânica Capítulo 4. Leis de conservação I<br />

Vimos que o Princípio 2 faz uso de uma quantidade<br />

vetorial conservada (constante no tempo) durante<br />

a interação entre dois ou mais corpos isolados.<br />

Após a introdução do conceito operacional<br />

de massa (inercial), esta quantidade conservada foi<br />

denominada de momentum linear total do sistema<br />

isolado. Vimos também, após a definição de força,<br />

que a variação temporal do momentum linear individual<br />

é igual à força agindo em um corpo. Portanto,<br />

se a força atuando em um corpo é nula, podemos<br />

afirmar que o vetor momentum linear deste<br />

corpo é constante no tempo, ou seja, é uma quantidade<br />

conservada. Este resultado está em pleno<br />

acordo com o Princípio 1. Naturalmente, estaremos<br />

interessados em um sistema físico constituído<br />

por dois ou mais corpos interagindo entre si. No<br />

entanto, por comodidade, vamos concentrar nossa<br />

atenção em apenas dois corpos interagentes. Não<br />

havendo forças externas, então a força resultante<br />

é nula, pois as forças internas cancelam-se aos pares.<br />

Entretanto, o momentum linear resultante não<br />

é nulo, pois não há cancelamento por pares para<br />

o momentum linear. Ao invés de cancelar, o momentum<br />

linear resultante permanece constante no<br />

tempo (veja o Teorema 6),<br />

⃗F = d dt ⃗p ⇒ ⃗p(t 1) = ⃗p(t 2 ) se ⃗ F = ⃗0. (4.36)<br />

Uma quantidade conservada é extremamente<br />

útil. Vejamos alguns exemplos. Considere dois corpos<br />

de massas iguais, m 1 = m 2 = m, movendo-se<br />

na mesma direção, mas em sentidos opostos, com<br />

velocidades ⃗v 2 = −⃗v 1 . Após um certo intervalo de<br />

tempo, estes dois corpos irão colidir entre si. O que<br />

sabemos? Sabemos que num certo instante t 1 , antes<br />

da colisão, o momentum linear total era nulo,<br />

⃗p(t 1 ) = ⃗p 1 (t 1 ) + ⃗p 2 (t 1 ) = m 1 ⃗v 1 (t 1 ) − m 2 ⃗v 1 (t 1 ) = 0.<br />

Supondo que estes dois corpos estejam isolados,<br />

então, devido à lei de conservação (4.36), o momentum<br />

linear total após a colisão deve permanecer<br />

com o mesmo valor que tinha antes da colisão.<br />

Assim, num certo instante t 2 depois da colisão devemos<br />

ter ⃗p(t 2 ) = 0. Portanto há duas possibilidades<br />

para o movimento posterior à colisão: (1) os<br />

dois corpos permanecem unidos em repouso (choque<br />

perfeitamente inelástico), ou (2) os dois corpos<br />

invertem completamente o sentido de suas velocidades,<br />

as quais continuam tendo módulos iguais (choque<br />

perfeitamente elástico).<br />

Numa situação “simétrica” à anterior, imagine<br />

um corpo em repouso. Para ser mais preciso, suponha<br />

que este corpo seja a partícula sub-atômica<br />

káon K 0 , a qual é eletricamente neutra (veja Aventuras<br />

das Partículas). Expontaneamente, o káon<br />

transforma-se (decai) em duas outras partículas,<br />

K 0 → π + + π − , denominadas de píons (de cargas<br />

opostas e massa iguais). Antes do decaimento, o<br />

momentum linear é nulo (káon em repouso). Após<br />

o decaimento, o momentum linear deve ser o mesmo<br />

de antes, pois o processo de decaimento não envolve<br />

forças externas. Portanto, os píons devem sair após<br />

o decaimento com velocidades opostas. Isto é o que<br />

observamos no laboratório.<br />

Nos dois exemplos anteriores, todos os resultados<br />

que obtivemos decorreram somente da lei de<br />

conservação (4.36) para o vetor momentum linear,<br />

independentemente do que ocorre exatamente no<br />

momento de uma colisão ou de um decaimento (ou<br />

de uma explosão). Se lembramos que os processos<br />

físicos envolvidos no momento da colisão são terrivelmente<br />

complexos, a lei de conservação (4.36) é<br />

“A Ferramenta”. Na verdade são três leis de conservação:<br />

uma para cada eixo independente. Entretanto,<br />

nem sempre há conservação do momentum<br />

linear nos três eixos simultaneamente. Pode haver<br />

uma força externa agindo somente em um eixo,<br />

ou somente em dois eixos. O resultado importante<br />

a ser gravado é: se uma componente da força resultante<br />

é nula, então o momentum linear naquela<br />

direção é conservado:<br />

F i = 0 ⇔ p i (t 1 ) = p i (t 2 )<br />

ou mv i (t 1 ) = mv i (t 2 ). (4.37)<br />

Esta é uma forma explícita do Teorema 6, isto é,<br />

escrita em termos de componentes.<br />

h<br />

A<br />

B<br />

θ<br />

g<br />

v B<br />

t = 0 t > 0<br />

Figura 4.1: O corpo A escorrega sem atrito sobre o<br />

corpo B, o qual desliza horizontalmente sem atrito.<br />

Ambos estão sob a ação da gravidade.<br />

v A<br />

54


Capítulo 4. Leis de conservação I<br />

4.5. Períodos<br />

Como exemplo, considere o sistema mostrado na<br />

Figura 4.1. Os dois corpos estão sob a ação da força<br />

gravitacional e não há atrito entre as superfícies.<br />

Então a força gravitacional agindo na vertical é a<br />

única força externa. Portanto, segundo o teorema<br />

da conservação do momentum linear na sua forma<br />

explícita (4.37), haverá conservação da componente<br />

horizontal do momentum linear,<br />

m A v A + m B v B = 0, (4.38)<br />

na qual v A e v B são as velocidades dos dois corpos<br />

imediatamente após perderem o contato entre si.<br />

Use agora a conservação da energia mecânica<br />

(despreze todo tipo de atrito e viscosidade) para<br />

encontrar outra relação que nos permita determinar<br />

completamente as velocidades procuradas.<br />

4.4.1 Exercícios<br />

Exercício 63<br />

Use a equação horária (4.29) de um oscilador<br />

harmônico para escrever a dependência temporal<br />

das energias cinética, potencial e mecânica. Mostre<br />

que a energia mecânica (4.28) é uma constante,<br />

(4.30). Faça um gráfico mostrando estas três energias<br />

para os seus valores preferidos de m e k (em<br />

MKS).<br />

Exercício 64<br />

Use a equação horária (4.32) de uma massa m em<br />

queda livre para escrever a dependência temporal<br />

das energias cinética, potencial e mecânica. Mostre<br />

que a energia mecânica (4.31) é uma constante,<br />

(4.33). Faça um gráfico mostrando estas três energias<br />

para os seus valores preferidos de m e h (em<br />

MKS).<br />

Exercício 65<br />

Resolva a equação diferencial (3.85) numericamente<br />

para encontrar a equação horária θ(t) da massa m<br />

no pêndulo mostrado na Figura 3.4. Mostre que a<br />

energia mecânica (4.34) é uma constante. Faça um<br />

gráfico mostrando as três energias, cinética, potencial<br />

e mecânica, para os seus valores preferidos de<br />

m e l (unidades em MKS).<br />

4.5 Períodos<br />

O oscilador harmônico e o pêndulo simples são<br />

exemplos de sistemas que exibem movimentos<br />

periódicos. A periodicidade destes movimentos são<br />

devidas à forma da energia potencial destes sistemas.<br />

Nem todo potencial produz movimentos<br />

periódicos. A força gravitacional nas proximidades<br />

da Terra, por exemplo, não produz um movimento<br />

periódico. Veremos que o potencial de Kepler é<br />

responsável pelo movimento periódico da Terra em<br />

torno do Sol.<br />

Naturalmente, quando temos a equação horária<br />

de um movimento periódico, como mostrado na Figura<br />

3.5, podemos determinar o seu período por<br />

inspeção direta do gráfico da equação horária. Entretanto,<br />

nem sempre é fácil determinar equações<br />

horárias resolvendo a segunda lei de Newton, uma<br />

equação diferencial de segunda ordem no tempo.<br />

Felizmente, pelo menos para sistemas conservativos,<br />

não precisamos resolver a segunda lei de Newton<br />

para obtermos a equação horária e em seguida<br />

obtermos períodos. Para sistemas conservativos,<br />

podemos usar o fato de que a energia<br />

mecânica é uma quantidade conservada para determinarmos<br />

períodos. Para isto, é crucial vermos<br />

que a expressão da energia mecânica constante é<br />

uma equação diferencial de primeira ordem para a<br />

posição. Ora, porque não utilizamos isto ao invés<br />

da segunda lei? Primeiro, porque a segunda lei se<br />

aplica sempre, enquanto que a conservação da energia<br />

mecânica é válida somente em sistemas conservativos.<br />

Segundo, embora esta equação diferencial<br />

resultante da conservação da energia mecânica seja<br />

de primeira ordem, ela é quadrática na primeira derivada<br />

da posição, portanto não-linear. Equações<br />

diferenciais não-lineares são mais “complicadas”.<br />

Vejamos como este procedimento pode ser colocado<br />

em prática. Vamos considerar inicialmente<br />

um potencial V qualquer. Em geral, este procedimento<br />

é muito utilizado quando o potencial<br />

depende apenas de uma variável, digamos V =<br />

V (x). Então, para sistemas conservativos, a energia<br />

mecânica (4.26) pode ser re-escrita como<br />

E = T + V = 1 2 m( dx) 2<br />

+ V (x). (4.39)<br />

dt<br />

Como E é uma constante, então esta equação é uma<br />

equação diferencial para x(t),<br />

√<br />

dx 2<br />

dt = √<br />

E − V (x). (4.40)<br />

m<br />

Esta equação pode ser resolvida pela técnica de se-<br />

55


4.5. Períodos Capítulo 4. Leis de conservação I<br />

paração de variáveis,<br />

√ m<br />

dt =<br />

2<br />

dx<br />

√<br />

E − V (x)<br />

. (4.41)<br />

Como o tempo aparece apenas no primeiro membro,<br />

podemos integrar independentemente os dois<br />

membros desta equação após especificarmos quem<br />

é V (x). Vejamos alguns exemplos.<br />

V (x)<br />

E<br />

x − 0 x<br />

x<br />

+<br />

Figura 4.2: Pontos de retorno x ± = ± √ 2E/k para<br />

o potencial harmônico V (x) = kx 2 /2, onde E é a<br />

energia mecânica.<br />

Vamos usar o oscilador harmônico para aprendermos<br />

a usar este procedimento. O potencial<br />

harmônico é V (x) = kx 2 /2 (Figura 4.2). Então,<br />

substituindo este potencial na equação diferencial<br />

(4.41), obtemos (faça o Exercício 66)<br />

com<br />

ω 0 =<br />

dt = 1 ω 0<br />

√<br />

k<br />

m ,<br />

dx<br />

√<br />

ε2 − x 2 , (4.42)<br />

ε2 = 2E<br />

mω0<br />

2 . (4.43)<br />

Integrando os dois lados de (4.42), obtemos (faça o<br />

Exercício 66)<br />

x(t) = ε cos(ω 0 t + ϕ), (4.44)<br />

na qual ϕ é uma constante. Lembrando que a energia<br />

mecânica do oscilador harmônico é uma constante<br />

que pode ser expressada em termos da amplitude<br />

A, expressão (4.30), então ε = A. Assim,<br />

re-obtivemos a equação horária (3.80) sem a necessidade<br />

de resolvermos a segunda lei de Newton.<br />

Isto justifica a importância dos teoremas de conservação.<br />

Mas e o período? Observe o gráfico da energia<br />

potencial mostrado na Figura 4.2. Neste gráfico,<br />

também é mostrado a energia mecânica (4.39), a<br />

qual é conservada na ausência de dissipações. Em<br />

especial, podemos ver que há dois pontos, x ± , onde<br />

a energia mecânica é igual à energia potencial. Nestes<br />

pontos, a energia cinética é nula,<br />

E = V (x ± ) = 1 2 kx2 ±<br />

⇒ x ± = ±√<br />

2E<br />

k . (4.45)<br />

Os pontos x ± satisfazendo E = V (x ± ) são denominados<br />

de pontos de retorno. Nestas posições, o vetor<br />

velocidade é nulo. No caso do sistema massa-mola,<br />

os pontos de retorno são as amplitudes máximas,<br />

x ± = ±A.<br />

Vejamos algumas observações importantes sobre<br />

pontos de retorno. (1) O vetor velocidade inverte<br />

seu sentido nos pontos de retorno. Para inverter<br />

o sentido, é necessário primeiro se anular. Assim,<br />

o vetor velocidade se anula nos pontos de retorno.<br />

(2) Havendo dois pontos de retorno, como mostrado<br />

na Figura 4.2, o movimento fica confinado na região<br />

delimitada pelos pontos de retorno.<br />

Para o oscilador harmônico, devido à forma parabólica<br />

do potencial, V (x) = kx 2 /k, quanto maior<br />

a energia mecânica, maior a região de confinamento.<br />

Devemos ressaltar que o valor da energia<br />

mecânica E pode ser escolhido livremente. Desta<br />

forma, para uma determinada energia mecânica<br />

E, a massa m no sistema massa-mola (oscilador<br />

harmônico) fica oscilando entre x − = −A e x + = A<br />

(pontos de retorno) com uma freqüência angular<br />

igual a ω 0 . Portanto, o período de uma oscilação é<br />

exatamente o tempo da massa m sair de x − , ir até<br />

x + e retornar a x − . Como o potencial é simétrico,<br />

V (−x) = V (x), então o tempo de sair de x − e ir<br />

até x + é metade do período. A equação diferencial<br />

(4.42) pode ser usada para calcularmos o período<br />

P (faça o Exercício (67)),<br />

com<br />

ω 0 =<br />

P = 2 ω 0<br />

∫ x+<br />

√<br />

k<br />

m ,<br />

e os pontos de retorno<br />

x −<br />

ε2 = 2E<br />

mω 2 0<br />

x ± = ±√<br />

2E<br />

k<br />

dx<br />

√<br />

ε2 − x 2 , (4.46)<br />

= 2E k , (4.47)<br />

= ±ε. (4.48)<br />

56


Capítulo 4. Leis de conservação I<br />

4.5. Períodos<br />

Esta integral pode ser resolvida efetuando a substituição<br />

x = ε cos θ (faça o Exercício 66). O resultado<br />

é o que esperávamos: P = 2π/ω 0 , ou seja, o sistema<br />

oscila com um período correspondente à sua<br />

freqüência angular natural ω 0 . Aprendemos então<br />

que a conservação da energia mecânica pode ser<br />

usada tanto para encontrarmos a posição em função<br />

do tempo, quanto para calcularmos períodos.<br />

O ponto x = 0 na Figura 4.2 é especial. Neste<br />

ponto, a força elástica, a derivada primeira do potencial,<br />

com o sinal trocado, F = −kx, é nula.<br />

Então, x = 0 é um ponto de equilíbrio (força nula).<br />

Também podemos ver que um deslocamento, tanto<br />

para a direita quanto para a esquerda, resulta numa<br />

força restauradora que tenta trazer a massa de<br />

volta para a posição de equilíbrio. Um ponto de<br />

equilíbrio com uma força restauradora agindo nas<br />

vizinhanças deste ponto é denominado de ponto de<br />

equiĺıbrio estável. Do ponto de vista matemático,<br />

um ponto de equilíbrio estável é sempre um ponto<br />

de mínimo da função energia potencial.<br />

Vamos aplicar o procedimento anterior a outros<br />

sistemas. Considere o pêndulo da Figura 3.4. A<br />

energia mecânica deste sistema está calculada em<br />

(4.34), a qual pode ser re-escrita numa forma mais<br />

conveniente E = T + V , onde<br />

com<br />

T = ε ω 0 ˙θ2 , V = ε(1 − cos θ), (4.49)<br />

ε = Iω 2 0, I = ml 2 , ω 0 =<br />

√ g<br />

l . (4.50)<br />

Note que a quantidade ε = Iω 2 0 tem dimensões de<br />

energia (faça o Exercício (67)). Veremos mais adiante,<br />

quando estivermos estudando o movimento de<br />

rotação em torno de um eixo fixo, que Iω 2 0/2 é a<br />

energia cinética devido a rotação de um corpo com<br />

momento de inércia I e velocidade angular ω 0 . Note<br />

a semelhança com a expressão da energia cinética<br />

mv 2 /2. Por isto, iremos usar ε como uma “ unidade”<br />

de energia. Note também que a massa m do<br />

pêndulo mostrado na Figura 3.4 executa um movimento<br />

de rotação em torno do eixo fixo que passa<br />

pela outra extremidade da haste do pêndulo.<br />

V/ε<br />

2<br />

E/ε<br />

−π<br />

θ<br />

θ − − π π<br />

2<br />

0<br />

2<br />

θ +<br />

π<br />

Figura 4.3: Pontos de retorno θ ± = ± cos −1 (−ϵ)<br />

para o potencial (4.49) de um pêndulo simples.<br />

A Figura 4.3 mostra os pontos de retorno,<br />

θ ± = ± cos −1 (−ϵ), ϵ = E ε<br />

1<br />

− 1, (4.51)<br />

para o potencial V (θ) = ε(1−cos θ) do pêndulo simples;<br />

confira a energia mecânica em (4.49). Como<br />

este potencial é periódico, apenas um período, contendo<br />

a posição de equilíbrio estável θ = 0, é<br />

mostrado na Figura 4.3. Note que este potencial<br />

também é uma função par, V (−θ) = V (θ). Isto<br />

nos permite calcular o período como o dobro do<br />

tempo entre gasto entre θ − e θ − . Isolando a primeira<br />

derivada em (4.49), assumindo uma energia<br />

mecânica constante (sistema conservativo, sem dissipação),<br />

temos<br />

P =<br />

√ ∫ 2<br />

θ+<br />

dθ<br />

√ . (4.52)<br />

ω 0 θ − ϵ + cos θ<br />

Esta integral deve ser resolvida numericamente<br />

(faça o Exercício 68).<br />

Note que a integral (4.52) é uma função da constante<br />

adimensional ϵ = E/ε − 1, a qual pode variar<br />

no intervalo real desde ϵ = −1 (E = 0, equilíbrio<br />

estável, fundo do potencial) até ϵ = 1 (E = 2ε,<br />

equilíbrio instável, topo do potencial; veja a discussão<br />

do próximo parágrafo). Esta integral é nula<br />

para ϵ = −1 (período nulo) e é infinita para ϵ = 1<br />

(período infinito). Portanto, em princípio, podemos<br />

dar à massa m uma energia inicial (θ = 0)<br />

exatamente igual a E = 2ε, a energia necessária<br />

para que a massa m consiga chegar até a posição<br />

de equilíbrio instável (θ = π). No entanto, a parte<br />

divertida disto é que ela leva um tempo infinito<br />

para chegar até lá! É muito interessante ver isto<br />

nas simulações feitas em computação algébrica.<br />

Para uma energia mecânica maior que a energia<br />

potencial máxima, E > 2ε, o movimento do<br />

57


4.5. Períodos Capítulo 4. Leis de conservação I<br />

pêndulo deixa de ser oscilatório e passa a ser uma<br />

rotação em torno de um eixo fixo. O período desta<br />

rotação também pode ser calculado a partir da conservação<br />

da energia mecânica (4.49) como sendo o<br />

tempo de uma volta completa (faça o Exercício 68),<br />

com<br />

P =<br />

√<br />

2<br />

2ω 0<br />

∫ 2π<br />

0<br />

dθ<br />

√<br />

ϵ + cos θ<br />

, (4.53)<br />

ϵ = E − 1, E > 2ε. (4.54)<br />

ε<br />

Mencionamos logo acima o termo “equilíbrio<br />

instável”. Vejamos o significado disto. Podemos<br />

ver que o potencial mostrado na Figura 4.3 possui<br />

mais de um ponto de equilíbrio (força nula). Em<br />

θ = 0, ponto de mínimo do potencial, temos um<br />

ponto de equilíbrio estável. Em θ = ±π, pontos<br />

de máximos, temos um tipo de equilíbrio diferente<br />

do estável. Segundo a discussão feita no final da<br />

Seção 4.3 (reveja também o Exercício 62), a componente<br />

tangencial da força peso (a única que realiza<br />

trabalho) pode ser escrita na forma<br />

⃗F θ = − ˆθ dV<br />

l dθ = −ε l sin θ ˆθ = −mg sin θ ˆθ. (4.55)<br />

Considere agora a vizinhança de θ = π. O versor<br />

ˆθ aponta sempre no sentido horário, como indicado<br />

pelo deslocamento do ângulo θ na Figura 4.3. Suponha<br />

que a massa m esteja em repouso em θ = π.<br />

Produza um deslocamento no sentido horário. Portanto<br />

este deslocamento terá o mesmo sentido do<br />

versor ˆθ. No entanto o ângulo θ será ligeiramente<br />

maior que π. Isto resulta numa força tangencial, segundo<br />

a expressão (4.55), no mesmo sentido do deslocamento<br />

dado, afastando cada vez mais a massa<br />

m da posição de equilíbrio θ = π. Produza agora<br />

um deslocamento no sentido anti-horário. Este deslocamento<br />

estará no sentido oposto ao versor ˆθ. No<br />

entanto o ângulo θ será ligeiramente menor que π.<br />

Isto resulta numa força tangencial, segundo a expressão<br />

(4.55), no mesmo sentido do deslocamento,<br />

afastando cada vez mais a massa m da posição de<br />

equilíbrio θ = π. Assim, a força agindo nas vizinhanças<br />

deste novo ponto de equilíbrio, θ = π, não<br />

é restauradora. Muito pelo contrário, esta força<br />

sempre afasta a massa m do ponto θ = π. Estas<br />

observações também valem para θ = −π. Um<br />

ponto de equilíbrio que não tem uma força restauradora<br />

em suas vizinhanças é denominado de ponto<br />

de equiĺıbrio instável.<br />

Matematicamente falando, um ponto de<br />

equilíbrio instável é um ponto de máximo da<br />

função potencial. Há também um terceiro tipo<br />

de equilíbrio, denominada de neutro (ou indiferente).<br />

A força é nula nas vizinhanças de um<br />

ponto de equilíbrio neutro. A diferença com o<br />

ponto de equilíbrio estável está no tamanho do<br />

deslocamento em torno do ponto de equilíbrio para<br />

mudar o estado do corpo. No equilíbrio estável,<br />

um deslocamento infinitesimal é suficiente para<br />

recolocar o corpo em movimento. No equilíbrio<br />

neutro, é necessário um deslocamento finito para<br />

recolocar o corpo em movimento. Também vale<br />

ressaltar que a função potencial deve exibir pelo<br />

menos um ponto de equilíbrio estável para haver<br />

um movimento periódico.<br />

4.5.1 Exercícios<br />

Exercício 66<br />

(I) Resolva a equação diferencial (4.42) integrando<br />

indefinidamente e independentemente os dois lados.<br />

Calcule a integral na posição efetuando a substituição<br />

x = ε cos θ. Não esqueça de colocar uma<br />

constante de integração. Re-arranje tudo para obter<br />

(4.44). (II) Efetue a integral definida (4.46)<br />

fazendo x = ε cos θ e mostre que o resultado é<br />

P = 2π/ω 0 .<br />

Exercício 67<br />

Mostre que as dimensões da quantidade ε = Iω 2 0,<br />

definida em (4.49), tem as mesmas dimensões de<br />

energia. Use a energia mecânica dada em (4.49)<br />

para escrever as integrais (4.52) e (4.53) que calculam<br />

o período para E ≤ 2ε e E > 2ε, respectivamente.<br />

Exercício 68<br />

Use a integral (4.52) para determinar o período do<br />

pêndulo usado para a construção das duas primeiras<br />

equações horárias mostradas na Figura 3.5. Depois<br />

use a integral (4.53) para determinar o período<br />

do pêndulo usado para a construção da terceira<br />

equação horária mostrada na mesma Figura 3.5.<br />

Use uma massa de 1 kg e demais valores contidos<br />

na discussão da Figura 3.5. Note que você precisa<br />

calcular a energia mecânica em cada caso. Despreze<br />

qualquer tipo de dissipação. Uma integral numérica<br />

no Maple deve ser feita usando o seguinte modelo:<br />

∫ b<br />

a<br />

f(x)dx = evalf(Int(f(x), x = a..b)). (4.56)<br />

58


Capítulo 4. Leis de conservação I<br />

4.6. Comentários gerais<br />

É imprescindível o uso da primeira letra maiúscula<br />

na função “Int”.<br />

4.6 Comentários gerais<br />

Definimos potência em (4.4) e não a usamos desde<br />

então. Potência é uma ferramenta fundamental em<br />

engenharia. Por isso, ela merece todo o nosso respeito.<br />

Vamos usar o pêndulo simples da Figura 3.4<br />

para ilustrar o conceito de potência. Vamos considerar<br />

duas situações: sem e com dissipação. Sem<br />

dissipação, o pêndulo troca energia pelo trabalho<br />

realizado pela componente tangencial (4.55) da<br />

força gravitacional, F ⃗ θ = −mg sin θ ˆθ. O vetor velocidade<br />

tangencial pode ser escrito na forma ⃗v = l ˙θ ˆθ<br />

(Exercício 62). Então a potência instantânea associada<br />

à força F ⃗ θ , segundo a definição (4.4), é<br />

P θ = ⃗ F θ · ⃗v = −mgl sin θ ˙θ = −Iω 2 0 sin θ ˙θ. (4.57)<br />

Esta é a mesma expressão que é obtida quando derivamos<br />

o trabalho realizado pela força F ⃗ θ desde<br />

θ = 0 até θ qualquer, pois este trabalho é menos<br />

a energia potencial (4.17), W = −Iω0(1 2 − cos θ).<br />

A potência (4.57) mede a taxa temporal da transferência<br />

de energia potencial para a massa m, a qual<br />

é transformada em energia cinética na mesma taxa<br />

(com o sinal invertido). Neste caso, sem dissipação,<br />

o sistema possui uma energia mecânica constante.<br />

Isto significa que este sistema está isolado, ou seja,<br />

não troca energia com o meio externo.<br />

Quando o sistema tem dissipação, a situação<br />

energética muda um pouco. Primeiro temos que<br />

modelar uma dissipação para o nosso pêndulo. Por<br />

conveniência, vamos usar uma força dissipativa proporcional<br />

à velocidade tangencial,<br />

⃗F d = −b⃗v = −bl ˙θ ˆθ (4.58)<br />

Note que a constante b tem dimensões de massa por<br />

tempo por comprimento. Com esta força dissipativa,<br />

a equação diferencial (3.85) é alterada para<br />

com<br />

ω 0 =<br />

¨θ + ω 2 0 sin θ + ω 1 ˙θ = 0, (4.59)<br />

√ g<br />

l , ω 1 = b ml = bl<br />

I , I = ml2 . (4.60)<br />

Usando a definição (4.4), podemos calcular a<br />

potência associada à força dissipativa,<br />

P = ⃗ F d · ⃗v = −bl 2 ˙θ2 = −Iω 1 l ˙θ 2 . (4.61)<br />

Esta é a taxa temporal com que o pêndulo perde<br />

energia para a sua vizinhança. Neste caso, a energia<br />

mecânica não é mais uma constante,<br />

dE<br />

dt<br />

= P. (4.62)<br />

É muito interessante ver isto nas simulações feitas<br />

em computação algébrica.<br />

Em mecânica clássica (Newtoniana), a energia<br />

mecânica é um número real, o qual pode variar<br />

continuamente em um certo intervalo. No caso do<br />

oscilador, E ∈ [0, ∞) (veja a Figura 4.2). Entretanto,<br />

este quadro muda drasticamente no mundo<br />

microscópico. O potencial harmônico, V (x) =<br />

kx 2 /k, é uma primeira aproximação para modelar<br />

uma ligação química em uma molécula diatômica:<br />

dois núcleos presos por uma mola. De fato,<br />

este modelo consegue explicar o comportamento<br />

mecânico de uma molécula diatômica quando seus<br />

núcleos estão em movimento em torno de suas<br />

posições de equilíbrio com amplitudes muito pequenas.<br />

Neste caso, estes movimentos serão oscilações<br />

harmônicas. Até aqui, nada de surpresas. A surpresa<br />

aparece quando olhamos para as possíveis<br />

energias mecânicas de uma molécula diatômica:<br />

seus possíveis valores formam um conjunto discreto<br />

de valores, não são mais contínuas, livres para serem<br />

escolhidas como no caso clássico. Esta discretização<br />

da energia é denominada de quantização da<br />

energia. As leis Newtonianas deixam de ser válidas<br />

no mundo microscópico. Elas são corrigidas pelas<br />

leis da mecânica quântica. No entanto, todas as leis<br />

de conservação que aprendemos no mundo clássico<br />

continuam válidas no mundo quântico. Isto reforça<br />

ainda mais a importância das leis de conservação.<br />

Veja só onde viemos dar água de beber aos nossos<br />

burrinhos: começamos praticamente do nada,<br />

procurando quantificar nossa noção intuitiva sobre<br />

esforços. Definimos o trabalho (4.1), apelando<br />

para a simplicidade. O trabalho mostrou-se mais<br />

ou menos satisfatório quando comparamos seus resultados<br />

com a nossa experiência cotidiana. No<br />

entanto, aquela definição de trabalho nos revelou<br />

ser melhor que a encomenda, pois conseguimos interpretá-la<br />

como um mecanismo de troca de energia,<br />

Teorema 7, nos possibilitando descobrir energia<br />

cinética, (4.6). Além disto, trabalho também nos<br />

revelou a energia potencial (4.8), para sistemas conservativos.<br />

Em seguida, fizemos duas descobertas<br />

magníficas: primeiro descobrimos que a força pode<br />

59


4.6. Comentários gerais Capítulo 4. Leis de conservação I<br />

ser obtida da energia potencial. Portanto, energia<br />

potencial passa a ter um papel fundamental:<br />

ela é responsável pelo comportamento dinâmico de<br />

qualquer sistema. Depois, descobrimos a lei de conservação<br />

da energia mecânica, a qual juntamente<br />

com a lei de conservação do momentum linear nos<br />

possibilita encontrar a solução de muitos problemas<br />

importantes em mecânica. Além disto, as leis de<br />

conservação são as únicas informações que aprendemos<br />

no mundo clássico (sistemas macroscópicos)<br />

que continuam válidas no mundo quântico (sistemas<br />

microscópicos). Leis de conservação são robustas.<br />

Solte a imaginação e formule muitas questões.<br />

Por exemplo, se a soma E = T + V é uma quantidade<br />

conservada em um sistema conservativo, e<br />

somente esta soma foi encontrada até o momento<br />

como uma quantidade conservada, o que podemos<br />

dizer da diferença T − V ? É fazendo perguntas que<br />

descobrimos o novo. A quantidade T − V tem uma<br />

importância fundamental. Se é importante, merece<br />

um nome: função Lagrangiana, L = T −V . vejamos<br />

alguns exemplos.<br />

A função Lagrangiana para o sistema massa-mola<br />

(força elástica) é<br />

é<br />

L = T − V = 1 2 mż2 − mgz. (4.67)<br />

Novamente, podemos ver que a segunda lei de Newton<br />

pode ser obtida a partir das equações de Euler-<br />

Lagrange (4.66), pois<br />

∂L<br />

= mż,<br />

∂ż<br />

(4.68)<br />

∂L<br />

= −mg.<br />

∂x<br />

(4.69)<br />

Até aqui, as equações de Euler-Lagrange (4.66)<br />

podem parecer apenas uma outra maneira de escrevermos<br />

a segunda lei de Newton. No entanto, esta<br />

formulação em termos da Lagrangiana L = T − V<br />

é muito mais que uma forma alternativa. A importância<br />

da formulação Lagrangiana reside no fato<br />

dela ser usada integralmente na física quântica, em<br />

substituição à segunda lei de Newton.<br />

L = T − V = 1 2 mẋ2 − 1 2 kx2 . (4.63)<br />

Agora considere (por um momento) x e ẋ como<br />

variáveis independentes. Desta forma, a função Lagrangiana<br />

(4.63) torna-se em uma função de duas<br />

variáveis, L = L(x, ẋ). Assim, podemos calcular as<br />

seguintes derivadas (parciais):<br />

∂L<br />

= mẋ,<br />

∂ẋ<br />

(4.64)<br />

∂L<br />

= −kx.<br />

∂x<br />

(4.65)<br />

Note que a derivada temporal de (4.64) é a definição<br />

de força mẍ e que (4.65) é exatamente a<br />

força elástica. Portanto, a segunda lei de Newton<br />

pode ser re-escrita na forma<br />

( )<br />

d ∂L<br />

dt ∂ẋ<br />

= ∂L<br />

∂x . (4.66)<br />

Esta forma de escrever a segunda lei de newton é<br />

conhecida por equações de Euler-Lagrange.<br />

Considere agora o caso da força gravitacional no<br />

sistema massa-Terra. A Lagrangiana deste sistema<br />

60


Capítulo 5<br />

Rotações<br />

5.1 Cinemática<br />

Estudamos até então as leis que controlam um tipo<br />

de movimento: o movimento de translação. No<br />

entanto, o movimento mais geral é composto de<br />

translação e rotação. Rotações estão presentes no<br />

nosso cotidiano: o disco rígido do seu computador<br />

está em rotação, as rodas de um automóvel estão<br />

em rotação, as hélices de um ventilador ou de um<br />

avião estão em rotação, a Terra está em rotação<br />

neste exato instante. Quais são as leis que controlam<br />

o movimento de rotação?<br />

Vamos iniciar nossos estudos com a parte cinemática<br />

do movimento de rotação. Precisaremos<br />

definir três quantidades vetoriais: deslocamento<br />

angular, velocidade angular e aceleração angular.<br />

Para simplificar nossa tarefa, vamos considerar<br />

um corpo rígido em rotação em torno de um<br />

eixo fixo. Mais tarde, poderemos permitir que o<br />

eixo de rotação esteja em movimento translacional.<br />

Corpo rígido, o que é um corpo rígido? Um<br />

corpo rígido é um sistema de partículas, discreto<br />

ou contínuo, onde as distâncias relativas entre<br />

partículas são mantidas fixas por forças internas. É<br />

importante termos em mente que as partículas que<br />

constituem um corpo rígido não apresentam qualquer<br />

tipo de movimento relativo entre elas. Desta<br />

forma, podemos nos concentrar em uma partícula<br />

qualquer do corpo rígido para definirmos nossas<br />

quantidades cinemáticas.<br />

A Figura 5.1 mostra de forma esquemática um<br />

corpo rígido em rotação em torno de um eixo<br />

fixo. No momento, não importa escrever o eixo de<br />

rotação em algum sistema de coordenadas. O fato<br />

importante aqui é que este eixo de rotação define<br />

uma direção particular no espaço. Vamos colocar<br />

um versor ˆn ao longo do eixo de rotação, como indicado<br />

na Figura 5.1.<br />

ˆn<br />

r<br />

dm<br />

dθ<br />

t + dt<br />

ds<br />

t<br />

ω<br />

l.r.<br />

Figura 5.1: Rotação de um corpo rígido em torno<br />

de um eixo fixo. dm é uma quantidade infinitesimal<br />

de massa deste corpo rígido. A linha de referência<br />

(l.r.) é perpendicular ao eixo de rotação.<br />

Além do eixo de rotação, também precisaremos<br />

de um segundo eixo fixo, perpendicular ao eixo de<br />

rotação, o qual denominaremos simplesmente de<br />

“linha de referência” (l.r.). Esta linha de referência<br />

é usada para medirmos o deslocamento angular θ<br />

da seguinte forma: em um certo instante t, nós<br />

prestamos atenção a uma determinada partícula de<br />

massa dm, localizada a uma distância r do eixo<br />

de rotação, como mostrado na Figura 5.1. Note<br />

que r é a distância de um ponto a uma reta (passando<br />

pelo eixo de rotação). Num instante posterior<br />

t+dt, nossa partícula encontra-se numa posição<br />

diferente, devido ao movimento de rotação. Como<br />

esta partícula (bem como todas as outras) está fixa<br />

no corpo rígido, a distância r dela até o eixo de<br />

61


5.1. Cinemática Capítulo 5. Rotações<br />

rotação não pode mudar. Assim, a nossa partícula<br />

executa um movimento circular de raio r. Assim, a<br />

posição de uma partícula qualquer no corpo rígido<br />

pode ser especificada por um único número (um<br />

grau de liberdade): o ângulo θ medido em radianos<br />

a partir da linha de referência. Se este ângulo é<br />

θ em t, então em t + dt ele é θ + dθ. Feito isto,<br />

temos tudo que precisamos para definirmos nossas<br />

grandezas cinemáticas.<br />

O vetor deslocamento angular é definido como<br />

sendo o vetor<br />

⃗θ = θ ˆn, (5.1)<br />

ou seja, um vetor na direção do eixo de rotação, de<br />

módulo igual ao deslocamento angular. O sentido<br />

do vetor deslocamento angular é dado pela regra<br />

da mão direita onde o indicador indica o sentido<br />

da rotação e o polegar indica o sentido do versor ˆn.<br />

Não podemos esquecer que o deslocamento angular<br />

deve ser calculado em radianos (π radianos = 180<br />

graus).<br />

Em geral, o ângulo de rotação θ = θ(t) terá uma<br />

dependência temporal complicada. Conseqüentemente,<br />

a taxa de variação temporal do vetor deslocamento<br />

angular é a velocidade angular e a taxa de<br />

variação temporal do vetor velocidade angular é a<br />

aceleração angular,<br />

⃗ω = d⃗ θ<br />

dt = ˙θˆn,<br />

⃗α = d⃗ω<br />

dt = ¨θˆn, (5.2)<br />

respectivamente. Observe que estes três vetores são<br />

paralelos ao eixo de rotação. Note também que o<br />

versor ˆn é independente do tempo (eixo de rotação<br />

fixo). Tome cuidado para não confundir a direção e<br />

o sentido do vetor velocidade angular com a própria<br />

rotação. A unidade de velocidade angular é radianos<br />

por segundo (rad/s) e da aceleração angular é<br />

radianos por segundo por segundo (rad/s 2 ). Radiano<br />

não tem dimensões; ele indica apenas que uma<br />

circunferência de raio unitário tem 2π subintervalos<br />

angulares medidos em radianos, equivalentes a 360<br />

subintervalos angulares medidos em graus (faça o<br />

Exercício 69).<br />

Note a semelhança entre as quantidades cinemáticas<br />

do movimento de translação com as<br />

quantidades cinemáticas do movimento de rotação.<br />

De fato, podemos ir além desta similaridade. Observe<br />

novamente a Figura 5.1. Nela, o deslocamento<br />

ds pode ser relacionado ao deslocamento angular dθ<br />

pela geometria plana:<br />

ds = r dθ. (5.3)<br />

Assim, podemos relacionar o módulo da velocidade<br />

tangencial v da nossa partícula com o módulo da<br />

velocidade angular ω,<br />

v = ds<br />

dt = r dθ = rω. (5.4)<br />

dt<br />

Naturalmente, derivando mais uma vez em relação<br />

ao tempo, lembrando que a distância r é independente<br />

do tempo, obteremos uma relação similar entre<br />

o módulo da aceleração tangencial a e o módulo<br />

da aceleração angular α,<br />

a = dv<br />

dt = r dω = rα. (5.5)<br />

dt<br />

Como a nossa partícula não pode movimentar-se<br />

na direção radial, a aceleração radial (centrípeta)<br />

não precisa ser considerada.<br />

Note que as relações (5.3), (5.4) e (5.5) são escalares.<br />

Podemos obter uma versão vetorial para<br />

elas? Certamente. Basta usarmos algumas propriedades<br />

do produto vetorial. Primeiro, observe<br />

que a velocidade tangencial e a velocidade angular<br />

são vetores perpendiculares entre si. Mesma<br />

observação para as acelerações. Podemos também<br />

definir um vetor “distância” ⃗r = rˆr, com o versor<br />

ˆr apontando (perpendicularmente) do eixo de<br />

rotação para a partícula. Este versor ˆr é perpendicular<br />

a todos os demais vetores (ˆr · ˆn = 0). Então<br />

temos duas trincas de vetores mutuamente ortogonais,<br />

{⃗v, ⃗r, ⃗ω} e {⃗a, ⃗r, ⃗α}. Também conhecemos a<br />

relação entre seus módulos, (5.4) e (5.5). Portanto,<br />

após um pouco de reflexão, podemos escrever<br />

d⃗s = −⃗r × d ⃗ θ, ⃗v = −⃗r × ⃗ω, ⃗a = −⃗r × ⃗α. (5.6)<br />

Certifique-se que as relações (5.4) e (5.5) podem<br />

ser obtidas de (5.6) e que o sentido de ⃗v e<br />

⃗a estão corretos. Vale observar que podemos escrever<br />

também uma relação vetorial entre os vetores<br />

deslocamentos somente numa forma infinitesimal.<br />

Mantenha sempre em mente que r aqui é a<br />

distância entre o ponto onde está a massa sendo observada<br />

e o eixo de rotação (distância de um ponto<br />

até uma reta). Lembre-se que também usamos r,<br />

em outras ocasiões, para indicar o módulo do vetor<br />

posição ⃗r em algum sistema de coordenadas. Deixaremos<br />

claro quando estaremos usando o vetor ⃗r<br />

62


Capítulo 5. Rotações<br />

5.2. Energia rotacional<br />

para indicar o vetor posição em relação a origem de<br />

algum sistema de coordenadas.<br />

Importante. Guarde a estrutura geral entre as<br />

grandezas cinemáticas translacionais e rotacionais:<br />

grandezas translacionais = produto vetorial entre<br />

as grandezas cinemáticas rotacionais e o vetor<br />

distância ⃗r, onde r = |⃗r| é a distância entre o eixo<br />

de rotação e uma partícula do corpo rígido.<br />

Não devemos esquecer que as grandezas cinemáticas<br />

rotacionais são definidas em termos de<br />

um eixo de rotação, o qual deve ser mantido fixo no<br />

espaço. Também não deve ser esquecido que todas<br />

as partículas do corpo rígido possuem as mesmas<br />

grandezas cinemáticas rotacionais, mas podem ter<br />

grandezas cinemáticas translacionais distintas. Por<br />

exemplo, quanto mais distante do eixo de rotação,<br />

maior é a velocidade tangencial de uma partícula.<br />

Como exemplo, temos a agência espacial européia,<br />

instalada na América do Sul (Guiana Francesa),<br />

próxima ao equador. Assim, parte da velocidade<br />

de escape será fornecida pelo movimento de<br />

rotação da Terra. Esta contribuição é máxima no<br />

equador.<br />

Outra aplicação das relações (5.6) é na industria<br />

fonográfica. No disco de vinil, a velocidade<br />

angular é mantida constante. Isto significa que um<br />

anel circular na borda interna contém o mesmo intervalo<br />

de tempo de informação musical que um<br />

anel na borda externa, pois ambos têm um deslocamento<br />

angular igual a 2π radianos. Uma vez<br />

que a velocidade angular é a mesma, o intervalo<br />

de tempo também será o mesmo. A desvantagem<br />

neste processo é a necessidade da informação no<br />

anel interno ser comprimida, pois o comprimento<br />

deste anel é menor. Em um CD, o mecanismo de<br />

rotação é construído de tal forma a manter a velocidade<br />

tangencial constante onde a leitura (via um<br />

laser) está sendo feita. Desta forma, a informação<br />

musical pode ser gravada de forma uniforme.<br />

5.2 Energia rotacional<br />

Se tem movimento, deve haver energia cinética.<br />

Neste caso, energia cinética rotacional. Supondo<br />

que o nosso corpo rígido da Figura 5.1 seja<br />

contínuo (discreto), então a nossa partícula deve<br />

ter uma massa infinitesimal (finita) dm (m i ) a uma<br />

distância r (r i ) do eixo de rotação. Caso o corpo<br />

rígido seja discreto, então ele é constituído por N<br />

partículas de massa m i (i = 1, 2, . . . , N). Em qualquer<br />

caso, a energia cinética total é a soma das<br />

energias cinéticas de cada partícula,<br />

T =<br />

∫ 1<br />

2 dm v2 =<br />

∫ 1<br />

2 dm (rω)2 = 1 2 Iω2 , (5.7)<br />

com<br />

∫ ∫<br />

I = r 2 dm, dm = M (caso contínuo), (5.8)<br />

I =<br />

N∑<br />

ri 2 m i ,<br />

i=1<br />

N∑<br />

m i = M (caso discreto). (5.9)<br />

i=1<br />

A quantidade I é denominada de momento de<br />

inércia.<br />

Note a semelhança da última expressão em (5.7)<br />

com a expressão da energia cinética de uma massa<br />

m em um movimento translacional (T = mv 2 /2).<br />

Desta semelhança aprendemos o significado físico<br />

do momento de inércia: ele mede a inércia rotacional.<br />

Quanto maior o momento de inércia de um<br />

corpo, mais difícil é alterar seu estado de rotação.<br />

Porém, temos aqui uma novidade importante: o<br />

momento de inércia definido em (5.8) depende da<br />

escolha do eixo de rotação (através da distância r<br />

até o eixo). Portanto, ao contrário da massa m,<br />

o momento de inércia I não é uma propriedade<br />

intrínseca de um corpo.<br />

Vejamos o que acontece quando nosso corpo<br />

rígido é composto por uma única partícula (pontual)<br />

de massa m. Neste caso o momento de inércia<br />

(5.8) é I = mr 2 . A energia cinética rotacional (5.7)<br />

pode ser escrita como 2T = mr 2 ω 2 = mv 2 , a qual<br />

é a forma usual da energia cinética do movimento<br />

translacional (circular). Lembre-se que o eixo de<br />

rotação deve ser mantido fixo.<br />

Vamos supor agora que o nosso corpo rígido contenha<br />

duas partículas de massas m 1 e m 2 , separadas<br />

por uma distância l. Vamos supor inicialmente<br />

que o eixo de rotação passe pela partícula<br />

m 1 . Neste caso, o momento de inércia deste sistema<br />

é<br />

I = m 1 0 2 + m 2 l 2 = m 2 l 2 . (5.10)<br />

Conseqüentemente, a energia cinética rotacional (a<br />

única energia cinética deste sistema) é<br />

T = 1 2 Iω2 = 1 2 m 2l 2 ω 2 = 1 2 m 2v 2 2. (5.11)<br />

Esta é a mesma energia cinética de uma única<br />

partícula de massa m 2 girando a uma distância l<br />

63


5.2. Energia rotacional Capítulo 5. Rotações<br />

do eixo de rotação. Vamos agora mudar o eixo de<br />

rotação para o centro de massa 1 do sistema, definido<br />

pela posição média (com pesos dados pelas<br />

massas)<br />

⃗R = 1 M<br />

(<br />

m1 ⃗r 1 + m 2 ⃗r 2<br />

)<br />

, M = m1 + m 2 , (5.12)<br />

onde ⃗r i são os vetores posições das massas m i em<br />

relação a algum sistema de coordenadas previamente<br />

escolhido de forma que |⃗r 1 − ⃗r 2 | = l Devemos<br />

tomar o cuidado de manter este novo eixo de<br />

rotação paralelo ao primeiro. Se os módulos r 1 e r 2<br />

satisfazem l = r 1 +r 2 , isto significa que a origem do<br />

nosso sistema de coordenadas foi colocada no centro<br />

de massa, ⃗ R = 0 (faça o Exercício 70). Com esta<br />

escolha, temos m 1 r 1 = m 2 r 2 (faça o Exercício 70).<br />

Aqui, tanto r 1 quanto r 2 são constantes no tempo.<br />

O momento de inércia neste caso,<br />

I 0 = m 1 r 2 1 + m 2 r 2 2 = m 1<br />

m 2<br />

(m 1 + m 2 )r 2 1, (5.13)<br />

é diferente do anterior calculado em (5.10), ou seja<br />

(faça o Exercício 70),<br />

I = I 0 + Mr 2 1. (5.14)<br />

Este resultado é importante e é conhecido como o<br />

teorema dos eixos paralelos. Como este sistema de<br />

duas partículas que utilizamos tem nada de especial,<br />

podemos esperar que este resultado seja válido<br />

para qualquer outro sistema. De fato, sem nos preocuparmos<br />

com uma demonstração formal, podemos<br />

enunciá-lo de forma geral.<br />

Teorema 9 (Eixos Paralelos)<br />

Seja M a massa total de um corpo rígido (contínuo ou<br />

discreto). Seja I 0 o momento de inércia em relação<br />

a um eixo de rotação passando pelo centro de massa.<br />

Seja I o momento de inércia em relação a um segundo<br />

eixo de rotação paralelo ao primeiro. Seja l a distância<br />

entre estes dois eixos de rotação. Então,<br />

I = I 0 + Ml 2 . (5.15)<br />

Note que o momento de inércia em relação ao<br />

centro de massa é o menor valor do momento de<br />

1 A importância do centro de massa é devido ao fato de<br />

podermos descrever o movimento de um sistema com massa<br />

total M sob a ação de uma força resultante (externa) ⃗ F<br />

como o movimento de uma massa pontual M, localizada no<br />

centro de massa, atuada por uma força ⃗ F = M ¨⃗ R<br />

inércia que um determinado corpo rígido pode ter.<br />

Note também o quanto estamos aprendendo usando<br />

sistemas simples e revendo conceitos fundamentais<br />

(cinemática e energia cinética). Por isto que conceitos<br />

fundamentais são fundamentais: eles formam<br />

a base do nosso conhecimento.<br />

Que tal um pouquinho de exercício em cálculo<br />

diferencial e integral usando integrais múltiplas?<br />

A tarefa matemática que executaremos aqui é o<br />

cálculo do momento de inércia de um corpo rígido<br />

contínuo. Naturalmente, faremos isto usando sistemas<br />

simples. A situação mais simples que podemos<br />

encontrar é uma distribuição linear de massa.<br />

Sendo mais específico, vamos considerar uma vareta<br />

delgada (ou um fio muito fino) com dois formatos:<br />

retilíneo e circular. Vamos considerar uma situação<br />

ideal no seguinte sentido: a única dimensão que realmente<br />

importa é o comprimento do fio, as outras<br />

duas dimensões são muito pequenas e serão ignoradas.<br />

Isto é o que é denominado de uma distribuição<br />

linear de massa. Suponha que o fio tenha um comprimento<br />

l e uma massa total M, independentemente<br />

de seu formato. A densidade linear de massa<br />

λ é definida da seguinte forma. Primeiro escolhemos<br />

um comprimento infinitesimal ds em qualquer<br />

lugar no fio. Neste comprimento infinitesimal ds<br />

cabe uma quantidade também infinitesimal dm de<br />

massa. A densidade linear de massa é definida pela<br />

razão massa/comprimento,<br />

λ = dm<br />

ds . (5.16)<br />

Conhecendo a densidade, podemos calcular imediatamente<br />

a massa total do corpo,<br />

∫ ∫ ∫<br />

M = dm = λ ds = λ(s) ds. (5.17)<br />

Esta integral deve ser ser feita em toda a região contendo<br />

massa, como indicado nos exemplos abaixo.<br />

Cabe aqui uma observação importante (independente<br />

da forma do fio). Podemos encontrar duas<br />

situações: uma onde a densidade linear λ = λ 0<br />

é uma constante, ou seja, a massa total está distribuída<br />

uniformemente sobre o fio, e uma outra<br />

onde a densidade linear λ = λ(s) não é constante,<br />

ou seja, a massa total não está distribuída de forma<br />

uniforme sobre o fio. No primeiro caso, com λ = λ 0<br />

constante, a massa total pode ser obtida imediata-<br />

64


Capítulo 5. Rotações<br />

5.2. Energia rotacional<br />

mente,<br />

∫ ∫ ∫ l<br />

M = dm = λ 0 ds = λ 0 ds = λ 0 l. (5.18)<br />

No segundo caso, densidade não-uniforme, a massa<br />

total só pode ser calculada após conhecermos explicitamente<br />

como a densidade depende do comprimento<br />

s ao longo do fio. Como exemplo, suponha<br />

que a densidade varie linearmente com o comprimento,<br />

λ = b s. Então, usando (5.17),<br />

∫ ∫ ∫ l<br />

M = dm = b s ds = b s ds = 1 2 b l2 . (5.19)<br />

Note que a integral indefinida em (5.17) torna-se<br />

uma integral definida na região contendo massa.<br />

Note também que obtivemos estes dois resultados<br />

sem a necessidade de especificar a forma do fio. No<br />

entanto, o formato do fio é indispensável no cálculo<br />

do momento de inércia.<br />

A fim de exemplificar o cálculo do momento de<br />

inércia, vamos considerar um fio retilíneo com a<br />

densidade não-uniforme λ = b s. Primeiramente,<br />

temos de escolher um eixo de rotação. Vamos escolher<br />

este eixo passando perpendicularmente por<br />

uma das extremidades do fio. O próximo passo é a<br />

escolha da origem de um bom sistema de coordenadas.<br />

Vamos escolher um sistema de coordenadas<br />

retangular (ortonormal) com a origem na intersecão<br />

do eixo de rotação com o fio. Podemos até colocar<br />

o fio sobre um dos eixos do nosso sistema de coordenadas,<br />

digamos em cima do eixo X. Com esta<br />

escolha, o eixo de rotação está sobre o eixo Z (ou<br />

sobre o eixo Y ) e ds = dx, com s = x variando entre<br />

0 e l. Então, usando a definição (5.8), o momento<br />

de inércia deste fio retilíneo não-uniforme é<br />

∫ ∫ l<br />

I = r 2 dm = x 2 bx dx = 1 4 bl4 . (5.20)<br />

0<br />

Podemos usar a massa total calculada em (5.19)<br />

para re-escrever o momento de inércia (5.20) em<br />

termos da massa total, I = Ml 2 /2. Este mesmo<br />

momento de inércia é Ml 2 /3 caso a densidade seja<br />

uniforme (faça o Exercício 71). Isto significa que o<br />

momento de inércia deste sistema com uma densidade<br />

uniforme é menor que o momento de inércia<br />

com uma densidade não-uniforme. Ora, isto nos<br />

dá uma oportunidade magnífica: podemos alterar<br />

o valor do momento de inércia simplesmente alterando<br />

a distribuição de massa ao longo do fio, mantendo<br />

a massa total constante.<br />

0<br />

0<br />

Vamos considerar agora um anel de raio a com<br />

uma densidade também linear λ = b s, onde s é um<br />

comprimento qualquer sobre o anel. Neste caso,<br />

a melhor escolha é um sistema polar (r, ϕ) de coordenadas<br />

devido à simetria circular do anel. Vamos<br />

colocar o eixo de rotação no centro do anel ao<br />

longo do eixo Z, com o anel no plano XY . Assim,<br />

a distância de uma massa infinitesimal até o eixo<br />

de rotação é r = a. O comprimento infinitesimal<br />

ds sobre o anel pode ser escrito na forma ds = a dϕ<br />

(geometria plana) e, conseqüentemente, s = aϕ.<br />

Usando a definição (5.8), temos<br />

∫<br />

I = r 2 dm =<br />

∫ 2π<br />

ϕ=0<br />

a 2 baϕ a dϕ<br />

= 2a 4 bπ 2 = 1 2 a2 bl 2 = Ma 2 , (5.21)<br />

onde usamos também a massa total (5.19) e o comprimento<br />

do anel l = 2πa. Neste caso, este resultado<br />

coincide com o valor do momento de inércia<br />

calculado com uma densidade uniforme (faça o<br />

Exercício 72). De fato, devido à simetria circular<br />

do anel, desde que o eixo passe pelo centro, perpendicularmente<br />

ao plano do anel, não importa como a<br />

massa está distribuída sobre o anel. Importa apenas<br />

o fato de todos os elementos de massa estarem<br />

à mesma distância do eixo de rotação.<br />

Até aqui consideramos apenas distribuições lineares.<br />

O caso mais simples de uma distribuição nãolinear<br />

é uma distribuição superficial plana, ou seja,<br />

um disco de raio a ou um quadrado de lado l. Enquanto<br />

que em uma distribuição linear nós necessitamos<br />

de apenas um grau de liberdade para especificar<br />

a densidade linear (o comprimento ao longo<br />

do fio, por exemplo), precisaremos de dois graus de<br />

liberdade para especificar a densidade superficial.<br />

A densidade superficial é construída assim: primeiro<br />

escolhemos uma área infinitesimal dA qualquer.<br />

Esta área infinitesimal contém uma quantidade<br />

infinitesimal dm de massa. A densidade superficial<br />

σ é definida como<br />

σ = dm<br />

dA . (5.22)<br />

De forma usual, a massa total deve ser calculada<br />

somando-se todas as massas infinitesimais,<br />

∫ ∫<br />

M = dm = σ dA. (5.23)<br />

65


5.3. Dinâmica rotacional e leis de conservação II Capítulo 5. Rotações<br />

Temos novamente duas possibilidades: ou a densidade<br />

superficial é constante, σ = σ 0 , e neste caso a<br />

massa total é<br />

∫ ∫<br />

M = dm = σ 0 dA = σ 0 A, (5.24)<br />

ou a densidade superficial é não-uniforme.<br />

No caso não-uniforme, precisamos saber explicitamente<br />

como a densidade superficial depende da<br />

posição na região contendo massa. Como exemplo,<br />

vamos considerar inicialmente uma distribuição de<br />

massa circular de raio a e com uma densidade<br />

dependente apenas da distância radial na forma<br />

σ = σ 0 r, com σ 0 constante. Como a distribuição<br />

de massa é um disco, as coordenadas polares (r, ϕ)<br />

constituem a melhor escolha. Uma área infinitesimal<br />

em coordenadas polares é escrita na forma<br />

dA = rdϕ dr (faça o Exercício 73). Assim, a massa<br />

total pode ser calculada usando a densidade superficial<br />

(5.24),<br />

∫ ∫ 2π<br />

M = dm = σ 0 dϕ<br />

0<br />

∫ a<br />

0<br />

r 2 dr<br />

= 2π 3 σ 0a 3 . (5.25)<br />

É importante notar que a integral indefinida em<br />

(5.24) tornou-se duas integrais definidas em (5.25)<br />

sobre toda a região contendo massa. Também note<br />

que estas duas integrais definidas puderam ser calculadas<br />

separadamente. Isto foi possível somente<br />

porque a densidade superficial σ = σ 0 r não mistura<br />

as variáveis polares r e ϕ.<br />

E o momento de inércia deste disco com uma distribuição<br />

de massa não-uniforme em relação a um<br />

eixo de rotação passando pelo centro do disco perpendicularmente<br />

ao plano do disco? (haja fôlego!)<br />

Neste caso, como o eixo de rotação passa pelo centro<br />

do disco, então podemos usar r como a coordenada<br />

radial. Assim, usando a definição (5.8), temos<br />

∫<br />

∫ 2π<br />

I = r 2 dm = σ 0 dϕ<br />

0<br />

∫ a<br />

0<br />

r 4 dr<br />

= 2π 5 σ 0a 5 = 3 5 Ma2 . (5.26)<br />

Novamente, este valor é maior que o valor Ma 2 /2<br />

do momento de inércia do mesmo disco com uma<br />

distribuição uniforme de massa (verifique). Isto é<br />

devido ao fato de haver uma concentração maior<br />

de massa nas regiões mais afastadas do eixo de<br />

rotação.<br />

Espero que estes exemplos tenham indicado a<br />

você o caminho das pedras. Dada uma distribuição<br />

de massa M, em geral ocupando um volume V ,<br />

a primeira providência é saber como a densidade<br />

ρ = dm/dV associada a esta distribuição, onde dV<br />

é o volume infinitesimal, depende da posição na<br />

região contendo massa. O próximo passo é expressar<br />

a massa total<br />

∫ ∫<br />

M = dm = ρ dV (5.27)<br />

em função dos parâmetros que caracterizam a densidade<br />

volumétrica e o próprio volume contendo a<br />

massa total. Em seguida, podemos calcular o momento<br />

de inércia usando a definição (5.8),<br />

∫ ∫<br />

I = r 2 dm = r 2 ρ dV, (5.28)<br />

onde r é a distância da massa infinitesimal dm até<br />

o eixo de rotação. Em geral, tanto a distância ¯r<br />

quanto a densidade ρ são funções da posição, isto<br />

é, funções das coordenadas de algum sistema de coordenadas<br />

adequado ao problema. Também não devemos<br />

esquecer que a integral indefinida em (5.28)<br />

será transformada em três integrais definidas após<br />

a escolha do sistema de coordenadas mais adequado<br />

à geometria do corpo rígido em consideração.<br />

5.3 Dinâmica rotacional e leis<br />

de conservação II<br />

Como podemos colocar um determinado corpo<br />

rígido em rotação em torno de algum eixo fixo?<br />

Imagine que desejamos colocar uma roda de bicicleta<br />

em rotação em torno do eixo que a mantém<br />

fixa na bicicleta. Sabemos da experiência que precisamos<br />

aplicar uma força (externa) para colocá-la<br />

em rotação. Podemos relacionar esta força aplicada<br />

com a aceleração angular?<br />

Podemos ver claramente que a força aplicada<br />

está transferindo energia à roda pelo mecanismo<br />

de trabalho, para que a mesma possa executar<br />

seu movimento de rotação. Também aprendemos<br />

nesta experiência que não podemos aplicar a força<br />

de qualquer maneira na roda para colocá-la em<br />

rotação. Por exemplo, quando aplicamos uma força<br />

66


Capítulo 5. Rotações<br />

5.3. Dinâmica rotacional e leis de conservação II<br />

na mesma direção da linha que passa pelo ponto<br />

de aplicação da força e pelo eixo de rotação (perpendicularmente),<br />

não produzimos rotações (veja a<br />

Figura 5.2). Em outras palavras, somente conseguiremos<br />

colocar a roda em rotação quando a força<br />

aplicada F ⃗ não for paralela ao vetor (distância) ⃗¯r.<br />

A Figura 5.2 ilustra estas duas situações: uma onde<br />

a força F ⃗ 1 é capaz de produzir uma rotação em<br />

torno do eixo fixo perpendicular ao plano da roda<br />

e passando pelo centro da roda. A força F ⃗ 2 não<br />

produz rotações.<br />

⃗¯r 1<br />

Figura 5.2: A força ⃗ F 1 é capaz de produzir uma<br />

rotação em torno do eixo fixo perpendicular ao<br />

plano da roda e passando pelo centro da roda. A<br />

força ⃗ F 2 é incapaz de produzir rotações.<br />

⃗¯r 2<br />

⃗F 1<br />

⃗F 2<br />

Note que ⃗¯r 1 × ⃗ F 1 ≠ 0 e ⃗¯r 2 × ⃗ F 2 = 0. Isto está<br />

sugerindo que a quantidade ⃗¯r × ⃗ F pode ter alguma<br />

relevância na dinâmica do movimento de rotação.<br />

Evidentemente, devemos realizar mais experimentos<br />

para determinarmos como o produto vetorial<br />

⃗¯r × ⃗ F está relacionado (se estiver) com a aceleração<br />

angular. Será o produto vetorial ⃗¯r × ⃗ F a quantidade<br />

equivalente à força no movimento translacional?,<br />

isto é, ⃗¯r× ⃗ F ∝ ⃗α? Apenas o experimento pode<br />

decidir. O experimento com a roda de bicicleta está<br />

indicando que sim. Este experimento também nos<br />

revela que se o produto vetorial ⃗¯r × ⃗ F é constante,<br />

então o movimento rotacional resultante é uniformemente<br />

acelerado, isto é, a aceleração angular é<br />

também constante. Este experimento também nos<br />

revela que o sentido do vetor aceleração angular ⃗α<br />

é determinado pelo produto vetorial ⃗¯r × ⃗ F . Portanto<br />

a constante de proporcionalidade entre ⃗¯r × ⃗ F<br />

e ⃗α deve ser positiva. Além disto, uma análise dimensional<br />

revela que esta constante de proporcionalidade<br />

deve ter as mesmas dimensões de momento<br />

de inércia. Outros experimentos similares<br />

irão nos mostrar que esta constante de proporcionalidade<br />

é, nada mais, nada menos, o próprio momento<br />

de inércia do corpo rígido. De fato, isto<br />

faz sentido. Aprendemos anteriormente que o momento<br />

de inércia mede a inércia do movimento rotacional.<br />

Desta forma, valendo-nos de experimentos<br />

e análise dimensional, concluímos que ⃗¯r × F ⃗ = I⃗α<br />

é o análogo da segunda lei de Newton para o movimento<br />

rotacional. Esta relação também justifica a<br />

escolha que fizemos anteriormente para a direção e<br />

o sentido de nossas grandezas cinemáticas rotacionais,<br />

pois o produto vetorial ⃗¯r × F ⃗ está sobre o eixo<br />

de rotação.<br />

Para entendermos melhor os resultados experimentais<br />

descritos acima e, principalmente, para<br />

aprendermos a controlá-los, vamos observar novamente<br />

uma determinada massa (pontual), discreta<br />

ou infinitesimal, em um corpo rígido qualquer,<br />

como mostrado na Figura 5.3. O objetivo é a construção<br />

de um modelo, com uma base matemática<br />

que seja elegante e, ao mesmo tempo, capaz de explicar<br />

detalhadamente os resultados experimentais<br />

envolvendo rotações.<br />

ˆn<br />

⃗¯r<br />

m i<br />

⃗v ⊥<br />

Figura 5.3: Rotação, em torno de um eixo fixo, de<br />

uma massa m i em um corpo rígido qualquer. Note<br />

que ˙⃗¯r = ⃗v + ⃗v ⊥ , mas como o corpo é rígido, então<br />

devemos ter ⃗v = 0.<br />

A Figura 5.3 mostra uma massa m i (ou dm se<br />

a distribuição é contínua) pertencente a um corpo<br />

rígido qualquer de massa total<br />

N∑<br />

∫<br />

M = m i (ou M = dm). (5.29)<br />

i=1<br />

Note na Figura 5.3 que estamos indicando por ⃗¯r<br />

⃗v <br />

⃗v<br />

67


5.3. Dinâmica rotacional e leis de conservação II Capítulo 5. Rotações<br />

o vetor “distância”, aquele vetor perpendicular ao<br />

eixo de rotação, o qual é usado no cálculo do momento<br />

de inércia. Esta notação diferenciada se faz<br />

necessária para distinguirmos o vetor distância ⃗¯r<br />

do vetor posição ⃗r, o qual coincidirá com o vetor<br />

distância apenas quando a origem do sistema<br />

de coordenadas concidir com a interseção do vetor<br />

distância com o eixo de rotação. Entretanto, no<br />

caso geral, o vetor distância ⃗¯r é a componente perpendicular<br />

do vetor posição ⃗r em relação ao eixo de<br />

rotação passando pela origem do sistema de coordenadas.<br />

Observe na Figura 5.3 que o vetor velocidade<br />

⃗v = ˙⃗¯r da massa m i pode ser decomposto numa<br />

componente radial ⃗v e numa componente tangencial<br />

⃗v ⊥ , ⃗v = ⃗v + ⃗v ⊥ . Naturalmente, a componente<br />

radial é nula em um corpo rígido, ⃗v = 0, pois a<br />

massa m i não está livre para movimentar nesta<br />

direção. No entanto, poderíamos ter uma única<br />

massa neste sistema. No caso de uma única massa,<br />

o vetor velocidade possui uma componente radial.<br />

Veremos em breve que precisaremos do produto<br />

vetorial ⃗¯r × ⃗v ⊥ , o qual sempre pode ser re-escrito<br />

na forma<br />

⃗¯r × ⃗v ⊥ = ⃗¯r × ⃗v, (5.30)<br />

mesmo no caso em que ⃗v ≠ 0, pois ⃗¯r × ⃗v = 0 por<br />

construção. Também podemos ver na Figura 5.3<br />

que<br />

⃗ω = ⃗¯r × ⃗v ⊥<br />

¯r 2 = ⃗¯r × ⃗v<br />

¯r 2 , (5.31)<br />

onde ⃗ω = ωˆn é o vetor velocidade angular. Para<br />

verificar este resultado, certifique-se que a direção,<br />

o sentido e o módulo (v = ¯rω) de ⃗ω, calculados<br />

por (5.31), estão corretos. Note que fizemos uso<br />

de (5.30) na última passagem em (5.31). É importante<br />

ter em mente também que ˙⃗¯r = ⃗v ⊥ , pois o<br />

movimento da massa m i é circular (de raio ¯r).<br />

Mantendo as observações (5.30) e (5.31) em<br />

mente, podemos prosseguir. Para isto, precisamos<br />

recordar a definição de energia cinética do movimento<br />

de translação. No movimento de translação,<br />

a energia cinética de um corpo (pontual) de massa<br />

m e velocidade ⃗v sempre pode ser re-escrita em termos<br />

do momentum linear ⃗p = m⃗v,<br />

T = 1 2<br />

1<br />

m⃗v · ⃗v = ⃗p · ⃗p, ⃗p = m⃗v. (5.32)<br />

2m<br />

Também sabemos que o momentum linear é uma<br />

quantidade importante porque a sua taxa de variação<br />

temporal é igual a força agindo na massa m<br />

(segunda lei de Newton),<br />

⃗F = d⃗p<br />

dt = ˙⃗p. (5.33)<br />

Também sabemos que (5.33) é uma equação diferencial<br />

de segunda ordem no tempo para o vetor<br />

posição ⃗r(t). Resolvendo esta equação diferencial,<br />

determinaremos a dependência temporal do vetor<br />

posição, ou seja, a equação diferencial (5.33) controla<br />

todo o comportamento dinâmico da massa m.<br />

Isto é o que sabemos sobre o movimento translacional.<br />

Podemos usar este conhecimento para entendermos<br />

melhor a dinâmica do movimento rotacional?<br />

Certamente podemos.<br />

Procedendo de forma análoga, podemos reescrever<br />

a energia cinética rotacional (5.7) do corpo<br />

rígido mostrado na Figura 5.3 numa forma similar<br />

a (5.32),<br />

T = 1 2<br />

I⃗ω · ⃗ω =<br />

1<br />

2I ⃗ L · ⃗L, ⃗ L = I⃗ω, (5.34)<br />

na qual I é o momento de inércia (5.8) e o vetor<br />

⃗L, denominado de momentum angular, é análogo ao<br />

momentum linear, como veremos logo em seguida.<br />

Se o momentum angular L ⃗ é realmente uma quantidade<br />

importante, com uma importância igual à<br />

importância do momentum linear, então, por analogia<br />

à segunda lei (5.33), a sua taxa de variação<br />

temporal<br />

˙⃗L = I⃗α, pois I é uma constante, deve<br />

controlar a dinâmica do movimento de rotação.<br />

Por outro lado, vimos no início desta seção que<br />

necessitamos de uma força externa para colocar um<br />

corpo rígido em rotação. Portanto, a taxa de variação<br />

temporal do momentum angular deve estar<br />

relacionada de alguma forma com a força externa.<br />

Esta relação pode ser encontrada escrevendo o momentum<br />

angular total L ⃗ = I⃗ω em termos do momentum<br />

angular de cada massa m i (ou dm) presente<br />

no corpo rígido,<br />

N∑<br />

N∑<br />

⃗L = I⃗ω = m i¯r i 2 ⃗ω = m i<br />

⃗¯r i × ⃗v i<br />

i=1<br />

i=1<br />

=<br />

N∑<br />

⃗¯r i × ⃗p i , (5.35)<br />

i=1<br />

onde usamos também o resultados (5.30)–(5.31).<br />

Não devemos esquecer que podemos escrever ⃗p i =<br />

68


Capítulo 5. Rotações<br />

5.3. Dinâmica rotacional e leis de conservação II<br />

m i ⃗v i somente quando este momentum linear aparecer<br />

no produto vetorial ⃗¯r i × ⃗p i , pois no movimento<br />

circular da massa m i temos ⃗p i = m i ⃗v i⊥ (não há movimento<br />

radial; corpo rígido). A última expressão<br />

em (5.35) nos permite escrever um momentum angular<br />

⃗¯r i × ⃗p i para cada massa m i ,<br />

⃗L = I⃗ω =<br />

N∑<br />

⃗L i , Li ⃗ = ⃗¯r i × ⃗p i . (5.36)<br />

i=1<br />

Agora estamos numa posição confortável para determinarmos<br />

a taxa de variação do momentum angular,<br />

˙⃗L = I⃗α =<br />

N∑<br />

˙⃗L i ,<br />

i=1<br />

˙⃗L i = ⃗¯r i × ˙⃗p i = ⃗¯r i × ⃗ F i , (5.37)<br />

onde ⃗ F i é a força externa atuando na i-ésima<br />

massa m i . As forças internas cancelam-se aos pares,<br />

sempre. Note que usamos também o fato de<br />

˙⃗¯r = ⃗v + ⃗v ⊥ = ⃗v ⊥ em um corpo rígido. O resultado<br />

(5.37) acaba de nos revelar o análogo da segunda<br />

lei de Newton para o movimento rotacional,<br />

com<br />

⃗τ = ˙⃗ L = I⃗α,<br />

∑<br />

N<br />

⃗τ = ⃗τ i , (5.38)<br />

i=1<br />

⃗τ i = ⃗¯r i × ⃗ F i . (5.39)<br />

A quantidade ⃗τ = ⃗¯r × ⃗ F , denominada de torque,<br />

faz o papel da força ⃗ F no movimento de translação.<br />

Note em (5.39) que, conhecendo as forças externas<br />

e o eixo de rotação, temos como calcular o torque<br />

de cada uma destas forças externas. Assim, ⃗τ = I⃗α<br />

ou a sua forma mais geral ⃗τ = ˙⃗ L faz o papel análogo<br />

da segunda lei de Newton, ⃗ F = m¨⃗r ou ⃗ F = ˙⃗p, para<br />

o movimento de translação.<br />

Observe que, se o momento de inércia é constante,<br />

é necessário uma aceleração angular para colocar<br />

um corpo rígido em movimento de rotação.<br />

Isto implica que o torque (5.38) deve ser diferente<br />

de zero, ou seja, deve haver pelo menos uma força<br />

externa ⃗ F i produzindo um torque ⃗¯r i × ⃗ F i diferente<br />

de zero para haver rotação a partir do repouso.<br />

Também podemos ver em (5.38) que um torque<br />

constante produz uma rotação uniformemente acelerada<br />

(aceleração angular constante).<br />

Mais algumas observações importantes antes<br />

de passarmos ao desenvolvimento de algumas<br />

aplicações. (1) Dada a relação ⃗τ = ⃗¯r × F ⃗ entre<br />

torque e força (externa), entendemos porque não<br />

é possível colocar a roda de bicicleta em rotação<br />

aplicando uma força ao longo da linha perpendicular<br />

ao eixo de rotação e que passa pelo ponto de<br />

aplicação da força (veja a Figura 5.2). Em outras<br />

palavras, o torque ⃗τ = ⃗¯r × F ⃗ é nulo quando a força<br />

⃗F i é paralela ao vetor ⃗¯r i . (2) Como ¯r i = ||⃗¯r i || é<br />

a distância entre a massa m i e o eixo de rotação,<br />

então tanto o momentum angular quanto o torque<br />

dependem da posição do eixo de rotação. (3) Note<br />

que momentum angular tem dimensões de energia<br />

vezes tempo e que, conseqüentemente, torque tem<br />

dimensões de energia. No entanto, não é correto<br />

dizer que torque é energia. Também não é correto<br />

dizer que trabalho é energia. Trabalho é um mecanismo<br />

de troca de energia. Torque é o mecanismo<br />

que permite um corpo rígido entrar em movimento<br />

de rotação em torno de um eixo fixo.<br />

Por falar em trabalho, devemos determinar uma<br />

expressão para a potência instantânea no movimento<br />

rotacional. Em muitas situações, o movimento<br />

de rotação ocorre em meios viscosos, os<br />

quais oferecem uma força dissipativa. Tendo dissipação,<br />

há troca de energia pelo mecanismo trabalho.<br />

A potência P mede justamente a taxa temporal<br />

desta troca de energia, P = dW/dt. Observe<br />

novamente a Figura 5.1. Acrescente a ela a<br />

força externa F ⃗ i agindo na i-ésima massa m i . Para<br />

que este corpo rígido altere seu estado de movimento<br />

rotacional, as forças externas F ⃗ i devem realizar<br />

trabalho durante o intervalo de tempo dt,<br />

dW i = F ⃗ i · d⃗¯r i . Lembre-se que apenas o módulo<br />

do vetor (distância) ⃗¯r i permanece constante e que<br />

˙⃗¯r i = ⃗v i , onde ⃗v i é a velocidade (tangencial) da<br />

massa m i . Assim, a potência instantânea correspondente<br />

é P i = dW i /dt = F ⃗ i · ⃗v i . Este resultado é<br />

simplesmente a potência instantânea do movimento<br />

translacional. No entanto, ela pode ser re-escrita<br />

numa forma mais conveniente ao movimento rotacional.<br />

Para isto, basta substituir a velocidade<br />

tangencial pela velocidade angular, usando (5.6),<br />

P i = ⃗ F i · ⃗v i = ⃗ F i · (⃗ω<br />

× ⃗¯r i<br />

)<br />

= ⃗¯r i · ( ⃗ Fi × ⃗ω )<br />

= ⃗ω · (⃗¯r i × ⃗ F i<br />

)<br />

, (5.40)<br />

onde utilizamos a propriedade (2.30) do produto<br />

misto entre três vetores (invariância por permutações<br />

circulares). A última expressão em (5.40)<br />

69


5.3. Dinâmica rotacional e leis de conservação II Capítulo 5. Rotações<br />

contém o torque na i-ésima massa m i ,<br />

P i = ⃗ω · (⃗¯r i × ⃗ F i<br />

)<br />

= ⃗τi · ⃗ω. (5.41)<br />

Desta forma, a potência instantânea total pode ser<br />

escrita como<br />

N∑<br />

P = ⃗τ i · ⃗ω = ⃗τ · ⃗ω. (5.42)<br />

i=1<br />

Sem dúvidas, fomos muito felizes na construção<br />

de um modelo capaz de explicar a dinâmica do movimento<br />

rotacional. O torque é uma quantidade<br />

rotacional importante, pois ela tem um papel similar<br />

ao papel desempenhado pela força no movimento<br />

translacional. A relação (5.38) entre torque<br />

e o momentum angular certamente justifica suas<br />

definições. As relações entre momentum angular e<br />

momentum linear, (5.36), bem como entre torque<br />

e força, (5.39), corroboram a importância destas<br />

duas quantidades angulares. Note também que a<br />

relação (5.39) entre torque e força justifica a escolha<br />

da direção (eixo de rotação) e sentido (regra da<br />

mão direita) para os vetores deslocamento, velocidade<br />

e aceleração angulares.<br />

Além de ser bem sucedido na descrição do movimento<br />

rotacional, este modelo construído via momentum<br />

angular e torque, em analogia com momentum<br />

linear e força, é matematicamente perfeito,<br />

pois ele nos fornece também uma importante lei de<br />

conservação: a lei de conservação do momentum angular.<br />

Podemos ver claramente em (5.38) que o momentum<br />

angular (total) será uma quantidade conservada<br />

(constante no tempo) sempre que o torque<br />

(total) for nulo. Esta é mais uma (na verdade três)<br />

entre as leis de conservação da mecânica.<br />

Teorema 10<br />

O momentum angular se conserva na ausência de torques,<br />

⃗τ = 0 ⇒ ˙⃗ L = 0.<br />

Como já mencionamos, estas leis de conservação<br />

continuam válidas, assim como as demais, mesmo<br />

no reino microscópico onde a física Newtoniana não<br />

se aplica, ou seja, a conservação do momentum angular<br />

também é verificada em mecânica quântica.<br />

Giroscópios<br />

Talvez uma das aplicações mais importantes da<br />

conservação do momentum angular no nosso cotidiano<br />

seja na indústria da navegação. A idéia é<br />

muito simples. Um disco é colocado em rotação<br />

numa região ausente de agentes dissipadores e de<br />

forças externas. De acordo com o Teorema 10, é<br />

necessário a ação de uma força externa para produzir<br />

um torque para modificar o eixo de rotação<br />

deste disco. Portanto na ausência de torques, o eixo<br />

de rotação deste sistema (perpendicular ao plano<br />

do disco) deve apontar para uma direção fixa (podemos<br />

chamar esta situação de inércia rotacional).<br />

Este sistema é denominado de giroscópio (veja a Figura<br />

5.4). Um giroscópio é uma peça fundamental<br />

em qualquer sistema preciso de navegação atual.<br />

Sem dúvidas, um giroscópio serve como um<br />

guia, um “navegador” no espaço vazio. Por exemplo,<br />

o telescópio Hubble (hubble.nasa.gov) é orientado<br />

por um mecanismo contendo dois giroscópios<br />

de 45 kg cada, girando em torno de eixos nãocoincidentes<br />

com uma velocidade angular que pode<br />

atingir 3000 rpm. Estas velocidades angulares podem<br />

ser modificadas eletronicamente para produzirem<br />

pequenas variações nos momenta angulares dos<br />

giroscópios. Como o momentum angular total precisa<br />

ser conservado, pois não há forças externas, o<br />

telescópio precisa girar para compensar as variações<br />

de momentum angular produzidas nos giroscópios.<br />

Desta forma, o telescópio pode manter um determinado<br />

alvo fixo com um erro de cinco milésimos<br />

de segundo. Isto é equivalente a mantermos iluminada<br />

uma moeda de um real localizada em Vitória<br />

(ES) com uma lanterna em cima do pão de açúcar<br />

no Rio de Janeiro (RJ).<br />

Outro exemplo importante é a sonda espacial<br />

GP-B (Gravity Probe-B), a qual está medindo<br />

o campo gravitacional ao redor da Terra (einstein.stanford.edu).<br />

Esta sonda está equipada com<br />

quatro giroscópios construídos de esferas de quartzo<br />

(crescidos no Brasil) com uma imperfeição de apenas<br />

20 nm (10 −9 m) em seu diâmetro e com uma<br />

massa uniformemente distribuída com uma imperfeição<br />

de apenas 2 ppm (parte por milhão), girando<br />

a 4000 rpm. Qualquer variação no espaçotempo<br />

produzirá uma variação nos eixos de rotação destes<br />

giroscópios.<br />

Um efeito muito interessante, denominado de<br />

precessão, surge quando um giroscópio é colocado<br />

na presença de uma força externa, por exemplo na<br />

superfície da Terra. Como sempre, vamos simplificar<br />

o máximo nosso giroscópio para entendermos<br />

o efeito de precessão. Nosso giroscópio será representado<br />

por uma massa M, distribuída uniforme-<br />

70


Capítulo 5. Rotações<br />

5.3. Dinâmica rotacional e leis de conservação II<br />

mente em um anel circular de raio R. Este anel<br />

está preso a uma extremidade de um eixo rígido de<br />

massa desprezível, em torno do qual o anel pode<br />

girar livremente. A outra ponta do eixo do anel<br />

está apoiada por um pivô (um ponto fixo). Este<br />

pivô permite que a ponta do eixo do anel apoiado<br />

por ele execute livremente um movimento de 360<br />

graus sem perda de contato. A Figura 5.4 ilustra<br />

este nosso giroscópio ideal.<br />

ˆn<br />

força externa M⃗g produz um torque,<br />

⃗τ = ⃗r × (M⃗g) = d⃗ L<br />

, τ = Mrg sin θ, (5.43)<br />

dt<br />

onde ⃗r é o vetor posição indicado na Figura 5.4.<br />

Pela propriedade do produto vetorial, o vetor torque<br />

em (5.43) é perpendicular ao vetor posição ⃗r e,<br />

portanto, perpendicular ao vetor momentum angular<br />

⃗ L. Como a variação d ⃗ L do momentum angular<br />

é proporcional ao vetor torque, então podemos concluir<br />

que a variação d ⃗ L é perpendicular a ⃗ L, como<br />

mostrado na Figura 5.5.<br />

ˆn 1<br />

θ<br />

⃗r<br />

anel<br />

⃗L + d ⃗ L<br />

ˆn 2<br />

M⃗g<br />

ˆn 1<br />

dφ<br />

⃗ L<br />

d ⃗ L<br />

Figura 5.4: Giroscópio ideal, formado por um anel<br />

de massa M que pode girar livremente em torno do<br />

eixo ˆn, na presença da força gravitacional. O eixo<br />

de rotação ˆn 2 é perpendicular ao eixo de rotação<br />

ˆn 1 .<br />

Podemos muito bem substituir o pivô por um<br />

cordão preso ao teto. O cordão é uma opção mais<br />

prática, pois sempre temos um pedaço de cordão<br />

no bolso e um bom pivô nem sempre está a disposição.<br />

Observe na Figura 5.4 que há dois eixos<br />

em torno dos quais nosso giroscópio pode girar livremente:<br />

um eixo de rotação ˆn 1 na direção do<br />

suporte (fixo) do pivô (ou do cordão dependurado)<br />

e um eixo de rotação ˆn 2 perpendicular a ˆn 1 e passando<br />

pelo pivô. O anel do giroscópio gira em torno<br />

do eixo ˆn com um velocidade angular ω. Seja I o<br />

momento de inércia do anel e r o comprimento do<br />

eixo passando pelo centro do anel. Na ausência de<br />

forças externas, este giroscópio tem um momentum<br />

angular ⃗ L = Iωˆn, o qual é conservado. Note que<br />

este vetor momentum angular é paralelo ao vetor<br />

⃗r.<br />

Como mostrado na Figura 5.4, a presença da<br />

Figura 5.5: Movimento de precessão do eixo ˆn do<br />

giroscópio mostrado na Figura 5.4, com θ = π/2,<br />

em torno do eixo ˆn 1 .<br />

Como L ⃗ · dL ⃗ = 0, temos então que o módulo<br />

L = Iω do vetor momentum angular é conservado<br />

(Faça o Exercício 74), apesar da ação da força externa.<br />

Apenas a direção do vetor momentum angular<br />

está mudando no tempo. Podemos dizer então<br />

que a força gravitacional ainda manteve intacta<br />

uma das três leis de conservação do momentum<br />

angular (o seu módulo), contidas no Teorema 10.<br />

Como L ⃗ · dL ⃗ = 0 e ˙L = 0, então a ponta do vetor<br />

momentum angular angular executa um movimento<br />

circular, como mostrado na Figura 5.5. Este movimento<br />

circular em torno do eixo ˆn 1 é denominado<br />

de precessão. A velocidade angular ω p deste movimento<br />

de precessão pode ser calculada facilmente<br />

com o auxílio da Figura 5.5, pois ||dL|| ⃗ = L dϕ. 2<br />

Assim,<br />

ω p = dϕ<br />

dt = 1 ||dL||<br />

L dt<br />

= 1 L⇑<br />

dL<br />

dt<br />

⇑ = 1 ||⃗τ||. (5.44)<br />

L<br />

2 Note que ||d ⃗ L|| é diferente de dL, pois ˙L = 0.<br />

71


5.4. As equações de Euler Capítulo 5. Rotações<br />

Usando o torque calculado em (5.43), temos a velocidade<br />

angular de precessão,<br />

ω p = Mgr<br />

Iω<br />

= gr<br />

K 2 1<br />

ω , (5.45)<br />

onde fizemos I = MK 2 , com K sendo um<br />

parâmetro característico do sistema, conhecido<br />

como “raio de giro”.<br />

Devemos notar que a velocidade angular de precessão<br />

(5.45) é inversamente proporcional à velocidade<br />

angular ω do giroscópio. Conseqüentemente,<br />

a velocidade de precessão aumenta quando a velocidade<br />

angular do giroscópio diminui. No limite<br />

ω → 0, temos, de acordo com (5.45), ω p → ∞.<br />

No entanto, observamos no experimento que o giroscópio<br />

não aumenta a sua velocidade de precessão<br />

quando a sua velocidade angular diminui. Ele simplesmente<br />

para de precessionar e cai quando ω → 0.<br />

Isto significa que (5.45) necessita de correções. Estas<br />

correções podem ser feitas levando-se em consideração<br />

a massa do eixo do anel, a qual foi ignorada<br />

até então. No entanto, estas correções seriam um<br />

refinamento desnecessário neste momento. Em um<br />

giroscópio real, a ponta do vetor momentum angular<br />

também apresenta um movimento oscilatório<br />

perpendicular ao plano de precessão. Este terceiro<br />

movimento é denominado de nutação.<br />

Aprendemos algo muito importante com este giroscópio<br />

ideal esquematizado na Figura 5.4: a variação<br />

do momentum angular produz um movimento<br />

de rotação. Isto é completamente análogo ao<br />

movimento de translação produzido pela variação<br />

do momentum linear (como acontece nos foguetinhos<br />

de água). Também podemos ver que o simples<br />

fato da roda estar girando impede que o centro de<br />

massa dela caia sob a ação da força gravitacional.<br />

Curiosamente, a própria força gravitacional é responsável<br />

por manter o eixo de rotação na posição<br />

angular θ sem que o centro de massa “caia”.<br />

5.4 As equações de Euler<br />

Nós aprendemos que as leis de Newton para a<br />

dinâmica translacional são válidas em qualquer referencial<br />

inercial. Também já mencionamos que as<br />

leis da mecânica Newtoniana em geral também são<br />

válidas em qualquer referencial inercial, e somente<br />

em referenciais inerciais. Nos parágrafos acima, estabelecemos<br />

as leis que governam a dinâmica do<br />

movimento rotacional e afirmamos que tais leis<br />

são análogas às leis de Newton para o movimento<br />

de translação. Sendo então leis integrantes da<br />

mecânica Newtoniana, então elas (as leis do movimento<br />

de rotação) devem ser válidas somente em<br />

referenciais inerciais. No entanto, nenhum referencial<br />

inercial foi especificado nas discussões acima<br />

sobre a dinâmica do movimento de rotação. Onde<br />

estará este referencial inercial? Veremos que ao estabelecermos<br />

uma resposta a esta pergunta, encontraremos<br />

algumas características muito importantes<br />

sobre momento de inércia até então desconhecidas.<br />

Estas características novas são indispensáveis<br />

ao tratamento de sistemas reais e, mesmo não sendo<br />

necessário, elas justificam a existência de um curso<br />

inteiramente dedicado a matrizes, denominado de<br />

Álgebra Linear. Veremos também que o momento<br />

de inércia, o qual é para nós um escalar (número)<br />

até então, é de fato uma matriz. Será o nosso primeiro<br />

encontro com uma matriz representando uma<br />

entidade física. Até então, temos usado escalares<br />

(como massa, carga, tempo e energia) e vetores<br />

(como posição, força, torque, etc.) na descrição<br />

do movimento na mecânica Newtoniana.<br />

Vamos considerar novamente um corpo rígido, no<br />

qual a distância entre dois de seus pontos permanece<br />

sempre fixa, girando em torno de um eixo passando<br />

pela origem do sistema de coordenadas inercial<br />

O. Por pura comodidade, passaremos a denotar<br />

vetores por letras em negrito ao invés de flechas<br />

sobre as letras. Seja r o vetor posição da massa<br />

dm neste sistema inercial, o qual faz um ângulo θ<br />

com o eixo de rotação representado pelo versor ˆn,<br />

conforme indicado na Figura 5.6.<br />

Podemos ver na Figura 5.6 que r ⊥ = r sin θ, a<br />

projeção do vetor posição r perpendicularmente ao<br />

eixo de rotação, é exatamente a distância da massa<br />

dm até o eixo de rotação. Note também que (se<br />

ainda não o fez, faça)<br />

ṙ = ω × r, ω = ωˆn. (5.46)<br />

Desta forma, não precisamos mais usar o vetor<br />

“distância” ⃗¯r da Figura 5.1, pois ¯r = r ⊥ . Vale<br />

a pena relembrar que devido à rigidez do nosso sistema,<br />

a massa dm está localizada a uma distância<br />

fixa da origem O, ou seja, r 2 = r · r é constante no<br />

tempo. Então, derivando esta expressão no tempo,<br />

teremos<br />

d<br />

dt r2 = 0 = d (r·r) = 2r·ṙ ⇒ r·ṙ = 0. (5.47)<br />

dt<br />

72


Capítulo 5. Rotações<br />

5.4. As equações de Euler<br />

Assim, qualquer massa dm em um corpo rígido executa<br />

um movimento circular de raio r ⊥ = r sin θ.<br />

Lembre-se que o vetor velocidade é perpendicular<br />

ao vetor posição somente em um movimento circular<br />

(aproveite o momento e refaça o Exercício 74).<br />

Também é possível mostrar (mas não o faremos<br />

aqui) que o eixo de rotação está a cada instante em<br />

repouso no referencial inercial O. Isto significa que<br />

o eixo de rotação não precisa estar absolutamente<br />

fixo, como vínhamos adotando.<br />

ˆn<br />

r ⊥<br />

r<br />

θ<br />

O O c<br />

dm<br />

Figura 5.6: Vetor posição r da massa dm em dois<br />

referenciais. O referencial O c está fixo no corpo<br />

rígido.<br />

A relação (5.46) é muito mais que uma simples<br />

relação entre grandezas cinemáticas. Vejamos o<br />

porquê disto. A observação especial aqui é que o<br />

vetor posição r descreve a posição da massa dm (ou<br />

de um ponto qualquer no corpo rígido) em dois sistemas<br />

de coordenadas: no referencial inercial O e<br />

em um outro referencial não-inercial O c . Este referencial<br />

não-inercial O c também tem a sua origem<br />

no mesmo ponto O e seus eixos estão solidários ao<br />

corpo rígido, ou seja, vistos do referencial inercial<br />

O, eles estão girando junto com o corpo rígido. Em<br />

algum tempo, por exemplo em t = 0, os eixos destes<br />

dois referenciais são coincidentes. Evidentemente,<br />

para quem está no referencial não-inercial O c , o vetor<br />

posição da massa dm, o qual denotaremos por<br />

r c , é independente do tempo, ṙ c = 0. O vetor velocidade<br />

angular ω é o mesmo nos dois referenciais,<br />

v<br />

ω = ω c , pois o eixo de rotação está instantaneamente<br />

em repouso no referencial inercial. Então<br />

a relação (5.46) está nos ensinando como escrever<br />

no referencial inercial a taxa de variação de um vetor<br />

fixo no corpo rígido, ou seja, fixo no referencial<br />

não-inercial girando com a velocidade angular ω,<br />

ṙ = ω × r c , ω = ωˆn. (5.48)<br />

Observe que as expressões (5.46) e (5.48) são<br />

idênticas, isto é, ω × r = ω × r c . Como a única<br />

característica especial do vetor posição r c é estar<br />

fixo no referencial não-inercial, então a taxa de variação<br />

Ȧ de um vetor qualquer A c em um referencial<br />

girando com a velocidade angular ω vista de<br />

um referencial inercial é<br />

Ȧ = ω × A c + Ȧc, ω = ωˆn. (5.49)<br />

Esta relação nos diz quem é a taxa de variação de<br />

um vetor no referencial inercial conhecendo apenas<br />

o próprio vetor e sua taxa de variação no referencial<br />

girante (não-inercial), ou seja, não precisamos conhecer<br />

explicitamente como este vetor depende do<br />

tempo no referencial inercial. Isto nos será muito<br />

útil logo adiante. Não devemos esquecer que estes<br />

dois referenciais devem possuir uma origem em comum<br />

e que seus eixos devem ser coincidentes em<br />

algum instante de tempo. Podemos ver de (5.49)<br />

que ˙ω = ˙ω c , o que justifica escrevermos o vetor<br />

velocidade angular com o mesmo símbolo nos dois<br />

referenciais (faça o Exercício 75).<br />

Procedendo como na Seção 5.3, vamos escrever a<br />

energia cinética total do nosso corpo rígido como a<br />

soma das energias cinéticas de cada massa dm,<br />

T = 1 ∫<br />

dm ṙ · ṙ. (5.50)<br />

2<br />

O vetor velocidade angular pode ser introduzido<br />

via qualquer uma das duas relações (5.46) e (5.48).<br />

Vamos escolher a primeira delas, isto é, com o vetor<br />

posição escrito no referencial inercial. Como este<br />

referencial é ortonormal (por construção), então<br />

ṙ · ṙ =<br />

3∑<br />

ẋ 2 i =<br />

i=1<br />

3∑ ( ) 2<br />

ω × r<br />

i , (5.51)<br />

i=1<br />

na qual estamos escrevendo o vetor posição na<br />

forma r = (x 1 , x 2 , x 3 ) = (x, y, z). Existe uma<br />

73


5.4. As equações de Euler Capítulo 5. Rotações<br />

forma compacta de escrevermos as componentes<br />

(<br />

ω × r<br />

)<br />

i do produto vetorial ( ω × r ) ,<br />

(<br />

ω × r<br />

)i = 3∑<br />

j=1 k=1<br />

3∑<br />

ε ijk ω j x k , (5.52)<br />

na qual (ε ijk ) é o tensor completamente antisimétrico<br />

de ordem três (ou tensor de Levi-Civita),<br />

definido pelas seguintes propriedades de suas componentes<br />

ε ijk (Faça o Exercício 76):<br />

⎧<br />

⎪⎨ +1 se (i, j, k) ∈ P par ,<br />

ε ijk = 0 se i = j ou i = k ou j = k,<br />

⎪⎩<br />

−1 se (i, j, k) ∈ Pímpar ,<br />

onde<br />

(5.53)<br />

P par = {(1, 2, 3), (3, 1, 2), (2, 3, 1)}, (5.54)<br />

Pímpar = {(1, 3, 2), (2, 1, 3), (3, 2, 1)}, (5.55)<br />

onde são as permutações pares e ímpares, respectivamente.<br />

Podemos ver então que o tensor de Levi-<br />

Civita troca de sinal sempre que uma permutação<br />

entre dois índices é efetuada. Isto justifica o adjetivo<br />

“completamente” anti-simétrico.<br />

Somas envolvendo produtos das componentes<br />

ε ijk do tensor de Levi-Civita possuem muitas propriedades<br />

interessantes. Em particular, iremos precisar<br />

desta,<br />

3∑<br />

ε ijk ε irs = δ jr δ ks − δ js δ kr , (5.56)<br />

i=1<br />

na qual (δ ij ) é o tensor completamente simétrico de<br />

ordem dois (ou tensor de Kronecker, ou simplesmente<br />

“delta” de Kronecker), definido pelas seguintes<br />

propriedades entre suas componentes δ ij (Faça<br />

o Exercício 77):<br />

δ ij =<br />

{<br />

1 se i = j,<br />

0 se i ≠ j,<br />

(5.57)<br />

Devemos agora substituir a expressão (5.52) em<br />

(5.51) e usar os resultados anteriores,<br />

ṙ · ṙ =<br />

=<br />

=<br />

=<br />

3∑<br />

3∑<br />

i=1(<br />

j=1 k=1<br />

3∑<br />

3∑<br />

j=1 k=1 r=1<br />

3∑<br />

3∑<br />

j=1 k=1 r=1 s=1<br />

3∑<br />

j=1 r=1<br />

3∑<br />

ε ijk ω j x k<br />

3∑<br />

r=1 s=1<br />

3∑<br />

)<br />

ε irs ω r x s<br />

3∑ 3∑<br />

( 3∑<br />

ε ijk ε irs<br />

)ω j x k ω r x s<br />

s=1 i=1<br />

3∑ 3∑ ( )<br />

ω j δjr δ ks x k x s − δ js δ kr x k x s ωr<br />

3∑ (<br />

ω j δjr r 2 )<br />

− x j x r ωr . (5.58)<br />

O “caminho das pedras” é usar um índice diferente<br />

em cada soma. Note que fizemos uso de (5.56) na<br />

penúltima linha de (5.58). Também fizemos uso<br />

da segunda parte do Exercício 77 na última linha<br />

de (5.58). Podemos perceber também que há um<br />

duplo produto matricial na última linha de (5.58)<br />

se imaginarmos (ω j ) como uma matriz linha e (ω r )<br />

como uma matriz coluna e (δ jr r 2 −x j x r ) como uma<br />

matriz quadrada. Entretanto, como dm = ρ dV ,<br />

onde ρ é a densidade, em geral depende das coordenadas<br />

x i , então é melhor definir esta estrutura matricial<br />

após a substituição do produto escalar (5.58)<br />

na energia cinética (5.50)<br />

T = 1 2<br />

3∑<br />

j=1 r=1<br />

3∑<br />

[∫<br />

ω j dm ( δ jr r 2 ) ]<br />

− x j x r ω r<br />

= 1 2<br />

3∑<br />

j=1 r=1<br />

3∑<br />

ω j I jr ω r , (5.59)<br />

na qual definimos<br />

∫<br />

I jr = dm ( δ jr r 2 )<br />

− x j x r . (5.60)<br />

A matriz I = (I jr ) formada pelos elementos I jr definidos<br />

em (5.60) é conhecida por tensor de inércia.<br />

Note que podemos trocar os índices do tensor de<br />

inércia sem alterá-los, I jr = I rj . Por isto, podemos<br />

dizer que este tensor (ou esta matriz) é simétrico<br />

(simétrica). Por se tratar de uma matriz, ou seja, é<br />

necessário especificar dois índices para especificarmos<br />

qualquer um de seus elementos, este tensor é de<br />

segunda ordem. Como dm = ρ dV , onde ρ é a densidade<br />

de massa do corpo rígido, o tensor de inércia<br />

depende apenas da geometria e da distribuição de<br />

massa do corpo rígido.<br />

74


Capítulo 5. Rotações<br />

5.4. As equações de Euler<br />

Também devemos notar que as componentes<br />

(5.60) estão escritas no referencial inercial e, portanto,<br />

dependem do tempo. No entanto, mencionamos<br />

no início deste parágrafo que o vetor velocidade<br />

angular podia ser introduzido na expressão (5.50)<br />

da energia cinética via qualquer uma das duas<br />

relações (5.46) e (5.48). Escolhemos a primeira.<br />

Caso tivéssemos escolhido a segunda relação, o tensor<br />

de inércia I continuaria a ter a mesma forma<br />

(5.60), com as coordenadas x i representando o vetor<br />

posição r c no referencial não-inercial. Neste<br />

referencial não-inercial, solidário ao corpo rígido,<br />

o tensor momento de inércia é independente do<br />

tempo. Isto nos mostra que mesmo não servindo<br />

para fazer dinâmica, este referencial não-inercial é<br />

útil para calcularmos as componentes do tensor de<br />

inércia de forma independente do tempo. Esta observação<br />

será muito útil logo adiante.<br />

Com o auxílio do tensor de inércia, a expressão<br />

(5.59) pode ser re-escrita ainda numa forma mais<br />

compacta,<br />

T = 1 2 ω · Iω = 1 2 ωT Iω. (5.61)<br />

Na primeira forma (primeira igualdade), o tensor<br />

de inércia I atua sobre o vetor velocidade angular<br />

ω para produzir um novo vetor L = Iω cujo produto<br />

escalar com ω nos dá a energia cinética (5.61).<br />

Na segunda forma (segunda igualdade), o vetor velocidade<br />

angular ω é interpretado como uma matriz<br />

coluna e ω T é a matriz transposta 3 , ou seja, uma<br />

matriz linha. A expressão (5.59) ou (5.61) para a<br />

energia cinética rotacional está nos informando que<br />

o tensor de inércia I está fazendo aqui o papel do<br />

nosso momento de inércia definido em (5.8).<br />

Mas não aprendemos que o momento de inércia<br />

definido em (5.8) é um escalar (um número)? Sim,<br />

de fato! Mas aqui ele é uma matriz. Logo abaixo<br />

mostraremos como estas duas informações podem<br />

ser reconciliadas. Aguarde apenas um instante.<br />

Mas no momento, o (tensor) momento de inércia<br />

é uma matriz simétrica. Esta é a primeira vez<br />

que uma entidade física é representada matematicamente<br />

por uma matriz, daí a importância de<br />

matrizes.<br />

E o momentum angular? Ele é o resultado da<br />

ação do tensor de inércia sobre o vetor velocidade<br />

3 A transposição é uma operação que troca linhas por colunas<br />

em uma matriz.<br />

angular? Sim, de fato,<br />

L = Iω (5.62)<br />

é o vetor momentum angular. Note que ao<br />

contrário do caso particular que estudamos anteriormente,<br />

este vetor momentum angular (5.62) não<br />

é, em geral, necessariamente paralelo ao vetor velocidade<br />

angular. Mais surpresas virão com exemplos.<br />

Uma observação importante antes de passarmos<br />

aos exemplos: note que a energia cinética é sempre<br />

positiva, T > 0 (o caso T = 0 não tem qualquer<br />

importância; repouso). Isto significa que o vetor<br />

L = Iω nunca poderá ser ortogonal ao vetor velocidade<br />

angular ω e nem pode ser nulo, Iω ≠ 0. Conseqüentemente,<br />

a matriz I possui uma inversa, pois<br />

Iω = L com L ≠ 0 implica em ω = I −1 L (Faça o<br />

Exercício 78). Portanto já conhecemos duas propriedades<br />

elementares da matriz momento de inércia:<br />

(1) ela é simétrica e (2) ela possui uma inversa.<br />

Como será visto no curso de Álgebra Linear, estas<br />

duas propriedades garante que esta matriz pode ser<br />

re-escrita numa forma diagonal (com todos os elementos<br />

fora da diagonal iguais a zero), em outras<br />

palavras, ela pode ser diagonalizada. Vejamos tudo<br />

isto através de um exemplo.<br />

x<br />

O<br />

z<br />

A<br />

Figura 5.7: Corpo rígido na forma de um cubo de<br />

arestas de comprimento l. Um dos eixos principais<br />

do momento de inércia passa pela origem e o ponto<br />

A.<br />

Vamos considerar um corpo rígido na forma de<br />

um cubo de arestas de comprimento l, conforme<br />

y<br />

75


5.4. As equações de Euler Capítulo 5. Rotações<br />

indicado na Figura 5.7. Por pura simplicidade,<br />

iremos calcular o momento de inércia no referencial<br />

solidário do cubo (embora estejamos usando a<br />

mesma notação do referencial inercial; outra comodidade).<br />

Também para simplificar um pouco mais,<br />

vamos supor uma densidade volumétrica constante,<br />

ρ = M/l 3 , com M sendo a massa total do cubo.<br />

Como o sistema cartesiano é o que se adapta melhor<br />

à simetria do cubo, então o volume infinitesimal é<br />

dV = dxdydz e dm = ρdV .<br />

As componentes do tensor momento de inércia<br />

são calculadas através da definição (5.60). Naturalmente<br />

a integral em (5.60) deve ser efetuada sobre<br />

toda a região contendo massa, ou seja, dentro do<br />

cubo. Assim, teremos três integrais, uma em cada<br />

direção. Cada uma destas integrais está definida<br />

no intervalo fechado [0, l], pois um dos vértices do<br />

cubo está na origem do nosso sistema de coordenadas,<br />

como pode ser visto na Figura 5.7.<br />

Por exemplo, para a componente I 11 , temos<br />

∫<br />

I 11 = dm [ (x 2 + y 2 + z 2 )δ 11 − x 2]<br />

= M ∫ l<br />

l 3 dx<br />

0<br />

= M ∫ l<br />

l 3 dx<br />

0<br />

∫ l<br />

0<br />

∫ l<br />

0<br />

dy<br />

dz<br />

∫ l<br />

0<br />

∫ l<br />

0<br />

∫ l<br />

dz (y 2 + z 2 )<br />

y 2 dy<br />

∫ l<br />

+ M ∫ l<br />

l 3 dx dy z 2 dz<br />

0 0 0<br />

= M ( )<br />

l 3 l 2 l3 3 + l2 l3 = 2 3 3 Ml2 . (5.63)<br />

Por simetria, devemos ter I 11 = I 22 = I 33 (Faça o<br />

Exercício 79). Similarmente, para o elemento I 12 ,<br />

temos<br />

∫<br />

I 12 = dm [ (x 2 + y 2 + z 2 )δ 12 − xy ]<br />

= M ∫ l<br />

l 3 dx<br />

0<br />

∫ l<br />

= − M ∫ l<br />

l 3 xdx<br />

0<br />

dy<br />

∫ l<br />

0 0<br />

∫ l<br />

0<br />

ydy<br />

dz (−xy)<br />

∫ l<br />

0<br />

dz<br />

= − 1 4 Ml2 . (5.64)<br />

novamente, por argumentos de simetria, devemos<br />

ter I 12 = I 13 = I 23 (Faça o Exercício 79). Assim, o<br />

tensor de inércia é representado no referencial solidário<br />

ao cubo pela matriz simétrica<br />

⎡<br />

⎤<br />

+8 −3 −3<br />

I = β ⎣−3 +8 −3⎦ , β = 1 12 Ml2 . (5.65)<br />

−3 −3 +8<br />

Note que esta matriz é simétrica e possui determinante<br />

diferente de zero, portanto possui uma inversa<br />

(calcule esse determinante e a inversa; veja<br />

também o Exercício 80).<br />

Mencionamos que a matriz representando o tensor<br />

de inércia pode ser re-escrita numa forma diagonal.<br />

Como isto é feito? Para diagonalizarmos<br />

a matriz (5.65), primeiro devemos responder à seguinte<br />

pergunta. Existe números λ e vetores u que<br />

satisfazem (matricialmente)<br />

Iu = λu ou ( I − λ1 ) u = 0, (5.66)<br />

na qual 1 é a matriz identidade? Isto é na verdade<br />

um sistema de três equações lineares para<br />

as três componentes do vetor u como incógnitas,<br />

para cada possível valor do número λ. O problema<br />

é que este número λ também é desconhecido. A<br />

solução vem junto com o problema, pois este sistema<br />

de equações lineares é homogêneo. Aprendemos<br />

(ou iremos aprender, isto não importa aqui)<br />

que um sistema homogêneo admite uma solução<br />

não-trivial (diferente de zero) somente quando o<br />

determinante da matriz ( I − λ1 ) for igual a zero<br />

(Faça o Exercício 81),<br />

∣<br />

∣I − λ1 ∣ = (λ − 2β)(λ − 11β) 2 = 0. (5.67)<br />

Este polinômio em λ é conhecido por polinômio característico.<br />

Assim, é possível somente a existência<br />

de três valores para λ, λ 1 = 2β, λ 2 = 11β e<br />

λ 3 = 11β. Estes valores são conhecidos como autovalores<br />

da matriz (5.65). Quando os autovalores<br />

aparecem repetidos, como λ 2 e λ 3 , eles são denominados<br />

de degenerados. O número de autovalores<br />

iguais é o grau da degenerescência. Neste caso, temos<br />

uma degenerescência de grau dois.<br />

Conhecendo os possíveis valores de λ (os autovalores),<br />

podemos determinar as componentes dos<br />

vetores u para cada valor de λ. Isto é feito substituindo<br />

os valores dos autovalores λ de volta em<br />

(5.66). Estes vetores u são denominados de autovetores.<br />

Como o sistema é homogêneo, haverá<br />

pelo menos uma componente destes vetores que<br />

não poderá ser determinada. Quando há degenerescências,<br />

o número de componentes indeterminadas<br />

é igual ao grau da degenerescência. Muito bem,<br />

76


Capítulo 5. Rotações<br />

5.4. As equações de Euler<br />

ao executarmos este procedimento, teremos (Faça<br />

o Exercício 81)<br />

u 1 = α 1 (1, 1, 1),<br />

u 2 = α 2 (−1, 0, 1),<br />

u 3 = α 3 (−1, 1, 0).<br />

(5.68)<br />

Estes vetores satisfazem (5.66) para quaisquer valores<br />

das constantes α, e são denominados de autovetores.<br />

Note que a estrutura geral em (5.66) é a ação<br />

do tensor de inércia em um de seus autovetores<br />

resultando no mesmo autovetor multiplicado pelo<br />

autovalor correspondente. Este tipo de estrutura,<br />

denominada de equação de autovalor–autovetor, tem<br />

um papel fundamental em ciências exatas em geral.<br />

Evidentemente, como os autovetores u 2 e u 3 correspondem<br />

ao mesmo autovalor λ = 11β, então podemos<br />

usar qualquer combinação linear de u 2 e u 3 .<br />

Esta observação é muito útil para escrevermos uma<br />

matriz ortogonal 4 U que diagonaliza o tensor de<br />

inércia (5.65), ou seja,<br />

I D = U −1 IU =<br />

⎡<br />

⎤<br />

⎣ 2β 0 0<br />

0 11β 0 ⎦ . (5.69)<br />

0 0 11β<br />

A receita para encontrarmos esta matriz ortogonal<br />

U é simples: ela é formado pelos autovetores do<br />

tensor de inércia. Os autovetores (5.68) formam as<br />

colunas da matriz ortogonal U. No entanto, temos<br />

um probleminha: os autovetores escritos na forma<br />

(5.68) não formam uma matriz ortogonal (Faça o<br />

Exercício 82). Isto se deve ao fato de haver uma<br />

degenerescência de grau dois. A solução é formar<br />

combinações lineares dos autovetores degenerados,<br />

u ′ 2 = a 1 u 2 + a 2 u 3 , u ′ 3 = b 1 u 2 + b 2 u 3 , (5.70)<br />

e determinar os valores das constantes a e b que<br />

tornam a matriz formada por estes novos autovetores<br />

uma matriz ortonormal. Uma possível escolha<br />

é a 1 = −1/ √ 2, a 2 = 0, b 1 = 1/ √ 6 e b 2 = −2/ √ 6.<br />

Com esta escolha, teremos<br />

⎡<br />

U =<br />

⎢<br />

⎣<br />

⎤<br />

1√ √2 1 √6 1<br />

3<br />

1√ −2<br />

3<br />

0 √<br />

⎥<br />

6<br />

1√ √−1<br />

√6 1<br />

3 2<br />

⎦ . (5.71)<br />

4 Uma matriz U é denominada de ortogonal quando a sua<br />

inversa U −1 for igual a sua transposta U T , isto é, U −1 =<br />

U T . Em outras palavras, é muito fácil calcular a inversa de<br />

uma matriz ortogonal.<br />

Note que normalizamos o autovetor u 1 antes de<br />

usá-lo. Esta matriz é ortogonal e diagonaliza o<br />

nosso tensor de inércia (5.65) de acordo com a receita<br />

(5.69) (Faça o Exercício 82).<br />

Uma matriz ortogonal tem uma outra propriedade<br />

igualmente importante: ela deixa o módulo<br />

de qualquer vetor inalterado. Por exemplo, calcule<br />

a ação da matriz (5.71) no vetor velocidade angular<br />

(visto como uma matriz coluna)<br />

⎡ ⎤<br />

ω 1<br />

ω = ⎣ω 2<br />

⎦ , ω ′ = Uω. (5.72)<br />

ω 3<br />

Agora calcule o módulo ao quadrado deste novo<br />

vetor ω ′ ,<br />

ω ′ · ω ′ = ω ′T ω ′ = ( Uω ) T<br />

Uω<br />

= ω T U T Uω = ω T ω = ω · ω, (5.73)<br />

na qual fizemos uso da propriedade de ortogonalidade<br />

U T = U −1 . Portanto este resultado nos<br />

mostra que uma matriz ortogonal sempre deixa o<br />

módulo de um vetor invariante.<br />

Em mecânica, os autovetores (5.68) ou (5.71) definem<br />

novos eixos mutuamente perpendiculares, os<br />

quais são denominados de eixos principais do tensor<br />

de inércia. Os autovalores são os momentos de<br />

inércia principais, I 1 = 2β e I 2 = I 3 = 11β. Há<br />

situações em que os três momentos de inércia principais<br />

são iguais e é neste caso (degenerescência tripla)<br />

que o momento de inércia pode ser considerado<br />

um número, como suposto no início da nossa discussão<br />

sobre a dinâmica do movimento rotacional<br />

(Faça o Exercício 83).<br />

A fim de calcularmos a energia cinética rotacional<br />

e o momentum angular mais facilmente para o<br />

sistema mostrado na Figura 5.7, vamos supor que o<br />

cubo esteja girando em torno do eixo x com uma velocidade<br />

angular de módulo ω, isto é, ω = (ω, 0, 0).<br />

Neste caso, a energia cinética (5.61) e o momentum<br />

angular (5.62) são,<br />

T = 1 3 Ml2 ω 2 ,<br />

L = Ml 2 ω(2/3, −1/4, −1/4),<br />

(5.74)<br />

respectivamente (Faça o Exercício 80). O ângulo<br />

entre ω e L é<br />

cos α = ω · L<br />

ωL = 8 √<br />

82<br />

⇒ α = 27.94 ◦ . (5.75)<br />

77


5.4. As equações de Euler Capítulo 5. Rotações<br />

Caso tivéssemos escolhido o vetor velocidade na<br />

direção de um dos eixos principais, por exemplo,<br />

na direção de u 1 (a diagonal do cubo passando pela<br />

origem), então ω e L seriam paralelos.<br />

Resta agora rediscutirmos as equações de movimento<br />

τ = ˙L. Elas continuam válidas? O torque<br />

τ ainda continua sendo calculado pelo produto<br />

vetorial τ = r × F? E o momentum angular,<br />

também continua sendo calculado pelo produto vetorial<br />

L = r × p? Quem é o ponto de referência?<br />

O ponto de referência deve ser sempre um ponto<br />

inercial, como a origem do referencial inercial O,<br />

por exemplo. Assim, r é o vetor posição do elemento<br />

de massa dm. Então, o momentum angular<br />

resultante, devido aos momenta lineares p = dm ṙ<br />

de cada massa dm, calculado em relação à origem<br />

do referencial inercial O, é<br />

∫<br />

L = dm r × ṙ. (5.76)<br />

Lembrando que as forças internas não contribuem<br />

para o torque resultante, então o torque resultante<br />

τ devido ao torque provocado pelas forças externas<br />

F = dm ¨r agindo em cada massa dm, calculado em<br />

relação à origem do referencial inercial O, é<br />

∫<br />

τ = dm r × ¨r. (5.77)<br />

Este torque pode ser re-escrito na seguinte forma:<br />

∫<br />

τ = dm r × ¨r = d ∫<br />

dm r × ṙ =<br />

dt<br />

˙L. (5.78)<br />

Portanto, as equações que governam o movimento<br />

de rotação continuam sendo<br />

τ = ˙L, L = Iω. (5.79)<br />

O único inconveniente em (5.79) é que o tensor de<br />

inércia calculado no referencial inercial O depende<br />

também do tempo e isto pode dar uma forma complicada<br />

para as equações de movimento. A solução<br />

deste problema está na relação (5.49). A solução é<br />

usar a relação (5.49) para calcularmos a taxa ˙L no<br />

referencial inercial conhecendo a taxa ˙L c no referencial<br />

não-inercial (solidário ao corpo rígido), onde o<br />

tensor de inércia é independente do tempo. Assim,<br />

τ = ω × L c + ˙L c , L c = I c ω. (5.80)<br />

Não devemos esquecer que tudo que está no lado<br />

direito em (5.81) deve ser calculado no referencial<br />

não-inercial. Desta forma, podemos re-escrever<br />

(5.80) sem a presença do momentum angular,<br />

τ = ω × Iω + I ˙ω, (5.81)<br />

na qual omitimos o índice do referencial nãoinercial<br />

no lado direito. As três equações de movimento<br />

contidas em (5.81) são conhecidas como<br />

equações de Euler. Elas desempenham o mesmo papel<br />

da segunda lei de Newton para as translações<br />

(Faça o Exercício 84).<br />

Podemos notar em (5.81) que somente quando<br />

o vetor velocidade angular ω for paralelo ao vetor<br />

momentum angular L, isto é, quando ω for proporcional<br />

a um dos autovetores do tensor de inércia I,<br />

pois L = Iω, é que teremos a expressão τ = ˙L = I ˙ω<br />

com a qual estávamos acostumados. Além disto,<br />

devemos nos lembrar que o tensor de inércia nesta<br />

expressão é calculado no referencial solidário ao<br />

corpo rígido, onde ele é independente do tempo,<br />

e que o torque τ está sendo observado no referencial<br />

inercial. Apenas o lado direito de (5.81) é que<br />

está sendo calculado no referencial não-inercial.<br />

Não seria interessante se pudêssemos calcular o<br />

torque em (5.81) também em relação a algum ponto<br />

especial no referencial do corpo rígido? Isto ajudaria<br />

muito em situações em que o centro de massa<br />

do corpo rígido está em movimento. De fato este<br />

ponto especial existe: ele é o centro de massa. Vejamos<br />

como isto é possível. Vamos denotar por R<br />

o vetor posição do centro de massa no referencial<br />

do corpo rígido. Seja u o vetor posição da elemento<br />

de massa dm relativo ao centro de massa. Então,<br />

r = u + R, onde r é o vetor posição da massa dm<br />

no referencial do corpo rígido (faça um esboço mostrando<br />

estes vetores). Todo mundo no corpo rígido.<br />

Da definição do vetor centro de massa (média ponderada<br />

pelas massas dm) temos<br />

∫ ∫<br />

MR = dm r = dm u + MR<br />

∫<br />

⇒ dm u = 0. (5.82)<br />

Este resultado nos permite escrever o vetor momentum<br />

angular em torno do centro de massa na se-<br />

78


Capítulo 5. Rotações<br />

5.4. As equações de Euler<br />

guinte forma,<br />

∫<br />

∫<br />

L = dm u × ṙ = dm u × ( ˙u + Ṙ)<br />

∫<br />

= dm u × ˙u. (5.83)<br />

Procedendo de forma semelhante, o torque em<br />

torno do centro de massa pode ser escrita na seguinte<br />

forma,<br />

∫<br />

∫<br />

τ = dm u × ¨r = dm u × (ü + ¨R)<br />

∫<br />

= dm u × ü = d L. (5.84)<br />

dt<br />

Portanto, podemos continuar usando τ = ˙L, com<br />

L e τ calculados em relação ao centro de massa,<br />

independentemente do centro de massa estar acelerado<br />

ou não. Isto significa que até mesmo o lado<br />

esquerdo de (5.81) pode ser calculado no referencial<br />

não-inercial do corpo rígido, mas este torque<br />

só pode ser calculado em relação a um ponto: o<br />

centro de massa (viva o centro de massa!).<br />

O fato de podermos calcular tanto o torque (em<br />

relação ao centro de massa) quanto o tensor de<br />

inércia no referencial do corpo rígido nos permite<br />

aplicar a segunda lei de Newton com facilidade,<br />

por exemplo, a um corpo rolando em um plano inclinado,<br />

onde o centro de massa certamente está<br />

acelerado. É por isto que, em geral, as equações<br />

(diferenciais) de Euler devem ser resolvidas juntamente<br />

com as equações (diferenciais) provenientes<br />

da segunda lei de Newton para o movimento translacional.<br />

Antes de alguns exemplos, uma observação sobre<br />

a taxa de variação da energia cinética (5.61) no<br />

tempo, ou seja, potência instantânea. Com o tensor<br />

de inércia calculado no referencial do corpo rígido,<br />

a derivada de (5.61) é<br />

˙ T = 1 2( ˙ω · Iω + ω · I ˙ω<br />

)<br />

= ω · I ˙ω, (5.85)<br />

na qual a última igualdade é possível somente porque<br />

o tensor de inércia I é representado por uma<br />

matriz simétrica e o produto escalar também é<br />

simétrico (repita este procedimento usando a representação<br />

matricial do vetor velocidade angular). O<br />

resultado em (5.85) é idêntico a ω · τ ,<br />

˙ T = ω · I ˙ω = ω · τ . (5.86)<br />

Esta expressão para a potência do movimento rotacional<br />

é análoga à expressão v ·F para a potência<br />

no movimento translacional.<br />

Vejamos alguns exemplos (simples) de como podemos<br />

utilizar as equações de Euler (5.81). Vamos<br />

considerar inicialmente um corpo rígido livre<br />

de torques. Esta situação é análoga à situação em<br />

que um corpo está em queda livre. Vamos supor<br />

também que o nosso corpo rígido esteja girando em<br />

torno de um de seus eixos principais, ou seja, vamos<br />

considerar que o vetor velocidade angular seja paralelo<br />

a um autovetor qualquer do tensor de inércia.<br />

Se o torque resultante é nulo, τ = 0, e se ω for paralelo<br />

a I, então a equação de Euler (5.81) nos diz que<br />

o vetor velocidade angular é constante no tempo,<br />

˙ω = 0, pois o tensor de inércia já é constante no<br />

tempo e Iω = λω, com λ constante. Caso o corpo<br />

rígido não esteja girando em torno de um de seus<br />

eixos principais, isto é, ω não será mais paralelo a<br />

I, então o vetor velocidade angular não será mais<br />

constante no tempo, pois teremos ω × Iω + I ˙ω = 0<br />

de (5.81). Note que nestes dois casos o vetor momentum<br />

angular no referencial inercial é constante<br />

no tempo, pois o torque resultante é nulo.<br />

O<br />

ˆk<br />

î<br />

ĵ<br />

r<br />

CM<br />

Rˆk<br />

Figura 5.8: Esfera rolando em um plano horizontal<br />

sem deslizar. O referencial O é inercial.<br />

Como um segundo exemplo, vamos considerar<br />

um corpo rígido com uma distribuição uniforme de<br />

massa M na forma de uma esfera de raio R. O<br />

momento de inércia desta esfera, em torno de seu<br />

centro de massa, isto é, com a origem do referencial<br />

não-inercial colocada no centro de massa, é proporcional<br />

à identidade, I = I1, ou seja, os autovetores<br />

do tensor de inércia são triplamente degenerados<br />

(veja o Exercício 83). Vamos supor também que<br />

esta esfera esteja rolando, sem deslizar, sobre um<br />

plano horizontal. Em princípio, podemos imaginar<br />

a necessidade de uma força externa F = (F x , F y , 0)<br />

ṙ<br />

F<br />

79


5.5. Exercícios Capítulo 5. Rotações<br />

paralela ao plano horizontal para manter a esfera<br />

rolando. Vamos supor que esta força seja constante<br />

e esteja sendo aplicada na parte superior da esfera<br />

conforme mostrado na Figura 5.8. Então esta força<br />

resultante produz um torque em torno do centro de<br />

massa da esfera,<br />

τ = Rˆk × (F x î + F y ĵ) = RF y î + RF x ĵ<br />

= ω × Iω + I ˙ω = Iω × ω + I ˙ω = I ˙ω, (5.87)<br />

onde aproveitamos também para usar as equações<br />

de Euler (5.81). Note que estamos usando o fato de<br />

podermos calcular inclusive o torque no referencial<br />

da esfera, desde que a origem esteja no centro de<br />

massa.<br />

Re-escrevendo (5.87) em termos das componentes,<br />

temos<br />

I ˙ω x = RF y , I ˙ω y = RF x , I ˙ω z = 0. (5.88)<br />

Além desta equações, temos que considerar o movimento<br />

de translação do centro de massa da esfera.<br />

O movimento de translação do centro de massa obedece<br />

a segunda lei de Newton F = M¨r, ou seja,<br />

F x = Mẍ, F y = Mÿ, ¨z = 0. (5.89)<br />

De fato, temos de escolher z = R para que a esfera<br />

permaneça no plano XY (horizontal). Não devemos<br />

esquecer que a condição de rolamento sem<br />

deslizamento impõe condições de vínculos na velocidade<br />

do centro de massa,<br />

ou seja<br />

ṙ = −Rˆk × ω = −Rω y î − Rω x ĵ, (5.90)<br />

ẋ = −Rω y , ẏ = −Rω x , ż = 0. (5.91)<br />

Ao invés de tentarmos resolver as equações acopladas<br />

(5.88), (5.89) e (5.91) formalmente, podemos<br />

improvisar. Por exemplo, podemos derivar as<br />

equações de vínculos (5.91) no tempo e eliminar as<br />

derivadas das componentes da velocidade angular<br />

usando (5.88) e, finalmente, eliminar as componentes<br />

da força usando (5.89), para obtermos<br />

ẍ = −R ˙ω y = − R2<br />

I F x = − MR2 ẍ<br />

I<br />

⇒ ẍ = 0 ⇒ ˙ω y = 0, (5.92)<br />

ÿ = −R ˙ω x = − R2<br />

I F y = − MR2 ÿ<br />

I<br />

⇒ ÿ = 0 ⇒ ˙ω x = 0. (5.93)<br />

Pronto, temos o que procurávamos: ¨r = 0, movimento<br />

retilíneo para o centro de massa, e ˙ω = 0,<br />

movimento uniforme para o vetor velocidade angular.<br />

Vamos às surpresas. Primeiro, note que a componente<br />

ω z não precisa ser nula, como poderíamos<br />

esperar. Segundo, note que não há necessidade de<br />

uma força externa para manter a esfera rolando.<br />

Para que ela permaneça rolando, basta ter uma velocidade<br />

angular inicial. Esta situação é completamente<br />

análoga ao movimento uniforme do centro<br />

de massa. Basta haver inércia (massa ou momento<br />

de inércia) para haver movimento.<br />

5.5 Exercícios<br />

Exercício 69<br />

Uma roda gira com uma aceleração angular constante<br />

de 3.5 rad/s 2 . A velocidade angular da roda<br />

é de 2 rad/s em t = 0. Suponha que a posição angular<br />

seja θ = 0 em t = 0. (a) Qual é o ângulo<br />

percorrido pela roda entre t = 0 e t = 2 s? (b)<br />

Qual é a velocidade angular em t = 2 s? Expresse<br />

também esta velocidade angular em voltas (ou revoluções)<br />

por minuto (rpm).<br />

Exercício 70<br />

Considere um corpo rígido formado por duas<br />

partículas de massas m 1 e m 2 separadas pela<br />

distância l. Colocando a origem do sistema de<br />

coordenadas no centro de massa (faça um desenho),<br />

(a) mostre que m 1 ⃗r 1 + m 2 ⃗r 2 = 0. Mostre<br />

também que os respectivos módulos satisfazem<br />

m 1 r 1 − m 2 r 2 = 0 e l = r 1 + r 2 . (b) Mostre então<br />

que a relação (5.14) é verdadeira. (c) Suponha<br />

que estas partículas sejam dois átomos de oxigênio,<br />

m = 2.66 × 10 −26 kg, separados pela distância<br />

(média) l = 1.21 × 10 −10 m. Calcule o momento de<br />

inércia em relação ao eixo passando pelo centro de<br />

massa (I 0 ) e também em relação a um eixo (paralelo<br />

ao primeiro) passando por um dos átomos de<br />

oxigênio (I ′ ). Calcule também as respectivas energias<br />

cinéticas.<br />

Exercício 71<br />

Mostre que o momento de inércia de uma distribuição<br />

de massa, homogênea e retilínea (fio), em<br />

relação a uma de suas extremidades (perpendicularmente),<br />

é Ml 2 /3, onde M é a massa total e l é<br />

o comprimento do fio. Use o Teorema 9 dos eixos<br />

80


Capítulo 5. Rotações<br />

paralelos para calcular o momento de inércia em<br />

relação a um eixo passando pelo centro de massa.<br />

Exercício 72<br />

Mostre que o momento de inércia de uma distribuição<br />

de massa, homogênea e circular de raio a<br />

(anel), em relação ao centro do anel (perpendicularmente<br />

ao plano do anel), é Ma 2 , onde M é a<br />

massa total e a é o raio do anel.<br />

Exercício 73<br />

Mostre que uma área infinitesimal dA em coordenadas<br />

polares (r, ϕ) é<br />

dA = rdϕ dr. (5.94)<br />

Faça um desenho mostrando todas as quantidades<br />

envolvidas.<br />

Exercício 74<br />

Considere uma quantidade vetorial A qualquer que<br />

esteja mudando no tempo, A = A(t). Suponha que<br />

o diferencial dA seja perpendicular a A, A·dA = 0.<br />

Mostre então que<br />

d(A · A) = 2A dA = 0. (5.95)<br />

Este resultado mostra que o módulo do vetor A é<br />

uma quantidade conservada (admitindo que A ≠<br />

0).<br />

Exercício 75<br />

Faça A = ω em (5.49) e mostre que ˙ω = ˙ω c . Use<br />

as propriedades do produto vetorial. Use também<br />

o fato de que ω é o mesmo nos dois referenciais.<br />

Exercício 76<br />

Efetue as somas em (5.52) explicitamente e mostre<br />

que<br />

( )<br />

ω × r<br />

1 = ω 2x 3 − ω 3 x 2 , (5.96)<br />

( )<br />

ω × r = ω 2 3x 1 − ω 1 x 3 , (5.97)<br />

( )<br />

ω × r = ω 3 1x 2 − ω 2 x 1 , (5.98)<br />

conforme previsto pela propriedade (2.25) e ilustrado<br />

pela Figura 2.4 (equivalente ao uso de determinantes).<br />

5.5. Exercícios<br />

Exercício 77<br />

Verifique explicitamente a propriedade (5.56). Efetue<br />

também explicitamente as seguintes somas:<br />

3∑<br />

k=1 s=1<br />

3∑<br />

k=1 s=1<br />

3∑<br />

δ jr δ ks x k x s = δ jr<br />

3∑<br />

x k x k = δ jr r 2 , (5.99)<br />

k=1<br />

3∑<br />

δ js δ kr x k x s = x j x r . (5.100)<br />

Exercício 78<br />

Mostre que a energia cinética (5.61) pode ser reescrita<br />

nas seguintes formas:<br />

T = 1 2 ω · Iω = 1 2 ω · L = 1 2 LI−1 L, (5.101)<br />

onde L é o momentum angular (5.62).<br />

Exercício 79<br />

Use a definição (5.60) para calcular todos os seis<br />

elementos do tensor de inércia para o cubo descrito<br />

na Figura 5.7 e no texto ao lado da figura. Compare<br />

seus resultados com (5.63) e (5.64).<br />

Exercício 80<br />

Use ω = (ω, 0, 0) e o tensor de inércia (5.65) para<br />

determinar a energia cinética (5.61) e o momentum<br />

angular (5.62). Recalcule também a energia<br />

cinética usando as expressões em (5.101). Calcule<br />

também o ângulo entre ω e L. Faça um desenho<br />

(bem feito) mostrando estes dois vetores juntamente<br />

com o cubo da Figura 5.7 e o respectivo<br />

sistema de coordenadas.<br />

Exercício 81<br />

Calcule o polinômio característico da matriz (5.65)<br />

e mostre que ele pode ser fatorado como mostrado<br />

em (5.67). Encontre o autovetor u 1 (determine<br />

suas componentes) correspondente ao autovalor<br />

λ 1 = 2β, Iu 1 = λ 1 u 1 . Normalize este autovetor,<br />

isto é, imponha que u 1 · u 1 = 1. Isto determina a<br />

constante arbitrária α 1 (α 1 = 1/ √ 3).<br />

Exercício 82<br />

Normalize os três autovetores apresentados em<br />

(5.68) e construa uma matriz U contendo estes autovetores<br />

normalizados como colunas,<br />

⎡<br />

⎢<br />

U = ⎣<br />

√1<br />

3<br />

√−1<br />

2<br />

√−1<br />

2<br />

√1<br />

3<br />

0 √2 1<br />

1 √<br />

3<br />

−1 √<br />

2<br />

0<br />

⎤<br />

⎥<br />

⎦ . (5.102)<br />

81


5.5. Exercícios Capítulo 5. Rotações<br />

Mostre que esta matriz não é ortogonal, ou seja,<br />

U −1 ≠ U T . Mostre que a matriz definida em (5.71)<br />

é ortogonal. Mostre também que esta matriz (5.71)<br />

diagonaliza o tensor de inércia (5.65) segundo a<br />

prescrição (5.69).<br />

Exercício 83<br />

Determine o tensor de inércia (5.60) de uma esfera<br />

maciça de massa M, densidade uniforme e raio R.<br />

Coloque a origem do referencial no centro da esfera.<br />

Exercício 84<br />

Escreva explicitamente as três equações de movimento<br />

que estão contidas na relação vetorial (5.81).<br />

Use o tensor de inércia na forma diagonal e deixe as<br />

expressões em termos das componentes do torque,<br />

do tensor de inércia e do vetor velocidade angular.<br />

82


Capítulo 6<br />

Gravitação<br />

Gravitação é a parte da ciência que estuda a<br />

atração mútua entre corpos materiais. A gravitação<br />

como ciência iniciou-se com o modelo heliocêntrico<br />

proposto por Copérnico em 1543. Suas<br />

idéias, extremamente revolucionárias para a época,<br />

influenciaram profundamente Kepler, o qual, após<br />

examinar um volume gigantesco de observações astronômicas<br />

durante o período de 1609 a 1619, foi<br />

capaz de descobrir as leis fundamentais do sistema<br />

heliocêntrico, conhecidas hoje como leis de Kepler.<br />

Detalhe importante, a idéia de atração mútua, isto<br />

é, da existência de uma força de atração entre massas<br />

foi introduzida por Newton, muito tempo depois.<br />

Newton foi capaz de deduzir as leis de Kepler<br />

em 1680 a partir de suas leis da mecânica, com<br />

base no conceito de força como causa de qualquer<br />

movimento. Ter associado o movimento de planetas<br />

a uma força foi um feito realmente impressionante<br />

para a época dele. Segundo Newton, os<br />

planetas seguem em suas órbitas estáveis em um<br />

espaço imutável, descrito pela geometria Euclideana,<br />

com um tempo absoluto, graças a uma força<br />

de atração agindo a distância e instantaneamente,<br />

a qual passou a ser conhecida por força gravitacional.<br />

Para haver esta força, corpos precisam ter<br />

massa (ou carga) gravitacional, a qual se mostrou<br />

idêntica à massa inercial aparecendo na segunda lei<br />

de Newton (veja os comentários feitos no final da<br />

Seção 3.2). Portanto, Newton foi o primeiro a explorar<br />

quantitativamente este fato curioso sobre a<br />

nossa natureza: a igualdade entre massa inercial<br />

que<br />

e massa gravitacional. Newton também realizou<br />

muitos experimentos para verificar esta igualdade,<br />

tendo constatado sua veracidade com uma precisão<br />

de uma parte em dez. É ainda mais surpreendente<br />

esta mesma igualdade entre massa inercial e<br />

massa gravitacional tenha também aberto as portas<br />

para o advento da teoria da relatividade geral<br />

de Einstein em 1915, a qual corrige e re-interpreta<br />

a teoria Newtoniana da gravitação.<br />

Na teoria da relatividade geral de Einstein, o<br />

tempo não é absoluto e nem o espaço é imutável.<br />

Agora tempo e espaço se juntam para formar um<br />

novo espaço: o espaçotempo. Este espaçotempo<br />

não é Euclideano, pois ele pode ser curvado. Einstein<br />

conseguiu descrever intrinsicamente (sem termos<br />

que sair dele) as propriedades geométricas<br />

deste espaçotempo, como curvatura e torção,<br />

através de um conjunto de equações diferenciais.<br />

Segundo Einstein, a forma do espaçotempo é ditada<br />

pela presença de matéria (e energia). Na<br />

ausência de matéria, o espaçotempo é plano, obedecendo<br />

a métrica de Minkowski da relatividade<br />

especial, com um subespaço tridimenional idêntico<br />

ao espaço Euclideano usado por Newton (veja o<br />

Capítulo 7). Segundo Einstein, a força Newtoniana<br />

é o efeito de movimentarmos num espaçotempo<br />

curvado pela presença do Sol e da Terra, ou seja,<br />

que gravidade é equivalente a um movimento acelerado.<br />

Naturalmente, muitos experimentos comprovam<br />

a existência de um espaçotempo curvo (nao-<br />

Euclideano) bem como os resultados previstos pelas<br />

equações de Einstein.<br />

Na Seção 6.1, veremos como um problema de<br />

dois corpos, representando um sistema solar formado<br />

pelo Sol e pela Terra, isolados no universo,<br />

pode ser reduzido ao problema de um único corpo.<br />

Na Seção 6.2, estudaremos a energia potencial de<br />

Kepler, a qual dá origem à força gravitacional Newtoniana.<br />

Veremos na Seção 6.3 como as leis de Kepler<br />

podem ser derivadas explicitamente da força<br />

83


6.1. Um sistema de dois corpos Capítulo 6. Gravitação<br />

gravitacional Newtoniana. Na Seção 6.4, introduziremos<br />

a noção de campo gravitacional, um ferramenta<br />

importante no estudo de muitas outras<br />

teorias, como o eletromagnetismo, para citar um<br />

exemplo. A Seção 6.5 apresenta, de forma qualitativa,<br />

as principais correções à teoria da gravitação<br />

Newtoniana.<br />

6.1 Um sistema de dois corpos<br />

Naturalmente, consideraremos aqui uma versão<br />

bastante simplificada do nosso sistema solar: o<br />

Sol com massa m 1 e um planeta apenas, a Terra,<br />

por exemplo, de massa m 2 , como indicado na Figura<br />

6.1. Por comodidade, consideraremos este sistema<br />

com isolado no universo.<br />

O<br />

⃗u 1 ⃗u 2<br />

m 1 m 2<br />

⃗r 1 ⃗R ⃗r 2<br />

⃗r<br />

Figura 6.1: Coordenadas de um sistema formado<br />

por dois corpos. O vetor R ⃗ indica a posição do<br />

centro de massa.<br />

Este é um sistema de dois corpos, o qual será<br />

mantido isolado dos demais corpos. Desta forma,<br />

teremos uma excelente oportunidade de aplicar as<br />

leis de Newton a um sistema constituído por mais<br />

de um corpo. Supondo que este sistema esteja isolado,<br />

então a força resultante ⃗ F R = ˙⃗ PR , onde ⃗ P R é<br />

o momentum linear resultante, é nula,<br />

⃗F R = ˙⃗ PR = m 1¨⃗r1 + m 2¨⃗r2 = 0. (6.1)<br />

Portanto, de acordo com o Teorema 6, há conservação<br />

do vetor momentum linear total,<br />

⃗P R = m 1 ˙⃗r1 + m 2 ˙⃗r2 ,<br />

˙⃗P R = 0. (6.2)<br />

Desta forma, de acordo com a primeira lei de Newton,<br />

este sistema solar ou está em movimento uniforme<br />

ou está em repouso. No entanto, os dois corpos<br />

que compõem este sistema estão interagindo<br />

através de uma força (gravitacional), como proposto<br />

pela primeira vez por Newton. Isto significa<br />

que há movimento relativo entre eles e que este<br />

movimento é não uniforme. Como podemos descrever<br />

este movimento relativo? Qual será o melhor<br />

sistema de coordenadas? Qual será a melhor maneira<br />

de encontrarmos as equações horárias destes<br />

dois corpos? Sabemos que o sistema (Sol-Terra)<br />

está em movimento uniforme. Assim, a primeira<br />

providência é eliminar esse movimento uniforme.<br />

A forma mais eficiente de fazermos isto é introduzindo<br />

o vetor posição centro de massa R ⃗ mostrado<br />

na Figura 6.1<br />

Em geral, em um sistema contendo N corpos discretos,<br />

o vetor posição centro de massa R ⃗ é definido<br />

por uma média ponderada de todas as posições individuais<br />

⃗r i com as respectivas massas m i atuando<br />

como pesos,<br />

⃗R = 1 M<br />

N∑<br />

N∑<br />

m i ⃗r i , M = m i . (6.3)<br />

i=1<br />

i=1<br />

A Figura 6.1 mostra o vetor posição centro de<br />

Massa para um sistema de dois corpos. Como veremos,<br />

este vetor posição do centro de massa tem<br />

um papel muito importante na dinâmica de um sistema<br />

de muitos corpos. Vale ressaltar que, sendo o<br />

sistema formado por uma distribuição contínua de<br />

massa, as massa e somas em (6.3) devem ser trocadas,<br />

respectivamente, por massas infinitesimais e<br />

integrais na região contendo massa,<br />

⃗R = 1 ∫<br />

∫<br />

dm ⃗r, M = dm, (6.4)<br />

M<br />

V<br />

onde dm é uma quantidade infinitesimal de massa<br />

ocupando o volume (ou área, ou comprimento) infinitesimal<br />

dV e ⃗r é o vetor posição da massa dm.<br />

A integração deve ser feita sobre toda a região V<br />

contendo massa e ρ = dm/dV é a densidade de<br />

massa nesta região. Cálculos de centros de massa<br />

de distribuições contínuas são exercícios excelentes<br />

de cálculo com muitas variáveis.<br />

Os novos vetores posições ⃗u i = ⃗r i − ⃗ R, relativos<br />

ao centro de massa, como mostrados na Figura 6.1,<br />

possuem uma propriedade interessante. Inserindo<br />

as coordenadas relativas ao centro de massa via<br />

V<br />

84


Capítulo 6. Gravitação<br />

6.1. Um sistema de dois corpos<br />

⃗r i = ⃗ R + ⃗u i na definição do centro de massa (6.3)<br />

(faça o Exercício 87), obtemos<br />

N∑<br />

m i ⃗u i = 0, ⃗u i = R ⃗ − ⃗r i . (6.5)<br />

i=1<br />

Este resultado será muito útil logo adiante.<br />

De volta ao nosso sistema inicial de dois corpos:<br />

o sistema solar Sol-Terra (veja a Figura 6.1). Segundo<br />

a definição (6.3), o centro de massa do sistema<br />

Sol-Terra é<br />

⃗R = 1 ( )<br />

m1 ⃗r 1 + m 2 ⃗r 2 , M = m1 + m 2 . (6.6)<br />

M<br />

A posição do centro de massa de um sistema de<br />

dois corpos pertence à curva de menor comprimento<br />

ligando estes dois corpos. Esta curva é denominada<br />

de geodésica. No caso de um espaço Euclideano,<br />

onde o nosso sistema solar está inserido, esta<br />

geodésica é uma reta. A demonstração deste resultado<br />

é um exercício muito interessante de Geometria<br />

Analítica (faça o Exercício 86). Contraste<br />

este resultado com um espaço formado por uma<br />

casca esférica, onde geodésicas são arcos de circunferências<br />

(as quais não são retas).<br />

Suponha que as massas em (6.6) sejam constantes.<br />

Então podemos multiplicar o centro de massa<br />

pela massa total e derivar o resultado no tempo,<br />

M ˙⃗ R =<br />

(<br />

m1 ˙⃗r1 + m 2 ˙⃗r2<br />

)<br />

= ⃗ PR . (6.7)<br />

Este resultado nos mostra que o nosso sistema contendo<br />

dois corpos pode ser visto como um único<br />

corpo de massa total M = m 1 + m 2 colocado na<br />

posição do centro de massa. Por isto, quando estamos<br />

interessados no movimento de um sistema<br />

de muito corpos como um todo, a posição centro<br />

de massa é importante. Naturalmente, derivando<br />

a (6.7) mais uma vez no tempo obtemos a segunda<br />

lei de Newton<br />

M ¨⃗ R =<br />

(<br />

m1¨⃗r1 + m 2¨⃗r2<br />

)<br />

= ˙⃗PR = ⃗ F R . (6.8)<br />

Este resultado reforça a interpretação onde trocamos<br />

o sistema de muitos corpos por um único corpo<br />

de massa M, igual à massa total do sistema, localizada<br />

na posição do centro de massa, atuada pela<br />

força resultante do sistema. Independentemente do<br />

movimento relativo dos corpos que compõem o sistema,<br />

o movimento do sistema como um todo pode<br />

ser descrito como o movimento de um único corpo<br />

localizado no centro de massa. Este resultado vale<br />

para um sistema com um número arbitrário de componentes.<br />

Esta re-interpretação em termos de um<br />

único corpo de massa total é útil para visualizarmos<br />

o movimento do centro de massa do sistema de<br />

muitos corpos, sem nos precupar com o movimento<br />

de seus constituintes. Note também que, estando<br />

o sistema de dois corpos isolado, então ⃗ F R = 0<br />

em (6.8), implicando que as forças internas agindo<br />

neste sistema precisam obedecer a terceira lei de<br />

Newton, m 1¨⃗r1 = −m 2¨⃗r2 . Isto significa que a força<br />

gravitacional proposta por Newton, a qual é atrativa,<br />

deve agir ao longo da reta que passa pelas<br />

massas m 1 e m 2 .<br />

Como o nosso sistema solar Sol-Terra está isolado,<br />

portanto o seu centro de massa está em movimento<br />

uniforme, vamos procurar uma maneira de<br />

visualizar o movimento interno de seus constituintes.<br />

Para isto, mudaremos a origem do nosso referencial<br />

inercial para a posição do centro de massa,<br />

onde as posições do Sol e da Terra são dadas pelos<br />

novos vetores posições ⃗u 1 e ⃗u 2 , respectivamente<br />

(veja a Figura 6.1). Este movimento interno pode<br />

ser muito bem compreendido analisando a energia<br />

cinética deste sistema. A energia cinética total de<br />

um sistema de N corpos (sem rotações) é a soma<br />

individual das energias cinéticas de cada corpo,<br />

T =<br />

N∑<br />

i=1<br />

1<br />

2 m i|| ˙⃗r i || 2 . (6.9)<br />

Introduzindo as coordenadas ⃗u i relativas ao centro<br />

de massa, definidas na Figura 6.1, ⃗r i = ⃗ R + ⃗u i , a<br />

energia cinética (6.9) pode ser re-escrita como<br />

T =<br />

N∑<br />

i=1<br />

+ 1 2<br />

1<br />

2 m i|| ˙⃗ R + ˙⃗ui || 2 = 1 2 M|| ˙⃗ R||<br />

2<br />

N∑<br />

i=1<br />

(<br />

m i || ˙⃗u<br />

∑ N )<br />

i || 2 + m i ˙⃗ui · ˙⃗ R. (6.10)<br />

i=1<br />

Como podemos ver, o último termo na energia<br />

cinética (6.10) contém a derivada de (6.5), a qual,<br />

conseqüentemente, é nula também. Portanto, a<br />

energia cinética total (6.10) pode ser escrita como<br />

T = 1 2 M|| ˙⃗ R|| 2 + 1 2<br />

N∑<br />

m i || ˙⃗u i || 2 . (6.11)<br />

i=1<br />

85


6.2. A energia potencial de Kepler Capítulo 6. Gravitação<br />

Este resultado nos ensina que a energia cinética<br />

total é a soma de duas parcelas: uma parcela é<br />

devido ao movimento do sistema como um todo,<br />

ou seja, de uma massa M na posição do centro<br />

de massa com velocidade ˙⃗ R. A outra parcela é<br />

a energia cinética do movimento relativo (ao centro<br />

de massa) dos corpos que constituem o sistema.<br />

Em outras palavras, há claramente dois movimento<br />

neste sistema que podem ser separados completamente.<br />

Isto nos permite concentrar nossa atenção<br />

em um deles, por exemplo, no movimento relativo<br />

ao centro de massa.<br />

Para o nosso sistema de corpos, usando (6.11),<br />

a energia cinética do movimento relativo ao centro<br />

de massa é<br />

T = 1 2 m 1|| ˙⃗u 1 || 2 + 1 2 m 2|| ˙⃗u 2 || 2 . (6.12)<br />

É extremamente interessante re-escrever esta energia<br />

cinética em termos do vetor posição ⃗r = ⃗u 2 −⃗u 1 ,<br />

ou seja, vamos mudar novamente a origem do nosso<br />

referencial inercial, desta vez para a posição do Sol<br />

⃗r 1 (veja a Figura 6.1). Usando o resultado (6.5)<br />

com N = 2, temos (faça o Exercício 88)<br />

⃗u 1 = − m 2<br />

M ⃗r, ⃗u 2 = m 1<br />

⃗r, (6.13)<br />

M<br />

onde M = m 1 + m 2 é a massa total. Vale observar<br />

que este resultado somente é válido para um<br />

sistema de dois corpos. Substituindo estes resultados<br />

na energia cinética (6.12), obtemos (faça o<br />

Exercício 89)<br />

T = 1 2 µ|| ˙⃗r|| 2 , µ = m 1m 2<br />

m 1 + m 2<br />

. (6.14)<br />

Note que a constante µ tem a dimensão de massa.<br />

Ela é denominada de massa reduzida do sistema de<br />

dois corpos. Esta massa reduzida nos permite reinterpretar<br />

a energia cinética do movimento relativo<br />

como sendo a energia cinética de um único<br />

corpo de massa reduzida µ, localizada pelo vetor<br />

posição ⃗r, cuja taxa de variação no tempo nos dá a<br />

velocidade ˙⃗r. Portanto, conseguimos um feito extraordinário:<br />

nós reduzimos um problema de dois<br />

corpos a um problema de um único corpo! Esta<br />

redução é possível apenas em um sistema de dois<br />

corpos. Como veremos, é importante que esta<br />

redução seja compatível também com a energia potencial<br />

deste sistema.<br />

6.2 A energia potencial de Kepler<br />

Newton propôs uma força atrativa entre corpos materiais<br />

que pudesse explicar as leis de Kepler: (1)<br />

a órbita de um planeta é uma elipse com o Sol em<br />

um de seus focos; (2) a área formada pela trajetória,<br />

pela reta que une os centros de massa do Sol e do<br />

planeta e uma reta fixa de referência (geralmente<br />

a reta contendo o semi-eixo maior da trajetória)<br />

é constante no tempo; e (3) a razão entre o cubo<br />

da distância média a (a ser definida) do planeta<br />

até o Sol e o quadrado do período orbital τ é uma<br />

constante para todos os planetas, a 3 /τ 2 = cte. A<br />

primeira lei estabelece que a trajetória é uma curva<br />

espacial plana, sem torção, fechada, dada por uma<br />

elipse, a qual é uma seção cônica muito bem conhecida.<br />

A segunda estabelece que um planeta deve<br />

ter uma velocidade tangencial maior quando estiver<br />

mais próximo do Sol. Estas duas leis dificilmente<br />

dão qualquer pista para Newton estabelecer<br />

a forma que deve ter a sua força gravitacional. No<br />

entanto, a terceira lei parece ser útil neste sentido.<br />

Suponha que uma órbita circular (um caso extremo<br />

de uma elipse) de raio a (igual à distância<br />

média), com o Sol de massa m 1 no centro. Seja v<br />

o módulo da velocidade tangencial do planeta de<br />

massa m 2 nesta órbita circular. Então o período<br />

τ desta órbita (movimento circular uniforme) é<br />

τ = 2πa/v (comprimento da trajetória circular pela<br />

velocidade uniforme). Num movimento circular de<br />

raio a, o módulo da aceleração (centrípeta) é v 2 /a,<br />

onde v é o módulo da velocidade tangencial. Esta<br />

acelaração multiplicada pela massa m 2 do planeta<br />

deve ser igual ao módulo da força (gravitacional)<br />

agindo sobre ela (segunda lei de Newton). Após<br />

algumas tentativas, Newton pôde supôr uma força<br />

gravitacional cujo módulo é Gm 1 m 2 /a 2 , isto para<br />

que a velocidade tangencial possa ser escrita como<br />

v = √ Gm 1 /a, onde G é uma constante universal.<br />

Substituindo esta velocidade na expressão do<br />

período, τ = 2πa/v, obtemos a terceira lei de Kepler,<br />

τ 2 /a 3 = 4π 2 Gm 1 . Isto significa que a força<br />

gravitacional atrativa procurada por Newton deve<br />

ter as seguintes características: (1) ser proporcional<br />

ao produto entre as duas massas; (2) ser inversamente<br />

proporcional ao quadrado da distância<br />

entre as massas; e (3) estar ao longo da reta que<br />

passa pelas duas massas. Colocando a origem de<br />

86


Capítulo 6. Gravitação<br />

6.2. A energia potencial de Kepler<br />

um sistema de coordenadas inercial na posição do<br />

Sol, massa m 1 , então as características acima podem<br />

ser re-escritas na forma<br />

⃗F = −K ˆr r 2 = −K ⃗r r 3 , K = Gm 1m 2 , (6.15)<br />

onde ⃗r é o vetor posição do planeta de massa m 2<br />

(veja a Figura 6.1) e G uma constante (universal),<br />

G = 6.673 × 10 −11 N·m 2 /kg 2 .<br />

A força gravitacional (6.15) agindo na massa m 2<br />

(Terra) é conservativa? A resposta depende do trabalho<br />

associado a esta força ser independente da<br />

trajetória ou não. Então, vamos calcular o trabalho<br />

que um agente externo deve realizar para levar<br />

a massa m 2 desde uma distância muito grande<br />

(tomando o devido cuidado para manter m 2 sempre<br />

longe de outros corpos além de m 1 ) até uma<br />

distância r de m 1 . Usando a definição (4.1), este<br />

trabalho é<br />

∫ r<br />

∆W = ⃗F · d⃗s, (6.16)<br />

∞<br />

onde d⃗s é o deslocamento infinitesimal sobre uma<br />

determinada trajetória. Como a força gravitacional<br />

(6.15) é radial, isto é, F ⃗ = F (r)ˆr, com r sendo<br />

uma das coordenadas esféricas, podemos escrever o<br />

deslocamento infinitesimal d⃗s como d⃗s = d⃗r + dΩ,<br />

⃗<br />

onde dΩ ⃗ é perpendicular ao deslocamento infinitesimal<br />

d⃗r na direção radial. Desta forma, o trabalho<br />

(6.16) pode ser reescrito como<br />

∆W =<br />

∫ r<br />

∞<br />

⃗F · d⃗r = −K<br />

∫ r<br />

∞<br />

dr<br />

r 2 = K r , (6.17)<br />

para uma trajetória qualquer. Note que a integral<br />

na segunda igualdade desta expressão não depende<br />

mais da orientação dos vetores força e posição, mas<br />

somente da distância r até a massa m 1 . Portanto,<br />

a força gravitacional (6.15) é conservativa. Isto implica<br />

que há uma função energia potencial neste<br />

sistema solar. Esta energia potencial, segundo a<br />

definição (4.8), é menos o trabalho realizado (6.17),<br />

V = − K r = − K<br />

√<br />

x2 + y 2 + z , (6.18)<br />

2<br />

onde x, y e z são as coordenadas do vetor posição<br />

⃗r da massa m 2 e K = Gm 1 m 2 . Por razões<br />

históricas, esta energia potencial é conhecida como<br />

a “energia potencial de Kepler”, embora Kepler<br />

não a tenha sugerido diretamente. Naturalmente,<br />

usando a prescrição (4.9), a força gravitacional<br />

(6.15) pode ser obtida desta energia potencial (Faça<br />

o Exercício 85).<br />

A energia potencial de Kepler (6.18) tem uma<br />

característica muito especial. Embora ela seja uma<br />

função de três variáveis (em coordenadas Cartesianas),<br />

podemos visualizar suas curvas de níveis<br />

V = cte < 0, as quais são superfícies esféricas de<br />

raio K/|V |. Isto significa que a energia potencial de<br />

Kepler tem o mesmo valor sobre uma casca esférica.<br />

Uma energia potencial com esta característica é<br />

dita possuir uma simetria esférica. Por isso, quando<br />

expressa em coordenadas esféricas, a energia potencial<br />

de Kepler se apresenta na sua forma mais<br />

simples possível, como pode ser vista na primeira<br />

igualdade em (6.18). Uma energia potencial com<br />

simetria esférica também é denominada de energia<br />

potencial central, ou seja, é uma energia potencial<br />

que depende apenas do módulo do vetor posição do<br />

objeto sob sua ação. Qualquer sistema com uma<br />

energia potencial com simetria esférica é melhor<br />

descrito em coordenadas esféricas. A relação entre<br />

coordenadas cartesianas (x, y, z) e coordenadas<br />

esféricas (r, θ, ϕ) está apresentada no Apêndice C.<br />

Sendo uma energia potencial central, a energia<br />

potencial de Kepler (6.18) permanece inalterada<br />

perante a redução do problema de dois corpos a<br />

um corpo que fizemos na Seção 6.1. Então, nosso<br />

sistema solar pode ser visto da seguinte maneira:<br />

um referencial de coordenadas com a origem em<br />

m 1 , como indicado na Figura 6.1, e um corpo de<br />

massa reduzida µ na posição ⃗r sujeito ao potencial<br />

(6.18). Assim, a energia mecânica do nosso sistema<br />

solar, sem a energia cinética do centro de massa, é<br />

com<br />

E = T + V = 1 2 µ|| ˙⃗r|| 2 − K r , (6.19)<br />

µ = m 1m 2<br />

m 1 + m 2<br />

, K = Gm 1 m 2 . (6.20)<br />

Como a energia potencial deste sistema é conservativa,<br />

então a energia mecânica (6.19) é uma quantidade<br />

conservada, isto é, Ė = 0. Desta forma, podemos<br />

usar o procedimento que aprendemos com o<br />

oscilador harmônico para calcular períodos (veja os<br />

detalhes na Seção 4.5) e, com um pouco de sorte,<br />

equações horárias, ou pelo menos uma equação para<br />

a trajetória.<br />

87


6.3. Trajetórias Capítulo 6. Gravitação<br />

6.3 Trajetórias<br />

Há mais quantidades conservadas neste problema,<br />

além da energia mecânica? Havendo mais quantidades<br />

conservadas, talvez haja mais simplificações.<br />

Afinal, temos uma energia potencial com simetria<br />

esférica, a qual produz uma força na direção radial,<br />

isto é, na direção do versor ˆr na Figura 6.1. De<br />

fato, dado estas condições, podemos mostrar que o<br />

vetor momentum angular<br />

⃗L = ⃗r × ⃗p, ⃗p = µ ˙⃗r, (6.21)<br />

é também uma quantidade conservada, ou seja, independente<br />

do tempo (faça o Exercício 90).<br />

Ter encontrado esta outra quantidade conservada<br />

é uma dádiva! De imediato, vemos que o movimento<br />

do nosso sistema solar deve ocorrer em um<br />

plano, pois L ⃗ é perpendicular a ⃗r e ⃗p, segundo a<br />

definição (6.21) e a definição (2.16) de produto vetorial.<br />

Então podemos escolher o ângulo ϕ em (C.1)<br />

igual a 90 graus. Esta escolha traz a trajetória para<br />

o plano XY e o momento angular para o eixo Z.<br />

Em termos operacionais, isto significa que as coordenadas<br />

esféricas (C.1) se reduzem a coordenadas<br />

polares (r, θ) dadas em (C.6). Esta redução ao sistema<br />

de coordenadas polares facilita bastante nosso<br />

trabalho. Por exemplo, da conservação do momentum<br />

angular (6.21) obtemos (Faça o Exercício 91)<br />

L = µr 2 ˙θ, ˙L = 0. (6.22)<br />

O módulo do vetor velocidade também pode ser<br />

escrito numa forma bastante simples (Exercício 91),<br />

v 2 = || ˙⃗r|| 2 = ṙ 2 + r 2 ˙θ2 . (6.23)<br />

Temos duas possibilidades neste momento. Primeiro,<br />

podemos usar (6.22) para eliminar a dependência<br />

da velocidade angular em (6.23), pois<br />

˙θ = L/µr 2 (faça o Exercício 92). Neste caso estaremos<br />

interessados na equação horária r = r(t), a<br />

qual é determinada resolvendo a equação diferencial<br />

proveniente da energia mecânica (6.19),<br />

E = 1 2 µṙ2 + U(r),<br />

U(r) = L2 1<br />

2µ r 2 − k r . (6.24)<br />

Note que esta expressão pode ser interpretada como<br />

a energia cinética de uma massa µ, com velocidade<br />

(escalar) ṙ, sujeita a uma energia potencial efetiva<br />

U (veja a Figura 6.2). Será a análise desta energia<br />

potencial que irá nos revelar as possíveis trajetórias<br />

do nosso sistema original.<br />

0<br />

E<br />

U<br />

r − r +<br />

Figura 6.2: Energia potencial efetiva (6.25) para<br />

o problema de dois corpos reduzido ao problema<br />

de um corpo na presença da energia potencial de<br />

Kepler. r ± são os pontos de retorno de trajetórias<br />

elípticas.<br />

Suponha que a energia mecânica tenha o valor<br />

dado na Figura 6.2 (linha horizontal vermelha).<br />

Quando a energia mecânica se iguala à energia potencial<br />

efetiva, temos as duas posições denotadas<br />

por r ± , denominadas de pontos de retorno. Nestas<br />

posições, a energia cinética T = E−U deve ser nula,<br />

o que implica ṙ = 0, ou seja, a trajetória não muda<br />

na direção radial. Como r − < r + , então quando o<br />

objeto atinge a distância mínima r − (periélio) da<br />

origem e começa a se afastar até atingir a distância<br />

máxima r + (afélio). Ao atingir a distância máxima,<br />

o objeto precisa se aproximar da origem novamente.<br />

Portanto, a trajetória gerada para aquele valor da<br />

energia mecânica deve ser uma curva fechada. Até<br />

o momento não podemos dizer exatamente qual<br />

curva é esta, apenas afirmar que é uma curva fechada.<br />

Seguindo o mesmo raciocínio, mostre que se<br />

a energia mecânica tiver o mesmo valor da energia<br />

potencial efetiva em seu ponto de mínimo, a curva<br />

será uma circunferência (calcule o valor do raio em<br />

função dos demais parâmetros constantes). Mostre<br />

também que a trajetória será uma curva aberta<br />

(contendo apenas um ponto de retorno) para uma<br />

energia mecânica positiva.<br />

A outra opção é usar (6.22) novamente para eliminar<br />

o parâmetro tempo em (6.24), pois de (6.22)<br />

temos dt = µr 2 dθ/L. Assim, (6.24) pode ser re-<br />

r<br />

88


Capítulo 6. Gravitação<br />

6.3. Trajetórias<br />

escrita como (Exercício 92)<br />

onde<br />

E =<br />

( ) 2 L2 dr<br />

2µr 4 + U(r), (6.25)<br />

dθ<br />

U(r) = L2 1<br />

2µ r 2 − k r , (6.26)<br />

é a nova energia potencial (ou a energia potencial<br />

efetiva). Esta forma é adequada para determinarmos<br />

a equação da trajetória diretamente, r = r(θ),<br />

sem a necessidade do parâmetro tempo. Lembre-se<br />

que a trajetória do nosso sistema é uma curva no<br />

plano XY cujos os pontos são os valores das coordenadas<br />

(r, θ). Em caso de dúvidas sobre trajetórias,<br />

faça uma revisão da Seção 2.5.<br />

Agora estamos em posição de determinarmos as<br />

famosas três leis de Kepler para o movimento planetário.<br />

Note que podemos fazer isto. Então vamos<br />

fazê-lo porque podemos. Primeiro a primeira lei, a<br />

qual diz respeito à forma da trajetória. Interessados<br />

na trajetória, então é melhor usarmos a energia<br />

mecânica na forma (6.25). Note que as variáveis r<br />

e θ podem ser isoladas em (6.25),<br />

dr<br />

dθ = √<br />

2µE<br />

L<br />

r 4 + 2µk<br />

2 L<br />

r 3 − r 2<br />

2<br />

dr<br />

= √<br />

. (6.27)<br />

r 2 2µE<br />

L<br />

+ 2µk<br />

2 L<br />

r −1 − r −2 2<br />

Esta integral do lado direito pode ser colocada<br />

numa forma canônica, isto é, numa forma em que<br />

ela possa ser encontrada em uma tabela de integrais,<br />

efetuando a transformação χ = 1/r. Com<br />

esta transformação, a expressão (6.27) pode ser colocada<br />

na seguinte forma (faça o Exercício 93):<br />

∫<br />

θ − θ 0 = −<br />

com<br />

dχ<br />

√<br />

α + β χ + γ χ<br />

2<br />

= − 1 √ −γ<br />

cos −1 (<br />

−√ β + 2γ χ<br />

β2 − 4αγ<br />

)<br />

, (6.28)<br />

α = 2µE<br />

L 2 , β = 2µk , γ = −1. (6.29)<br />

L2 Desta forma, invertendo o arco cosseno em (6.28) e<br />

fazendo algumas manipulações algébricas para eliminar<br />

as constantes α, β e γ, temos a equação da<br />

trajetória (faça o Exercício 94):<br />

com<br />

C = µk<br />

L 2 ,<br />

1<br />

r = C[ 1 + e cos(θ − θ 0 ) ] , (6.30)<br />

√<br />

e = 1 + 2 E ϵ , ϵ = µk2<br />

L 2 . (6.31)<br />

Esta é a equação de uma cônica com um dos focos<br />

na origem. Isto significa que o nosso vetor posição<br />

tem sua origem em um dos focos, ou seja, que o<br />

nosso sol está em um dos focos. Note que a constante<br />

ϵ em (6.30) tem dimensões de energia e serve<br />

como uma “unidade” de energia para o nosso sistema<br />

solar. A constante e é a excentricidade da<br />

seção cônica. Dependente do valor da excentricidade<br />

e, a seção cônica (6.30) pode ser classificada<br />

em quatro trajetórias:<br />

e > 1 ⇒ E > 0, hiperbóle,<br />

e = 1 ⇒ E = 0, parábola,<br />

e < 1 ⇒ E < 0, elipse,<br />

e = 0 ⇒ E = −ϵ, circunferência.<br />

(6.32)<br />

Note que a energia mecânica precisa ser negativa<br />

para haver trajetórias fechadas, pois neste caso haverá<br />

dois pontos de retorno para a coordenada r,<br />

conforme indicado na Figura 6.2. Estes pontos de<br />

retorno significam que a trajetória correspondente<br />

será fechada. Pontos de retorno diferentes (iguais)<br />

implicam em uma trajetória elíptica (circular). É<br />

comum nos referirmos a este sistema com energias<br />

negativas como um sistema ligado.<br />

A Figura 6.2 mostra a energia potencial efetiva<br />

U(r), definida em (6.24), e uma energia mecânica<br />

negativa. Nos pontos de retorno, devemos ter<br />

ṙ = 0. Portanto, substituindo ṙ = 0 em (6.24) e<br />

resolvendo a equação do segundo grau resultante<br />

em r, obtemos<br />

r ± = − k (1 ± e). (6.33)<br />

2E<br />

Estes valores correspondem aos pontos de retorno<br />

em uma elipse (E < 0): r − é a menor distância da<br />

Terra ao Sol (periélio) e r + é a maior distância da<br />

Terra ao Sol (afélio). O semi-eixo maior de uma<br />

elipse é<br />

a = r + + r −<br />

2<br />

= − k<br />

2E , (6.34)<br />

89


6.3. Trajetórias Capítulo 6. Gravitação<br />

ou<br />

r ± = a(1 ± e). (6.35)<br />

Note que, em t = 0, temos θ(0) = θ 0 . Substituindo<br />

esta informação na equação da trajetória (6.30),<br />

teremos r(0) = r 0 = 1/C(1 + e). Esta última<br />

condição inicial também pode ser escrita como<br />

r 0 = r − , ou seja, escolhemos o periélio como<br />

posição inicial (Exercício 95). Então Kepler<br />

tinha razão, as órbitas de nossos planetas são<br />

elipses. Geralmente, encontramos na literatura<br />

especializada (veja www.solarviews.com ou<br />

www.nssdc.gsfc.nasa.gov/planetary) informações<br />

contendo o valor da excentricidade e, do semi-eixo<br />

maior a e da massa m 2 de cada planeta do sistema<br />

solar. Conhecendo também a massa do Sol,<br />

m 1 = 1.989 × 10 30 kg, e a constante universal da<br />

gravitação, G = 6.6726 × 10 −11 Nm/kg 2 , então os<br />

valores da energia mecânica E e do momentum<br />

angular L podem ser determinados. É importante<br />

manter em mente que o semi-eixo maior da órbita<br />

da Terra é usado como unidade de distância<br />

astronômica, 1 UA=a = 1.496 × 10 11 m.<br />

A segunda lei de Kepler está contida na conservação<br />

do momentum angular (6.22). Para vermos<br />

isto, precisamos apenas de um pouquinho de<br />

geometria plana. Em um intervalo infinitesimal de<br />

tempo dt, a Terra varre uma área também infinitesimal<br />

dA, delimitada pelas posições r e r+dr. Esta<br />

área é aproximadamente a área de um triângulo<br />

retângulo, dA = r(rdθ)/2. Então a taxa de variação<br />

desta área no tempo é<br />

dA<br />

dt = 1 dθ<br />

r2<br />

2 dt = L 2µ . (6.36)<br />

Como L é constante, (6.22), então Kepler estava<br />

correto, os planetas em suas órbitas elípticas varrem<br />

áreas iguais em tempo iguais, ou seja, a taxa<br />

(6.36) é constante.<br />

Vimos em exemplos anteriores que a conservação<br />

da energia pode servir tanto para calcularmos<br />

equações horárias quanto para calcularmos<br />

períodos. A situação aqui não é diferente. Podemos<br />

usar a energia mecânica na forma (6.24) para<br />

determinarmos o período de órbitas elípticas. Isolando<br />

as variáveis r e t em (6.24) e integrando<br />

independentemente os dois lados, temos (faça o<br />

Exercício 95)<br />

√ µ<br />

t =<br />

2<br />

√ µ<br />

=<br />

2k<br />

∫ r<br />

r 0<br />

∫ r<br />

r 0<br />

dr<br />

√<br />

E + k r − L2<br />

2µ<br />

1<br />

r 2<br />

rdr<br />

√<br />

. (6.37)<br />

− 1<br />

2a r2 + r − 1 2 a(1 − e2 )<br />

Note que estamos usando t 0 = 0 e r(0) = r 0 = r − .<br />

Introduzindo uma nova variável ψ,<br />

r = a(1 − e cos ψ), (6.38)<br />

a integral (6.37) pode ser re-escrita na forma (faça<br />

o Exercício 96)<br />

√ ∫ µa<br />

3 ψ<br />

t =<br />

(1 − e cos ψ) dψ<br />

k 0<br />

√<br />

µa<br />

3<br />

= (ψ − e sin ψ). (6.39)<br />

k<br />

Obtivemos assim uma equação horária para a<br />

variável intermediária ψ,<br />

√<br />

k<br />

ω 0 t = (ψ − e sin ψ), ω 0 =<br />

µa 3 . (6.40)<br />

No entanto, ela é uma equação transcendental para<br />

ψ, ou seja, é impossível isolar (anliticamente) ψ em<br />

(6.40). Apesar disto, ela é útil para calcularmos o<br />

período. Podemos ver, comparando (6.35) e (6.38),<br />

que ψ = π quando estamos no afélio (r = r + ) e<br />

ψ = 0 quando estamos no periélio (r = r − ). O<br />

período τ é o tempo de uma volta completa, ou<br />

seja, devemos variar o ângulo ψ desde ψ = 0 até<br />

ψ = 2π. Assim, ω 0 τ = 2π, de onde podemos obter<br />

τ 2 = 4π2 µ 4π 2<br />

k a3 =<br />

G(m 1 + m 2 ) a3 . (6.41)<br />

Novamente Kepler tinha razão: o quadrado do<br />

período é realmente proporcional ao cubo do semieixo<br />

maior da órbita elíptica. No entanto, devemos<br />

observar que a constante de proporcionalidade<br />

depende também da massa de cada planeta. No<br />

nosso sistema solar, a massa do sol (m 1 ) é muito<br />

maior que a massa de quase todos os planetas (m 2 ).<br />

A única exceção é Júpiter, com uma massa de<br />

0.1 % da massa do Sol. Então é razoável usarmos<br />

m 1 + m 2 ≈ m 1 .<br />

90


Capítulo 6. Gravitação<br />

6.4 O campo gravitacional<br />

6.5 Correções<br />

6.6 Exercícios<br />

Exercício 85<br />

Aplique o operador gradiente (C.5) ao potencial<br />

gravitacional (6.18), considerando o potencial em<br />

coordenadas esféricas (polares), V = V (r), ou seja,<br />

você deve usar a segunda expressão em (6.18).<br />

Note que este potencial não depende das coordenadas<br />

angulares θ e ϕ. Repita este procedimento<br />

usando o operador gradiente (4.10) em coordenadas<br />

cartesianas. Neste caso você deve ver o potencial<br />

como uma função das coordenadas cartesianas,<br />

V = V (x, y, z), ou seja, você deve usar a última expressão<br />

em (6.18). Mostre que em ambos os casos<br />

a força gravitacional (6.15) pode ser obtida usando<br />

a prescrição (4.9).<br />

Exercício 86<br />

Observando a Figura 6.1, mostre que a equação da<br />

reta ⃗ X que passa por m 1 e m 2 pode ser na forma<br />

⃗X = ⃗r 1 + λ⃗r = ⃗r 2 + λ ′ ⃗r, ⃗r = ⃗r 2 − ⃗r 1 . (6.42)<br />

Re-escrevendo esta equação em coordenadas cartesianas,<br />

usando a primeira igualdade, por exemplo,<br />

podemos isolar a constante λ,<br />

λ = X − x 1<br />

x 2 − x 1<br />

= Y − y 1<br />

y 2 − y 1<br />

= Z − z 1<br />

z 2 − z 1<br />

. (6.43)<br />

Desta forma, dado as coordenadas do centro de<br />

massa (6.6), precisamos verificar que ⃗ R = ⃗ X satisfaz<br />

as condições em (6.43). Verifique isto substituindo<br />

⃗ X = ⃗ R, com as componentes do vetor centro<br />

de massa escritas na forma (6.6), que as razões<br />

(6.43) são de fato constantes com λ = m 2 /M, confirmando<br />

assim que o centro de massa de um sistema<br />

de dois corpos sempre está na reta que passa<br />

pelos dois corpos.<br />

Exercício 87<br />

Substitua o vetor posição ⃗r i por ⃗r i = R ⃗ + ⃗u i na<br />

definição do centro de massa (6.3) e mostre que o<br />

resultado (6.5) está correto. As novas coordenadas<br />

⃗u i são denominadas de coordenadas relativas<br />

ao centro de massa. Elas estão representadas na<br />

Figura 6.1.<br />

6.6. Exercícios<br />

Exercício 88<br />

Mostre que, para N = 2 (dois corpos), usando a<br />

relação ⃗r = ⃗u 2 − ⃗u 1 mostrada na Figura 6.1 e (6.5),<br />

os resultados (6.13) podem ser obtidos. Substitua<br />

os resultados (6.13) em (6.12) e obtenha, a menos<br />

da energia cinética do centro de massa, a energia<br />

cinética (6.14) escrita em termos do vetor posição<br />

⃗r.<br />

Exercício 89<br />

Substituindo as relações (6.13) na energia cinética<br />

(6.12), obtenha os resultados exibidos em (6.14).<br />

Exercício 90<br />

Derive o momentum angular ⃗ L = ⃗r ×⃗p, com ⃗p = µ ˙⃗r<br />

sendo o momentum linear da massa reduzida µ,<br />

em relação ao tempo e mostre que esta derivada é<br />

uma constante. Use o fato da força (gravitacional)<br />

agindo na massa reduzida ser a taxa de variação<br />

temporal do momentum linear e que esta força está<br />

na mesma direção do vetor ⃗r. Use também o fato<br />

do produto vetorial (×) entre vetores paralelos ou<br />

anti-paralelos ser nulo.<br />

Exercício 91<br />

Mostre que, fazendo ϕ = π/2, as coordenadas<br />

esféricas (C.1) do vetor posição ⃗r = x î+yˆȷ se reduzem<br />

a coordenads polares x = r cos θ e y = r sin θ.<br />

Agora tanto o raio r e o ângulo θ dependem do<br />

tempo. Mostre que o vetor velocidade ⃗v = ˙⃗r e seu<br />

módulo são<br />

⃗v = (ṙ cos θ − r ˙θ sin θ) î<br />

+ (ṙ sin θ + r ˙θ cos θ)ˆȷ, (6.44)<br />

v 2 = || ˙⃗r|| 2 = ṙ 2 + r 2 ˙θ2 . (6.45)<br />

Mostre também que o módulo do momentum angular<br />

⃗ L = ⃗r × ⃗p, com ⃗p = µ ˙⃗r, é<br />

L = µr 2 ˙θ. (6.46)<br />

Exercício 92<br />

Substitua a velocidade escalar || ˙⃗r|| 2 encontrada em<br />

(6.45) na expressão da energia mecânica (6.19) para<br />

obter (6.24). Agora isole ˙θ em (6.46) e mostre,<br />

multiplicando tudo pelo diferencial dt, que<br />

dt = µ L r2 dθ. (6.47)<br />

Substitua o diferencial dt em (6.24) pela expressão<br />

(6.47) e encontre (6.25).<br />

91


6.6. Exercícios Capítulo 6. Gravitação<br />

Exercício 93<br />

Aplique a transformação de coordenadas χ = 1/r<br />

na expressão (6.27). Primeiro mostre que<br />

χ = 1 r<br />

⇒<br />

dχ = − dr<br />

r 2 . (6.48)<br />

Depois mostre que a integral da expressão (6.27) na<br />

nova variável χ pode ser escrita na forma canônica<br />

(6.28) com o auxílio das constantes (6.29).<br />

Exercício 94<br />

Troque χ por 1/r na solução (6.28) para obtermos<br />

a equação da trajetória (6.30). Use as constantes<br />

(6.29) para efetuar as devidas simplificações e verifique<br />

que os valores da constante C em (6.30),<br />

bem como da excentricidade e e da unidade “astronômica”<br />

de energia ϵ estão corretos.<br />

Exercício 97<br />

Use os valores reais de G (constante universal da<br />

gravitação Newtoniana) m 1 (massa do Sol), m 2<br />

(massa do planeta), e (excentricidade) e a (semieixo<br />

maior) para calcular em unidades MKS a energia<br />

mecânica (E), o módulo do momentum angular<br />

(L) e o período (τ) de cada planeta no nosso sistema<br />

solar. Observe a ordem de grandeza destas quantidades.<br />

Desenhe também suas órbitas e potenciais.<br />

Quantidades astronômicas pode ser obtidas no portal<br />

www.solarviews.com.<br />

Exercício 95<br />

Mostre, usando (6.34) e (6.30), que a energia<br />

mecânica E e o momentum angular L estão relacionados<br />

com os parâmetros de uma elipse através<br />

das expressões<br />

onde<br />

a = − k<br />

2E ⇒ E k = − 1<br />

2a ;<br />

√<br />

e = 1 + 2 EL2<br />

µk 2 ⇒ L2<br />

µk = 1 (6.49)<br />

C ,<br />

1<br />

C = a(1 − e2 ) = a(1 + e)(1 − e) = r +r −<br />

a . (6.50)<br />

Agora substitua estas relações na primeira integral<br />

em (6.37) para obter a segunda. Aproveite para<br />

mostrar também que, usando (6.30), temos r = r −<br />

em θ = 0 e r = r + em θ = π.<br />

Exercício 96<br />

Mostre que r = a(1 − e cos ψ) implica em (i) dr =<br />

ae sin ψ dψ, (ii) r 0 = a(1 − e cos 0) = a(1 − e) = r −<br />

e (iii)<br />

− 1<br />

2a r2 + r − 1 2 a(1 − e2 ) = 1 2 ae2 sin 2 ψ. (6.51)<br />

Use este resultado em (6.37) para chegar na integral<br />

(6.39). Aproveite para mostrar também que,<br />

usando r = a(1 − e cos ψ), temos r = r − em ψ = 0<br />

e r = r + em ψ = π.<br />

92


Capítulo 7<br />

Relatividade restrista<br />

7.1 Transformações de Galileu<br />

Os dois princípios estabelecidos anteriormente ou,<br />

equivalentemente, as três leis de Newton, são<br />

válidas em qualquer sistema inercial de referência.<br />

Vamos considerar então o seguinte problema de interesse<br />

prático. Suponha dois referenciais inerciais<br />

quaisquer, digamos O e Ō. Considere o referencial<br />

Ō em movimento (uniforme) em relação a O. Para<br />

simplificar um pouco, vamos supor que os eixos espaciais<br />

X, Y e Z do referencial O coincidam com<br />

os eixos espaciais ¯X, Ȳ e ¯Z do referencial Ō em<br />

t = ¯t = 0. Vamos supor também que suas origens<br />

coincidam em t = ¯t = 0. Vamos também considerar<br />

um movimento para o referencial Ō ao longo<br />

do eixo X, mantendo o eixo ¯X sempre paralelo ao<br />

eixo X, com velocidade constante V ⃗ = V î. Veja a<br />

Figura 7.1.<br />

Y<br />

V ¯t<br />

Ō<br />

Ȳ<br />

O<br />

X<br />

x<br />

Figura 7.1: Transformações de coordenadas entre<br />

referenciais inerciais que preservam a definição de<br />

força.<br />

Muito bem.<br />

¯x<br />

V<br />

¯X<br />

Imagine agora que algum experimento<br />

mecânico seja realizado (no vácuo) no referencial<br />

em movimento Ō. Este experimento pode<br />

ser (1) o lançamento de um objeto sob a ação da<br />

gravidade ou (2) o movimento de um corpo preso a<br />

uma mola (oscilador harmônico sem atrito) ou (3)<br />

o movimento de um corpo sob a ação da gravidade,<br />

porém preso a uma extremidade de uma haste inextensível<br />

e de massa nula com a outra extremidade<br />

fixa (pêndulo simples sem atrito), para citar apenas<br />

três possibilidades que iremos discutir em detalhes.<br />

Como os resultados desses experimentos em<br />

Ō serão vistos no referencial em repouso O? Todos<br />

os números serão os mesmos? As trajetórias serão<br />

as mesmas? Eventos simultâneos em Ō serão vistos<br />

também simultaneamente no outro referencial O?<br />

Esta última questão tem um papel fundamental nas<br />

discussões seguintes.<br />

Para respondermos estas questões, precisaremos<br />

saber primeiramente como relacionar coordenadas<br />

nestes dois referenciais. Esta relação deve cumprir<br />

uma exigência fundamental: ela deve preservar a<br />

definição de força, ou seja, o produto massa por aceleração<br />

deve ser o mesmo ou proporcionais nos dois<br />

referenciais, pois estes dois referenciais são inerciais.<br />

Isto é conhecido como o princípio da relatividade<br />

de Galileu-Newton:<br />

Princípio 3<br />

Todas as leis da mecânica são as mesmas em todos<br />

os sistemas inerciais de referência.<br />

Note que Galileu e Newton não ousaram incluir<br />

as demais leis físicas, além daquelas da mecânica,<br />

neste princípio. Este princípio garante a validade<br />

apenas das leis da mecânica. Portanto, se queremos<br />

aderir ao princípio da relatividade de Galileu-<br />

Newton, a única possibilidade de efetuarmos uma<br />

93


7.2. Transformações de Lorentz Capítulo 7. Relatividade restrista<br />

transformação de coordenadas que preserve a definição<br />

de força (portanto as leis da mecânica) é<br />

uma relação linear no tempo,<br />

x = γ(¯x + V ¯t), (7.1)<br />

quando o referencial Ō estiver movimentando-se no<br />

sentido positivo do eixo X (veja a Figura 7.1), ou<br />

¯x = γ(x − V t), (7.2)<br />

quando o referencial O estiver movimentando-se no<br />

sentido negativo do eixo ¯X. Note que mantivemos<br />

a mesma constante γ nestas duas relações. Isto<br />

é razoável, pois estamos considerando que o nosso<br />

espaço (o palco de todos os movimentos) vazio seja<br />

homogêneo e isotrópico. Ser homogêneo significa<br />

que a constante γ não pode depender da posição.<br />

Em um espaço isotrópico, a constante γ não pode<br />

depender nem da direção nem do sentido de qualquer<br />

vetor. Conseqüentemente, se γ tiver qualquer<br />

dependência com a velocidade V ⃗ do referencial Ō,<br />

poderá ser apenas uma dependência em seu módulo<br />

V = || V ⃗ ||. Até aqui, não mencionamos qualquer<br />

relação entre t e ¯t, a não ser que x = ¯x em t = ¯t = 0.<br />

Quem são os possíveis valores de γ?<br />

Galileu e Newton fizeram a hipótese de que o<br />

tempo fluía uniformemente nos dois referenciais, ou<br />

seja, t = ¯t sempre. Isto significa que o tempo é absoluto,<br />

o mesmo em todos os referenciais inerciais,<br />

ou seja, um evento que é simultâneo em um referencial<br />

inercial, o será em todos os demais. Newton<br />

acreditava também que a existência de um tempo<br />

absoluto estivesse ligado com a existência de um<br />

Deus supremo. Fazendo t = ¯t em (7.1) e (7.2) e<br />

depois somando estes dois resultados, obteremos<br />

x + ¯x = γ(x + ¯x) ⇒ γ = 1. (7.3)<br />

Também fazendo x = ¯x em t = ¯t = 0, determinamos<br />

γ = 1. Note que derivando (7.1) ou (7.2) com<br />

γ = 1 obtemos a conhecida regra de composição<br />

para velocidades,<br />

As transformações<br />

¯v = v ± V. (7.4)<br />

¯x = x − V t, ȳ = y, ¯z = z, ¯t = t, (7.5)<br />

são conhecidas por transformações de Galileu. Estas<br />

transformações não afetam as massas dos corpos.<br />

Conseqüentemente, o produto massa por aceleração<br />

é mantido invariante. Naturalmente, estas<br />

transformações têm sido verificadas em todos os<br />

casos (discutiremos alguns casos em detalhes mais<br />

mais adiante), exceto quando o valor de V é comparável<br />

ao valor da velocidade da luz. Surpreso?<br />

Isto é realmente surpreendente!<br />

7.2 Transformações de Lorentz<br />

A discrepância entre as transformações de Galileu<br />

(7.5) e experimentos foi consagrada no experimento<br />

de Michelson-Morley, realizado pela primeira vez<br />

um pouco antes de 1900. Vejamos primeiro o que a<br />

teoria de Galileu-Newton prediz quando luz é emitida<br />

na origem do referencial O e observada no referencial<br />

em movimento Ō. No instante t, a luz<br />

está na posição x = ct em O. Então, no referencial<br />

Ō, ela estará em ¯x¯t = x − V t = ct − V t = ¯c¯t,<br />

segundo as transformações (7.5). Portanto, no referencial<br />

Ō a luz deve ser vista com uma velocidade<br />

menor, ¯c = (c − V ). Entretanto, o experimento de<br />

Michelson-Morley diz que a velocidade da luz é a<br />

igual a c em todos os referenciais inerciais! Pronto,<br />

a confusão estava feita. Albert A. Michelson, por<br />

achar que havia alguma coisa errada em seu experimento,<br />

continuou a repeti-lo por quase 30 anos! Ele<br />

recebeu o prêmio Nobel em 1907 (por estas medidas<br />

e por ter inventando o interferômetro, um instrumento<br />

ótico de alta precisão).<br />

No entanto, ainda em 1905, um jovem físico, até<br />

então completamente desconhecido, disse de forma<br />

arrebatedora que a nossa natureza é assim: a velocidade<br />

da luz no vácuo é independente do movimento<br />

da fonte, ou seja, ela é uma constante universal.<br />

Simples assim. E concluiu: então não poderemos<br />

ter γ 2 = 1 e nem ¯t = t! Portanto, devemos ter<br />

x = ct em O e ¯x = c¯t em Ō. Substituindo estes valores<br />

em (7.1) e (7.2), teremos (faça o Exercício 100)<br />

ct = γ(c + V )¯t, c¯t = γ(c − V )t, (7.6)<br />

da qual resulta<br />

γ 2 = 1<br />

1 − β 2 , β = V c . (7.7)<br />

Podemos ver então que no limite de baixa velocidade<br />

V ≪ c, β → 0 e γ 2 → 1, que é o seu valor<br />

no caso clássico (7.3) se escolhermos γ positivo em<br />

(7.7). No entanto, quando V é comparável a c,<br />

temos de usar as relações (7.1) e (7.2). Relações<br />

94


Capítulo 7. Relatividade restrista<br />

7.3. Mecânica relativística<br />

similares para o tempo visto nos dois referenciais<br />

podem ser obtidas eliminando-se ou x ou ¯x em (7.1)<br />

e (7.2) (faça o Exercício 101),<br />

¯x = γ(x − β ct), c¯t = γ(ct − β x), (7.8)<br />

x = γ(¯x + β c¯t), ct = γ(c¯t + β ¯x). (7.9)<br />

com ȳ = y e ¯z = z. Estas transformações foram<br />

apresentadas por Einstein em 1905 e são conhecidas<br />

por transformações de Lorentz (por motivos<br />

relacionados, mas anteriores a 1905). Em (7.8), expressamos<br />

as coordenadas do referencial Ō em termos<br />

das coordenadas do referencial O. As transformações<br />

inversas (7.9) são obtidas simplesmente<br />

mudando o sinal de V (e, conseqüentemente, de<br />

β = V/c também).<br />

Como as relações (7.8)–(7.9) misturam as<br />

posições espaciais com o tempo (e vice-versa), o<br />

tempo não é absoluto, como Galileu e Newton imaginaram.<br />

Isto significa que um acontecimento não<br />

precisa ser simultâneo em todos os referenciais inerciais.<br />

Isto trará conseqüências ainda mais surpreendentes,<br />

como veremos a seguir.<br />

Naturalmente, as transformações (7.8) são<br />

idênticas às transformações (7.5) quando V ≪ c<br />

(ou β ≪ 1). Este é outro exemplo onde uma teoria<br />

foi devidamente corrigida: para velocidades baixas<br />

podemos usar a mecânica Newtoniana; para velocidades<br />

altas devemos usar a mecânica Einsteiniana<br />

(ou mecânica relativística).<br />

7.3 Mecânica relativística<br />

O princípio da relatividade de Galileu-Newton<br />

agora deve ser enunciado na seguinte forma:<br />

Princípio 4 (Relatividade de Einstein)<br />

1. Todas as leis físicas são as mesmas em todos os<br />

sistemas inerciais de referência;<br />

2. A velocidade da luz no vácuo tem o mesmo valor<br />

em todos os sistemas inerciais, independentemente<br />

do movimento de sua fonte.<br />

Este princípio é conhecido como o princípio da relatividade<br />

de Einstein. Note que ele é mais geral<br />

que o princípio da relatividade de Galileu-Newton<br />

(Princípio 3), pois agora todas as leis físicas foram<br />

devidamente incluídas.<br />

Vejamos algumas previsões da mecânica relativística.<br />

Suponha um relógio em repouso no referencial<br />

Ō. Permanecendo sempre no mesmo lugar<br />

(¯x 1 = ¯x 2 ), marque um determinado intervalo<br />

de tempo, ¯T = ¯t 2 − ¯t 1 , usando um relógio que esteja<br />

em repouso no referencial Ō. Este intervalo de<br />

tempo medido com o relógio em repouso será denominado<br />

de tempo próprio. No outro referencial O,<br />

este relógio usado para medir o tempo próprio será<br />

visto em movimento. Portanto os dois eventos que<br />

determinaram o intervalo de tempo ¯T serão vistos<br />

nos instantes t 1 e t 2 (em posições diferentes), correspondendo<br />

a um intervalo T = t 2 − t 1 . Usando<br />

as transformações (7.9) em T , encontraremos (faça<br />

o Exercício 102)<br />

T = γ ¯T . (7.10)<br />

Esta relação nos mostra que T > ¯T , ou seja, que o<br />

relógio em movimento em relação ao referencial O,<br />

é mais lento, pois o intervalo de tempo T medido<br />

é maior. Naturalmente, os observadores solidários<br />

ao referencial Ō chegarão à mesma conclusão a respeito<br />

de um relógio usado para medir um tempo<br />

próprio em O. Há nenhuma contradição nisto. O<br />

tempo próprio será sempre o menor intervalo tempo<br />

e pode somente ser medido em apenas um referencial<br />

inercial. Todos os demais referenciais inercias<br />

distintos irão medir intervalos de tempo maiores,<br />

porém haverá nenhuma concordância de valores entre<br />

eles.<br />

Suponha que temos uma régua em repouso no<br />

referencial Ō de comprimento ¯L = ¯x 2 − ¯x 1 . Este é<br />

o comprimento próprio (régua em repouso). Uma<br />

vez que a régua está em repouso, as posições ¯x 1 e<br />

¯x 2 das extremidades podem ser efetuadas em tempos<br />

diferentes. No outro referencial O, esta régua<br />

será vista em movimento, por isto a leitura de suas<br />

extremidades deverão ser efetuadas no mesmo instante<br />

de tempo (t 1 = t 2 ). Assim, o comprimento da<br />

régua medido em O será L = x 2 (t)−x 1 (t). Usando<br />

as transformações (7.8) em ¯L, encontraremos (faça<br />

o Exercício 102)<br />

L = ¯L<br />

γ . (7.11)<br />

Esta relação nos mostra que L < ¯L, pois γ > 1.<br />

Portanto, a régua em Ō será vista com um comprimento<br />

menor no outro referencial O. Este resultado<br />

é conhecido por contração espacial. Naturalmente,<br />

os observadores solidários ao referencial<br />

Ō chegarão à mesma conclusão a respeito de uma<br />

95


7.3. Mecânica relativística Capítulo 7. Relatividade restrista<br />

régua usada para medir um comprimento próprio<br />

em O. Novamente, há nenhuma contradição nisto.<br />

O comprimento próprio será sempre o maior comprimento<br />

próprio e pode somente ser medido em<br />

apenas um referencial inercial. Todos os demais referenciais<br />

inercias distintos irão medir comprimentos<br />

menores, porém haverá nenhuma concordância<br />

de valores entre eles.<br />

Naturalmente, estas previsões estão confirmadas<br />

em experimentos usando partículas elementares<br />

em aceleradores de partículas (para saber mais sobre<br />

partículas elementares, consulte Aventuras das<br />

Partículas, mantido pelo Instituto de Física Teórica<br />

(IFT), Unesp). Em particular, visite o site Experimento<br />

com Múons.<br />

Que acontece se um determinado corpo estiver<br />

movendo-se na velocidade da luz c (como a própria<br />

luz), digamos em O? A velocidade dele será diferente<br />

de c em Ō? Vejamos. A componente x<br />

da velocidade deste corpo em O é definida como<br />

v x = dx/dt. Então, usando as transformações<br />

(7.8)–(7.9), teremos<br />

v x = dx<br />

dt<br />

=<br />

d¯x + β cd¯t<br />

d¯t + β c d¯x = ¯v¯x + V<br />

1 + V ¯v¯x<br />

c 2 . (7.12)<br />

Fazendo o mesmo para as demais componentes, teremos<br />

v y = dy<br />

dt =<br />

v z = dz<br />

dt =<br />

¯vȳ<br />

( ), (7.13)<br />

γ 1 + V ¯v¯x<br />

c 2<br />

¯v¯z<br />

( ). (7.14)<br />

γ 1 + V ¯v¯x<br />

c 2<br />

Note que v x = c implica em ¯v¯x = c. A regra<br />

relativística de composição de velocidades (7.12)–<br />

(7.14) nunca fornece uma velocidade relativa maior<br />

que a velocidade da luz.<br />

Ȳ<br />

O<br />

Y<br />

c<br />

Ō<br />

¯X<br />

V<br />

Figura 7.2: Colisão entre um corpo A e dois fótons<br />

vista por dois referenciais inerciais em movimento<br />

relativo com V ≪ c. O corpo A está em repouso<br />

no referencial O e tem massa M neste referencial.<br />

Há ainda outras revelações extraordinárias, mas<br />

por falta de espaço, ficarão para uma outra oportunidade.<br />

Entretanto, deve ser mencionado que Einstein<br />

também descobriu uma relação entre massa e<br />

energia, ∆E = ∆mc 2 . Tudo que tem massa, possui<br />

esta energia armazenada. Até a descoberta desta<br />

relação, acreditava-se na conservação da energia e<br />

da massa separadamente. Hoje sabemos que massa<br />

e energia são manifestações de uma mesma quantidade<br />

física (ainda sem nome!). Seguindo o próprio<br />

Einstein, é instrutivo realizarmos uma derivação<br />

elementar desta equivalência entre massa e energia.<br />

Consideremos um sistema como ilustrado na<br />

Figura 7.2. O corpo A está em repouso no referencial<br />

O e tem massa M. Neste mesmo referencial,<br />

observamos dois fótons (luz) movendo-se na mesma<br />

direção mas em sentidos opostos, os quais serão absorvidos<br />

pelo corpo A. Supondo que cada fóton tenha<br />

uma energia igual a E/2, o corpo A terá sua<br />

energia aumentada por ∆E = E.<br />

O fóton é uma partícula curiosa, pois ele não<br />

tem carga elétrica e nem massa de repouso, isto é,<br />

um fóton parado em algum referencial inercial teria<br />

massa nula. No entanto ele só pode estar em movimento!<br />

Mas como, se ele não tem massa? Mesmo<br />

não tendo massa, o fóton pode ter então uma quantidade<br />

de movimento não-nula! O momentum linear<br />

do fóton é a sua energia dividida pela velocidade<br />

da luz (no vácuo). Devemos lembrar que o<br />

fóton carrega a menor quantidade de energia em<br />

um feixe luminoso. Esta energia depende apenas<br />

da freqüência ν da luz e da constante de Planck<br />

A<br />

c<br />

X<br />

96


Capítulo 7. Relatividade restrista<br />

7.4. Exercícios<br />

Assim, antes da colisão, o momentum linear total<br />

h = 6.626 × 10 −34 J s, E = hν. Por isso ele é<br />

sin(α) = V Mostre que as transformações de Galileu (7.5) deixa<br />

c ≃ α. (7.15) a definição de força invariante.<br />

denominado de quantum de luz (ou da radiação<br />

eletromagnética). O fóton foi descoberto por Max<br />

Planck em 1900 (Planck recebeu o prêmio Nobel em<br />

1918 por esta descoberta). Coube a Einstein esclarecer<br />

a natureza corpuscular (composta de muitos<br />

no eixo Ȳ é<br />

( )<br />

P (a)<br />

E<br />

ȳ = MV + 2<br />

2c sin(α) = MV + E V. (7.16)<br />

c2 Na verdade, tanto a massa M do corpo A quanto<br />

fótons) da luz em 1905 (ele recebeu o prêmio Nobel<br />

em 1921, por esta e outras contribuições à física Ō deveriam sofrer correções relativísticas. No en-<br />

o momentum linear do fóton E/2c no referencial<br />

teórica).<br />

tanto, estas correções são muito pequenas se β =<br />

V/c é pequeno, que é o caso em questão (veja o<br />

Exercício 105). Bem e depois da absorção? Vimos<br />

Ȳ<br />

no referencial em repouso O que o corpo A permanece<br />

V<br />

em repouso após a colisão dos dois fótons.<br />

Portanto ele terá de continuar com a velocidade V<br />

c<br />

c<br />

α α<br />

ao longo do eixo Ȳ , como indicado na Figura 7.3,<br />

mesmo após a colisão (explique). Após a colisão<br />

A<br />

os dois fótons não existem mais, devido à absorção.<br />

Então, para não concluirmos que os fótons não existiam<br />

antes da colisão devido à conservação do momentum<br />

linear, ou então para não concluirmos que<br />

Ō<br />

¯X<br />

Figura 7.3: Colisão entre um corpo A e dois fótons a conservação do momentum linear está errada, temos<br />

de supor uma massa M ′ ≠ M para o corpo<br />

vista pelo referencial inercial em movimento Ō com<br />

V ≪ c.<br />

A após a colisão. Assim, após a colisão o momentum<br />

linear total no eixo<br />

De volta ao nosso experimento: uma colisão entre<br />

três corpos, onde um dos corpos (o corpo A) existem forças externas, então<br />

Ȳ é P (d)<br />

ȳ = M ′ V . Como<br />

o momentum linear deve ser conservado, pois não<br />

absorve dois fótons. Estes três corpos estão isolados.<br />

Portanto, de acordo com o Princípio 2 há<br />

conservação do momentum linear. Como o momentum<br />

MV + E c 2 V = M ′ V, (7.17)<br />

linear é um vetor, devemos olhar para as de onde obtemos<br />

três direções em cada referencial. No referencial<br />

O onde o corpo A está em repouso, a componente ∆M = M ′ − M = E<br />

no eixo X do momentum linear total é nula, pois<br />

c 2 = ∆E<br />

c 2 . (7.18)<br />

cada fóton tem p x = E/2c, mas em sentidos opostos<br />

e o corpo A está em repouso. Depois da colisão,<br />

eles são completamente absorvidos e, por simetria,<br />

o corpo A continua em repouso. Portanto há conservação<br />

Este resultado está nos dizendo que a energia ∆E,<br />

absorvida ou liberada por um corpo, é equivalente<br />

a uma variação ∆m na sua massa de repouso m,<br />

∆E = ∆m c 2 . Veja o Exercício 107 para um exem-<br />

da componente X do momentum linear. plo numérico. Esta relação entre massa e ener-<br />

Como não há movimento nas demais direções (Y e<br />

Z), não precisamos nos preocupar como as respectivas<br />

componentes do momentum linear.<br />

Passemos agora para o outro referencial inercial<br />

em movimento, Ō. Este referencial está em movimento<br />

uniforme ao longo eixo Y , como indicado na<br />

gia explica porque o fóton tem uma quantidade de<br />

movimento, mesmo não tendo uma massa de repouso<br />

(como a nossa): ele tem energia. Não poder<br />

(nunca) estar parado, é o preço que ele paga por<br />

não ter uma massa de repouso, ele precisa estar<br />

sempre em movimento.<br />

Figura 7.2. Neste referencial em movimento, nosso<br />

experimento é visto como mostrado na Figura 7.3.<br />

Considerando que o fator β = V/c é pequeno, podemos<br />

7.4 Exercícios<br />

aproximar o ângulo α pelo seu seno, Exercício<br />

98<br />

97


7.4. Exercícios Capítulo 7. Relatividade restrista<br />

Exercício 99<br />

Suponha que no referencial em movimento Ō haja<br />

um corpo preso a uma mola. Use as transformações<br />

de Galileu (7.5) para encontrar a equação diferencial<br />

equivalente a (3.15) no referencial O.<br />

Exercício 100<br />

Mostre que as relações (7.6) estão corretas e determine<br />

explicitamente o valor de γ.<br />

Exercício 101<br />

Determine explicitamente a última relação em (7.8)<br />

e em (7.9) a partir das relações (7.1) e (7.2).<br />

Exercício 102<br />

Determine explicitamente as relações (7.11) e<br />

(7.10).<br />

átomo radioativo. Pela descoberta de átomos radioativos,<br />

ela e seu marido (Pierre) receberam o<br />

prêmio Nobel em 1903. Vale mencionar que ela recebeu<br />

um segundo prêmio Nobel em 1911 pela descoberta<br />

do polônio. O processo de desintegração do<br />

polônio é<br />

216 Po → 212 Pb + 4 He. (7.20)<br />

Sabendo que a massa atômica do chumbo 212 Pb é<br />

211.991872 u.m.a. e a do hélio é 4.002602 u.m.a.<br />

e que uma unidade de massa atômica (u.m.a) vale<br />

1.66×10 −27 kg, calcule a energia liberada neste processo<br />

radioativo. Esta energia é usada na forma de<br />

energia cinética pelos produtos da reação nuclear.<br />

Exercício 103<br />

Determine explicitamente os resultados (7.12),<br />

(7.13) e (7.14).<br />

Exercício 104<br />

Quais são os valores relativos das contrações espacial<br />

(¯L/L) e temporal ( ¯T /T ) quando β = 0.3,<br />

β = 0.6, β = 0.9 e β = 0.99?<br />

Exercício 105<br />

Mostre, usando a série de Taylor (B.2), que a<br />

correção relativística (7.7) pode ser escrita na forma<br />

de uma série de potências quando β ≪ 1,<br />

γ ≃ 1 + 1 2 β2 + 3 8 β4 . (7.19)<br />

Exercício 106<br />

Considere a partícula denominada de múon. Ela<br />

tem a mesma carga de um elétron, mas uma massa<br />

cerca de 200 vezes maior. Ela é instável: após o<br />

tempo ¯L = 2.2 µs (medido no referencial do múon<br />

(lento) em laboratórios) ela se transforma (decai)<br />

em outras partículas. Quando o múon vem de raios<br />

cósmicos, ele penetra nossa atmosfera com uma velocidade<br />

V = 0.99c e chega até o solo em grandes<br />

quantidades. Explique como o múon chega até a<br />

superfície (poucos kilômetros abaixo do início da<br />

nossa atmosfera).<br />

Exercício 107<br />

O isótopo 216 Po do átomo de polônio, número<br />

atômico Z = 84, massa atômica 216.001889 u.m.a.,<br />

foi descoberto por Marie Curie em 1893, como um<br />

98


Apêndice A<br />

Análise dimensional<br />

Uma pequena pausa para uma discussão sobre<br />

unidades. Unidades são importantes. Por exemplo,<br />

qual é a unidade de força que iremos utilizar?<br />

De forma geral, procuraremos expressar todas<br />

as nossas quantidades físicas em unidades derivadas<br />

de quatro grandezas fundamentais: comprimento<br />

(L) em metros (m), tempo (T) em segundos<br />

(s), massa (M) em kilogramas (kg) e carga elétrica<br />

(Q) em Coulombs (C). Existe uma notação especial<br />

para analisarmos as dimensões de uma determinada<br />

grandeza física. Vejamos alguns exemplos.<br />

O vetor posição ⃗r tem dimensão de comprimento<br />

(L). Então escrevemos matematicamente esta informação<br />

como<br />

[⃗r] = L.<br />

(A.1)<br />

O vetor velocidade tem dimensões de comprimento<br />

por tempo, então<br />

[ ] d⃗r<br />

[⃗v] = = L dt T = LT−1 . (A.2)<br />

O vetor aceleração tem dimensões de comprimento<br />

por tempo ao quadrado, então<br />

[ d 2 ]<br />

⃗r<br />

[⃗a] =<br />

dt 2 = L T 2 = LT−2 . (A.3)<br />

Seguindo estes modelos, a análise dimensional da<br />

definição do vetor força na segunda lei de Newton<br />

nos fornece<br />

[ ⃗ F ] = [ m⃗a ] = ML<br />

T 2 = MLT−2 = N, (A.4)<br />

na qual N é uma unidade de força, denominada<br />

de Newton. Também da segunda lei de Newton, o<br />

momentum linear tem dimensões<br />

[⃗p] = [ m⃗v ] = ML<br />

T = MLT−1 . (A.5)<br />

Neste caso, não há um nome para a unidade de<br />

momentum.<br />

Quais são as dimensões da constante C e aparecendo<br />

na expressão para a força elétrica<br />

⃗F e = C e<br />

Qq<br />

r 2 ˆr<br />

(A.6)<br />

entre duas cargas elétricas Q e q? Seguindo o modelo<br />

anterior, temos<br />

[C e ] = NL 2 Q −2 = ML 3 T −2 Q −2 . (A.7)<br />

Por completeza, devemos mencionar que cargas<br />

magnéticas nunca foram observadas. No entanto<br />

quando dois fios conduzindo correntes elétricas I 1<br />

e I 2 estão a uma distância ρ, podemos medir uma<br />

força entre eles,<br />

⃗F m = 2C m<br />

I 1 I 2<br />

ρ<br />

ˆρ. (A.8)<br />

Esta força é conhecida como lei de Biot-Savat. As<br />

dimensões da constante C m são<br />

onde<br />

[C m ] = NT 2 Q −2 = MLQ −2 , (A.9)<br />

[I] =<br />

[ ] dQ<br />

= QT −1 . (A.10)<br />

dt<br />

Note que usamos também a definição de corrente:<br />

carga por tempo. Podemos notar também então<br />

que a razão C e /C m tem a mesma dimensão de velocidade<br />

ao quadrado. De fato, Maxwell mostrou<br />

que no vácuo, a velocidade da luz é<br />

√<br />

Ce<br />

c = .<br />

(A.11)<br />

C m<br />

Os valores destas constantes (no vácuo) são: C e =<br />

8.987551788 × 10 9 N·m 2 /C 2 e C m = 10 −7 N·s 2 /C 2 .<br />

99


Capítulo A. Análise dimensional<br />

Portanto, medindo as constantes C e e C m podemos<br />

calcular a velocidade da luz. Este resultado está<br />

entre os mais surpreendentes acerca da nossa natureza.<br />

As surpresas não param aqui, há ainda mais<br />

alguns fatos marcantes sobre o comportamento da<br />

luz. Veja também a discussão na seção sobre as<br />

transformações de Galileu e Lorentz.<br />

Para completar, as dimensões de campo<br />

magnético têm um nome especial: Tesla. Vamos<br />

usar a força de Lorentz,<br />

⃗F = q ⃗v × ⃗ B,<br />

(A.12)<br />

produzida por uma carga q em movimento, para<br />

determinar as dimensões do campo magnético ⃗ B,<br />

[ ⃗F<br />

]<br />

[ B] ⃗ = = MQ −1 T −1 = Tesla. (A.13)<br />

q⃗v<br />

100


Apêndice B<br />

Séries de Taylor<br />

Como calculamos (geralmente usando uma calculadora)<br />

senos, cossenos, exponenciais, funções<br />

transcendentais em geral? Ou então, suponha que<br />

conhecemos o valor de uma função f(x) e de suas<br />

derivadas em um determinado ponto x 0 ,<br />

f (k)<br />

0 = f (k) (x 0 ) = d(k)<br />

dx k f(x) ∣<br />

∣∣∣x=x0<br />

,<br />

(B.1)<br />

com f (0)<br />

0 = f 0 = f(x 0 ). Suponha também que seja<br />

muito difícil calcular o valor deta mesma função<br />

em um outro ponto ¯x, vizinho a x 0 , ¯x = x 0 + ∆x,<br />

mesmo que ∆x = ¯x − x 0 seja muito pequeno.<br />

Bem, nesta situação seria muito conveniente se<br />

pudéssemos calcular f(¯x), mesmo que de forma<br />

aproximada, em termos de uma série de potências<br />

em ∆x = ¯x − x 0 com coeficientes dependentes apenas<br />

dos valores conhecidos f (k) (x 0 ). Quando a<br />

função f(x) é analítica (contínua e com todas as<br />

derivadas contínuas em x 0 ), esta série existe,<br />

f(¯x) =<br />

∞∑<br />

k=0<br />

f (k) (x 0 )<br />

k!<br />

(¯x − x 0 ) k . (B.2)<br />

Hora de colocar a mão na massa. (1) Escreva<br />

uma rotina em Maple para calcular a série de Taylor<br />

(B.2) de uma função arbitrária. (2) Teste sua<br />

rotina com as principais funções trigonométricas<br />

(seno, cosseno e tangente) e com a função exponencial.<br />

(4) Nestes testes, mostre no mesmo gráfico a<br />

função originalmente definida no Maple e pelo menos<br />

quatro séries de Taylor correspondentes. (5)<br />

Faça uma animação mostrando como é o comportamento<br />

da série de Taylor em função da quantidade<br />

de termos na série para cada caso. Lembre-se: o<br />

trabalho dignifica e faz bem ao caráter.<br />

B.1 Exercícios<br />

Exercício 108<br />

Determine a expressão para um termo genérico da<br />

série de Taylor em torno de x e escreva explicitamente<br />

os quatro primeiros termos não-nulos para<br />

cada uma das seguintes funções:<br />

f(x) = e x ,<br />

g(x) = cos(x),<br />

h(x) = sin(x).<br />

(B.3)<br />

Exercício 109<br />

Escreva uma rotina em computação algébrica para<br />

calcular a série de Taylor com N termos para uma<br />

funçào dada. Verifique sua rotina comparando-a<br />

com os resultados do Exercício 108.<br />

Exercício 110<br />

Use a rotina feito no Exercício 109 para colocar em<br />

um mesmo gráfico, a função original e as seis séries<br />

de Taylor correspondentes a N = 2, N = 4, N =<br />

10, N = 20, N = 50 e N = 100 termos. Observe<br />

que para valores pequenos do número de termos N,<br />

a série de potências coincide com a função original<br />

apenas numa região muito pequena, denominada<br />

de “raio de convergência” da série.<br />

Exercício 111<br />

Mostre que qualquer número complexo z = x + i y,<br />

de módulo r = √ zz ∗ = √ x 2 + y 2 , com z ∗ = x − i y<br />

(conjugado), pode ser representado por<br />

com<br />

z = r ( cos θ + i sin θ),<br />

r = √ x 2 + y 2 , tan θ = y x .<br />

(B.4)<br />

(B.5)<br />

101


B.1. Exercícios<br />

Capítulo B. Séries de Taylor<br />

Use o Exercício 108 para mostrar que<br />

e iθ = ( cos θ + i sin θ).<br />

(B.6)<br />

102


Apêndice C<br />

Coordenadas polares<br />

Seja ⃗r = x î + yˆȷ + z ˆk um vetor posição qualquer<br />

descrito em um sistema coordenadas Cartesianas<br />

ortonormais. As componentes Cartesianas x, y e z<br />

podem ser escritas em coordenadas esféricas r, ϕ e<br />

θ,<br />

x = r sin ϕ cos θ,<br />

y = r sin ϕ sin θ,<br />

z = r cos ϕ,<br />

(C.1)<br />

onde 0 ≤ ϕ ≤ π é o ângulo formado pelo vetor<br />

posição ⃗r e o eixo Z, 0 ≤ θ ≤ 2π é o ângulo formado<br />

pela projeção do vetor posição ⃗r no plano XY e o<br />

eixo X. Note que uma casca esférica de raio R é<br />

descrita em coordenadas esféricas fazendo r = R<br />

constante e variando os ângulos ϕ e θ.<br />

Os elementos infinitesimais de área e volume<br />

também são muito utilizados. O elemento de área<br />

sobre uma esfera de raio r pode ser calculado como<br />

a área de um retângulo infinitesimal,<br />

dA = r sin ϕ dθ rdϕ = r 2 sin ϕ dϕ dθ.<br />

(C.2)<br />

O elemento de volume corresponde ao volume infinitesimal<br />

de base dA e altura dr,<br />

da mão direita, ˆr = ˆϕ ∧ ˆθ e permutações circulares.<br />

O versor ˆr está na direção e no mesmo sentido<br />

do vetor posição ⃗r. Por isto, custuma-se denominar<br />

este versor de radial. O versor ˆϕ é tangente<br />

ao grande círculo (de raio r = ||⃗r||) passando pela<br />

ponta do vetor ⃗r. O versor ˆθ é tangente ao círculo<br />

de raio r sin ϕ, também passando pela ponta do vetor<br />

⃗r.<br />

Note que as transformações (C.1) para ϕ constante<br />

e igual π/2 são as coordenadas polares no<br />

plano XY<br />

x = r cos θ,<br />

(C.6)<br />

y = r sin θ.<br />

O operador gradiente (C.5) neste caso é dado por<br />

∇ = ˆr ∂ ∂r + ˆθ ∂<br />

r ∂θ .<br />

(C.7)<br />

A derivada parcial em relação a ϕ não aparece porque<br />

esta coordenada está fixa. Assim, a sua derivada<br />

parcial a vê como uma constante.<br />

dV = dA dr = r 2 dr sin ϕ dϕ dθ.<br />

(C.3)<br />

O operador gradiente em coordenadas esféricas é<br />

escrito na seguinte forma:<br />

∇ = î ∂<br />

∂x +ˆȷ ∂ ∂y + ˆk ∂<br />

∂k ,<br />

= ˆr ∂ ∂r + ˆϕ<br />

r<br />

∂<br />

∂ϕ +<br />

ˆθ ∂<br />

r sin ϕ ∂θ ,<br />

(C.4)<br />

(C.5)<br />

onde os versores (ˆr, ˆϕ, ˆθ) também formam uma<br />

trinca de versores ortonormais obedecendo a regra<br />

103

Hooray! Your file is uploaded and ready to be published.

Saved successfully!

Ooh no, something went wrong!